FIRST AIDTM Q&A for the NBDE PART II

JASON E. PORTNOF, DMD, MD Attending Oral and Maxillofacial Surgeon Beth Israel Medical Center Jacobi Medical Center Assistant Professor Departments of and Otorhinolaryngology, Head and Neck Surgery Albert Einstein College of Medicine Bronx, New York TIMOTHY LEUNG, DMD, MHSc, MD Private Practice Oral and Maxillofacial Surgery Toronto, Ontario, Canada Oral and Maxillofacial Surgery Residency Program Department of Dentistry, Oral and Maxillofacial Surgery Jacobi Medical Center Bronx, New York MELVYN S. YEOH, DMD, MD Resident, Department of Oral and Maxillofacial Surgery New York Presbyterian Hospital Weill Cornell Medical College New York, New York

New York / Chicago / San Francisco / Lisbon / London / Madrid / Mexico City

Milan / New Delhi / San Juan / Seoul / Singapore / Sydney / Toronto Copyright © 2010 by The McGraw-Hill Companies, Inc. All rights reserved. Except as permitted under the United States Copyright Act of 1976, no part of this publication may be reproduced or distributed in any form or by any means, or stored in a database or retrieval system, without the prior written permission of the publisher.

ISBN: 978-0-07-161373-6

MHID: 0-07-161373-0

The material in this eBook also appears in the print version of this title: ISBN: 978-0-07-161372-9, MHID: 0-07-161372-2.

All trademarks are trademarks of their respective owners. Rather than put a trademark symbol after every occurrence of a trademarked name, we use names in an editorial fashion only, and to the benefi t of the trademark owner, with no intention of infringement of the trademark. Where such designations appear in this book, they have been printed with initial caps.

McGraw-Hill eBooks are available at special quantity discounts to use as premiums and sales promotions, or for use in corporate training programs. To contact a representative please e-mail us at [email protected].

NOTICE Medicine is an ever-changing science. As new research and clinical experience broaden our knowledge, changes in treatment and drug therapy are required. The authors and the publisher of this work have checked with sources believed to be reliable in their efforts to provide information that is complete and generally in accord with the standards accepted at the time of publication. However, in view of the possibility of human error or changes in medical sciences, neither the authors nor the publisher nor any other party who has been involved in the preparation or publication of this work warrants that the information contained herein is in every respect accurate or complete, and they disclaim all responsibility for any errors or omissions or for the results obtained from use of the information contained in this work. Readers are encouraged to confirm the information contained herein with other sources. For example and in particular, readers are advised to check the product information sheet included in the package of each drug they plan to administer to be certain that the information contained in this work is accurate and that changes have not been made in the recommended dose or in the contraindications for administration. This recommendation is of particular importance in connection with new or infrequently used drugs.

TERMS OF USE

This is a copyrighted work and The McGraw-Hill Companies, Inc. (“McGrawHill”) and its licensors reserve all rights in and to the work. Use of this work is subject to these terms. Except as permitted under the Copyright Act of 1976 and the right to store and retrieve one copy of the work, you may not decompile, disassemble, reverse engineer, reproduce, modify, create derivative works based upon, transmit, distribute, disseminate, sell, publish or sublicense the work or any part of it without McGraw-Hill’s prior consent. You may use the work for your own noncommercial and personal use; any other use of the work is strictly prohibited. Your right to use the work may be terminated if you fail to comply with these terms.

THE WORK IS PROVIDED “AS IS.” McGRAW-HILL AND ITS LICENSORS MAKE NO GUARANTEES OR WARRANTIES AS TO THE ACCURACY, ADEQUACY OR COMPLETENESS OF OR RESULTS TO BE OBTAINED FROM USING THE WORK, INCLUDING ANY INFORMATION THAT CAN BE ACCESSED THROUGH THE WORK VIA HYPERLINK OR OTHERWISE, AND EXPRESSLY DISCLAIM ANY WARRANTY, EXPRESS OR IMPLIED, INCLUDING BUT NOT LIMITED TO IMPLIED WARRANTIES OF MERCHANTABILITY OR FITNESS FOR A PARTICULAR PURPOSE. McGraw-Hill and its licensors do not warrant or guarantee that the functions contained in the work will meet your requirements or that its operation will be uninterrupted or error free. Neither McGraw-Hill nor its licensors shall be liable to you or anyone else for any inaccuracy, error or omission, regardless of cause, in the work or for any damages resulting therefrom. McGraw-Hill has no responsibility for the content of any information accessed through the work. Under no circumstances shall McGraw-Hill and/or its licensors be liable for any indirect, incidental, special, punitive, consequential or similar damages that result from the use of or inability to use the work, even if any of them has been advised of the possibility of such damages. This limitation of liability shall apply to any claim or cause whatsoever whether such claim or cause arises in contract, tort or otherwise. DEDICATION

JEP would like to dedicate this book to his two best friends: his beautiful wife, Courtney, and his amazing son, Justin. Thank you for making life wonderful!

TL would like to dedicate this book to his wife Josephine and new baby daughter Evelyn.

MSY would like to dedicate this book to his wife, Sophia, and son, Max, who tolerate his mood swings and behavior and still love him in spite of it and to his parents, who have been supportive in all his pursuits. This page intentionally left blank CONTENTS

Contributors vii

Faculty Reviewers xi Test-Taking: Getting Prepared Physically and Mentally xv

Chapter 1 Pharmacology 1 Questions 2 Answers 10 Chapter 2 Operative Dentistry and 25 Questions 26 Answers 40 Chapter 3 Oral and Maxillofacial Surgery and Pain Control 53 Questions 54 Answers 71 Chapter 4 Orthodontics 95 Questions 96 Answers 117 Chapter 5 Pediatric Dentistry 135 Questions 136 Answers 140 Chapter 6 Endodontics 145 Questions 146 Answers 153 Chapter 7 Periodontics 163 Questions 164 Answers 168 Chapter 8 Radiology 173 Questions 174 Answers 182 Chapter 9 Pathology 191 Questions 192 Answers 202 Chapter 10 Patient Management, Public Health, Ethics, and Biostatistics 221 Questions 222 Answers 227

Index 233

About the Authors 240

v This page intentionally left blank CONTRIBUTORS

Anayo O. Adachie, DMD, MD Resident, Oral and Maxillofacial Surgery Beth Israel Medical Center, Jacobi Medical Center, Albert Einstein College of Medicine Bronx, New York

Jose Ignacio Alamo, Jr., DMD Resident, Senior General Practice Department of Dentistry & Oral and Maxillofacial Surgery New York Presbyterian Hospital—Weill Cornell Medical College New York, New York

David M. Alfi, DDS, MD Resident, Oral and Maxillofacial Surgery Division of Oral and Maxillofacial Surgery New York Presbyterian Hospital New York, New York

Will R. Allen, DMD Resident, Oral and Maxillofacial Surgery Beth Israel Medical Center/Jacobi Medical Center/ Albert Einstein College of Medicine Bronx, New York

LoanAnh Bui Student Class of 2010 SUNY Stony Brook School of Dental Medicine Stony Brook, New York

Margaret LeeAnn Clark Student Class of 2009 New York University College of Dentistry New York, New York

Jennifer Frangos, DDS Intern, Department of Oral and Maxillofacial Surgery Baltimore College of Dental Surgery University of Maryland Baltimore, Maryland

vii viii CONTRIBUTORS

Daniel Fruchter DMD Resident, General Practice Department of Dentistry Jacobi Medical Center Bronx, New York

Evan Hershkowitz, DDS Resident, Pediatric Dental Department of Dentistry Jacobi Medical Center Bronx, New York

Jamie J. M. Hwang, DMD Private Practice, General Dentistry New York, New York

James A. Kraus, DMD Chief Resident, Oral and Maxillofacial Surgery Tufts Medical Center Boston, Massachusetts

Lindsey J. Krecko, DDS Chief Resident, General Practice Dentistry Illinois Masonic Medical Center Department of Dentistry Chicago, Illinois

Andrew W. C. Lee, MSc, DDS, MD Resident, Oral and Maxillofacial Surgery Department of Dentistry & Oral and Maxillofacial Surgery New York Presbyterian Hospital—Weill Cornell Medical College New York, New York

Katherine Lee, DDS Intern, Oral and Maxillofacial Surgery Department of Dentistry Jacobi Medical Center Bronx, New York

Won Sang Lee, DDS Resident, Oral and Maxillofacial Surgery Department of Dentistry Beth Israel Medical Center, Jacobi Medical Center, Albert Einstein College of Medicine Bronx, New York CONTRIBUTORS ix

Avi Malkis, DDS Student Class of 2010 New York University College of Dentistry New York, New York

Mariya Mamkin, DDS Attending Dentist, Department of Dental and Oral Medicine New York Hospital of Queens Queens, New York

Pankti Mehta, DDS Private Practice, General Dentistry Danbury, Connecticut

Keith Murtagh, DDS, FAAOMP Resident, Oral and Maxillofacial Surgery Brookdale University Hospital and Medical Center Brooklyn, New York

Nona Naghavi, DDS Resident, Orthodontics Jacksonville University Jacksonville, Florida

Niral N. Parikh, DDS, BDS Resident, Oral and Maxillofacial Surgery Department of Dentistry Beth Israel Medical Center, Jacobi Medical Center, Albert Einstein College of Medicine Bronx, New York

Debbie M. Parnes, DMD Resident, Department of Orthodontics Temple University Philadelphia, Pennsylvania

Clifford Salm, DMD Beth Israel Medical Center Private Practice, Oral and Maxillofacial Surgery New York, New York

Dara J. Sandler, MS, RD, CDN Private Practice, Registered Dietitian New York, New York x CONTRIBUTORS

Jonathan W. Shum, DDS, MD Resident, Oral and Maxillofacial Surgery Department of Dentistry & Oral and Maxillofacial Surgery New York Presbyterian Hospital—Weill Cornell Medical College New York, New York

Robert M. Spriggel, DDS, Capt., USAF, DC Clinical Dentist 1st Special Operations Medical Group, Dental Squadron Hurlburt Field, Florida

David E. Webb, DDS, Capt, USAF Resident, Oral and Maxillofacial Surgery Wilford Hall Medical Center Lackland Air Force Base San Antonio, Texas

Marta Williams, DDS Intern, Oral and Maxillofacial Surgery Department of Dentistry Jacobi Medical Center Bronx, New York

Stacy B. Wolf, DDS Student Class of 2009 New York University College of Dentistry New York, New York FACULTY REVIEWERS

Victor M. Badner, DMD, MPH North Bronx Healthcare Network Chairman, Department of Dentistry/OMFS Dental Public Health Residency—Site Director Jacobi Medical Center Bronx, New York

Daniel Buchbinder, DMD, MD Chief, Division of Oral and Maxillofacial Surgery OMFS Residency Program Director Head and Neck Institute Department of Otolaryngology Beth Israel Medical Center New York, New York

Stuart A. Caplan, DDS, MS Assistant Professor, Diagnostic Sciences Nova Southeastern University College of Dental Medicine Fort Lauderdale, Florida

Kenneth B. Cooperman, DMD Private Practice, Orthodontics New York, New York Assistant Professor of Dentistry Albert Einstein College of Medicine Jacobi Medical Center Bronx, New York

Dennis Davis, DMD, MS Adjunct Professor, Department of Periodontology University of Florida College of Dentistry Gainesville, Florida Private Practice Lady Lake, Florida

Jennifer Blake Eli, DMD Private Practice, General Dentistry Lake Forest, California

xi xii Faculty Reviewers

Francisco Eraso, DMD, MS, ABO Radiologist, Oral & Maxillofacial Adjunct Associate Professor of Orthodontics Department of Orthodontics and Oral Facial Development Indiana University School of Dentistry Oral & Maxillofacial Imaging Center—Indianapolis Indianapolis, Indiana

Andrew Forrest, DMD, MS Clinical Assistant Professor Department of Periodontics College of Dental Medicine University of Florida Gainesville, Florida

Andrew A. C. Heggie, MBBS, MDSc, FFDRCS, FACOMS, FRACDS (OMS) Section Chief, Oral and Maxillofacial Surgery Department of Plastic and Maxillofacial Surgery Royal Children’s Hospital Associate Professor Department of Pediatrics University of Melbourne Melbourne, Victoria, Australia

Christine Heng, DDS, MPH Captain, United States Public Health Service Federal Correctional Institution Danbury, Connecticut Department of Dentistry/OMFS Jacobi Medical Center Bronx, New York

Marcela Herrera, DMD Clinical Assistant Professor Department of Preventive and Restorative Sciences University of Pennsylvania School of Dental Medicine Philadelphia, Pennsylvania

Jason P. Hirsch, DMD MPH Private Practice, Pediatric Dentistry Plantation, Florida

Laurance Jerrold, DDS, JD, ABO Dean and Program Director Jacksonville University School of Orthodontics Jacksonville, Florida Faculty Reviewers xiii

Kevin Kohler, DMD Private Practice, Family and Cosmetic Dentistry Boise, Idaho

Keith S. Margulis, DDS Diplomate, American Board of Pediatric Dentistry Assistant Director—Advanced Education in Pediatric Dentistry North Bronx Healthcare Network Jacobi Medical Center Bronx, New York

Christopher Phelps, DMD Private Practice, General Dentistry Charlotte, North Carolina

Gol Pourkhomami, DMD Family and Cosmetic Dentistry Adjunct Faculty Arizona School of Dentistry and Oral Health A.T. Still University Mesa, Arizona

Morton Rosenberg, DMD Professor of Oral and Maxillofacial Surgery Tufts University School of Dental Medicine Associate Professor of Anesthesia Tufts University School of Medicine Tufts Medical Center Boston, Massachusetts

Michelle Segal, DMD Private Practice, General and Cosmetic Dentistry Miami, Florida

Robert M. Sorin, DMD Clinical Instructor in Surgery Attending Dentist Division of Dentistry, Oral and Maxillofacial Surgery New York Presbyterian Hospital—Cornell Campus New York, New York

James D. Toppin DDS, MBA Clinical Assistant Professor, Oral and Maxillofacial Pathology, Radiology and Medicine New York University College of Dentistry New York, New York xiv Faculty Reviewers

Robert A. Uchin, DDS Dean, College of Dental Medicine Health Professions Division Nova Southeastern University Fort Lauderdale, Florida

Farhad Yeroshalmi, DMD, FAAPD Diplomate, American Board of Pediatric Dentistry Program Director, Pediatric Dentistry Residency Program Jacobi Medical Center Bronx, New York TEST-TAKING: GETTING PREPARED PHYSICALLY AND MENTALLY

Preparation advice from Dara Sandler, MS, RD who is a Clinical Nutritionist in New York, New York:

Although the anticipation and uncertainty of stepping into the testing center to take the Dental Board Exam can be taxing on the mind, it can also take its toll physically. You should regard nourishment, hydration, and rest equally as important as being academically prepared. You will enhance and sustain your ability to focus and perform by providing your body with the necessary vitamins and minerals. Eating a balanced meal that consists of protein, complex carbohydrates, fat, and B vitamins prior to sitting down for the test improves your memory function and increases your mood. Proteins—such as eggs, milk products, and nuts—keep your blood sugar steady, a function of both keeping your brain alert and your hunger satisfied over long periods of time. Complex carbohydrates (3 g or more per serving) that exist in foods such as whole grains, oatmeal, legumes, fruits, and vegetables will also help to sustain your energy. Fats, such as Omega-3 fatty acids, are important to include in your pretest meal because they help with blood circulation, memory, and your immune system. Nuts and seeds, vegetable oil, avocado, and different types of fish are good sources of Omega-3s. Foods that contain B vitamins, such as whole grains, eggs, and milk products, are extremely important in maintaining your mental energy. Combining all these nutrients into a well-balanced meal will allow you to sustain your energy and keep you sharp and focused. A breakfast of eggs, cheese, whole- grain toast, and yogurt will meet all the criteria and carry you through the test; cottage cheese, fruit, and whole-grain crackers, or a protein smoothie made with peanut butter, milk, and fruit are also great choices. During the test, smart snacking will supplement your balanced meal and allow you to maintain focus. Small snacks containing protein and complex carbohydrates are your best options: peanut butter on a banana, whole-wheat crackers with cheese, or cottage cheese and nuts. Finally, hydration is a key factor in rounding out your day. Stress can cause mild dehydration; make sure to drink plenty of water before and during the test. It is recommended that adults drink eight 8-ounce glasses of fluid each day. Following these simple guidelines will help your mind stay razor sharp and enable you to concentrate on your exam rather than on your stomach. Take a break from studying and take a trip to your local market to plan your exam-day diet. You owe it to yourself!

xv This page intentionally left blank CHAPTER 1 Pharmacology

1 2 Chapter 1: Pharmacology

QUESTIONS

1. A 63-year-old male patient with a history of in- A. Oxygen

Pharmacology frequent dental visits reports to your office with B. Epinephrine the complaint of swollen gums. He has a his- C. Nitroglycerin tory of high blood pressure, high cholesterol, and D. Sugar was in the hospital 2 years ago after suffering a E. Hydrocortisone silent myocardial infarction. He also has a his- tory of asthma. He is currently taking lisinopril, 4. A 15-year-old male patient presents to your office nifedipine, atenolol, simvastatin, and albuterol for an initial dental visit. While reviewing the pa- as needed. Which of the following medications tient’s medical history with him and his parents, is the most likely cause of his hyperplastic gin- he reveals that during a recent basketball game giva? he suffered from chest pain and fainted. After several tests, including an echocardiogram, he A. Lisinopril was diagnosed with hypertrophic cardiomyopa- B. Nifedipine thy. He also has a history of exercise-induced C. Atenolol asthma. He has an albuterol inhaler but cannot D. Simvastatin remember the name of the medication he was E. Albuterol recently put on for his heart condition. Which 2. A 52-year-old overweight male patient presents of the following beta-blockers is he most likely to your office with erosion of the lingual surfaces taking? of all his maxillary teeth. The patient also says he A. Propranolol suffers from heartburn, dysphagia, and a chronic B. Labetalol cough. The patient’s physician has prescribed C. Metoprolol him cimetidine and told him to take over-the- D. Nadolol counter antacids. Which of the following is the E. Timolol most likely mechanism of the therapeutic effect of cimetidine? 5. A 55-year-old female patient was referred to your office by her physician because she is prepar- A. Eradicates Helicobacter pylori ing to receive an infusion of zoledronic acid. B. Neutralizes stomach acid Her physician told her to see a dentist to deter- C. Reduces GI motility and acid secretion mine if she has any teeth that may need to be ex- D. Blocks histamine-2 receptors on parietal tracted in the near future. If so, these should be cells extracted prior to the infusion because of the risk E. Blocks proton pumps on parietal cells of poor bone healing (osteonecrosis) that some- 3. A female patient presents to your office with fa- times occurs in patients receiving infusions of cial and torso obesity, facial hair growth, and zoledronic acid. Prior to the infusion, the pa- a deepened voice. The patient explains these tient was taking an oral form of this type of drug. changes occurred when she started taking a com- What is another major side effect of the oral mon drug to treat Crohn’s disease, and subse- form that her physician may have warned her quently she recently discontinued the drug use. about? While performing a surgical extraction, the pa- A. Anemia tient suddenly starts breathing at a fast rate, B. Xerostomia sweating, and complains of nausea and chills. C. Edema You recognize that the patient is experiencing a D. Esophageal perforation crisis and reach for your Emergency Kit. Which E. Orthostatic hypotension of the following contents of your Emergency Kit would be most appropriate to alleviate this crisis? Questions: 1-10 3 Pharmacology

6. In rats, a new anticonvulsant drug has an lethal few months back, he acquired an infection while dose (LD)1 of 20 mg, and LD50 of 30 mg, an working at a homeless shelter in the nearby city effective dose (ED)50 of 5 mg, and an ED99 of 10 and was admitted to the hospital after develop- mg. What is the experimental drug’s therapeutic ing pneumonia. His most recent chest x-ray was index and safety index, respectively? negative, but he still has a positive skin test. On the basis of the disease, this patient has been A. 2 and 1.5 treated for and the intraoral image taken dur- B. 1.5 and 2 ing the examination, what oral mucosal lesion C. 6 and 2 does this patient have? D. 2 and 6 E. 3 and 4 F. 4 and 3

7. A 40-pound child presents to your office for a pulpotomy and placement of a stainless steel on tooth #L. What would be the approxi- mate maximum recommended dose of 2% lido- caine? A. 1 carpule B. 2 carpules C. 3 carpules D. 4 carpules E. 5 carpules

8. Your 55-year-old male patient is scheduled for three quadrants of periodontal surgery. Each sur- A. Linea alba gical procedure is scheduled 4 weeks apart. This B. Nonhealing ulcer patient is allergic to penicillin and has a history C. Perioral vesicles of a mitral valve prolapse. On the basis of the D. Hairy leukoplakia current guidelines, which prescription would be E. Lichen planus most appropriate to adequately premedicate this 10. An elderly patient presents to your office with patient for all of his visits? a long list of medications that she is currently A. Amoxicillin 2-g tablets; Disp: 3 tablets; Sig: taking to treat the following conditions: history take 1 tablet 30–60 minutes prior to dental of stroke, depression, arthritis, and glaucoma. In procedure the clinical examination, you note multiple cer- B. Amoxicillin 2-g tablets; Disp: 6 tables; Sig: vical caries and severe dry mouth. Which of the take 2 tablets 30–60 minutes prior to dental following medications is the most likely cause of procedure xerostomia in this patient? C. Clindamycin 600-mg tablets; Disp: 3 tablets; A. Amitriptyline Sig: take 1 tablet 30–60 minutes prior to den- B. Aspirin tal procedure C. Pilocarpine D. Clindamycin 600-mg tablets; Disp: 6 tablets; D. Nabumetone Sig: take 2 tablets 30–60 minutes prior to E. Physostigmine dental procedure E. No prescription necessary

9. Your patient presents to your office and reports that he recently completed a 2-month course of the following medications: isoniazid, rifampin, ethambutol, and pyrazinamide. He states that a 4 Chapter 1: Pharmacology

11. An emergency patient presents to your office possible substance interactions. When planning with trismus, fever, and a swelling on one side for a surgical procedure, which of the following of his lower face. The patient questionnaire he alternative medications might you want to dis- filled out in the waiting room states that he has continue to prevent excessive bleeding in this

Pharmacology no medical issues, takes no medications, and has patient? a history of alcohol abuse but has been sober for A. Ginseng one month. During the examination, you ob- B. Echinacea serve extensive decay on #18 and a small abscess C. St. John’s Wort in the apical region of #18. The tooth is hopeless, D. Ginkgo Balboa but before extraction, you decide to place the pa- E. Capsaicin tient on an antibiotic to clear up the acute intrao- ral infection. Which of the following antibiotics 14. During a routine medical history interview, your would you want to avoid because of the patient’s patient tells you he is taking celecoxib for os- social history and the potential disulfiram-like teoarthritis. He has been on this medication effect? for the past year. Compared to traditional non- A. Clarithromycin steroidal anti-inflammatory drugs (NSAIDs) like B. Metronidazole ibuprofen, celecoxib: C. Clindamycin A. Is a more effective analgesic for acute post- D. Doxycycline surgical pain E. Rifampin B. Produces greater anti-inflammatory re- sponses in osteoarthritis patients 12. One of your friends, who is also a patient, is in C. Produces less water and sodium retention your office for a routine visit. While in your of- D. Produces less gastrointestinal ulcers and fice, she asks if you can write a prescription for bleeds with chronic use one of the drugs her primary care physician nor- E. Is more effective at fever reduction mally prescribes for an existing condition. Your friend, the patient, says she does not have much 15. You received a call from the emergency depart- time to make an appointment with her primary ment at your local hospital concerning a pe- care physician so she would really appreciate diatric patient that you recently treated. The if you could write a script for the pharmacist. patient is a 6-year-old boy who was brought to When faced with this question, as a practicing the emergency department in a comatose state. dentist, it is important to keep in mind that you His parents state that he recently visited your of- can legally write prescriptions for which of the fice for extraction of a grossly decayed primary following drugs? tooth. The parents state that they had given him a A. Oral contraceptives few pain pills to help with the discomfort follow- B. Blood pressure lowering drugs ing the extraction. Physical examination of the C. Analgesic agents boy revealed “comatose, hepatomegaly, and yel- D. Antidiabetic agents low sclera.” A computerized tomography (CT) E. Amphetamines scan revealed cerebral edema. Laboratory re- F. Antiasthmatics sults showed elevated alanine aminotransferase (ALT), aspartate aminotransferase (AST), and 13. A patient presents to your office for an initial prothrombin time (PT). The emergency depart- visit. When reviewing the patient’s medical his- ment physician believes that treatment with the tory, the patients tells you that she is taking 81 mg antidote N-acetylcysteine is necessary but has of aspirin once a day and several alternative med- called you to confirm whether you prescribed ications. She does not specify which alternative any medications. What agent is most likely in- medications she is taking because she assumes volved in the symptoms of this patient? they are not necessary for you to know. In fact, it is important for dentists to know about all medi- A. Aspirin cations and supplements to protect patients from B. Acetaminophen Questions: 11-21 5 Pharmacology

C. Codeine A. Drug Z is a liver microsomal enzyme inducer D. Celecoxib B. Drugs X and Z formed a chelation product E. Ibuprofen in the intestine C. Drug Z increased the glomerular filtration 16. A patient presents to your office with the fol- rate lowing current medication list: warfarin, aspirin, D. Drug Z has a positive inotropic and chrono- and clotrimazole. Before beginning the proce- tropic effect dure you had planned, you decide it is worth- E. Drugs X and Z are highly bound to plasma while to look up further information on these proteins drugs. Warfarin is metabolized in the liver and has a volume of distribution of approximately 19. A patient presents to your office for a surgical 0.14 L/kg. You are slightly concerned that this extraction. Following the procedure, you pre- patient may experience increased bleeding be- scribe the patient tylenol #3 (acetaminophen cause of these properties along with the con- + codeine). The patient is taking several other comitant medications he is taking. Which of the drugs concurrently. Which of the following following properties of a drug is the least likely drugs is most likely to inhibit the effectiveness to contribute to a significant adverse interaction of codeine? with another drug? A. Terazosin A. The drug has a low therapeutic index B. Glipizide B. The drug has minimal protein binding C. Lisinopril C. The drug undergoes extensive cytochrome D. Fluoxetine P450 metabolism E. Lithium D. The drug is an enzyme inducer E. The drug has a low volume of distribution 20. A patient with uncontrolled asthma may be tak- ing multiple medications to attempt to control 17. Many people use herbal supplements as alter- the disease. Which of the following best de- nate therapy for a variety of conditions. It is im- scribes the mechanism of action of montelukast? portant for physicians and dentists to know what A. Beta-2 adrenergic agonist medications their patients are taking, including B. Leukotriene receptor antagonist herbal supplements. St. John’s Wort is an exam- C. Lipoxygenase pathway inhibitor ple of a supplement commonly used to relieve D. Muscarinic receptor antagonist depression. The simultaneous ingestion of the E. Corticosteroid used to reduce inflammation herbal St. John’s Wort has been shown to de- F. Suppresses the cough reflex crease the half-life of oral contraceptives because St. John’s Wort: 21. A patient presents to your office with complaints A. Blocks the active secretion of oral contracep- of a burning tongue. She is overweight and says tive her average fasting blood glucose level is 326 B. Induces CYP-3A4 in the gut and liver mg/dL. She also states that her most recent C. Increases stomach acidity HbA1c was 9%. The patient was prescribed sev- D. Blocks both estrogen and progesterone re- eral medications by her physician but claims that ceptors most of them she does not take because she for- E. Decreases serotonin levels in the brain gets. Which of the following medications should the patient be taking? 18. After taking Drug X for 5 days, steady free blood A. Levothyroxine levels average about 20 ug/mL. Drug Z is then B. Hydrocortisone introduced while the patient continues to take C. Insulin Drug X, and the free blood levels of Drug X D. Omeprazole increase to 100 ug/mL. What could have ac- E. Ranitidine counted for this? 6 Chapter 1: Pharmacology

22. A patient presented to your office for an emer- A. Vitamin A gency visit. One side of the patient’s face was B. Vitamin B swollen, and on examination, a small abscess was C. Vitamin C present apical to tooth #3. The abscess was in- D. Vitamin D

Pharmacology cised and drained, penicillin was prescribed, and E. Vitamin K root canal treatment was recommended. The next day, the patient calls your office and com- 27. A 50-year-old man complains of heartburn. plains of swollen lips and a developing rash. The Which one of the following medications will best response to this situation is to: help prevent acid secretion in the stomach? A. Tell the patient not to worry; this is a normal A. Omeprazole reaction B. Cimetidine B. Prescribe epinephrine C. Famotidine C. Instruct the patient to discontinue the peni- D. Ketoconazole cillin and prescribe amoxicillin instead E. Ranitidine D. Instruct the patient to discontinue the peni- cillin and prescribe clindamycin instead 28. Which one of the following benzodiazepines does not require Phase I metabolism by the 23. A pediatric patient is scheduled to have multiple liver? restorative procedures carried out in the operat- A. Alprazolam ing room under anesthesia. The drug chosen for B. Oxazepam anesthesia is fentanyl. Should this patient show C. Midazolam signs of respiratory depression, what would be D. Triazolam the appropriate reversal agent? E. Diazepam A. Epinephrine B. Naloxone 29. Continuous infusion of medication A is given to C. Flumazenil a 65-kg patient. The pharmacokinetic parame- = D. Clonidine ters are as follows: clearance 9 mL/min/kg, vol- = = E. Hydrocortisone ume of distribution 65 L, half-life 4 hours. How long will it take for the medication level to 24. Which drug is most appropriate to treat oral can- reach 93.75% of steady state? didiasis? A. 8 hours A. Penicillin B. 10 hours B. Amphotericin B C. 14 hours C. Nystatin D. 16 hours D. Tetracycline E. 18 hours E. Valacyclovir 30. Which one of the following is an ester local anes- 25. Which of the following local anesthetics is most thetics? likely to cause an allergic reaction? A. Bupivacaine A. Lidocaine B. Lidocaine B. Propoxycaine C. Mepivacaine C. Prilocaine D. Tetracaine D. Mepivicaine E. Prilocaine E. Bupivacaine 31. A 75-year-old smoker with history of chronic ob- 26. Accidental ingestion of warfarin can lead to ele- structive disease presents to clinic with blood vated PT and PTT. Which one of the following pressure of 160/96 mm Hg. Which one of the is the most appropriate treatment? Questions: 22-38 7 Pharmacology

following medications is contraindicated in this C. Protamine patient? D. Flumazenil E. Insulin A. Atenolol B. Metoprolol 35. Which one of the following is true regarding C. Propranolol paroxetine? D. Esmolol E. Acebutolol A. It specifically inhibits serotonin reuptake B. It is used to treat mania 32. A new antibiotic is being tested in a clinical trial. C. It can be safely taken with monoamine oxi- The following pharmacokinetic parameters are dase inhibitor (MAOI) listed as follows: Clearance = 200 mL/min, vol- D. GI upset is not a common side effect ume of distribution (Vd) = 50 L, half-life = E. Sexual dysfunction is rarely reported 6 hours. Assuming that the drug is being admin- istered intravenously, what loading dose (LD) 36. Which one of the following beta-lactam antibi- should be given to the patient to reach a plasma otics has antipseudomonal properties? concentration (Cp) of 5 mg/L? A. Penicillin V A. 50 B. Penicillin G B. 100 C. Piperacillin C. 150 D. Amoxicillin D. 200 E. Ampicillin E. 250 37. Which one of the following statements is true 33. A 60-year-old female patient with history of hy- regarding heparin? pertension was recently put on antihypertensive A. Its effect cannot be reversed with protamine medications. She is now reporting recent onset sulfate of polyuria and polydipsia. The patient does not B. It can bind and activate antithrombin III, re- appear to have any other medical conditions. sulting in the inactivating thrombin and fac- Which one of the following medication is most tor Xa likely of what the patient is taking? C. It can lead to decrease in PTT, which is a A. Hydrochlorothiazide clinical measure used to follow the anticoag- B. Enalapril ulant effect caused by heparin C. Terazosin D. It decreases the synthesis of vitamin K coag- D. Methyldopa ulation factors E. Diltiazem E. There are no applications of heparin in pul- monary embolism 34. A 20-year-old man with no prior significant med- ical history visits his oral surgeon for wisdom 38. Which one of the following statements is true tooth extraction with IV sedation. He was given regarding warfarin? IV midazolam during the procedure. Midway A. Follow PTT in determining its clinical effect through the surgery, he suddenly becomes ag- B. It inhibits the synthesis of vitamin K- itated and combative and exhibits involuntary dependent clotting factors (III, VIII, XI, XII) movements. The surgeon determines that this C. It is teratogenic is likely due to disinhibition reaction to the D. Protein C and S levels are not affected benzodiazepine. Which one of the following E. It is false that warfarin is found in rat poison medication can be used to reverse the effect of midazolam? A. Naloxone B. Naltrexone 8 Chapter 1: Pharmacology

39. What do prazosin, doxazosin, and terazosin have 43. Which one of the following statements is true in common? regarding sulfonylureas? A. They are all alpha-2 receptor blocker that A. They include glipizide and glyburide causes a decrease in peripheral vascular re- B. They stimulate insulin release from alpha Pharmacology sistance cells of pancreas and binding of insulin to B. They are diuretics used to control hyperten- target tissue and inhibit release of glucagons sion C. It can decrease hepatic gluconeogenesis C. Side effects of the drugs include orthostatic D. It is the first-line medication for type I dia- hypotension betes mellitus D. They are noncompetitive alpha-receptor E. Lactic acidosis is a concern for sulfonylureas blockers. overdose E. They rarely cause headache or dizziness 44. Which one of the following is not an antifungal? 40. A 50-year-old patient with recent diagnosis of hy- A. Nystatin pertension presents to your clinic complaining B. Amphotericin B of a sudden increase in the size of his tongue C. Ketoconazole and slight difficulty in breathing. He was re- D. Esomeprazole cently started on an antihypertensive medica- E. Griseofulvin tion. Which one of the following drug classes may have caused the patient’s presentation? 45. N-acetylcysteine is used as an antidote for over- A. Beta-blocker dose of which of the following medications? B. Alpha 1-blocker A. Acetaminophen C. Thiazide diuretic B. Ibuprofen D. Angiotensin-converting enzyme (ACE) in- C. Celecoxib hibitor D. Indomethacin E. Calcium channel blocker E. Naproxen

41. Which one of the following is true regarding tri- 46. All of the following statement is true regarding cyclic antidepressants (TCAs)? NSAIDs, except: A. They include phenelzine, isocarboxazid, A. Aspirin irreversibly inhibits cox-1 and cox-2 and tranylcypromine cyclooxygenases B. They inhibit reuptake of norepinephrine B. All NSAIDs, except aspirin, are reversible in- and serotonin into presynaptic nerve termi- hibitors of cox-1 and cox-2 cyclooxygenases nals C. Prostaglandin synthesis is rarely affected C. They are only used in depression D. They are used for inflammation, analgesia, D. Arrhythmias are a concern with TCA over- and antipyrexia dose E. Side effects include dyspepsia, GI ulcers, and E. Orthostatic hypotension rarely occurs renal failure

42. Which one of the following anticonvulsants is 47. Which one of the following is true about H1 known to cause neural tube defects in unborn antihistamines? fetus? A. They include cimetidine and diphenhy- A. Phenytoin dramine B. Carbamazepine B. They are rarely used for allergy C. Gabapentin C. Some are more sedating than others D. Benzodiazepines D. loratadine is a first-generation H1 antihis- E. Valproic acid tamine E. They can be used for motion sickness Questions: 39-55 9 Pharmacology

48. All of the statements below about glucocorti- 52. After 0.6 g of Drug A was administered intra- coids are true, except: venously, the plasma concentration is 100 g/L. What is the correct volume of distribution of A. They function by binding to an extracellular Drug A in liters? receptor B. They decrease the production of leuko- A. 6 trienes and prostaglandins B. 60 C. Inhibition of phospholipase A2 is a key to C. 0.167 their mechanism of action D. 166.67 D. They are contraindicated in Cushing syn- E. 0.006 drome E. They are commonly used in inflammation 53. Which of the following is correct regarding side and immunosuppression effects of angiotensin-converting enzyme (ACE) inhibitors? 49. Which one of the following is true about opi- A. Hypertension oids? B. Hyperkalemia A. They can interact with ␮, ␦, and/or ␬ opioid C. Cough receptor in PNS and enteric nervous system D. Cerebral edema B. They can cause respiratory depression E. Depression C. They can cause diarrhea D. All can cause mydriasis 54. The plasma concentration of drug X is 150 mg/L, E. Naloxone is an example of an opioid and it is eliminated from the body at a rate of 3 mg/min. Which of the following is the clear- 50. Misoprostol can be used for the following, ance rate of drug X? except: A. 0.2 L/min A. Abortion B. 0.02 L/min B. Induction of labor C. 2 L/min C. Prevention of NSAID-induced peptic ulcer D. 50 L/min disease E. 200 L/min D. Rheumatoid arthritis E. Maintenance of patent ductus arteriosus 55. All of the following statements are correct re- (PDA) in newborns with certain congenital garding penicillin, except: cardiovascular defects A. Penicillin is a bactericidal antibiotic that works by binding to penicillin-binding pro- 51. All of the following statements are true regarding tein, and blocking transpeptidase from bioavailability, except: crosslinking peptidylglycine in bacterial cell A. Route of administration is important in de- wall termining bioavailability B. Not all penicillins are effective against Pseu- B. High hepatic first pass effect decreases domonas aeruginosa bioavailability C. Amoxicillin can cover both Escherichia coli C. Degradation of drug prior to absorption in- and Haemophilus influenzae creases bioavailability D. Combined with clavulanic acid or tazobac- D. Hydrophilic drugs are less able to cross tam, penicillins possess anti-beta lactamase lipid-rich cell membranes than hydrophobic activity drugs E. Penicillin is a bacteriostatic antibiotic that E. IV administration yields 100% bioavailability works by inhibiting bacterial protein syn- thesis 10 Chapter 1: Pharmacology

ANSWERS

1. The correct answer is B. Nifedipine is an ex- of drugs is used to control hypercholesterolemia

Pharmacology ample of a calcium channel blocker (CCB). and coronary artery disease. Adverse effects of These drugs work by blocking calcium entry statins include muscle pain, with an increased into smooth muscle cells of the heart and its risk of muscle toxicity (rhabdomyolysis) when associated arteries. Calcium is responsible for these drugs are taken concomitantly with drugs heart contraction and narrowing of the arteries, that inhibit the CYP3A4 enzyme system, includ- and its inhibition results in vasodilation of coro- ing erythromycin and azole antifungals. nary and peripheral blood vessels and decreased Albuterol is an example of a beta-2 adrener- contraction force (negative inotropic effect) and gic agonist, meaning that it stimulates beta-2 re- rate (negative chronotropic effect). CCBs are ceptors located on the muscles surrounding the used for treating hypertension, angina, and ar- airways. Stimulation of these receptors induces rhythmias. Side effects of CCBs include gin- bronchodilation. It is taken via inhalation to treat gival hyperplasia and orthostatic hypotension. acute asthmatic attacks. Side effects include anx- Other drugs that cause gingival hyperplasia in- iety, tremors, fast heart rate, and throat irritation. clude phenytoin and cyclosporine. Lisinopril is an example of an ACE in- 2. The correct answer is D. Based on the signs hibitor; is used for treating hypertension and and symptoms that the patient described above heart failure, prevention of strokes, and kidney is experiencing, he is most likely suffering from a failure; and improves survival after heart attacks. gastrointestinal (GI) problem. All of the answer ACE inhibitors work by inhibiting the enzyme choices are mechanisms of drugs that are used ACE, which catalyzes the conversion of an- to treat GI upset (i.e., duodenal ulcers, gastroe- giotensin I to angiotensin II, a potent vasocon- sophageal reflux disease (GERD), or Zollinger– strictor and stimulant of aldosterone. As a re- Ellison syndrome) and choice D best describes sult, blood vessels dilate and sodium and water the mechanism of cimetidine, an antihistamine retention is reduced. In addition, ACE inhibi- H2 blocker. H2 receptor blockers complete with tion blocks the breakdown of bradykinin, a va- histamine only in the GI tract and consequently sodilator. The side effects of ACE inhibitors can interrupt the signal for the parietal cell to secrete be remembered using the mnemonic “CAPTO- acid, resulting in overall decreased acid secre- PRIL” (another example of an ACE inhibitor), tion. When a patient is on cimetidine, it is also which stands for: cough, angioedema, potassium important to be aware of potential drug inter- excess, taste changes, orthostatic hypotension, actions that may occur because it is a CYP3A4 pregnancy contraindication, rash, indomethacin inhibitor and can increase levels of drugs metab- inhibition, liver toxicity. Gingival hyperplasia is olized in the liver by the CYP3A4 enzyme. Other not on this list. H2 blockers (i.e., ranitidine, famotidine) are not Atenolol is an example of a beta adrener- associated with these potential interactions. gic receptor blocker. Beta blockers inhibit beta Helicobacter pylori is a bacterium that is asso- adrenergic substances from affecting the sympa- ciated with gastritis, duodenal or stomach ulcers thetic nervous system. In the heart, their effect is (especially recurrent ulcers), and stomach can- to lower cardiac output and decrease the work- cer, including MALT lymphoma. Infection with load of the heart, resulting in reduced peripheral this bacterium is most likely acquired from con- pressure. These drugs are used for hypertension, taminated food or water and must be treated with angina, arrhythmias, and migraine headaches. antibiotics. Because H. pylori is capable of devel- Side effects include orthostatic hypotension and oping resistance to commonly used antibiotics, difficulty breathing, particularly in asthmatics. usually two or more antibiotics are prescribed Simvastatin is an example of HMG-CoA re- along with a proton pump inhibitor (PPI) and/or ductase inhibitors (statins) that block the rate- bismuth-containing compound to eradicate this limiting step in cholesterol synthesis. This class bacterium. Answers: 1-4 11 Pharmacology

Antacids are used to treat GI upset associated which would temporarily relieve the symptoms with heartburn (dyspepsia) and acid indigestion. of adrenal insufficiency. Antacids work by neutralizing excess stomach Oxygen is useful in every emergency, with the acid (increasing pH). Antacids work relatively exception of hyperventilation. Oxygen would quickly and are meant for short-term relief. The have been helpful to get the patient through the differences in how fast antacids work and for how symptoms of the adrenal crisis; however, it would long they provide relief depends on what ingredi- not have been sufficient to alleviate the crisis. ents (aluminum, calcium, magnesium, sodium Epinephrine is the drug of choice to treat ana- bicarbonate) they contain. phylaxis in the dental chair. Signs of anaphy- Antimuscarinic drugs (i.e., atropine) are re- laxis would include flushing and itching of the sponsible for decreased gastric secretions and de- skin, feelings of anxiety, rapid pulse, and diffi- creased GI motility. Because of these effects, in culty breathing. Epinephrine is typically given addition to being used to treat heartburn and acid intramuscularly and has a very rapid onset and indigestion, they may also be used to treat diar- short duration of action. Epinephrine can also rhea, to prevent secretions (gastric and salivary) be used to treat an asthma attack that does not preoperatively, and to prevent/reduce motion initially respond to a beta-2 agonist such as al- sickness. buterol. Proton pump inhibitors (i.e., omeprazole) are Nitroglycerine is used in the case of acute also used to treat acid-related GI conditions and angina attack or myocardial infarction in the work by blocking the H+/K+ ATPase pumps on dental office. It is given sublingually, as a tablet parietal cells that are directly responsible for se- or spray, and has a rapid onset. When a patient creting H+ ions into the stomach. By targeting experiences signs of angina, one dose should be the terminal step in acid secretion, PPIs are more given. If relief does not occur, second and third potent than H2 blockers at reducing acid secre- doses can be given at 5-minute intervals. It is also tion. useful to have an automated external defibrilla- tor (AED) available in the office in the case of 3. The correct answer is E. Crohn’s disease is a cardiac arrest. chronic inflammatory condition that can affect Sugar, either in an oral form or injectable the entire digestive system. There is no cure for form, is indicated for the management of hy- this disease; its symptoms can be minimized and poglycemia. Conscious patients will be able to sent into remission with anti-inflammatory med- take an oral source of sugar, such as fruit juice, ications and immune modulators. On the basis whereas unconscious patients will require IM of the symptoms this patient has experienced as glucagon or IV dextrose. When treating diabetic a side effect of taking a drug to treat her Crohn’s patients, it is important to ask about their most disease, it is likely that she had been taking a recent HBA1c and daily blood glucose levels and systemic corticosteroid to reduce inflammation. to be aware of their risk of hypoglycemia, as often Long-term use of corticosteroids can suppress times patients will take their medications for di- the ability of the adrenal glands to produce nat- abetes and then skip a meal prior to their dental ural cortisol and as a result when these drugs visits. are discontinued abruptly, the body can display symptoms of adrenal insufficiency (adrenal cri- 4. The correct answer is C. Beta-blockers inhibit sis), like the patient above did during the dental endogenous catecholamines (i.e., epinephrine) procedure, because of a lack of natural corti- from stimulating beta adrenergic receptors, part sol. To prevent the above scenario, gradually ta- of the sympathetic nervous system. There are two per the dose when discontinuing corticosteroids types of beta receptors, beta-1 and beta-2, located and take an extra dose prior to stressful situations mainly in the heart and the lungs, respectively. for up to 2 years after discontinuing the drug. When beta-1 receptors are blocked, the result is The best answer choice would be Hydrocor- reduced cardiac output and decreased workload tisone (2-mL dose), an artificial corticosteroid, of the heart because of negative chronotropic and inotropic effects. When beta-2 receptors are 12 Chapter 1: Pharmacology

blocked, the result is bronchoconstriction. On to intake of any other substance and to sit or the basis of the cardiac effects, beta blockers stand for at least 30 minutes after ingestion of are indicated for the treatment of hypertrophic the drug. cardiomyopathy, a condition in which the car- Drug-induced hemolytic anemia may occur

Pharmacology diac muscle is hypertrophic without any obvious when a drug causes the immune system to react cause. All of the answer choices are examples against its red blood cells. Drugs that may have of beta blockers; however, because this patient this effect include: cephalosporins and peni- also has asthma, it would be advisable to use a cillins, levodopa (used to treat Parkinson dis- cardioselective beta blocker, specific for beta-1, ease), methyldopa (an alpha-2 agonist used to which will not cause the unwanted side effect of treat hypertension), quinidine, and some anti- bronchospasm, sometimes observed when non- inflammatory drugs. selective beta blockers are taken. The only ex- Xerostomia, or dry mouth, is not a disease, ample above of a cardioselective beta blocker is rather a symptom of other medical condi- metoprolol. tions or a side effect of numerous medications. Propranolol is a nonselective beta blocker, The major classes of drugs that cause xeros- also referred to as a beta-adrenergic blocking tomia are antihistamines, antidepressants, anti- agent, beta-adrenergic antagonist, or a beta- cholinergics, antihypertensives, antipsychotics, antagonist. In general, beta blockers are used anti-Parkinson agents, diuretics, sedatives, and for hypertension, cardiac arrhythmias, chest pain anorexiants. (angina), migraine headaches, and glaucoma. Edema is defined as the retention of fluid Labetalol is a mixed alpha-1 and beta blocker. in an organ or in the body that results in By also blocking alpha-1, there is an additional swelling. Drugs that may cause edema include vasodilation effect. Because of the additional chemotherapy agents (such as rituximab that is alpha-1 blockade, it is important to be aware of also used for rheumatoid arthritis) and corticos- the risk of orthostatic hypotension when the pa- teroids (such as prednisone). tient leaves the dental chair. Labetalol is used Orthostatic hypotension (or postural hy- to treat hypertension and is specifically indi- potension) occurs when there is a sudden drop cated to treat pregnancy-induced hypertension. in blood pressure. This side effect is often seen Pregnancy-induced hypertension (HTN) is of- in patients taking drugs that have an alpha- ten associated with pre-eclampsia, a condition 1 blocking effect. Drugs with this effect in- in which there is increased protein in the urine clude those used to treat hypertension, such (proteinuria). as alpha-blocking agents (i.e., terazosin), mixed Nadolol and timolol are nonselective beta alpha- and beta-blocking agents (i.e., carvedilol, blockers. labetalol), calcium channel blockers (i.e., nifedi- pine, verapamil), and antidepressants, both tri- 5. The correct answer is D. Zoledronic acid is cyclics (i.e., amitriptyline) and monoamine an example of a bisphosphonate used to treat oxidase inhibitors (i.e., phenelzine). conditions of the bones, such as osteoporosis, osteopenia, Paget disease, or hypercalcemia of 6. The correct answer is C. The therapeutic in- malignancy. Bisphosphonates are antibone re- dex (TI) is defined as a number, LD50/ED50, sorbers that work by inhibiting the actions of the ratio comparing the lethal dose of a drug for osteoclasts. Side effects of bisphosphonates in- 50% of the population to the effective dose of clude: irritation of the esophagus (potentially a drug for 50% of the population. A drug with causing perforations), headaches, muscle and a high (or broad) index can typically be given joint pain, constipation, diarrhea, flatulence, with greater safety than one with a low (or nar- dysphagia, and risk of osteonecrosis. Choice D row) index. Digoxin, for example, has a very low is therefore the best answer. To best reduce the TI and requires close monitoring of the patient’s incidence of esophageal perforation, it is recom- blood levels to minimize the occurrence of ad- mended to take the oral drugs in the morning verse reactions. The therapeutic index is usually with a full glass of water at least 30 minutes prior calculated from data obtained from experiments Answers: 5-7 13 Pharmacology

with animals. The comparison of the LD50 and Safety index = LD1/ED99 ED is most significant when the dose-effect 50 ED99 curves from which these median doses are in- 100 ferred are parallel (see graph below). The ther- Sleep Death apeutic index can also be calculated using data that does not include lethal effects and instead applies the median dose causing minimally toxic 50 effects (TD ). The safety index is defined as 50 LD1 % Population a number, LD1/ED99, the ratio comparing the lethal dose of a drug in 1% of the population to the effective dose in 99% of the population. The 0 1 10 0.1 100 safety index indicates by what percentage of it- 0.01 1000 0.001 10000 self a dose must be exceeded to produce a lethal 0.0001 100000 Dose effect in just 1% of the population. Clinically, the safety index is more practical than the thera- peutic index, and unlike the therapeutic index, In choice E, these values were determined by the determination of the safety index does not de- calculating LD50/ED99 and LD1/ED50, of which pend on the parallelism of the dose-effect curves neither are the correct formulas for therapeutic from which the LD1 and ED99 are inferred. On index and safety index. the basis of the following calculations, choice C Answer choice F is the reverse of those in is correct. TI = LD50/ED50 = 30 mg/5 mg = 6. choice E and again, neither of the formulas used Safety index = LD1/ED99 = 20 mg/10 mg = 2. are representative of therapeutic index or safety index.

Therapeutic index = LD50/ED50 7. The correct answer is C. The first step is to deter- mine the local anesthetic concentration of 2% 100 lidocaine present in one carpule on the basis Sleep Death of one carpule containing 1.8 mL of fluid. The following formula is used: 2% = 2 g/100 mL = = × ED50 LD50 2,000 mg/100 mL 20 mg/mL; 20 mg/mL 50 1.8 mL/carpule = 36 mg/carpule. There are

% Population 36 mg/carpule in a 2% solution of lidocaine. The next step is to take into account the max- 0 imum recommended dosage (MRD) for lido- 1 10 caine, which is 2.0 to 3.2 mg/lb (or 4.5–7.0 0.1 100 0.01 1000 0.001 10000 × 0.0001 100000 mg/kg). For a 40-pound child, 40 lb 2.0 to 3.2 Dose mg/lb = 80 to 128 mg. On the basis of this range and the knowledge that there are 36 mg/carpule in a 2% lidocaine solution, the following formu- In answer choice A, these values were de- las are used: 80 mg/36 mg/carpule = 2.2 carpules termined by calculating the ED99/ED50 and and 128 mg/36 mg/carpule = 3.5 carpules. The LD50/LD1, of which neither are the correct for- high end of the acceptable range is 3.5 carpules; mulas for therapeutic index and safety index. therefore, three carpules would be an acceptable In answer choice B, these values are the re- volume to administer. verse of those in choice A and again, neither of One and two carpules would be acceptable the formulas used are representative of therapeu- amounts of anesthetic to give; however, it would tic index and safety index. not be the maximum recommended dose. Answer choice D is incorrect because the val- Four and five carpules are greater than the ac- ues are determined by reversing the formulas for ceptable range for the maximum recommended therapeutic index and safety margin. dose. 14 Chapter 1: Pharmacology

8. The correct answer is E. The American Heart easily be spread in crowded areas such as home- Association guidelines for antibiotic prophylaxis less shelters. The medications listed above are were revised in April 2007. According to the typically given together for 2 months in the case new guidelines, patients requiring prophylaxis of active infection, and after 2 months, only two

Pharmacology include those with a history of prosthetic cardiac drugs are continued. Active infection is deter- valve, history of previous infective endocarditis, mined by a positive skin test, an abnormal chest history of congenital heart disease, and history x-ray, and sputum containing TB bacteria. Treat- of cardiac transplantation resulting in cardiac ment can continue for months or years depend- valvulopathy. It is no longer recommended to ing on the patient. If a patient has TB in an inac- prescribe antibiotics for those patients with a tive state, an antibiotic, typically isoniazid, may history of mitral valve prolapse. The guidelines be given alone. Rifampin is metabolized largely also define which procedures do and do not re- in the liver by the CYP450 enzyme system, and it quire antibiotic prophylaxis. Periodontal surgery is, therefore, important to be aware of the effect it would require antibiotic prophylaxis, along with can have on the metabolism of other drugs me- any procedure that involves manipulation of tabolized by CYP450 enzymes, including oral gingival tissue, the periapical region of teeth, contraceptives, warfarin, and azole antifungals. and the perforation of the oral mucosa. Proce- One of the oral signs of TB is a chronic nonheal- dures that do not require antibiotic prophylaxis ing ulcer, most commonly on the dorsum of the include: routine anesthetic, injections through tongue, which occurs following lung infection. noninfected tissue, taking dental radiographs, The differential diagnosis of a tuberculosis ulcer placement of removable prosthodontic or or- should also include: aphthous ulcers, traumatic thodontic appliances, adjustment of orthodon- ulcers, syphilitic ulcers, and malignancy. The tic appliances, placement of orthodontic brack- diagnosis can be confirmed by the presence of ets, shedding of deciduous teeth, and bleeding acid-fast bacilli in the specimen. from trauma to the lips or the oral mucosa. The Linea alba is an example of a mucosal lesion guidelines did not change for those patients with caused by trauma, typically cheek biting. It ap- a history of prosthetic joint replacement. pears as a raised white line in the buccal mucosa, In the case that a patient has one of the four usually along the plane of occlusion. Under a conditions requiring antibiotic prophylaxis and microscope, it appears as hyperkeratosis. It is a had no allergy to penicillin, 2 g of amoxicillin variation of normal and therefore requires no would be an appropriate prescription to cover pharmacologic treatment. the patient for the three different surgical proce- Perioral vesicles are associated with the her- dures. pes simplex virus. Primary infection is predom- Four grams of amoxicillin would be more inant among children. The virus cannot be than necessary to give for antibiotic prophylaxis cured with medications and remains latent in prior to each procedure. the trigeminal ganglion. Pharmacologic treat- Patients who have one of the four condi- ment for herpes virus is aimed at treating the tions requiring antibiotic prophylaxis and who symptoms. Topical agents to treat the vesicles are also allergic to penicillin require 600 mg include 1% penciclovir cream. In addition, an- of clindamycin 1 hour prior to their surgical tiviral medications, such as acyclovir (mecha- procedures. Twelve hundred milligram of clin- nism: inhibits viral DNA synthesis), valacyclovir, damycin would be more than necessary to give and famciclovir, can be effective when given sys- for antibiotic prophylaxis prior to each proce- temically and when initiated early; however, the dure. symptoms usually resolve on their own in 10 to 14 days. 9. The correct answer is B. The patient described Hairy leukoplakia is associated with the above was taking antitubercular medications Epstein–Barr virus and occurs mostly in peo- to treat tuberculosis (TB), a bacterial infec- ple infected with HIV. Hairy leukoplakia is an tion caused by Mycobacterium tuberculosis.It opportunistic infection that presents as a non- is caused by inhalation of the bacteria and can painful white plaque along the lateral borders Answers: 8-10 15 Pharmacology

of the tongue. Hairy leukoplakia does not always to allow for normal clotting following the synthe- require treatment. When it is treated, the follow- sis of new platelets. Other applications of aspirin ing medications may be used: systemic antivirals are: (1) an analgesic for minor aches and pains, (to slow down EBV), and topical agents such (2) an antipyretic to reduce fever, and (3) an anti- as podophyllum resin solution and tretinoin inflammatory medication. Adverse effects of as- (retinoic acid). pirin include GI ulcers, stomach bleeding, pro- Lichen planus is thought to be caused by cyto- longed bleeding, tinnitus, and an increased risk toxic cell-mediated hypersensitivity and is some- of Reye syndrome (characterized by acute en- times associated with hepatitis C. Lichen planus cephalopathy and fatty liver) in children with appears as a white, lacy rash (Wickham’s striae) viral diseases. on the buccal mucosa. Certain drugs, such as Pilocarpine is a nonselective cholinergic ag- those containing arsenic, bismuth, or gold, can onist. It has a direct effect on muscarinic acetyl- cause a response in the oral mucosa that is choline receptors and can be applied topically. It indistinguishable from lichen planus. In addi- produces rapid miosis and contraction of ciliary tion, long-term use of antimalaria drugs, such muscles and is used by patients with glaucoma. It as quinidine, may produce lichen planus. No also stimulates the secretion of large amounts of treatment is required for asymptomatic lesions. saliva and sweat and is used to treat xerostomia, If symptomatic (itchiness, unpleasant appear- rather than cause xerostomia. Adverse effects of ance), topical steroid creams, antihistamines, or pilocarpine relate to its nonselective nature and systemic steroids can be used. If the question include: excessive sweating and salivation, bron- above described a patient with hepatitis C, he chospasm and increased bronchial mucous se- may have been taking the following antiviral cretion, bradycardia, hypotension, and diarrhea. medications: interferon (peginterferon) or rib- Nabumetone is an example of a nonsteroidal avirin. anti-inflammatory drug (NSAID). NSAIDs work by inhibiting the enzyme cyclooxygenase, result- 10. The correct answer is A. Amitriptyline is an ing in lower concentrations of prostaglandins, example of a tricyclic antidepressant (TCA). chemicals produced by the body that are re- TCAs work by inhibiting the reuptake of nore- sponsible for inflammation, pain, and fever. pinephrine and serotonin at nerve terminals. Nabumetone may be taken once a day for long Their application is based on the theory that term by patients with osteoarthritis and rheuma- abnormal levels of these neurotransmitters may toid arthritis. The most common side effects of relate to depression in some patients. TCAs nabumetone affect the GI system. To reduce in general have an anticholinergic effect, and the incidence of GI ulceration with chronic use therefore adverse reactions may include: xeros- of this drug, a prostaglandin analog (i.e., miso- tomia, urinary retention, constipation, increased prostol) may also be prescribed. Prostaglandin ocular pressure, and mydriasis. Other classes of analogs must be used with caution because of drugs associated with xerostomia include: an- their potential as an abortive agent or labor in- ticholinergics, anticonvulsants, antihistamines, ducer. Other drugs used to treat osteoarthri- antihypertensives, antiparkinsonians, antipsy- tis include: aspirin or acetaminophen, selective chotics, antispasmodics, appetite suppressants, COX-2 inhibitors, prednisone injections, and barbiturates, bronchodilators, decongestants, di- topical capsaicin. uretics, hypnotics, muscle relaxants, and opioid Physostigmine is an indirect acting cholin- analgesics. ergic agonist that exerts its effect by reversibly Aspirin is the most popular antiplatelet drug inhibiting cholinesterase from breaking down and is available over-the-counter. Doses range acetylcholine, thereby increasing the level and from 81 to 325 mg/day to prevent myocardial duration of action of acetylcholine in the neu- infarctions and occlusive strokes in high-risk romuscular junction. It has several clinical people. Aspirin works by irreversibly inhibiting applications, including glaucoma, urinary reten- platelet aggregation. Because of its irreversible tion, reversal of neuromuscular junction block- nature, it needs to be discontinued for 5 to 7 days age, and myasthenia gravis. Researchers have 16 Chapter 1: Pharmacology

also found some potential for its use in treating is Clostridium difficile-associated diarrhea (the depression and Alzheimer disease. Adverse reac- most frequent cause of pseudomembranous col- tions include the potential to cause SLUDGE, itis, a potentially lethal condition). This side ef- an acronym for a cholinergic syndrome with the fect can occur with other antibiotics as well but

Pharmacology following characteristics: salivation, lacrimation, most commonly occurs with clindamycin use. urination, diarrhea, GI cramping, and emesis. Because clindamycin concentrates in bone, it is often used in dentistry to treat periapical bony 11. The correct answer is B. Disulfiram is used in infections. the treatment of chronic alcoholism by caus- Doxycycline is a member of the tetracycline ing an acute sensitivity to alcohol. The un- class of antibiotics, another group of antibiotics derlying mechanism is the accumulation of that works by inhibition of bacterial protein acetaldehyde in the blood because of inhibition synthesis. Significant adverse effects of doxycy- of hepatic aldehyde dehydrogenase by disulfi- cline include photosensitivity and discoloration ram. Acetaldehyde is one of the major causes of teeth and inhibition of bone growth when of the symptoms of a “hangover.” Within min- taken during development in infants and chil- utes of its accumulation, the individual can dren. become acutely ill with a severe “hangover,” Rifampin is antituberculosis drug. The most experiencing generalized flushing, nausea, vom- serious adverse effect of rifampin is related to its iting, headache, and more serious symptoms, hepatotoxicity. such as accelerated heart rate and circulatory collapse. There are several other drugs that can 12. The correct answer is C. Analgesic agents, produce this disulfiram-like effect including: such as narcotics, are commonly prescribed by phenacetin, antibiotics, such as nitrofurantoin, dentists to treat emergencies and preoperative cephalosporins, and metronidazole, anti-angina and postoperative pain. Dentists may prescribe medication (nitrates), and first-generation sul- medications and controlled substances only for fonylureas, such as chlorpropamide and tolbu- dental-related conditions. The ADA defines den- tamide. Metronidazole works by inhibiting tistry as follows: “. . . the evaluation, diagnosis, DNA bacterial synthesis, exclusively against prevention and/or treatment (nonsurgical, surgi- strict anaerobic infections. cal or related procedures) of diseases, disorders Clarithromycin is in the class of macrolide and/or conditions of the oral cavity, maxillofacial antibiotics that exert their effects by inhibit- area and/or the adjacent and associated struc- ing bacterial protein synthesis. Clarithromycin tures and their impact on the human body; pro- should be prescribed with caution to patients tak- vided by a dentist, within the scope of his/her ing other drugs that interact with the CYP3A4 education, training and experience, in accor- enzyme, found largely in the liver. Because dance with the ethics of the profession and ap- clarithromycin is a CYP3A4 substrate, it may plicable law.” Any medication that falls within cause increased serum levels of other drugs the scope of this definition can be prescribed by that are CYP3A4 substrates, including several a dentist. Under no circumstances may a dentist benzodiazepines, calcium channel blockers, im- prescribe anything outside the course of his/her munosuppressants, statins, SSRIs, TCAs, azole practice of dentistry. To do so is a serious viola- antifungals, and others such as mirtazapine, tion of the laws and regulations, and a dentist nefazodone, venlafaxine, terfenadine, cisapride, may be disciplined by the Board. A dentist must theophylline, and warfarin. It is also possible for avoid self-prescribing, and any medications pre- other drugs, CYP3A4 inhibitors and CYP3A4 in- scribed for office staff or family members must ducers, to cause increased or decreased levels of be solely for dental-related conditions and must clarithromycin, resulting in adverse reactions or be clearly and specifically documented in the ineffectiveness, respectively. patient’s file. Clindamycin is an antibiotic that works by Oral contraceptives, antihypertensives, an- inhibiting bacterial protein synthesis. The most tidiabetic agents, amphetamines, and antiasth- severe common adverse effect of clindamycin matics do not fall within the scope of dentistry Answers: 11-14 17 Pharmacology

and therefore should not be prescribed by a den- with postherpetic neuralgia. Capsaicin works by tal practitioner. depleting substance P, a neurotransmitters that signals pain. Side effects of capsaicin include a 13. The correct answer is D. Ginkgo balboa is burning sensation in the area it is applied, red- mainly used as a memory and concentration en- ness, and skin blisters. hancer and to treat circulatory disorders. Like many alternative medications, there is much 14. The correct answer is D. Celecoxib is an ex- conflicting research on its effectiveness. Some ample of a selective COX-2 inhibitor. This laboratory studies have shown that ginkgo balboa class of drugs works by blocking the facultative improves blood circulation by dilating blood ves- COX-2 isoenzyme but sparing the constitutive sels and blocking the effects of platelet-activating COX-1 isoenzyme. Both cyclooxygenase en- factor. Because of its potential effect to im- zymes are responsible for the conversion of prove blood flow, ginkgo balboa is often taken arachidonic acid to prostaglandins. The main by patients with intermittent claudication (pain distinction is that prostaglandins whose pro- caused by inadequate blood flow to the legs). Be- duction requires the COX-1 isoenzyme are re- cause ginkgo balboa decreases the aggregation sponsible for protection of the gastric mucosa, ability of platelets, there is some concern that whereas the prostaglandins whose synthesis in- its use, particularly when combined with other volves COX-2 are responsible for inflammation anticoagulants, such as aspirin, puts patients at and pain. Traditional NSAIDs work by inhibit- increased risk of bleeding. Of special concern is ing both COX-1 and COX-2, thereby decreas- the increased risk of intracranial hemorrhage. ing inflammation and pain and decreasing cy- Ginseng has multiple traditional uses includ- toprotection of the GI tract. Selective COX-2 ing: improving recovery from illness and well- inhibitors work only to decrease inflammation being, treating erectile dysfunction, hepatitis C, and pain. Chronic use of traditional NSAIDs and symptoms associated with menopause, low- (6–12 months) in the treatment of osteoarthri- ering blood sugar, and controlling blood pres- tis and rheumatoid arthritis is associated with sure. Given the diversity of its uses, it may be approximately 1% to 3% yearly incidence of taken by many patients who believe in the use of clinically significant GI ulcers. With chronic complementary medicine. The most common dosing, celecoxib reduces the incidence of GI side effects of ginseng are headaches and GI up- perforations, ulcerations, and bleeds by approx- set. An important precaution in the use of gin- imately 50% to 60% compared with conven- seng should be given to patients with diabetes tional NSAIDs. An area of concern when tak- because of its potential to lower blood sugar. ing selective COX-2 inhibitors is the implication Echinacea is traditionally used to treat or pre- that COX-1 is working unopposed, which means vent colds or the flu by boosting the immune it is continually contributing to the synthesis system. Side effects include allergic reactions or of thromboxane, involved in platelet aggrega- GI upset. tion, thereby increasing the risk of thrombotic St. John’s Wort is one of the most commonly events. used herbal supplements in the United States. Traditional NSAIDs and celecoxib are It is used to help mild-to-moderate depression. equally effective as analgesics for acute postsur- It can enhance the effects of anesthesia, and be- gical pain than celecoxib since both drugs in- cause St. John’s Wort induces CYP3A4 in the hibit the desired target—COX-2 prostaglandins. gut and liver, it may decrease the metabolism of Some sources states that traditional NSAIDs other CYP3A4 substrates, which includes at least are actually more effective and are comparable half of marketed medications, including some to narcotics as an analgesic without the CNS- anesthetics. mediated side effects. Capsaicin is the active ingredient in chili pep- Traditional NSAIDs and celecoxib also pro- pers. It is usually used in a topical cream form duce equal anti-inflammatory responses in os- to relieve pain and itching. For instance, a den- teoarthritis patients, although celecoxib may tal practitioner may recommend it to patients be preferred because of the tendency for 18 Chapter 1: Pharmacology

osteoarthritis patients to use these drugs for long to children with chicken pox or influenza B in- term. fection. Reye syndrome may be misdiagnosed as Traditional NSAIDs and celecoxib have sim- a drug overdose and blood work would be help- ilar effects on renal function as measured by ful to confirm the diagnosis. The treatment for

Pharmacology glomerular filtration rate, creatinine clearance, Reye syndrome should include: hypertonic IV and urinary and serum sodium and potassium glucose solutions and mechanical ventilation. values. Selective COX-2 inhibition may tran- Codeine is an example of a narcotic used as siently increase sodium retention. an analgesic agent and sometimes for its anti- Traditional NSAIDs and celecoxib have sim- tussive and antidiarrheal properties. Narcotics ilar antipyretic qualities. are centrally acting drugs that work by bind- ing to opiate receptors (opiate agonist). In the 15. The correct answer is B. The most likely case of codeine, which is a prodrug, its active cause of these symptoms is an overdose of ac- metabolites, including morphine, bind to the etaminophen (APAP). APAP is used as an anal- opiate receptors. Codeine is converted to mor- gesic and antipyretic. Its exact mechanism is phine via the CYP2D6 enzyme, and because unknown. APAP is metabolized in the liver of its interaction with the CYP450 enzyme sys- by CYP2E1 to N-acetyl-benzoquinoneimine tem, its serum levels are susceptible to changes (NAPQI), a toxic by-product, which is later in- by interactions with other drugs (i.e., CYP2d6 activated by glutathione. In the case of an over- inducers). A potentially serious side effect of dose, glutathione stores are depleted and NAPQI codeine toxicity is respiratory depression, and in damages hepatocytes. Signs of liver toxicity in- the case of a codeine overdose, the reversal agent, clude hepatomegaly, jaundice, GI bleeding, and naloxone, should be given. Codeine, like other encephalopathy. Death may occur because of narcotics, can cause a physical or psychological cerebral edema, sepsis, or multiorgan failure. dependence as well as tolerance to the drug to Treatment may include the ingestion of acti- develop. vated charcoal or the antidote, N-acetylcysteine. Celecoxib is a variation of an NSAID and These are best given within 12 hours before the is classified as a selective COX-2 inhibitor. It liver damage becomes irreversible. If treatment is used in patients with gastric ulcers when the is not initiated quickly enough, liver failure may inhibition of COX-1 should be avoided. It has ensue requiring a liver transplant. It is important similar anti-inflammatory effects as traditional for parents to pay close attention to the dosages of NSAIDs. According to some research, when medications that they provide to their children. celecoxib is used for long term, it is capable of For children younger than 12 years and/or less producing cardiotoxicity. than 50 kg in weight, the maximum daily dosage Ibuprofen is a traditional NSAID that works of acetaminophen is 80 mg/kg (not to exceed a by nonselectively inhibiting COX-1 and COX- cumulative daily dose of 2.6 g). Weight-based 2. Ibuprofen has analgesic, antipyretic, and anti- oral dosing for children is 10 to 15 mg/kg every inflammatory effects. The major concern with 4 to 6 hours with a maximum of five doses per the overuse of ibuprofen is the increased risk of 24-hour period. GI ulcerations and GI bleeding and nephrotox- Aspirin is an irreversible inhibitor of COX-1. icity. It serves as an analgesic, antipyretic and, anti- inflammatory agent and inhibits platelet aggre- 16. The correct answer is B. A drug with minimal gation. Adverse effects of aspirin include: GI ir- protein binding is least likely to contribute to ritation, tinnitus, vertigo, hypersensitivity, and a significant adverse drug interaction. On the bleeding complications. An additional adverse other hand, drugs that are highly protein bound effect that many parents may not be aware of is can be displaced by other drugs that are highly its ability to serve as a trigger for Reye syndrome, protein bound when taken simultaneously. For a highly lethal disease. To avoid Reye syndrome, instance, warfarin is typically 99% plasma pro- aspirin (and its derivatives) should not be given tein bound, meaning that only 1% of the drug Answers: 15-18 19 Pharmacology

is free and therefore only 1% is active. Drugs zodiazepines, azole antifungals, erythromycin like aspirin or other salicylates will compete for and clarithromycin, antiretrovirals, statins, and the protein receptor sites that warfarin has a ten- SSRIs. dency to bind to. This competition will cause St. John’s Wortdoes not block active secretion displacement of warfarin, resulting in greater of the oral contraceptive, does not affect stomach availability of the drug and therefore an in- acidity, does not block estrogen and progesterone creased effect, which in warfarin’s case can result receptors, and does not decrease serotonin levels in increased bleeding. in the brain. The exact mechanism by which St. Therapeutic index (TI) represents the margin John’s Wort functions to decrease depression is of safety of a drug. Drugs with a low (or narrow) unclear. However, it is believed that one way TI have a small margin of safety. TI is equivalent the herbal product is thought to exert its effect is to LD50/ED50, or the lethal dose in 50% of pa- by inhibiting the reuptake of serotonin, thereby tients compared with the effective dose in 50% increasing levels of serotonin in the brain. of patients. If a drug has a low TI, such as a TI of 3, this means that if only three times as much as 18. The correct answer is E. If Drugs X and Z both the effective dose is taken, there is a 50% chance have a tendency to be highly bound to plasma for a lethal effect. proteins, then when Drug Z is introduced there Drugs that are metabolized by the CYP450 would be an increased competition for protein enzyme system in the liver are subject to changes binding sites. This would cause displacement in metabolism when taken concomitantly with of Drug X from plasma proteins, and the result other drugs that modify the activity of the would be increased free Drug X in the plasma, CYP450 system. Many drugs may increase or resulting in increased free blood levels of Drug decrease the activity of CYP450 by acting as an X. An example of this interaction occurs when inducer or inhibitor, respectively, and resulting aspirin is introduced in a patient who is currently in depletion or accumulation of the drug in ques- taking warfarin. tion, respectively. Drugs that induce liver microsomal enzymes Drugs that are enzyme inducers, as in those increase their activity. If Drug Z was an in- associated with the CYP450 system, have the po- ducer, and assuming that Drug X was a substrate tential to speed up metabolism of a drug, thereby of liver microsomal enzymes, then increasing depleting the presence of the drug. the activity of the enzymes would speed up the The volume of distribution is used to describe metabolism of Drug X. If Drug X was metab- the quantity of medication in the plasma and olized more rapidly, decreased blood levels of the rest of the body. Drugs with a low volume of Drug X would be expected. distribution have a higher concentration in the If Drugs X and Z formed a chelation product plasma and the potential for greater activity. in the intestine, this would result in decreased free levels of Drug X. For example, when a pa- 17. The correct answer is B. St. John’s Wort is a tient taking tetracycline also takes an antacid (or CYP3A4 inducer. As an inducer, St. John’s Wort ingests any food product containing divalent and causes the CYP3A4 enzyme (part of the CYP450 trivalent cations), the components of the antacid enzyme system) to become more active. Drugs will form a chelation product with tetracycline. that are substrates of CYP3A4 are therefore me- When tetracycline is bound to another molecule tabolized more rapidly, and their blood levels through chelation, this translates to decreased are reduced. In the case of oral contraceptives, free levels of tetracycline, resulting in decreased if inadequate amounts of hormone are present, absorption of tetracycline and decreased effec- the overall effectiveness of the oral contraceptive tiveness. may be reduced. Patients taking St. John’s Wort If Drug Z increased the glomerular filtration may require increased dosages of other drugs. rate, and it was assumed that Drug X was ex- Other examples of CYP3A4 substrates include creted via the kidneys, this would result in more calcium channel blockers, cyclosporine, ben- rapid excretion of Drug X. If the elimination of 20 Chapter 1: Pharmacology

Drug X is increased, then the free blood levels Glipizide belongs to the class of sulfonylurea of Drug X are decreased. drugs used by patients with type II diabetes. Sul- If Drug Z has a positive inotropic and fonylureas work by increasing the amount of chronotropic effect, this would have little effect insulin release from beta cells of the islets of

Pharmacology on the blood levels of Drug X. Positive inotropic Langerhans of the pancreas. These drugs are in- drugs (examples include digoxin, epinephrine, effective if there is no insulin available, as in and amrinone) increase the strength of muscle type I diabetes. Some sulfonylureas are metabo- contraction of the heart. Positive chronotropic lized by the CYP450 enzyme system in the liver, drugs (examples include atropine, dopamine, and their metabolism, like codeine, can be in- and epinephrine) increase heart rate. While fluenced by inducers and inhibitors of this sys- both of these types of agents may increase tem. Some sulfonylureas are bound to plasma blood flow through the body, they have no proteins and can be displaced by other drugs, effect on the concentration of drugs in the resulting in increased free blood levels of sul- blood. fonylureas. Glipizide has no effect on codeine. Lisinopril inhibits angiotensin-converting 19. The correct answer is D. Codeine is a nar- enzyme (ACE) that catalyzes the conversion cotic with analgesic, antidiarrheal, and antitus- of angiotensin I to angiotensin II. Effects in- sive properties. It is classified as a Schedule III clude vasodilation, decreased aldosterone secre- drug and has the potential for drug abuse and tion, decreased water and sodium retention, and dependence. Codeine is readily absorbed from increased bradykinin. Drugs that interact with the GI tract, is not bound to plasma proteins, lisinopril include diuretics and NSAIDs, result- is eliminated primarily through the kidneys, ing in hypotension and renal damage, respec- reaches its peak analgesic effect within 2 hours, tively. Lisinopril has no effect on codeine. An- and persists between 4 and 6 hours. Codeine is giotensin II receptor antagonists (“sartan” drugs) considered a prodrug. In the liver, codeine is are an improvement on this class of drugs; by di- converted to the active compounds morphine rectly blocking angiotensin II, they eliminate the and codeine-6-glucuronide, which exert their unwanted bradykinin-induced side effects asso- effects by binding to opiate receptors. This re- ciated with ACE inhibitors. action is catalyzed by the cytochrome P450 Lithium is an antipsychotic commonly used enzyme, CYP2D6. Drugs that inhibit CYP2D6 by patients with bipolar disorder. Lithium has a prevent this conversion to active compounds narrow therapeutic index and can cause serious from occurring, thereby reducing the overall renal and CNS toxicity. Metronidazole inhibits effectiveness of codeine. Examples of strong lithium clearance from the kidneys, resulting in CYP2D6 inhibitors include selective-serotonin high lithium blood levels. Renal toxicity can oc- reuptake inhibitors (SSRIs), such as fluoxetine cur when diuretics and lithium are taken con- (choice D) and paroxetine, other antidepres- currently because of excessive sodium depletion. sants, such as bupropion, terbinafine, and quini- Lithium has no effect on codeine. dine. Terazosin is an alpha-adrenergic blocker. By 20. The correct answer is E. Montelukast (singu- blocking the alpha-1 receptor, terazosin causes lair) is a leukotriene receptor antagonist used vasodilation, thereby lowering blood pressure, for the prophylactic and chronic treatment of and smooth muscle relaxation, particularly in asthma and seasonal allergies. Leukotrienes are the prostate. Terazosin is indicated for hyperten- released by inflammatory cells and cause the sion and benign prostate hyperplasia. The most following symptoms in asthma: hypersecretion important drug interactions to be aware of in a of mucus, decreased mucus transport and mu- patient taking terazosin is if the patient is tak- cosal accumulation, bronchoconstriction and ing additional antihypertensives or vasodilators airway narrowing, and infiltration of inflamma- as this will increase risk of hypotension and or- tory cells in the airway wall. These symptoms thostatic hypotension. Terazosinhas no effect on are decreased by drugs that competitively inhibit codeine. the binding of leukotrienes to their receptors. Answers: 19-24 21 Pharmacology

Montelukast is not useful for the treatment of choice would therefore be insulin. Insulin is acute asthma attacks. taken subcutaneously by patients type I diabetes Albuterol is an example of a short-acting beta- who lack endogenous insulin. Other antidia- 2 adrenergic agonist, which causes smooth mus- betic drugs that are effective only in patients with cle relaxation resulting in bronchodilation. It type II diabetes include: sulfonylureas (increases can be used for acute asthma attacks. Albuterol insulin release), biguanides, such as metformin is most often given in the inhaled form using a and glitazones (increase sensitivity to insulin), nebulizer. and alpha-glucosidase inhibitors (slow digestion Zileuton is an example of a lipoxygenase of starch). pathway inhibitor. This drug blocks leuko- triene synthesis by competitively inhibiting 22. The correct answer is D. The symptoms that 5-lipooxygenase, an enzyme that catalyzes the patient describes are representative of an al- formation of leukotrienes from arachidonic acid lergic reaction to a drug, in this case, peni- via oxygenation. Zileuton is not useful for the cillin. Clindamycin is a lincosamide antibiotic, treatment of acute asthma attacks. Zileuton is typically used to treat anaerobic infections. In a weak inhibitor of CYP1A2 and therefore has patients with hypersensitivity to penicillins, clin- significant drug interactions with CYP1A2 sub- damycin may be used to treat infections caused strates such as warfarin and theophylline. Zileu- by susceptible aerobic bacteria as well. It is also ton also causes elevations of liver enzymes (AST, used to treat bone and joint infections, partic- ALT). ularly those caused by Staphylococcus aureus. Ipratropium is an anticholinergic agent. By Amoxicillin would be an inappropriate alterna- blocking muscarinic receptors, this drug inhibits tive as it is from the same family of antibiotics as bronchoconstriction and mucus secretion. It penicillin. does not diffuse through the blood, preventing any potential systemic side effects, including car- 23. The correct answer is B. Naloxone is the drug diac effects and is therefore safe to use in pa- of choice for the treatment of opioid overdose. tients with heart disease. It is not as powerful as Naloxone is a ␮-opioid receptor competitive an- the beta-2 adrenergic agonists but can be used tagonist. Naloxone has a high affinity for the ␮- along with a beta-2 agonist in the case of acute opioid receptor in the central nervous system asthma attacks. It is sometimes associated with and is capable of competitively displacing opi- dry mouth. oids from the receptor. Flumazenil is used to Triamcinolone is an example of a glucocor- treat benzodiazepine overdose. It reverses the ticoid. Glucocorticoids are effective in treating effects of benzodiazepines by competitively in- asthma because of their anti-inflammatory ef- hibiting the GABAA receptor. Clonidine treats ␣ fects. Inhaled forms of glucocorticoids eliminate high blood pressure by stimulating the 2 recep- systemic side effects, but occasional reports of lo- tors in the brain, which results in a decrease in cal candidiasis do occur. sympathetic tone. Clonidine has also been used Benzonatate is an antitussive agent that works to ease withdrawal symptoms associated with the by depressing the medullary cough center and long-term use of substances, such as narcotics, exerting an anesthetic action on the respiratory alcohol, and nicotine, but not for acute reversal mucosa. It is unrelated to narcotic antitussive of narcotic overdose. It is mainly used to address agents such as codeine. the sympathetic nervous system response to opi- ate withdrawal, like hypertension and tachycar- 21. The correct answer is C. On the basis of the dia, in the initial days of withdrawals. symptoms described and the laboratory values provided, this patient is suffering from diabetes 24. The correct answer is C. Nystatin is a polyene and malnutrition. The patient is not taking antifungal effective against molds and yeasts the appropriate medications to treat these con- including the Candida spp. The drug has mini- ditions and therefore has higher than normal mal mucosal absorption and is thus used as a top- HbA1c and glucose values. The best answer ical treatment for oral candida. The other answer 22 Chapter 1: Pharmacology

choices are ineffective against yeast organisms. are 4 × 4 hours, which is a total of 16 hours. Each Amphotericin B is also a polyene antifungal but successive half-life brings the level of medication is used intravenously to treat systemic fungal in- A closer to 100%. fections.

Pharmacology 30. The correct answer is D. Tetracaine is a long- 25. The correct answer is B. Allergic reactions to acting ester local anesthetics. The rest of the op- local anesthetics typically involve the ester anes- tions are all amides. thetics. The sensitivity is usually because of a reaction against the preservative found in es- 31. The correct answer is C. Propranolol is a non- ter local anesthetics called para-aminobenzoic selective beta blocker that is contraindicated in acid (PABA). Among the answer choices, only patients with COPD or asthma. It can cause propoxycaine is an ester anesthetic. The others bronchoconstriction by blocking the beta 2 re- are amide local anesthetics. ceptors. Acebutolol, atenolol, esmolol, metopro- lol, and betaxolol are all beta 1 selective blockers 26. The correct answer is E. Warfarin interferes that are safer to use for this patient. with synthesis of vitamin K-dependent coagu- lation factors (II, VII, IX, X). This could lead to 32. The correct answer is E. The loading dose (LD) increased clotting time for both extrinsic (PT) is a function of the volume of distribution (Vd) and intrinsic (PTT) coagulation pathways. Vita- and the desired drug level in the plasma. In this mins A, B, C, and D are not directly involved in case, LD = Vd × Cp = 50 L × 5 mg/L = the coagulation cascade. 250 mg.

27. The correct answer is A. Omeprazole is an ir- 33. The correct answer is A. Hydrochlorothiazide reversible inhibitor of H+/K+ ATPase of the is a thiazide diuretic that can lead to increase in parietal cells used to treat peptic ulcer dis- urination. The rest of the options are all antihy- ease. Cimetidine, famotidine, and ranitidine are pertensives that work through different mecha- H2-receptor antagonists that block histamine- nisms. Diltiazem is a calcium channel blocker. induced acid secretion. Ketoconazole is an an- Enalapril is an ACE inhibitor. Methyldopa is tifungal medication. a centrally acting alpha receptor agonist. Ter- azosin is a peripherally acting alpha receptor 28. The correct answer is B. Oxazepam is a blocking agent. benzodiazepine that does not require phase I metabolism by the liver. Therefore, it can be 34. The correct answer is D. Flumenazil is a benzo- useful in patients with liver failure. Phase I diazepine antagonist used to treat in the case of reactions occur via oxidation, reduction, hy- overdose. Naloxone and naltrexone are opioid drolysis cyclization, or decyclization reactions receptor antagonists. Protamine is an antidote to either inactivate or activate drugs. Phase for heparin overdose. Insulin is used to treat hy- II reactions involve methylation, syphation, perglycemia. acetylation, and glucuronidation. The smooth endoplasmic reticulum of the liver cell in 35. The correct answer is A. Paroxetine is a selec- the principal location of drug metabolism. Lo- tive serotonin reuptake inhibitor that is used razepam and temazepam are the two other ben- commonly in the management of depressive zodiazepines whose levels are not significantly disorders. It is preferred over other antide- affected by the liver. Alprazolam, midazolam, pressants because of less severe side effects. triazolam, and diazepam all require phase I Common side effects include GI upset, de- metabolism by the liver. creased libido, and sexual dysfunction. Taking SSRI in combination with MAOi can increase 29. The correct answer is D. It takes one half-life to the risk of having life-threatening serotonin syn- get to 50% of steady state, two to reach 75%, and drome (autonomic instability, somatic, and cog- four to reach 93.75%. Therefore, four half-lives nitive symptoms). Answers: 25-45 23 Pharmacology

36. The correct answer is C. Piperacillin along with clude cough, hyperkalemia, rash, taste changes, carbenicillin and ticarcillin are beta-lactamase orthostatic hypotension, and liver toxicity. ACE resistant antibiotics. They have extended spec- inhibitors are contraindicated during pregnancy trum against many Gram-negative bacilli, in- because of their potential for teratogenicity. cluding Pseudomonas aeruginosa. The remain- ing antibiotics are all penicillin or its derivatives 41. The correct answer is B. TCAs (e.g., amitripty- that do not have antipseudomonal effects. line, imipramine, nortriptyline, desipramine, trimipramine, doxepin, and clomipramine) can 37. The correct answer is B: Heparin can bind and inhibit reuptake of norepinephrine and sero- activate antithrombin III, resulting in the inac- tonin into presynaptic nerve terminals. Besides tivating thrombin and factor Xa. This leads to usage in depression, TCAs can be used in the increase in PTT, a marker for the heparin’s an- treatment of neuropathy and panic disorders. Its ticoagulant effect. It is not related to synthesis of side effects include antimuscarinic, antiadrener- vitamin K coagulation factors, unlike warfarin. gic, and antihistamine effects such as dry mouth, Clinical applications of heparin include pul- orthostatic hypotension, and sedation. Serious monary embolism, ischemic myocardial infarc- adverse effects include coma, convulsion, and tion, and deep venous thrombosis. Overdose of cardiotoxicity (arrhythmias). heparin can be reversed with protamine sulfate. 42. The correct answer is E. Valproic acid is an an- 38. The correct answer is C. Warfarin has been used ticonvulsant that blocks sodium channels and as a rat poison. However, it also has clinical ap- increases GABA concentration. It is also used plication in humans for long-term anticoagula- for rapid cycling and mixed affective disorders as tion. It acts by inhibiting the synthesis of vitamin mood stabilizer. Side effects include hepatotox- K-dependent clotting factors (II, VII, IX, X), as icity, hemorrhagic pancreatitis, thrombocytope- well as proteins C and S. This affects the extrin- nia, and neural tube defects. sic coagulation pathway; thereby, one can fol- low the PT or INR levels for its clinical effect. 43. The correct answer is A. Sulfonylureas include Warfarin is known to be teratogenic; therefore, a glipizide, glyburide, tolbutamide, and chlor- pregnancy test should be performed on patients propamide. They stimulate insulin release from with childbearing potential. Vitamin K is used beta cells of the pancreas and binding of in- as a treatment for warfarin overdose. sulin to target tissues. They also inhibit release of glucagon. Metformin is a biguanides that can 39. The correct answer is C. Prazosin, doxazosin, decrease hepatic gluconeogenesis. Lactic acido- and terazosin are competitive alpha-1 receptor sis is a concern for metformin overdose. Both antagonists that are used to lower peripheral vas- types of medications are used commonly in the cular resistance. They can be used to control treatment of type II diabetes mellitus. hypertension and urinary retention. Side effects of these medications commonly include postu- 44. The correct answer is D. Esomeprazole (nex- + + ral hypotension, headache, and dizziness. ium) is an H /K ATPase inhibitor that is used in the treatment of GERD or peptic ul- 40. The correct answer is D. Patient here presents cer disease. The rest are all antifungal that work with angioedema, a side effect of ACE in- through various different mechanisms. hibitors such as captopril, enalapril, and lisino- pril. ACE inhibitors decrease the conversion of 45. The correct answer is A. Acetaminophen is used angiotensin I into angiotensin II, thereby pre- as an analgesic and antipyretic. Its mechanism is venting vasoconstriction and resultant increase not fully known, possibly related to inhibition of in aldosterone. It is commonly used in the treat- cyclooxygenases in the CNS. Hepatotoxicity is ment of hypertension, especially in the context a great concern with the use of acetaminophen of other comorbidities such as diabetes melli- in context of overdosing or patients with hepatic tus. Other adverse effects of ACE inhibitors in- failure. N-acetylcysteine allows an increase in 24 Chapter 1: Pharmacology

supply of intracellular glutathione via elevated 51. The correct answer is C. Degradation of drug intracellular cysteine, thereby decreasing the for- prior to absorption decreases bioavailability. mation toxic metabolites. Bioavailability is a measure of how much drug reaches the circulatory system and is available

Pharmacology 46. The correct answer is C. Nonselective NSAIDs at the site of action. Factors influencing the are commonly used for inflammation, analge- bioavailability of a drug include: route of admin- sia, and antipyrexia. They can inhibit both cox-1 istration (IV > oral), degradation of drug prior to and cox-2 cyclooxygenases (aspirin irreversibly, absorption (decreases bioavailability), efficient others reversibly). This can lead to a decrease gastrointestinal absorption (increases bioavail- in prostaglandin synthesis. Common side effects ability), drug solubility (hydrophobic better ab- include dyspepsia, GI ulcers, and renal failure. sorbed than hydrophilic), hepatic first pass effect Aspirin is also associated with Reye syndrome. (decreases bioavailability), and drug chemistry.

47. The correct answer is C. There are two gen- 52. The correct answer is A. Volume of distribution erations of H1 antihistamine (first generation: (Vd) is defined as total amount of drug in the diphenhydramine, dimenhydrinate; second gen- body/plasma drug concentration = 600 mg ≈ eration: fexofenadine, loratadine). They are 100 mg/L = 6L. commonly used for allergies. Second-generation medications are less sedating than the ones 53. The correct answer is C. ACE inhibitors, such from the first-generation. Toxicities include an- as captopril, enalapril, and lisinopril, block ticholinergic and antiadrenergic effects. angiotensin-converting enzyme, resulting in the decrease in conversion of angiotensin I into an- 48. The correct answer is A. Glucocorticoids (e.g., giotensin II, hence a potent vasoconstrictor. Ad- prednisone, triamcinolone, dexamethasone, and verse effects of ACE inhibitors include cough, hydrocortisone) are used in many different clin- angioedema, proteinuria, hyperkalemia, taste ical situations, including inflammation, im- changes, orthostatic hypotension, fetal toxicity, munosuppression, Addison disease, and asthma. and rash. They function by binding to intracellular cyto- plasmic receptor, phospholipase, thereby indi- 54. The correct answer is B. Clearance is defined rectly inhibiting production of leukotrienes and as rate of elimination/plasma drug concentra- prostaglandins. tion = 3 mg/min/150 mg/L = 0.02 L/min.

49. The correct answer is B. Opioids (e.g., mor- 55. The correct answer isE. Penicillin is a bacte- phine, heroin, codeine, and fentanyl) interact ricidal antibiotic (not bacteriostatic) that works with opioid receptor in the CNS and GI. They by binding to penicillin-binding protein and are used to treat for pain, cough, and diarrhea. blocking transpeptidase from crosslinking pep- Side effects include respiratory depression, con- tidylglycine in bacterial cell wall. Not all stipation, miosis, and CNS depression. Opioid penicillins are effective against Pseudomonas use can lead to abuse and dependence. Nalox- aeruginosa. Antipseudomonal penicillins in- one is an opioid receptor antagonist. clude piperacillin, carbenicillin, and ticarcillin. Amoxicillin has an extended spectrum that 50. The correct answer is D. Misoprostol is a is able to cover E. coli, Haemophilus in- synthetic prostaglandin E1 analog that can be fluenza, Proteus mirabilis, Listeria monocyto- used for abortion, labor induction, prevention genes, and Salmonella. By adding clavulanic acid of NSAID-induced gastric ulcers, and mainte- or tazobactam to penicillins, the antibiotics are nance of PDA in certain cardiac congenital de- rendered beta-lactamase resistant. fects. CHAPTER 2 Operative Dentistry and Prosthodontics

25 26 Chapter 2: Operative Dentistry and Prosthodontics

QUESTIONS

1. A 13-year-old female patient presents with yel- A. Radiograph low, pitted enamel on all permanent teeth. Her B. Fully or partially disappears when wet mother reports that all of the patient’s baby teeth C. Caries detector had a similar appearance. The patient has ex- D. Transillumination cellent hygiene and no caries but is concerned E. Sharp explorer about her appearance. What is the best treatment to address this patient’s chief complaint? 4. As caries progresses from enamel to dentin and toward the pulp interproximally, the radio- A. Composite bonding graphic appearance resembles which of the fol- B. Facial veneers lowing? C. Full crown coverage D. No dental treatment, but refer to a clinic for A. V shape in the enamel with the apex toward treatment of an eating disorder the DEJ and V shape in dentin with apex E. No treatment is necessary toward the pulp B. V shape in enamel with apex toward the prox- 2. A 27-year-old female presents with pain on the imal surface and V shape in dentin with the lower right side in the area of the second premo- apex toward the pulp, resulting in a diamond lar. The HIV-positive patient has smoked two Prosthodontics appearance packs of cigarettes per day for 15 years and is not C. V shape in enamel with the apex toward the interested in quitting smoking. She has multi-

Operative Dentistry and DEJ and V shape in dentin with the apex to- ple amalgam and composite restorations in all ward the DEJ, resulting in a bowtie appear- quadrants. Clinically, the tooth resembles a pri- ance mary mandibular second molar and has a large D. V shape in enamel with the apex toward the occlusal composite with open margins. Radio- proximal surface and V shape in dentin with graphic examination reveals widely divergent the apex toward the DEJ mesial and distal roots and radiolucency in the furcation. What is the most appropriate treat- 5. In a carious lesion in which caries has extended ment for this patient? into dentin, which of the following zones con- A. Extract the tooth and place an implant tains infected dentin and is not capable of rem- B. Root canal therapy, , full cov- ineralization? erage crown A. Normal dentin C. Removal of existing composite, direct pulp B. Subtransparent dentin cap, and composite restoration C. Transparent dentin D. Extract the tooth and replace with three-unit D. Turbid dentin fixed partial denture E. Eburnated dentin E. No treatment is necessary 6. Which of the following is NOT a parafunctional 3. A 25-year-old male patient presents to your of- movement? fice for a routine checkup. He has no signif- icant medical history. Dental history includes A. Swallowing orthodontic treatment 10 years ago and several B. Bruxism occlusal amalgam restorations that which are C. Nail biting 15- to 17-years old. D. Clenching You notice white spots on the facial surfaces of teeth #8 and #9. Which of the following diag- 7. A 28-year-old asymptomatic male patient nostic tools would best aid in differentiating the presents with active caries and active periodontal white lesions as incipient caries versus hereditary disease. What is the best treatment for a posterior enamel hypocalcification? endodontically treated premolar? Questions: 1-14 27

A. Veneer B. Extracting remaining anterior teeth and then B. Metal–ceramic crown fabricating RPD would be best for esthetics C. MO amalgam C. Parallel to the proximal surfaces of the abut- D. Extraction ment teeth adjacent to the space E. Fixed partial denture D. Parallel to the mandibular anterior teeth due F. Partial-coverage restoration to occlusion E. An RPD should not be considered 8. A patient moves his mandible in a right lateral excursion. To which direction will the nonwork- 12. A patient with a 22-mm working length (WL) ing condyle move? will need a post after the endodontic treatment is completed. Which of the following statement A. Down, forward, and medially and Dentistry Operative is true concerning post placement? B. Down, forward, and laterally

C. Up, back, and down A. A 3-mm apical seal must be maintained Prosthodontics D. Up, back, and out B. As post length increases, retention decreases E. None of the above C. Endodontically treated tooth should not be treated with posts 9. At the wax try-in, the maxillary plane appears to D. A 5-mm apical seal must be maintained be correct, yet when the patient closes lightly in E. A post less than 3-mm long is acceptable centric relation, the mandibular teeth appear to show too much. The patient’s lips do not contact 13. A 68-year-old male presents with severe loss of unless the patient forces them to do so. What is tooth structure on an abutment for an exist- the most likely error that has occurred? ing removable partial denture. A crown is in- dicated for the tooth. Which of the following are A. Excessive vertical dimension of occlusion important considerations when preparing a re- B. Inaccurate face-bow transfer movable partial denture abutment to receive a C. Over closed vertical dimension of occlusion crown? D. Excessive amount of freeway space A. Path of draw 10. During the insertion of a complete lower den- B. Location of rests ture, you observe that there is a lift in the denture. C. Orientation of guiding planes You observe the denture becomes dislodged with D. Placement of porcelain on metal finish lines constriction of the genioglossus muscle. What E. All of the above. portion of the denture lifted from position be- cause of contraction of the genioglossus muscle? 14. An endodontically treated posterior tooth has 3 mm of circumferential clinical crown remain- A. Entire denture lifts ing. How many millimeters in length are nec- B. Entire denture dislodges essary to forego a post, using an amalgam core C. Anterior portion lifts build up in prepared post spaces? D. None of the above A. Greater than 3 mm 11. A 44-year-old male presents with missing teeth B. Less than 3 mm #6 to #9. After being presented with many treat- C.5to6mm ment options including rockets power dancers D. 2 mm (RPD), dental implants, and , the patient E. Greater than 2 mm elects for the fabrication of an RPD. Where is the path of insertion going to allow for the best es- thetics minimizing space between artificial and natural teeth? A. Parallel to the most distal maxillary molar or most posterior tooth 28 Chapter 2: Operative Dentistry and Prosthodontics

15. A patient with a complete upper denture and a D. Type II: Crowns and FPDs bilateral lower partial denture presents in your E. Type II: RPDs office for the first time in 5 years. What can you expect to find after 5 years without RPD main- 19. A 56-year-old patient presents with moderate tenance? decay on tooth #30. After observing the bitew- ing and periapical radiograph of this tooth, you A. Loss of vertical dimension of occlusion determine that a PFM is indicated. Since the (VDO) and retrognathic facial appearance restoration is in an area where the margin is of B. Decrease in VDO and prognathic facial ap- low esthetic concern, you decide to prepare the pearance tooth with a supragingival margin. All of the fol- C. Premature posterior contacts and retrog- lowing are advantages of a supragingival margin, nathic facial appearance except: D. Increase in VDO and premature posterior contacts A. Easily finished margins E. Decrease in VDO and prognathic facial ap- B. Cleansable of margins pearance C. Less impingement of gingival tissue during impressions 16. Mechanical consideration in crown preparation D. Improved clinical access for evaluation of design is an important factor that could dictate margins at recall longevity of the fixed prosthesis. All of the follow- E. Increased retention

Prosthodontics ing are categories for mechanical consideration, except: 20. A 25-year-old patient presents in your office to

Operative Dentistry and restore a fractured maxillary central incisor. Af- A. Providing retention form ter evaluation, you determine that the tooth has B. Providing resistance form insufficient structure for an operative restoration C. Preventing deformation of the restoration and that an all-ceramic crown is indicated. What D. Geometry of tooth preparation type of finish line will you prepare? 17. Recognition of wall convergence angles in A. Feather edge crown preparation will determine if undercuts B. Chamfer are produced when taper is too small and if re- C. Shoulder tention is undermined when taper is too large. D. Knife edge What degree of wall convergence during crown E. Bevel preparation will give the prosthesis optimal re- tention? 21. In March 2003, the Journal of the American Dental Association revised the classification sys- A. 1 to 3 degrees tem of dental alloys. What are the requirements B. 3 to 6 degrees for a high noble alloy? C. 4 to 8 degrees D. 6 to 9 degrees A. Noble metal content ≥ 40% (gold + plat- inum group) and gold ≥ 60% 18. Certain restorations (inlay, crowns, fixed par- B. Noble metal content ≥ 60% (gold + plat- tials, removable partials) require alloys intended inum group) and gold ≥ 40% for their use according to physical strength. In C. Noble metal content ≥ 25% (gold + plat- turn, this affects our selection of casting alloy. inum group) and gold ≥ 40% Which of the following correctly pairs the cast- D. Noble metal content ≥ 40% (gold + plat- ing alloy with its intended restoration? inum group) E. Noble metal content ≥ 60% (gold + plat- A. Type III: Complex inlays inum group) B. Type IV: Simple inlays C. Type III: Crowns and FPDs Questions: 15-28 29

22. The pontic design selection with the most es- (e.g., impregum), but they also come with some thetic advantage for a 39-year-old female with disadvantages. Which of the following is not a an anterior bridge, teeth #7–#9, is: disadvantage? A. Saddle-ridge-lap A. Hydrophobicity B. Conical B. Temperature sensitivity C. Sanitary/hygienic C. Latex can interfere with setting D. Modified ridge-lap D. Surface detail E. Modified sanitary E. Wettability F. Ridge lap 27. Gypsum-bonded and phosphate-bonded in-

23. Retraction cord is often used with a hemo- vestments each have their requirements to and Dentistry Operative static agent to capture an accurate impres- be selected as an ideal investment material.

sion. Which hemostatic agents minimize tissue Phosphate-bonded investments are more stable Prosthodontics damage? at high temperatures and therefore better suited for casting metal–ceramic alloys. What is the A. Fe (SO ) and AlCl 2 4 3 3 correct order (first step, second step, . . . last step) B. ZnCl and AlCl 2 3 of the “lost-wax casting” technique? C. Fe2(SO4)3 and ZnCl2 D. (SO4)2 and ZnCl2 I. Casting E. Epinephrine and Fe2(SO4)3 II. Investing III. Wax pattern 24. Ceramic shade matching is an important step IV. Material selection (e.g., gypsum-bonded, in achieving natural esthetics in dental tooth phosphate-bonded) restoration. What is the most important dimen- V. Spruing sion of color for dental restorations? A. I, V, IV, III, and II A. Color B. III, IV, II, and I B. Hue C. III, II, I, V, and IV C. Value D. V, III, IV, II, and I D. Chroma E. IV, V, III, II, and I E. Metamerism F. V, III, II, and I

25. A 20-year-old female presents for her initial den- 28. The addition of resin to tal examination and prophylaxis. During the ex- in the 1990s was introduced to combine desir- amination, you observe wear on the occlusal able properties of both luting agents. Which of surfaces of many posterior and anterior teeth. following is not an advantage of a resin-modified The wear on the posterior teeth are cup like and glass ionomer-luting agent? smooth, and on the anterior, it has smoothed out A. Low microleakage the lingual anatomy of the incisors. This is most B. Luting ceramic restorations typical of which type of wear? C. Fluoride release A. Attrition D. Low solubility B. Erosion E. Reduce postoperative sensitivity C. Abrasion D. Abfraction E. Bruxism

26. Addition silicone (aka polyvinylsiloxane) is one of many impression materials used for crown and bridge final impressions. Addition silicone has greater dimensional stability than polyethers 30 Chapter 2: Operative Dentistry and Prosthodontics

29. Alginate (irreversible hydrocolloid) is a hydro- 32. It is important to select an esthetic tooth shape colloid consisting of a solution of alginic acid and to consider alignment features when fab- and has a physical state that is changed by an ir- ricating any type of prosthesis. Considered in reversible chemical reaction forming insoluble treatment planning and evaluation of patients calcium alginate. The setting reaction is repre- smile analysis, what is the acceptable midline sented by the following formula: deviation? + + → H2O Na alginate Ca2SO4 Ca alginate A. 1 mm gel (insoluble) + NaSO4 B. 2 mm What are the methods for decreasing setting C. 3 mm time of the alginate? D. 4 mm E. 5 mm A. Increase water temperature, mix more rapidly, decrease water:powder ratio 33. An articulator is a mechanical instrument used B. Decrease water temperature, mix less to represent the temporomandibular joint and rapidly, decrease water:powder ratio mandible. A classification system was developed C. Increase water temperature, mix less rapidly, to determine the variable movements an articu- decrease water:powder ratio lator can produce. Which of the following is the D. Decrease water temperature, mix more most commonly used type of articulator? rapidly, increase water:powder ratio A. Class I Prosthodontics 30. A 51-year-old female patient presents to your B. Class II office requiring full mouth extractions. You ex- C. Class III Operative Dentistry and plain the treatment options to her, and she elects D. Class IV upper and lower complete dentures. You explain E. Class V to the patient that fabricating an immediate den- ture (I/D) to insert after the full mouth extrac- 34. Articulators are further distinguished by design. tions is a great option to control bleeding and What is the advantage an ARCON (ARticula- inflammation. What is the time frame an imme- tor, CONdyle) design has over a non-ARCON diate denture can be used? design? A. An I/D can continue to be used as a prosthesis A. The ARCON design has condylar elements for the rest of the patients life in the upper member B. An I/D should only be used for about 1 to B. The ARCON design has condylar elements 2 days post oral surgery in the lower member C. An I/D can be used up to 1 full year post oral C. The non-ARCON design has the condylar surgery inclination of the mechanical fossae at a D. An I/D can be used up to 6 months post oral fixed angle to the maxillary occlusal plane surgery D. The ARCON design has the condylar incli- E. An I/D is not a viable option for full mouth nation of the mechanical fossae at a variable extractions angle to the maxillary occlusal plane as the articulator opens 31. Fabricating a complete denture requires many E. None of the above important steps from impression taking to tooth shape and shade selection. What are the three 35. A 68-year-old male presents to your office for factors for successful fit of a complete denture complete upper and lower denture try-in. At once it has been fabricated? insertion and after some adjustments, the pa- A. Stability, resiliency, and temperature tient continually complains of an uncomfortable B. Stability, support, and retention clicking when he talks. Which of the following C. Retention, resiliency, and temperature could be contributing to the clicking described D. Support, temperature, and retention by the patient? E. Retention, flexibility, and support Questions: 29-39 31

I. Cheek biting I. Removal of unacceptable dentures from the II. Excess VDO mouth for an extended period of time before III. Habit impressions IV. Lack of tissue adaptation at posterior palatal II. Reline the dentures with a tissue condi- seal tioner III. Educate the patient A. I and IV IV. Massage of the tissues and warm saline B. I rinses C. II and III D. II A. I and IV E. I, II, and III B. I and II F. All of the above C. II and III and Dentistry Operative D. I and III

36. A 54-year-old female presents for an initial ex- E. III and IV Prosthodontics amination. She admits to periodontal issues in F. All of the above the past and you observe this in the panoramic examination upon evaluation. You recommend 38. During the denture border molding and final the extraction of her remaining teeth and fabrica- impression appointment, it is important to ob- tion of complete upper and lower dentures. You serve which support and relief areas are for pri- discuss the physiologic nature of the maxillary mary support in both maxilla and mandible. and mandibular bone because of the possibility What are the primary denture support areas on of a reline or even a fabrication of new dentures the maxilla and mandible? in the future. Which statement below best de- A. Rugae, residual ridge scribes the reaction of maxillary and mandibular B. Residual ridge, buccal shelf bone to edentulism? C. Residual ridge, residual ridge A. The maxillary ridge resorbs superiorly and D. Rugae, buccal shelf anteriorly, while the mandibular ridge re- E. Incisive papilla, residual ridge sorbs inferiorly and posteriorly F. Incisive papilla, buccal shelf B. The maxillary ridge resorbs inferiorly and posteriorly, while the mandibular ridge re- 39. A 51-year-old male patient presents for the final sorbs superiorly and anteriorly impression appointment for the fabrication of C. The maxillary ridge resorbs superiorly and a complete upper denture. As you remove the posteriorly, while the mandibular ridge re- final impression from the patient’s mouth in the sorbs inferiorly and posteriorly custom tray, which of the following structures D. The maxillary ridge resorbs superiorly and are you attempting to accurately capture? posteriorly, while the mandibular ridge re- I. Residual ridge sorbs inferiorly and anteriorly II. Labial and buccal frenum E. The maxillary ridge resorbs superiorly and III. Labial and buccal vestibules anteriorly, while the mandibular ridge re- IV. Tuberosities sorbs inferiorly and anteriorly V. Hamular notches VI. Posterior palatal seal 37. A 70-year-old patient presents for her recall ap- VII. Fovea point. She has been wearing her complete den- VIII. Hard palate tures for over 15 years and is ready for a new set of IX. Rugae complete dentures. As you evaluate her oral mu- X. Incisive papilla cosa during your routine cancer screening, you A. I, II, III, IV, and V observe that her oral mucosal tissue is erythema- B. II, IV, VI, VIII, and X tous likely from overuse of her denture. Which C. I, III, V, VII, and XI of the following are issues should you address D. VI, VII, VIII, IX, and X with the patient? E. All of the above 32 Chapter 2: Operative Dentistry and Prosthodontics

40. Border molding is the shaping of the denture 43. In 1 hour, you will see your first complete den- border areas with impression materials typically ture patient as a “real dentist.” In your excitement compound. The shape and size of the vestibule and just to keep on the safe side, you review the is duplicated by either functional and/or man- proper steps for each appointment from begin- ual manipulation of the soft tissue adjacent to ning to end for the fabrication of a complete up- the borders. Rubber base (polysulfide) is an elas- per and lower denture. Which of the following tomeric impression material for taking impres- complete denture appointments is the longest sions after border molding has been completed appointment? on the custom tray. What are the advantages of A. Diagnosis and preliminary impressions using rubber base for this purpose? B. Border molding and final impressions I. Long setting time C. Wax rim try-in, VDO, CR II. Long working time D. Tooth setup try-in III. Flexible and tear resistant E. Delivery/insertion IV. Very unpleasant odor and taste F. 24-hour follow-up V. Highest permanent deformation A. I, III, and IV 44. During the delivery appointment of your pa- B. I, II, and III tient’s complete dentures, you are always sure to C. III, IV, and V give proper instructions for oral tissue and den- D. II and III ture care. You remind your patient that the den- Prosthodontics E. II and V ture should be kept off oral tissue for at least 6 to 8 hours each day, preferably when sleeping.

Operative Dentistry and 41. While anterior tooth selection is based on shape, Which of the following are the benefits of resting size, and shade, the posterior tooth selection is the oral tissue? based on cusp angle. Which one of the follow I. Blood and lymph flow is improved occlusal schemes is correctly paired with its cusp II. Cheek and tongue function will massage angle? the mucosa III. The constant film of stagnant saliva be- A. Anatomic: 0 degrees tween the dentures and the mucosa is re- B. Semi-anatomic: 33 to 45 degrees moved C. Nonanatomic/flat: 10 to 20 degrees IV. Bruxing will not impinge on denture sup- D. Semi-anatomic: 10 to 20 degrees porting tissue E. Anatomic: 33 to 45 degrees A. I and II 42. You are at the tooth try-in appointment where B. I, II, and III you will evaluate if you have captured the cor- C. II, III, and IV rect VDO for your patient’s dentures. Which of D. III and IV the following are VDO guidelines that you can E. I and IV follow to aid in your evaluation? F. All of the above I. Parallelism of residual ridges II. Phonetics 45. A 62-year-old male presents to your office for the III. Patient’s proprioception first time in several years. He has complete den- IV. Appearance tures that have been feeling loose for the last few V. Comparison to old dentures months. Upon examination, you observe that the denture borders do not extend to cover all the A. I, III, and V supporting soft tissue. The patient is not ready B. II, III, and IV for the fabrication of a new complete denture be- C. II, III, IV, and V cause he is comfortable with the way the teeth D. I, III, IV, and V come together. What is the best treatment for E. All of the above this patient? Questions: 40-49 33

A. Stress that fabricating new dentures is the removable prosthesis prior to the appointment only treatment option and are concerned with the factors that a major B. Reline the denture connector requires to be stable in the patient’s C. Instruct the patient on how to use denture mouth. Which of the following are requirements adhesive for a functional major connector? D. Rebase the denture I. Be rigid E. Tell the patient that he will adjust to his ex- II. Join clasps, rests, and indirect retainers isting dentures with time III. Provide vertical support and protect the soft tissue 46. A 35-year-old female presents for a comprehen- IV. Reciprocating stabilization sive oral examination. She admits that this is her V. Provide means of obtaining indirect reten- and Dentistry Operative first dental visit in 15 years when they extracted tion where indicated all four third molars. Upon examination, you ob-

VI. Provide an opportunity of positioning den- Prosthodontics serve decay in several maxillary anterior teeth, ture bases where needed and #4, #5, #12, and #13 require root canal ther- VII. Maintain patient comfort apy. The mandibular anterior teeth have heavy VIII. Connect parts of the prosthesis from one calculus. After explaining all of her treatment side of the arch to the other options, the patient elects to extract all the teeth requiring endodontic treatment and restoring A. I, II, III, IV, V, and VI them with an upper partial denture. What is B. II, III, V, VI, and VII the Kennedy classification for the partial denture C. I, III, IV, V, VII, and VIII you will design for this patient’s final dentition D. I, III, V, VI, VII, and VIII after extractions? E. III, IV, VI, VII, and VIII A. Kennedy Class I, Mod 1 F. All of the above B. Kennedy Class II, Mod 2 49. A 43-year-old male presents to your office for a C. Kennedy Class III, Mod 1 comprehensive dental examination. In your re- D. Kennedy Class IV, Mod 2 view of the panoramic radiograph, you observe E. Kennedy Class III, Mod 2 periodontal disease in the lower anterior denti- F. Kennedy Class I, Mod 2 tion. During your clinical examination, you are + 47. Which of the following is false concerning surprised to only observe 1 mobility on teeth Applegate’s rules governing the application of #22, #23, #25, and # 26. In addition, your patient Kennedy Classification? will require the extraction of his last remaining molar #19. Following a gross debridement and A. If a third molar is missing and is not to be the extraction of tooth #19, your patient will have replaced, it is not considered in the classifi- teeth #20 to #28 remaining. Which of the fol- cation lowing major connectors will be the most likely B. If a second molar is missing and is not to be indicated for your mandibular partial denture replaced, it is not considered in the classifi- design? cation I. Lingual bar C. The most posterior edentulous area always II. Lingual plate determines the classification III. Double lingual bar or Kennedy bar D. Edentulous areas other than those determin- IV. Labial bar ing the classification are referred to as modi- fications and are designated by their number A. I E. There can be only one modification area in B. I and II Class IV arches C. II and III F. All of the above are true D. I, II, and III E. II, III, and IV 48. A patient presents to your office for a frame- F. All of the above work try-in this afternoon. You are evaluating the 34 Chapter 2: Operative Dentistry and Prosthodontics

50. A combination clasp is a circumferential clasp D. Must be placed in the rest seats that transmit that has a retentive arm made of wrought metal, applied forces through the long axis of the thus making it more flexible than a similar abutment tooth arm constructed from cast alloy. The recipro- E. A distal extension partial denture uses the cating arm, made of cast metal, must be rigid to mechanical advantage of leverage by moving counteract the forces generated by the flexible the fulcrum line further from the force wrought metal. What part of the retentive is the F. Cingulum rests designed on maxillary ca- most flexible? nines are of great advantage

A. The terminal half 53. During a recall appointment, your patient states B. The middle third that he has finally decided to take your recom- C. The terminal third mendation of extraction of his upper posterior D. The shoulder half teeth. You discuss with him the sequence of E. The proximal third his treatment plan that includes partial upper 51. Which of the following are not basic require- and lower dentures. After extractions, root canal ments for clasp design? therapy, and periodontal therapy, you must sur- vey and contour natural teeth and crowns. What I. Retention is the modification sequence one should follow II. Support to complete tooth preparations for final impres- III. Rigidity

Prosthodontics sions? IV. Stability V. Reciprocation A. Heights of contour, parallel guiding planes,

Operative Dentistry and VI. Durability retentive contours, rests VII. Encirclement B. Rests, retentive contours, heights of contour, VIII. Esthetic parallel guiding planes IX. Passivity C. Retentive contours, rests, parallel guiding X. Restorability planes, heights of contour D. Heights of contour, retentive contours, rest, A. III, VI, VIII, and X parallel guiding planes B. II, IV, V, VI, and IX E. Parallel guiding planes, heights of contour, C. I, III, IV, V, and VI retentive contours, rests D. I, III, VI, and VII F. Parallel guiding planes, heights of contour, E. V, VI, and 1X rests, retentive contours F. None of the above are basic requirements for a clasp design 54. A 33-year-old male presents with pain. Upon ex- 52. A 30-year-old female patient presents to your amination, you see severely decayed teeth #6 to practice with no upper molars. You present all #11 with edematous exudate from the buccal her treatment options, and she chooses the op- mucosa of several teeth. You recommend the tion of upper partial dentures. There are many extraction of all upper anterior teeth #6 to #11 factors to consider when designing a removable as they are unrestorable. What Kennedy classifi- prosthesis. At this time, you are most concerned cation is this partially edentulous arch? about indirect retention. Which of the following A. Kennedy Class IV, Mod 2 statements is not true of an indirect retainer? B. Kennedy Class I, Mod 1 A. It prevents the distal extension from moving C. Kennedy Class IV away from the underlying tissue during func- D. Kennedy Class II, Mod 2 tion E. Kennedy Class I, Mod 2 B. The ideal location is determined by an imagi- F. Kennedy Class II nary line drawn perpendicular to the fulcrum 55. The design for a lower partial denture will line and as anterior as possible require wrought metal clasps on the direct C. Must be flexible Questions: 50-64 35

retainers. You evaluate the patient’s mounted A. 0.5 mm cast with an undercut gauge. What is the un- B. 1.0 mm dercut required for the retentive arm of a c-clasp C. 1.5 mm direct retainer? D. 2.0 mm E. 2.5 mm A. 0.005 inch B. 0.010 inch 60. All of the following are characteristics of a C. 0.015 inch Class V (five) composite preparation except one. D. 0.020 inch Which one is the EXCEPTION? E. 0.030 inch A. All internal line angles should be rounded

56. All of the following are true for Class II amal- B. Retention provided by undercuts made into and Dentistry Operative gam preparations, except one. Which one is the the axial walls

EXCEPTION? C. Cavosurface margins when placed on Prosthodontics enamel should be beveled A. Proximal retention locks if placed, should be D. Outline form of the prep is determined by entirely in dentin, and are deeper gingivally the extent of the carious lesion than occlusally E. A butt joint is not indicated when finishing B. The axiopulpal line angle is beveled or the preparation on root surfaces rounded to reduce stress C. The uninvolved proximal wall should con- 61. Which of the following marginal designs is the verge slightly toward the occlusal best for cast gold restorations? D. The pulpal depth aids in the resistance form of the preparation A. Shoulder E. The buccal and lingual walls should con- B. Shoulder bevel verge to provide mechanical retention to the C. Chamfer restorative material D. Shoulder with retention groves

57. All of the following are advantages of glass 62. What are the requirements of a good provisional ionomer cements except one. Which one is the restoration? EXCEPTION? A. Provide pulpal protection A. Anticariogenic B. Be able to be easily cleaned B. Biocompatibility C. Have nonimpinging margins C. Bond chemically to tooth structure D. All of the above D. Set via an acid–base reaction E. More soluble in the conditions of the oral 63. What is the recommended prosthodontic treat- cavity than cements ment of a patient missing four maxillary in- cisors with severe ridge resorption and limited 58. All of the following techniques can decrease the finances? setting time for Gypsum except one. Which one A. No treatment of the following is the EXCEPTION? B. Fixed bridgework A. Increasing the powder to liquid ratio C. Maxillary removable partial denture B. Decreasing the rate and time of spatulation D. Maryland bridge C. Contamination of the mixture D. Decreasing the mixing temperature 64. The most rigid palatal connector is: E. Increasing the rate and time of spatulation A. Anterior–posterior palatal bar connector B. Palatal palate 59. What is the ideal amount of remaining dentin C. Palatal horseshoe-shaped connector thickness that is required for an amalgam prepa- D. Single palatal bar ration? 36 Chapter 2: Operative Dentistry and Prosthodontics

65. “T” and “D” sounds are formed by: A. Both statements are TRUE B. Both statements are FALSE A. The tongue protruding slightly between the C. The first statement is TRUE, the second maxillary and mandibular anterior teeth statement is FALSE B. Contact of the tip of the tongue with the D. The first statement is FALSE, the second anterior palate and lingual surfaces of the statement is TRUE maxillary teeth C. The lips only 71. A stimulus on a tooth such as cold or air dry- D. None of the above ing can cause fluid movement to become rapid, which is then interpreted as pain to the nerve 66. Which of components of im- endings in the pulp. This theory is known as pression paste functions as an accelerator of the the: setting time? A. Pulpal inflammation theory A. Rosin B. Hydrodynamic theory B. Resinous balsam C. Thermal sensitivity theory C. Oil of cloves D. Thermal expansion theory D. Calcium chloride 72. Which of the following teeth retain sealants the 67. When fabricating dentures, posterior teeth that best? are set edge to edge may cause: Prosthodontics A. Permanent first molars A. Lip biting B. Primary second molars

Operative Dentistry and B. Tongue biting C. Permanent second premolars C. Cheek biting D. Permanent second molars D. Gagging E. Primary first molars 68. Change in dentinal structure as a response to 73. Which of the following teeth retain sealants the slowly progressive or mild irritations such as me- worst? chanical abrasion or chemical erosion is known as: A. Permanent first molars B. Primary second molars A. Physiologic dentinal sclerosis C. Permanent second premolars B. Reactive sclerosis D. Permanent second molars C. Reparative dentin E. Primary first molars D. Smear layer 74. All of the following can cause damage to the 69. All of the following describe the compositional pulp during cavity preparation except for: and structural aspects of dentin EXCEPT: A. Heat during cutting A. Contains 70% inorganic hydroxyapatite B. Pressure applied B. Contains 18% organic type I collagen and C. Size of bur 12% water D. Amount of time of cutting C. Organic and inorganic constituents of dentin E. Amount of tooth reduction are homogenous in structure D. Dentin is a vital and dynamic tissue due to 75. All of the following cavity classifications can in- the odontoblastic processes serving as a direct volve any teeth except for: connection to the vital pulp A. Class I 70. Primers serve as the adhesion-promoting agents B. Class II and contain both hydrophobic and hydrophilic C. Class V monomers dissolved in organic solvents such as D. Class VI acetone or ethanol. Questions: 65-82 37

76. Conservative tooth preparation is a fundamental full coverage porcelain crown. Which of the fol- principle of sound restorative dentistry. Which of lowing statements about crown preparation of the following strategies regarding tooth prepara- an all ceramic crown is incorrect? tion will result in an unsatisfactory result? A. The recommended facial surface reduction A. Preparing the occlusal surface anatomically for an all ceramic crown is 1 mm of reduction to create uniform thickness in the restoration in two planes anatomically following contour B. Axial wall preparation should be minimal at surface of tooth approximately 6-degree taper B. Incisal reduction of 1.5 mm including lateral C. Crown preparations should utilize partial ce- and protrusive excursions ramic coverage rather than full ceramic cov- C. A lingual chamfer preparation of 1-mm erage in nonesthetic areas depth and Dentistry Operative D. Use of a shoulder margin on all full metal D. Axial wall taper of 6 degrees is ideal to help

crown restorations preserve tooth structure and provide reten- Prosthodontics tion form for the restoration 77. A patient presents to your office with tooth #14 with a large restoration and recurrent decay. You 80. An insurance company has asked you to consult make the determination that the tooth will re- on a claim for an implant supported fixed pros- quire a full metal crown. Which of the following theses. The insurance company is seeking to de- statements would be incorrect regarding your termine if an implant is appropriate in a patient treatment plan for the preparation of this tooth? and has asked you which if any of the following patients are candidates for implant placement A. The functional cusp should be reduced to followed by a fixed prosthesis: provide 1.5-mm clearance B. The nonfunctional cusp should be reduced A. A patient with uncontrolled diabetes to provide 1.5-mm clearance B. A patient with a history of nasopharyngeal C. The axial reduction should allow 0.5 mm of cancer that required head and neck radiation metal thickness at the margin to include the maxilla and mandible D. The functional cusp bevel should be placed C. A patient who is 7-months pregnant at 45 degrees to the long axis D. An 80-year-old female with no medical prob- lems 78. You are treatment planning a patient who has se- E. None of the above lected a PFM (Porcelain Fused to Metal) crown restoration for tooth #8. The patient would like to 81. When preparing a tooth for a composite restora- know how much tooth structure will be removed tion, the dimension of the preparation is solely during your PFM preparation. Which one of the determined by the access and: following statements about your reduction for A. Removing sound tooth structure to obtain tooth preparation would be incorrect? the most optimal preparation A. The desired incisal reduction for a PFM B. The extent of the caries crown is 2.0 mm in intercuspal position and C. Removing sound tooth structure to obtain in excursive movements mechanical retention B. A labial reduction of 1.2 to 1.5 mm (one D. The shade of the tooth. plane) is mandatory for PMF preparations C. The lingual concavity is prepared to allow 82. What are the structural components of compos- 1-mm clearance if the contacts are to be lo- ite? cated on metal A. Polymer matrix and filler particles D. Shoulder must extend at least 1-mm lingual B. A coupling agent and initiator to proximal contact area C. Polymer matrix and coupling agents D. All of the above 79. A patient from your practice is a singer with a high esthetic requirement and is requesting a 38 Chapter 2: Operative Dentistry and Prosthodontics

83. When preparing an anterior tooth for a metal– 88. There are four zones of carious enamel, the ceramic crown, the ideal preparation requires zones of incipient lesions. Which of the four that approximately of tooth structure reduc- zones is the largest? tion is needed. A. Translucent zone A. 1.0 mm B. Dark zone B. 0.8 mm C. Body zone C. 2.5 mm D. Surface zone D. 1.5 mm E. Alfi zone

84. What is an advantage of using a cast-gold restora- 89. All of the following are associated with Rampant tion? caries except: A. Gold is a strong material and rarely fractures A. Acute onset B. A cast-gold restoration has esthetics superior B. Rapid progression to other restorations C. Children C. The impression material that is used for cast- D. Deep and wide gold restorations is less expensive E. Associated with pain D. Gold has a wear rate that is similar to porce- lain 90. Which of the following are known as high-risk factors for caries? Prosthodontics 85. When restoring an endodontically treated tooth, A. Visible cavitation what are the first two things that must be consid-

Operative Dentistry and B. Restoration in past 3 years ered? C. Exposed roots A. The canal configuration and if a post will be D. Deep pit and fissures necessary E. A and B B. Which restoration is indicated and if a post F. A, B, and C will be necessary G. All of the above C. The canal configuration and which restora- tion is indicated 91. Which of the following bone graft materials D. Which restoration is indicated and the func- area osteoconductive, osteoinductive, and os- tion of the restored tooth teogenic? A. Xenograft 86. The following are true of dental caries except: B. Alloplast A. The critical pH is 5.5 C. Allograft B. Incipient caries are able to remineralize D. Autogenous graft C. Frank caries describe caries that have pro- E. Both D and C gressed into dentin D. Dentin caries progress faster than enamel 92. All of the following are considered to be drugs caries used for oral conscious sedation except: E. Saliva is carioprotective A. Triazolam (Halcionr ) B. Fentanyl (Duragesicr ) 87. The most common cariogenic bacteria respon- r C. Lorazepam (Ativan ) sible for coronal caries are: r D. Diazepam (Valium ) A. Streptococcus mutans and Lactobacilli E. All of the above can be used B. Actinomyces viscosus C. Streptococcus sanguis 93. Single-tooth implants show a decreased risk of D. Veillonella caries to the adjacent teeth BECAUSE of the E. Actinomyces naeslundii patients improved ability to clean the proximal surfaces of these teeth. Questions: 83-95 39

A. Both the statement and the reason are correct C. The first statement is TRUE, and the second and related statement is FALSE B. Both the statement and the reason are correct D. The first statement is FALSE, and the second but NOT related statement is TRUE C. The statement is correct, but the reason is NOT 95. In a Provisional Fixed Partial Denture that is D. The statement is NOT correct, but the rea- seated immediately after the extraction of a son is correct tooth, which of the following pontic forms will E. NEITHER the statement NOR the reason is result in tissue blanching and provide the best correct support to the surrounding papillae while also providing for the best esthetic outcome follow- prtv etsr and Dentistry Operative 94. Defects in color vision primarily affect 8% to ing healing of the extraction site? 10% of the female population. Types of defects

A. Sanitary (hygienic) pontic Prosthodontics in color vision include achromatism, dichroma- B. Conical pontic tism, and anomalous trichromatism. C. Ovate pontic A. Both statements are TRUE D. Modified ridge lap pontic B. Both statements are FALSE E. None of the above 40 Chapter 2: Operative Dentistry and Prosthodontics

ANSWERS

1. The correct answer is C. The patient has amel- wide at the enamel surface and converges with ogenesis imperfecta, a rare hereditary condition the apex toward the DEJ. Then in dentin, caries that results in a reduced amount of enamel, progresses faster because of less mineral content. which is softer than normal enamel and is yellow V-shaped caries has a broad base at the DEJ and in color and pitted. Both primary and perma- converges to the apex toward the pulp. nent teeth are affected. The dentin and pulps are normal, and there is no increase in rate of 5. The correct answer is D. Turbid dentin is the caries. The only necessary treatment is cosmetic only zone listed, which is infected dentin. Bac- treatment, which is best accomplished by full terial invasion has occurred in dentinal tubules coverage crowns. Answer choice D refers the pa- of turbid dentin. Normal, subtransparent, and tient to an eating disorders clinic. This would be transparent dentin are all affected dentin and inappropriate in this case, but if the patient ex- are capable of remineralization. The dentin in hibits enamel erosion on the lingual surfaces of an arrested or remineralized lesion is eburnated the teeth, referral to a therapist who specializes or sclerotic dentin. in treatment of eating disorders may be appro- priate. 6. The correct answer is A. Swallowing, masti- cation, and speech are considered functional Prosthodontics 2. The correct answer is D. The patient has a con- movements of the mandible. Parafunctional genitally missing permanent right mandibular movements are sustained activities that occur be- Operative Dentistry and second premolar and an over-retained primary yond the normal functions of those mentioned second molar in that space. Primary teeth are above. Bruxism is initiated at a subconscious likely to exhibit furcation canals, and if infec- level diurnal, nocturnal, or both. It is the sus- tion is present radiographically in the furcation, tained grinding, rubbing together, or gnashing the most appropriate treatment is to extract the of the teeth with greater than normal chewing tooth. Ideal replacement of the missing tooth in force. Nail biting is an example of parafunctional a healthy patient would be to place a single-unit activity. Parafunctional activity can cause exces- implant. However, this patient is HIV positive sive wear, widening of the PDL, and mobility, and smokes two packs per day. Both are likely to migration, or fracture of the teeth. Clenching is impede healing, so placement of an implant is the pressure and clamping of the jaws and teeth not the most desirable treatment option. A three- together, frequently associated with acute ner- unit fixed partial denture would be the most ap- vous tension or physical effort. propriate treatment. 7. The correct answer is C. It is recommended 3. The correct answer is B. Incipient caries and to restore the patient to a functional status by enamel hypocalcification appear similar clini- eliminating caries and controlling periodontal cally. Toproperly diagnose, wet the surface of the disease prior to treatment planning with fixed tooth. If the lesion fully or partially disappears, prosthodontics. Veneers, crowns, fixed partial and drying it again causes it to reappear, the di- dentures, and partial coverage restorations are agnosis is smooth surface incipient caries. Nei- contraindicated with both active caries and ac- ther radiograph nor transillumination will show tive periodontal disease. Extraction is a final facial incipient lesions. Caries detector will ap- treatment option but may not be required at pear similar with both lesions. Both lesions will this point. Caries control and treatment plan- appear hard and smooth, so a sharp explorer will ning prior to restoration of edentulous space(s) not diagnose. is recommended. Endodontically treated tooth often require cast post and core fabrication prior 4. The correct answer is A. In interproximal or to fixed restorations. smooth surface caries, demineralization starts Answers: 1-16 41

8. The correct answer is A. The working condyle fracture. If the root is short, a 3-mm apical seal is rotational, while the nonworking condyle may be considered acceptable. demonstrates translational movement. As post length increases, retention increases. However, the relationship is not linear. A post 9. The correct answer is A. Vertical dimension is that is too short will fail, whereas one that is too a combination of relaxed muscles, lips at rest, long may damage the seal of the root canal fill or varying freeway space, harmony between lower risk root perforation if the apical third is curved and middle one-third of the face, ability to speak or tapered. Posts less than 3 mm in length are without bite rims contacting, tongue room for unacceptable. making the “th” sound, satisfaction of the pa- tient’s tactile sense, and a consistent rest position 13. The correct answer is E. All are considerations prtv etsr and Dentistry Operative measurement. for fabrication of a crown under an existing RPD. Since the maxillary plane “appears” close, Path of draw or withdrawal is planning both and there is light contact near centric rela- guide planes and reciprocal planes surfaces, as Prosthodontics tion, face bow is not the error. Remember the well as areas that require survey lines in the gin- problem is the straining lips to contact each gival third. Location of rests (at least 1mm deep) other. will help you determine RPD design and eval- Answer choice C would be true if lips were uate where rest seat will be in new crown. Ori- contacting “too much,” and there was a lack of entation of guide planes is important because of harmony with lower and middle one-third of the path of draw. Finally, it would be unesthetic if face. porcelain were to chip because of the stress from Freeway space is the space between the oc- retentive metal arms of RPD design. cluding surfaces of the maxillary and mandibu- lar teeth when the mandible is in physiologic 14. The correct answer is A. If more than 3 to 4 mm position. of coronal tooth structure remains, use of a post in the root for retention is not necessary, and this 10. The correct answer is C. avoids the chance of perforation. Answer choices B, D, and E are incorrect because more tooth 11. The correct answer is C. The best anterior es- structure is required for a secured core without thetic will be achieved by using the adjacent a post. tooth to a space for path of withdrawal because the contour of the artificial tooth can be ma- 15. The correct answer is B. All patients using re- nipulated to fit existing natural tooth. Extraction movable prosthodontics should visit his or her of the remaining anterior teeth may be an op- dentist no less than one time a year, preferably tion if these remaining teeth are compromised. every 3 months. Although occlusion with the lower mandibular teeth is a consideration for esthetics, it is not be- 16. The correct answer is D. Geometry of tooth cause of its path of draw. All treatment options preparation is a subcategory of retention and re- should be considered and presented to the pa- sistance form. It is not considered a category tient. of mechanical consideration. Providing reten- tion form can be further divided to include (1) 12. The correct answer is D. Most endodontic texts magnitude of the dislodging forces, (2) geome- advocate maintaining a 5-mm apical seal. Ide- try of the tooth preparation, (3) roughness of the ally, the post should be as long as possible with- fitting surface of the restoration, (4) materials out jeopardizing the apical seal or the strength being cemented, and (5) film thickness of the or integrity of the remaining root structure. luting agent. Providing resistance form can be If the post is shorter than the coronal height further divided to include (1) magnitude and di- of the clinical crown of the tooth, the prognosis rection of the dislodging forces, (2) geometry of is unfavorable. Stress is distributed over a smaller the tooth preparation, and (3) physical properties surface area and more likely to cause radicular of the luting agent. Preventing deformation of 42 Chapter 2: Operative Dentistry and Prosthodontics

the restoration can be further divided to include is overprepared. Bevels are rarely indicated ex- (1) alloy selection, (2) adequate tooth reduction, cept for full-gold crowns, inlay and onlay prepa- and (3) margin design. rations. Often this type of finish line is used to relieve a lip when overprepared with a chamfer 17. The correct answer is B. Three to six degrees finish line. has been shown to have the optimal retention in crown preparation. One to three degrees is too 21. The correct answer is B. small a degree of convergence; therefore, a possi- bility of undercuts exists. Four to eight degrees is Revised Classification System for Alloys incorrect because any angle greater than 6 will for Fixed Prosthodontics undermine retention. Likewise, 6 to 9 degrees will undermine retention. Classification Requirement High noble alloys Noble metal content ≥ 60% (gold + platinum group∗) 18. The correct answer is C. Crowns and FPDs are and gold ≥ 40% best fabricated with Type III alloys. Complex in- lays are best fabricated with Type II alloys. RPDs Titanium and Titanium ≥ 85% and pinledges are fabricated with Type IV alloys. titanium alloys Noble alloys Noble metal content ≥ 25% Nickel-chromium alloys are stronger than Type ∗ (gold + platinum group ) IV alloys. Some patients may present with an al- Predominantly Noble metal content < 25% + ∗ Prosthodontics lergic reaction to nickel-chromium. Simple in- base alloys (gold platinum group ) lays are best fabricated with Type I alloys. Type ∗ III alloys are used to fabricate Crowns and FPDs. Metals of the platinum group are platinum, palla- Operative Dentistry and dium, rhodium, iridium, osmium, and ruthenium. 19. The correct answer is E. When the margin preparation is taken subgingivally, there is an 22. The correct answer is D. A modified ridge lap increase in surface area; this in turn increases is recommended in areas of high esthetic con- retention. Keeping the margin above the gin- cern (i.e., anterior teeth). This design is moder- gival tissue decreases the complications of tooth ately easy to clean. Pontic design is classified into preparation. Patients are able to keep the restora- two categories, mucosal contact and no mucosal tion clean with daily oral hygiene if the margin contact. A modified ridge lap is categorized by in maintained above the gingival tissue. Impres- mucosal contact. sion material will have better access to margins Saddle-ridge lap is not recommended pontic if maintained supragingivally. The clinician will design because its concave fitting surface over- have direct visual access for evaluation of the laps the residual ridge of missing tooth. This tight PFM during recall if the margin in maintained fit with the tissue does not allow for adequate hy- supragingivally. giene and will accumulate plaque causing tissue inflammation. 20. The correct answer is C. All-ceramic crowns, A conical pontic is easy for the patient to such as those often used in areas of high esthetic keep clean. It has only one point of contact with consideration, are best prepared with a shoulder the gingival tissue as the design is convex, heart finish line. The shoulder finish line is used to shaped. This type of design would be recom- minimize risk of porcelain fracture. Featheredge mended in the posterior region where esthetics is usually contraindicated when preparing for ce- is not a high concern. ramic crowns because of over-contouring of the The sanitary/hygienic design is categorized crown near the margin. It typically does not pro- by no mucosal contact; therefore it is most vide sufficient bulk on the restoration, although hygienic. However, with poor esthetics, it is it is the most conservative type of finish line. typically considered for posterior areas with in- Chamfer finish lines are best suited for margins creased bone loss. It is the least “toothlike.” prepared for metal restorations. This type of fin- The modified sanitary is a modification of the ish line can cause a lip on the margin if the tooth sanitary design, but its archway design between Answers: 17-30 43

the abutment teeth allows this design to increase It is also a disadvantage of polyethers, causing ex- connector size while decreasing the stress to the pansion when contacted with moisture. pontic. 27. The correct answer is B. A wax pattern is made 23. The correct answer is A. Ferric sulfate and alu- on the die. A sprue is attached to the wax pat- minum chloride are good hemostatic agents that tern. There is no such spruing step. A sprue is minimize tissue damage. Zinc chloride causes a cast metal or plastic acting as a channel, con- tissue necrosis and is no longer used. Potas- necting the casting to a sprue button. Investment sium aluminum sulfate is less effective than material selected once the choice of alloy is se- epinephrine. Epinephrine causes vasoconstric- lected. Once the investment of material is in- tion and should be used with caution especially serted, reaching the wax pattern and has time to prtv etsr and Dentistry Operative in lacerated tissue. It may cause tachycardia, so set, the wax pattern is eliminated by burn out. it is contraindicated in cardiac patients. Finally, the casting send melted alloy into in- vestment space, producing a replica of the wax Prosthodontics 24. The correct answer is C. Value is the relative pattern. lightness/whiteness or darkness/blackness, and brightness of a color. Hue, value, and chroma 28. The correct answer is B. All ceramic restora- are the three dimensions of color. Hue is the ac- tions are contraindicated from using R-MGI lut- tual color. Chroma is the amount of saturation ing agents because of associated risk of fracture. of the hue. Metamerism is when two objects ap- Low microleakage is an advantage of R-MGIs. pear to color match under one light source but Fluoride release is an advantage of R-MGIs. Low not another. solubility is an advantage of R-MGIs. Reduced sensitivity is an advantage of R-MGIs. Luting 25. The correct answer is B. Erosion (acid ero- agents containing phosphoric acid have a his- sion) is wear because of chemical means (e.g., tory of postoperative sensitivity such as with zinc bulimia, GERD). It does not involve bacterial phosphate. action-producing defects (deep facial and cervi- cal wedge-shaped depressions). Attrition is nor- 29. The correct answer is A. Since set irreversible mal wear of occlusal and/or incisal surfaces of hydrocolloid is largely water, it will absorb (im- opposing teeth during mastication but can turn bibition) and give off (syneresis) liquid causing a excessive with parafunction. Abrasion is abnor- distortion in the impression. Therefore, the algi- mal wear because of mechanical process other nate impression should be poured immediately. than mastication (e.g., toothbrush). Biomechan- Be sure to always follow manufacturer’s instruc- ical loading forces that lead to flexure fatigue tions. degradation at a distant location on the tooth cause abfraction. Bruxism is not a type of wear 30. The correct answer is D. As inflammation de- but is rather the parafunctional habit of grinding creases, mucosal tissue heals, and bone resorbs teeth. and remodels. As a result, the I/D may not sup- port or retain itself on the ridge. This may lead to 26. The correct answer is D. PVS has excellent pain or fracture of the denture. The denture will surface detail. Along with dimensional stabil- need to be relined or be remade after 6 months. ity, these are the advantages of using a PVS ma- An immediate denture should never used as terial for final impressions. Hydrophobicity is a a complete denture. Once time has allowed the disadvantage of PVS. Polyether’s advantages are tissue to heal properly, a better impression can low permanent deformation, dimensional sta- be taken to fabricate a complete denture meant bility second only to PVS, and hydrophilicity. for a use over a longer term. Complete dentures Temperaturesensitivity is a disadvantage of PVS. also will not last forever but are fabricated for Polyether’s main disadvantage is rigidity during multi-year use. removal of impression. Latex gloves will retard An immediate denture is used to control in- PVS setting. Wettability is a disadvantage of PVS. flammation and protect the surgical site from 44 Chapter 2: Operative Dentistry and Prosthodontics

further trauma that may occur within 1 to 2 days 35. The correct answer is C. Excess VDO is usu- post oral surgery. In addition, an I/D will aid ally the primary problem that can be corrected to maintain VDO and a more natural esthetic by resetting the tooth setup to decrease the VDO while tissue heals. to a more comfortable and stable height. Click- Depending on the patients healing process, ing is also commonly the result of habit. If the a year may be too long. Six months is typically problem is habit, this is more difficult to correct. enough time for bone remodeling to occur in The dentist should refrain from using porcelain the maxilla and the mandible. teeth. Check biting is caused by insufficient VDO 31. The correct answer is C. Stability is the resis- and horizontal overlap of posterior teeth. This tance of the denture base against lateral forces. can be corrected by resetting tooth setup prior Support is the resistance to the forces directed to processing. The lack of posterior palatal seal against the tissues. Retention is the resistance to adaptation may lead to gagging. A reline will be the dislodgement of the denture base away from required to achieve a proper seal. You may also the tissues. consider the posterior length of the denture if Temperatureshould not affect a denture once gagging is a problem and reduce to a comfort- it has been fabricated. Resiliency and flexibility able length. are more important fit factors for partial den- tures. 36. The correct answer is D. The direction of ridge

Prosthodontics resorption for the maxilla is superior and poste- 32. The correct answer is B. Studies show that mid- rior, while the mandible is inferior and anterior. lines up to 2 mm off center are not noticeable This discussion with the patient is important so Operative Dentistry and unless they become canted obliquely. the patient understands that the denture does not last forever. 33. The correct answer is C. The semiadjustable (Class III) articulator provides diagnostic infor- 37. The correct answer is F. All of the answer mation while minimizing clinical adjustment choices are reasonable actions to be taken with at the try-in appointment. It simulates condylar this patient. Patient education regarding denture pathways by using averages or mechanical equiv- care and use is particularly important because alents for all or part of the motion. Class I articu- without it, the patient will recreate the same mis- lators are nonadjustable and only accept a single takes in denture care and use. registration. Class II articulators are also non- adjustable; however, they accept a horizontal 38. The correct answer is B. The residual ridge is and vertical registration. Movements are not ori- the primary maxillary area for denture support ented to the TMJ. Class IV articulators are fully and the buccal shelf is the primary mandibular adjustable. They accept a three-dimensional dy- area for denture support. namic registration. These instruments allow for The secondary area for denture support in orientation of casts to the TMJ and simulation maxilla is rugae. Relief areas are the incisive of mandibular movements. Class V articulators papilla, prominent midline suture, and areas of do not exist. the residual ridges that are highly displaceable. The secondary area for denture support in the 34. The correct answer is B. The ARCON is mandible is the residual ridge. Relief areas are anatomically correct, which makes the under- the sharp spiny ridges with overlying displace- standing of mandibular movements easier. The able tissue, mental foramen if exposed by se- condylar element is in the lower member, and its vere resorption, sharp mylohyoid ridges, tori, and condylar inclination to maxillary occlusal plane prominent genial tubercles. angle is fixed. The non-ARCON design has its condylar elements in its upper member, and its 39. The correct answer is E. Landmarks of the condylar inclination to maxillary occlusal plane mandible that should be accurately captured are angle changes as the articulator opens. the residual ridge, labial and lingual frenum, Answers: 31-45 45

buccal frenae, buccal shelf areas, external The patient should feel comfortable with the oblique lines, retromolar pads, lingual sulci, and denture. Teeth should not prematurely contact retromylohyoid spaces. each other and should not interfere with the pa- tient’s proprioception. 40. The correct answer is D. The advantages of us- Check the patient’s profile. An increased ing polysulfide are the long working time, its VDO would cause strained facial expression. A flexibility, and its tear-resistant properties. Long decreased VDO would cause a drooping at the setting time, unpleasant odor, and highest per- corners of the mouth and a prognathic appear- manent deformation are disadvantages of poly- ance. sulfides. Evaluating and measuring an old denture can Final impressions are beaded with rope wax be one of the most important tools to determin- and Dentistry Operative and boxed with boxing wax to create master casts ing a patient’s VDO. with proper land areas along the borders of the impression and an adequate base. 43. The correct answer is E. The delivery/insertion Prosthodontics An interim denture base is fabricated on the appointment is typically the longest because all master cast to support the occlusal rim. The oc- discrepancies in tissue adaptation, border ex- clusal rim is made of wax to help with jaw rela- tension, VDO, occlusal harmony, and esthetic tion records and the setting of the teeth. The rim value must be addressed. When the corrections is contoured and adjusted in the mouth. are accompanied by patient education and in- structions, the adaptation should be a pleasant 41. The correct answer is E. Thirty-three to 45 de- learning experience for the patient. It is very grees is the correct angle for an anatomical poste- important to have a 24-hour follow-up to ad- rior tooth setup. There are six factors relevant to just undetected errors. Delivery should not be selection of posterior teeth: (1) occluso-gingival schedule on Friday unless your office is open on length, (2) mesiodistal width, (3) buccolingual Saturdays. width, (4) shade, (5) type of occlusal surface, and (6) material. 44. The correct answer is F. These are all benefits of The semi-anatomical posterior setup is 10 to resting the oral tissue from the constant contact 20 degrees, while the nonanatomical setup is 0 of the denture on it. degree. Anatomical setup is used for easier pen- etration of food, better esthetics, interdigitating 45. The correct answer is D. Rebasing is the replace- cusps offer a guide for jaw closure, and can be ment of the entire denture base while keeping ground to harmonize with the TMJ and jaw clo- the same denture teeth in their current occlusal sure. The nonanatomical set-up is used as a sim- relationship. This option can also be utilized pler technique requiring less instrumentation. It when the denture base has been fractured or has is also used when closure is in more than one po- become stained or discolored. sition and adapts more easily to Class II and III Although the fabrication of new dentures jaw relationships. would be the best option, especially if dentures have been used over a long period, personal and 42. The correct answer is E. Parallelism of the resid- economical issues may make this option unrea- ual ridges is an excellent guide if the ridges sonable. have not experienced excessive bone resorption. Relining is the replacement of the intact sur- Once casts have been mounted, the maxillary face of the denture base with a new layer of ma- and mandibular casts can be evaluated for par- terial to make up for loss of supporting tissue. allelism, which will aid in VDO approximation. It is important to understand that a visual refer- The closest speaking space, or 1 mm, is con- ence of the loss of supporting tissue is important sidered when the patient speaks. Special con- before recommending this treatment option. sideration should be given to “ch,” “s,” and “j” The use of denture adhesive should never be sounds. Teeth should not touch during normal the solution to an ill-fitting denture. conversation. 46 Chapter 2: Operative Dentistry and Prosthodontics

The maxilla and mandible will continue to retainers, indirect retainers, auxiliary rests, and resorb, thus causing the denture to become even denture bases. more unstable over time. The types of maxillary major connectors in- clude: 46. The correct answer is C. 1. Single posterior palatal bar Kennedy Class I—Bilateral distal extension 2. Palatal strap Kennedy Class II—Unilateral distal extension 3. Anterior posterior or double, palatal bar Kennedy Class III—All tooth supported 4. Horseshoe or U-shaped connector Kennedy Class IV—Single anterior area 5. Closed horseshoe or anterior posterior palatal crossing the midline. This classification strap (IV) cannot have a modification. An easy 6. Complete palatal coverage method to determine modification is to count the number of missing spaces (spans 49. The correct answer is C. A lingual plate and not teeth) in a single arch and subtract that Kennedy bar will both provide support, such as number by 1. with a splint, to anterior teeth that have lost sup- port of the bone. They also provide additional 47. The correct answer is E. Applegate’s rule 8 states indirect retention when most, if not all, posterior that there can be no modification areas in Class teeth are being replaced with the partial denture. IV arches. The following are Applegate’s rules: Remember that the major connector itself is not

Prosthodontics Rule 1—Classifications should always follow an indirect retainer; however, the support given rather than precede any extractions of teeth that by the rests on the anterior teeth supply indirect

Operative Dentistry and might alter the original classification. retention. Since the bar is contacting all anterior Rule 2—If a third molar is missing and is not teeth, the force is distributed along the bar to all to be replaced, it is not considered in the classi- teeth contacted, thus reducing total force on a fication. Answer B is Rule 4, Answer C is Rule 5, single tooth. The lingual plate should be con- and Answer D is Rule 6. All are true statements sidered before the Kennedy bar because of food and therefore the incorrect answers. entrapment between the double bar and tongue Rule 3—If a third molar is present and is to annoyance with the Kennedy bar. be used as an abutment, it is considered in the The lingual bar is the most commonly used classification. major connector. Its advantage is its simplicity Rule 4—If a second molar is missing and is and its minimal contact of oral tissue. not to be replaced, it is not considered in the The labial bar is indicated for patients with classification. severe lingually inclined lower anterior teeth Rule 5—The most posterior edentulous area and/or premolars where a traditional lingualized always determines the classification. bar cannot be fabricated. This major connector Rule 6—Edentulous areas other than those should only be used when absolutely necessary determining the classification are referred to as and all other options have been exhausted. modifications and are designated by their num- ber. 50. The correct answer is C. A retentive clasp is di- Rule 7—The extent of the modification is not vided into three parts: the proximal, middle, and considered, only the number of additional eden- terminal. The terminal third is placed beneath tulous areas. the height of contour, allowing it to engage at the undercut. 48. The correct answer is D. Answer choices II Although there is a variable amount of flex- and IV describe minor connector functions. Mi- ibility in the middle part, as the wrought wire nor connectors connect all the remaining com- transforms from rigid (proximal) to flexible (ter- ponents of the RPD to the major connector minal), the terminal end is still more flexible. and provide stress distribution. These remain- The proximal part is the most rigid and is posi- ing components include clasp assemblies, direct tioned above the height of contour. Answers: 46-57 47

51. The correct answer is A. Rigidity, durability, es- restoration. By having the buccal and lingual thetic, and restorability are not requirements for walls converge occlusally will lock the restora- clasp design. The six basic requirements for clasp tion in place. Proximal retention locks should be design are as follows: placed 0.2 mm into the DEJ. Entirely in dentin 1. Retention—provide retention against dis- maintaining enamel support, regardless of the lodgement axial depth. If the retention locks are placed en- 2. Support—property of clasp to resist displace- tirely into the axial wall, there is no effective re- ment in a gingival direction tention obtained, and there is the increased risk 3. Stability—resistance to horizontal displace- of pulpal exposure. ment Beveling or rounding the axiopulpal line an-

4. Reciprocation—to resist horizontal forces ex- gle increases the bulk of and decreases the stress and Dentistry Operative erted on the tooth by the retentive arm concentration within the restorative material. This aids in the resistance form of the prepa- 5. Encirclement—each clasp must encircle Prosthodontics more than 180 degrees of the abutment tooth ration. 6. Passivity—for insertion and removal of partial The uninvolved proximal wall should be denture slightly obtuse (6 degrees) and diverge toward the occlusal. This will provide adequate sup- 52. The correct answer is C. An indirect retainer port and prevent undermining of the uninvolved must be rigid. If the retainer were to flex, forces marginal ridge and fracture of the restoration. would be multiplied instead of dissipated. The The resistance form aids in the resistance of remaining statements are true regarding indirect the restoration and the tooth to fracture as a re- retainers. Indirect retainers are most often incor- sult of occlusal forces. The pulpal depth of the porated when there is a unilateral or bilateral restoration is preferred to be a minimum of 2.0 distal extension. mm as measured from the central fossa and en- tirely into an even layer of hard dentin. 53. The correct answer is E. After examination, di- agnosis, and the treatment planning phases, the 57. The correct answer is E. Glass ionomer ce- sequence of mouth preparation appointments ments are made of a polyacrylic acid liquid and must be planned with the goal of conserving as an acid-soluble calcium fluoroaluminosilicate much time as possible. glass powder. Glass ionomer releases fluoride over a sustained period of time, which aids in 54. The correct answer is C. Kennedy Class IV is a the remineralization of tooth structure and has single, but bilateral (crossing the midline), eden- been shown in studies to inhibit the progres- tulous area located anterior to the remaining sion of secondary caries. Glass ionomer bonds teeth. chemically to tooth structure by having the car- Kennedy Class I—Bilateral distal extension boxyl groups of the polyacids chelated by the cal- Kennedy Class II—Unilateral distal extension cium, which is in the apatite of the enamel and Kennedy Class III—All tooth supported dentin. Enamel has a higher inorganic compo- Kennedy Class IV—Single anterior area sition than dentin, and thus the bond strength crossing the midline. This classification to enamel is higher. Calcium hydroxide liners (IV) does not have a modification. should only be used if the remaining dentin thickness is less than 0.5 mm. This is done 55. The correct answer is D. Wrought alloy clasps to protect these deep areas from direct contact are placed in an undercut of 0.020 inch. Chrome with unset glass ionomer. Glass ionomer shows metal cast clasps are placed in an undercut of mild pulpal effects that tend to subside within 0.010 inch. Golf cast clasps are placed in an un- a month’s time. When glass ionomer cements dercut of 0.015 inch. are compared with zinc phosphate cements, glass ionomer cements have shown a lower sol- 56. The correct answer is C. Retention form of the ubility in the environment of the oral cavity. preparation prevents dislodging of the amalgam Mechanical properties with the exception of a 48 Chapter 2: Operative Dentistry and Prosthodontics

lower elastic modulus are similar in comparison is very deep and they will provide thermal insu- as well. lation, thus protecting the pulp. With composite restorations, a base is not needed for thermal in- 58. The correct answer is B. The setting time is the sulation; however, with small pulpal exposures, time needed from the start of mixing to the calcium hydroxide is still used for its ability to time for a material to reach a state of hardness. aid in the formation of secondary dentin bridges. The penetration test is used to determine the A small amount of calcium hydroxide can be end point of the reaction. The setting time of placed on a pinpoint exposure and covered with Gypsum can be manipulated as to increase or a liner (usually, a zinc phosphate) to provide decrease. The following chart below shows ways pulpal protection. This is a direct pulp cap pro- to decrease the setting time of Gypsum: cedure. An indirect pulp cap is when a small amount of carious dentin is left behind and then Variable Decrease setting time covered with calcium hydroxide. This procedure P:L ratio By ⇑ P:L ratio, have less liquid utilizes the antimicrobial properties of calcium per unit volume at start of hydroxide. mixing Rate and time Longer and more rapid the 60. The correct answer is E. Composite restorations of spatulation plaster is mixed, the shorter should have all internal line angles rounded; this setting time that can be allows for better banking of the composite mate- achieved Prosthodontics Contamination If impurities are added to rial. It is easier to place composite into rounded of P:L mixture, it will decrease the line angles than sharp ones. Remember, sharp set time; this can be done by internal line angles are needed for retention Operative Dentistry and the addition of slurry in gold restorations. Micromechanical bonding water/terra alba. This is the through acid etching provides primary retention addition of small amounts of for composite restorations when needed addi- set Gypsum in watery mixture tional retention can be obtained. Undercuts can Temperature Temperature has shown to be made into the gingivo-axial and inciso-axial have minimal effect; it, lines angles with a small round bur; this will however, slightly shortens provide a form of mechanical retention. This is the set time of the reaction usually done for larger carious lesions. The cavo- surface margins when placed on enamel should be beveled. This is true as beveling will allow the 59. The correct answer is D. The pulpal depth of an acid-etch attack to occur at the ends of enamel amalgam preparation should extend to a mini- rods, allowing for better adhesion and retention mum of 0.5 mm into sound dentin. This will aid of the restorative resin material. Beveling also in the resistance form of the cavity preparation. aids in preventing microleakage. Outline form The ideal remaining amount of dentin to pro- of the prep is determined by the extent of the car- vide thermal insulation to the pulp is 2.0 mm. If ious lesion; this holds true for all restorative ma- the remaining dentin thickness is less than 2.0 terials. A butt joint is indicated when the prepa- mm, a base or liner should be used to replace ration does not end on enamel. This allows for the destroyed dentin. increased retention and a better seal when bond- ZOE (zinc oxide and eugenol) cannot be ing to cementum or root dentin. used on restorations where bonding to enamel or dentin is needed since the eugenol will in- 61. The correct answer is C. A chamfer margin pro- terfere with the curing phase (polymerization) vides the best support of a cast gold restoration. of resin-based composites. Picking a desirable A shoulder is used for porcelain jacket and all base for a deep restoration is required to replace ceramic crowns. A shoulder bevel is used for the amount of dentin that has been destroyed. A proximal boxes of inlays and occlusal shoulders base is needed when the restoration to be placed of mandibular three-fourth crowns. Answers: 58-72 49

62. The correct answer is D. All of the state- 67. The correct answer is C. Posterior denture teeth ments are requirements of a good provisional that are set edge to edge can cause cheek bit- restoration. The restoration must be fabricated of ing. Lip biting after denture fabrication can be material that prevents the conduction of temper- caused by reduced muscle tone or a large ante- ature extremes. The temporary restoration must rior horizontal overlap. Tongue biting may be be made of material and possess contours that the caused by having the denture teeth set too far patient will be able to clean easily. It is of great lingually. Gagging may be caused by a denture’s importance that gingival margins of the tempo- posterior palatal seal extended too far poste- rary not impinge upon the gingival tissues. riorly.

63. The correct answer is C. This patient needs to 68. The correct answer is D. Physiologic dentinal prtv etsr and Dentistry Operative function and needs a replacement for esthetics; sclerosis is the natural aging process of dentin. therefore, no treatment would be appropriate. In Reparative dentin is produced in the pulp cham- this case, a fixed bridge prosthesis would be a very ber at the lesion site in response to insults such Prosthodontics long span over this edentulous gap, which re- as caries, dental procedures, or attrition. quires sufficient abutments to adequately place The smear layer is any debris, calcific in na- force and support. In addition, the severe bone ture, produced by reduction or instrumentation resorption would compromise result of both an of dentin, enamel, or cementum. FPD and a Maryland bridge. Because of all of the above, a removable partial denture would be 69. The correct answer is C. Dentin has un- the best choice in this instance. evenly distributed intertubular and peritubular dentin and is therefore heterogeneous in nature. 64. The correct answer is A. An anterior posterior Enamel, however, is almost entirely homoge- palatal bar connector can be used in almost nous in nature. any maxillary partial denture to attain symme- try. A palatal palate is a thin broad connector 70. The correct answer is A. Primers contain hy- that may be used for simple edentulous area and drophilic monomers that have an affinity for full palatal coverage. A palatal horseshoe-shaped the exposed collagen fibril arrangement and hy- connector should only be used when a large non- drophobic properties for the copolymerization operable torus exists. The use of a single palatal with adhesive resins. bar is limited to tooth borne restorations for bi- lateral short-span edentulous areas. 71. The correct answer is B. The hydrodynamic the- ory proposes that when a stimulus causes the slow 65. The correct answer is B. Contact of the tip of fluid movement to become more rapid, nerve the tongue with the anterior palate and lingual endings in the pulp are deformed, and this is surfaces of the maxillary teeth produce “T” and interpreted as pain. Stimuli such as tooth prepa- “D” sounds. “P” and “B” sounds are produced by ration, air drying, and application of cold have the lips only. “Th” sounds are produced by the been suggested as causes of sudden, rapid move- tongue protruding slightly between the maxillary ment. and mandibular anterior teeth. 72. The correct answer is C. Premolars retain 66. The correct answer is D. Calcium chloride ac- sealants best because the patient is older and celerates the setting time. Rosin facilitates the it is easier to get better isolation for moisture speed of the reaction and results in a smoother, control to place the sealant. It is harder to more homogenous product. Resinous balsam is achieve adequate isolation on primary teeth and used to increase flow and improve mixing prop- in younger patients. Permanent second molars erties. Oil of cloves is used in preference to are also noted for difficulty of isolation and mois- eugenol because it reduces burning sensation ture control. of the soft tissues of the mouth. 50 Chapter 2: Operative Dentistry and Prosthodontics

73. The correct answer is A. Permanent first mo- 78. The correct answer is B. The labial reduction lars erupt when the patient is young. It is more should be in two planes (cervical and facial difficult to get optimal isolation for restoration planes). A minimum of 1.2 mm in required to placement at this young age. Sealants are rarely create a satisfactory appearance, but 1.5 is pre- placed on primary first molars and primary sec- ferred. The incisal reduction for a PMF crown is ond molars. 1.5 to 2.0 mm. The lingual reduction is 1 mm in excursion if contacts are in metal, but 1.5 mm of 74. The correct answer is C. The size of the bur reduction is necessary if contact is in porcelain. does not matter in this situation. Heat and pres- sure are the most common causes of pulp dam- 79. The correct answer is C. A lingual chamfer age during cavity preparation. The longer the is insufficient in this situation. A full porcelain cutting time, the more heat is generated, and restoration will require a shoulder margin. Ade- the more damage can be caused. Greater tooth quate reduction is imperative for an all-ceramic reduction would lead a cavity preparation to be restoration. The facial surface requires a two- closer in proximity to the pulp. Proximity to the plane anatomical reduction of 1-mm depth. The pulp chamber increases the potential for pulpal incisal edge requires 1.5-mm reduction in inter- damage. cuspation and excursive movements. The lin- gual surface of the tooth must be reduced 1.0 75. The correct answer is B. Class II classification is mm to provide adequate relief for porcelain cov-

Prosthodontics for interproximal lesions on posterior teeth only. erage Cavity Classes I, V, and VI can involve both an- terior and posterior teeth. 80. The correct answer is D. Age is not an abso- Operative Dentistry and lute contraindication for implant placement or 76. The correct answer is D. A shoulder margin restoration. Patients with acute or terminal ill- is unnecessary for a full metal crown; a cham- ness are not candidates for implant placement. fer margin is sufficient. The use of depth cuts Patients with unrealistic expectations, poor mo- along anatomical planes of the occlusal surface tivation, or poor oral hygiene are not sound can- will ensure adequate reduction and prevent over- didates for implants placement. Elective dental reduction. The final preparation of the occlusal work on pregnant patients is not indicated. Im- surface should follow a similar anatomical con- plant placement in patients who have received tour of the crown prior to reduction. Axial re- IV bisphosphonates or head and neck radiation duction should be approximately 6 degrees and is relatively contraindicated because of the risks should be slightly convergent to provide the max- of bisphosphonate-related necrosis and osteora- imum retention for the restoration. Increased dionecrosis, respectively. taper results in excessive reduction of tooth struc- ture. The use of PFM or partial ceramic cov- 81. The correct answer is B. The extent of caries erage in nonesthetic areas will preserve tooth is important because that is all that needs to be structure. removed without removing sound tooth struc- ture. Removing sound tooth structure is unnec- 77. The correct answer is B. The recommended di- essary when performing a composite restoration, mensions for a complete cast crown are a mini- and doing so would not produce the most mum of 1-mm clearance on nonfunctional cusps optimal preparation. A composite restoration and 1.5 mm on functional cusps. The functional does not require mechanical retention. The cusp bevel must be at a flatter angle than the cus- preparation does not correspond with the tooth pal plane to provide adequate reduction, which shade. should be placed at about 45 degrees to the long axis of the tooth. A chamfer margin on a full 82. The correct answer is D. The polymer matrix metal crown allows for a 0.5-mm metal thick- is the phase to which the other ingredients get ness, which provides adequate strength for the added. The filler particles are or restoration. glass that improves the physical properties of the Answers: 73-91 51

matrix. The coupling agent assists in the adhe- properties than those in dentin; the mineral con- sion of the matrix to the filler particles. The ini- tent in dentin is relatively sparse and arranged in tiator is what allows the polymerization of the a more tubular structure, allowing for more rapid composite to be activated. progression of caries (answer D).

83. The correct answer is D. 1.5 mm is the proper 87. The correct answer is A. Actinomyces visco- amount of reduction because 0.3 to 0.5 tooth sus is the most common cariogenic bacteria in structure reduction is needed to accommodate root surface or smooth surface caries (answer the metal and an additional 1.0 mm is needed for B). Streptococcus sanguis is the earliest organ- the porcelain. 0.8 and 1.0 mm are not enough re- ism found in dental plaque (answer C). A good duction to accommodate both metal and porce- way to remember common cariogenic bacte- prtv etsr and Dentistry Operative lain. 2.5 mm is too much reduction. ria: SALIVA: S. mutans, sanguis. A. viscosus. Lactobacilli. Veillonella. A. naeslundii. 84. The correct answer is A. Gold is a strong ma- Prosthodontics terial. As opposed to other materials, gold rarely 88. The correct answer is C. The four zones of in- fractures. When used in restorations, it has the cipient lesions are listed above in correct order greatest functional longevity. Gold esthetics are A to D. These characterize separate zones seen inferior to other materials as gold does not look in a sectioned enamel lesion. The translucent like natural teeth. The impression material used zone is the deepest zone and is named for its ab- is dictated by the operator, not the restoration. sent or composition-less appearance seen under Gold has a wear rate that is actually similar to polarized light. The dark zone represents rem- enamel. Gold, unlike porcelain, will not cause ineralization and is named after its characteristic wear on the opposing teeth to be accelerated inability to transit polarized light. The body zone is the largest zone and represents a demineral- 85. The correct answer is B. It is important to know ization phase. The surface zone is the outermost which restoration is going to be the best for the zone and seems to be unaffected by the caries. specific tooth, and depending on the amount of remaining tooth structure, if a post will be 89. The correct answer is D. Rampant caries are necessary. The canal configuration is not going rapidly progressing and widespread caries. They to affect the final outcome of how the tooth is are often the result of histological disadvan- restored. The function of the restored tooth is tages, poor hygiene, drug abuse, radiation, high not one of the first things to be considered. sugar diets, or disadvantaged saliva. They present acutely (A), are most often associated with pain 86. The correct answer is C. Frank Caries describe (E), are seen in children (C), and result in large caries that have progressed just into the denti- cavitation because of its deep and narrow pre- noenamel junction (DEJ). Carious lesions oc- sentation. cur when a mass of bacteria adhere to the tooth surface forming a dental plaque. The plaque bac- 90. The correct answer is E. While all of the above teria feast on refined carbohydrates, metabolize are known risk factors for caries, A and B are the sugars, and produce acidic byproducts. The high-risk factors, B and C are moderate-risk fac- acid lowers the pH of the plaque adherent to the tors. Other high-risk factors for caries include tooth; when the pH drops to 5.5, demineraliza- visible plaque, frequent between meals snacks, tion begins to take place; this is known as the inadequate saliva, and dental appliances. critical pH (answer A). Early lesions are capa- Moderate-risk factors include: interproximal ble of remineralization or arrest if the pH is in enamel lesions, other white spots or discol- favor of building and mineral content like fluo- orations, and recreational drug use. ride is abundant. Incipient caries describe caries that have not progressed farther than enamel; 91. The correct answers is D. Autogenous graft. these are reversible and capable of remineraliza- Xenograft and alloplastic grafts are only osteo- tion (answer B). Caries in enamel have different conductive and will not grow new bone without 52 Chapter 2: Operative Dentistry and Prosthodontics

the presence of surrounding cells, such as os- 94. The correct answer is D. The first statement is teoblasts, that will form bone. They only act as a FALSE and the second statement is TRUE. scaffold to which new bone can be added onto. Defects in color vision primarily affect 8% Allografts can be osteoconductive and os- to 10% of the MALE population, for example, teoinductive, meaning that they act as a scaf- about 10% of all males are color blind. fold onto which new bone forms and their bone The second statement is TRUE. Achroma- matrix contains inducing agents that cause new tism is the complete lack of hue sensitivity. bone to form. Dichromatism is sensitivity to two of the primary Only autogenous bone is osteoconductive, os- hues. Anomalous trichromatism is sensitivity to teoinductive, and osteogenic, meaning in addi- all three primary hues with a deficiency or ab- tion to acting as a scaffold for new bone to adhere normality of one of the three primary pigments to and having a matrix with inducing agents to in the retinal cones. stimulate new bone, it also has the capability of forming new bone on its own, without the aid of 95. The correct answer is C. Ovate pontic. surrounding growth factors. Sanitary (hygienic) pontics have the best ac- cess for oral hygiene but usually provide you with 92. The correct answer is B. Fentanyl. the worst esthetic outcomes once the gingival tis- Triazolam, lorazepam, and diazepam are all sues have healed. benzodiazepines that can be delivered by an oral Conical pontics typically provide for good ac-

Prosthodontics route and are routinely used for oral conscious cess for oral hygiene but again tend to have poor sedation. esthetic outcomes with the final prosthesis Fentanyl is considered to be an IV drug used Modified ridge lap pontics are moderately Operative Dentistry and in IV sedation easy to clean and can provide good esthetics. Ovate pontics: generally accepted to provide 93. The correct answer is A. Both the statement and the best esthetics for the final prosthesis because the reason are correct and related. of its support of the gingival architecture around Takenstraight from advantages of single tooth the extraction site following healing. implants section of the “Contemporary Implant Dentistry” textbook by Dr. Carl Misch, 3rd edi- tion. CHAPTER 3 Oral and Maxillofacial Surgery and Pain Control

53 54 Chapter 3: Oral and Maxillofacial Surgery and Pain Control

QUESTIONS

1. Which of the following statements about the D. All the above temporomandibular joint (TMJ) is/are true? E. None of the above I. It is a synovial joint II. It is a ginglymoarthrodial joint 4. Regarding common bacteria found in oral III. It allows for translational (gliding) move- infections, the following statements are true ment except: IV. Posterior TMJ dislocations are more com- I. Aerobic bacteria compose roughly 75% of mon than anterior dislocations oral infections II. Anaerobic bacteria involved in oral infec- A. I, II, and III tions include Neisseria spp. and the Bac- B. II and IV teroides spp. C. IV only III. Odontogenic infections involve the pre- D. All the above dominance of one bacterium E. None of the above IV. Alpha hemolytic Streptococcus spp. are Gram-positive cocci in pairs 2. Trigeminal neuralgia presents with: I. Generally presents bilaterally A. I, II, and III II. Common complaints include stabbing, B. II and IV burning, shocking pain C. IV only III. It is uncommon for the pain to be triggered D. All the above by benign events E. None of the above IV. Carbamazepine is more effective than amitriptyline 5. A patient presents to your clinic with a diagno- sis of an abscess. Which characteristics support A. I, II, and III your diagnosis? B. II and IV I. History of pain and swelling for 3 days C. IV only II. History of pain and swelling for 1 week D. All the above III. Predominance of anaerobic bacteria with E. None of the above a well-circumscribed border IV. Predominance of aerobic bacteria with a 3. An elderly patient with a history of atrial fib- diffuse border rillation and recent cardiac stent placement presents to your clinic for mandibular lingual A. I, II, and III bony exostosis removal. The procedure occurs B. II and IV

Oral and Maxillofacial without incident, but you are concerned be- C. IV only

Surgery and Pain Control cause of the potential immediate postoperative D. All the above complications that can occur in this situation. E. None of the above I. You are worried about infection II. You are worried about a pathologic frac- 6. Necrotizing fasciitis: ture. I. Is a slowly progressing infection of the skin III. You are worried that the patient will have and fascia difficulty eating II. Is a rapidly progressing infection of the skin IV. You are worried about postoperative and muscle bleeding III. Has a low mortality rate with administra- tion of antibiotics A. I, II, and III IV. Is a rapidly progressing infection of the skin B. II and IV and fascia C. IV only Questions: 1-12 55

A. I, II, and III A. I, II, and III B. II and IV B. II and IV C. IV only C. IV only D. All the above D. All the above E. None of the above E. None of the above

7. A patient presents for dental implant evalua- 10. What is the correct sequence for Guedel’s tion. You do an oral examination and notice stages of anesthesia? that he is completely edentulous and has mini- A. Amnesia and analgesia; delirium and ex- mal alveolar bone. You consider bone augmen- citement; surgical anesthesia; medullary tation surgery. The following are true, except: paralysis I. An allograft is composed of tissue obtained B. Delirium and excitement; amnesia and from an animal such as a cow or pigs analgesia; surgical anesthesia; medullary II. Augmentation of an atrophic mandible paralysis prior to implant placement is indicated in C. Amnesia and analgesia; surgical anesthesia; a patient with less than 10 mm of bone delirium and excitement; medullary paral- height ysis III. Smokers, alcoholics, and uncontrolled di- D. Amnesia and analgesia; delirium and abetics are an absolute contraindication to excitement; medullary paralysis; surgical dental implants anesthesia IV. An autograft is an excellent choice for bone augmentation. Donor sites include 11. You are performing full-mouth extractions for the mandibular ramus, tibial plateau, an- a patient when he starts to complain of tinni- terior, and/or posterior hip tus, circumoral numbness, and appear drowsy. A. I, II, and III What do you do next? B. II and IV Control Pain and Surgery

A. You continue with the procedure because Maxillofacial and Oral C. IV only the patient is likely anxious D. All the above B. Discontinue the procedure and send the pa- E. None of the above tient home C. The patient is probably feeling some pain 8. Clark’s rule of pediatric dosing of local anes- and is anxious. The patient needs more lo- thetics is: cal anesthetic A. Used to calculate the therapeutic dose D. Discontinue the procedure; monitor the pa- B. Defined by (weight of child in kg/150) tient for local anesthetic toxicity (maximum adult therapeutic dose) C. Not applicable to local anesthetics that have 12. Regarding the mechanism of action for local epinephrine anesthetics, the following are true, except: D. Only calculated by a complex algorithm A. An inactive nerve cell has a resting mem- E. None of the above brane potential of −50 to −70 mV B. During the repolarization of the peripheral 9. Which of the following relate to the esters group nerve, potassium ions pass out of the cell to local anesthetics? restore the resting cell to resting membrane I. The esters local anesthetics include novo- potential caine, procaine, and bupivacaine C. During depolarization, there is an influx of II. All are metabolized by the microsomal Ca2+ P450 enzymes of the liver D. Nonionized molecules enter through nerve III. Esters are metabolized by the liver cell membranes more readily than ionized IV. Esters are metabolized by pseudocho- molecules linesterase in the plasma 56 Chapter 3: Oral and Maxillofacial Surgery and Pain Control

13. Infection causes tissue to be more resistant to 17. The proper treatment for an avulsed tooth that the anesthetic effect of local anesthetics be- occurred less than 25 minutes ago would in- cause: clude which of the following? A. This results in a decrease in the ionized A. Minimize contact with root and replant pri- form of the local anesthetic; in turn, the lo- mary teeth cal anesthetic has difficulty penetrating the B. Placement of a rigid splint for 4 to 6 weeks cell membrane of the neuron C. Replant and splint for 7 to 10 days B. Infected tissue has many bacteria that cre- D. Immediate pulpectomy and reimplantation ate buffers that hinder the diffusion of local anesthetic toward the nerve cell 18. Nonrigid splints are recommended for all of C. This results in an increase of the nonionized the following situations except: form of the local anesthetic; in turn, the lo- A. Subluxation of a tooth or a group of teeth cal anesthetic effect is diminished because B. Avulsion. Tooth outside of socket less than of an increased refractory period because 30 minutes of the abundance of local anesthetic in the C. Nonrigid splints should be made with thin- neuron ner, 28 gauge, rather than thicker, 20 gauge, D. This results in an increase in the ionized stainless steel wire form of the local anesthetic at the expense D. Mid-root fracture of the unionized form of the local anesthetic 19. In decreasing order of frequency, what is the 14. What is the amount of local anesthetic in a car- correct sequence for mandible fractures? tridge with 3% local anesthetic and a volume of 2 mL? A. Symphyseal/parasymphyseal>body> angle>ramus>coronoid>alveolar A. 60 mg/mL B. Symphyseal/parasymphyseal>condyle> B. 20 mg /mL alveolar>coronoid C. 30 mg C. Symphyseal/parasymphyseal>angle> D. 60 mg body>condyle D. Symphyseal/parasymphyseal>ramus> 15. What is the amount of epinephrine (mg) in a angle>coronoid 5-mL vial labeled with 1:100,000 epinephrine? A. 0.01 20. Signs and symptoms of alveolar osteitis include B. 0.05 all of the following except: C. 0.1 A. Pain commencing 2 to 5 days following the D. 0.5 extraction B. Accompanied by a foul taste or smell Oral and Maxillofacial 16. In methemoglobinemia, all of the following are

Surgery and Pain Control C. It is a self-limiting condition that will im- true except: prove and resolve with time A. Hemoglobin is oxidized to methemoglobin D. Treat with antibiotics and pain medications B. Methemoglobin cannot bind and carry oxygen 21. A dentist removes a maxillary premolar and no- C. Methemoglobin levels can only be detected tices that she has created an oroantral commu- by arterial blood draws nication, which is 4 mm in diameter. Which of D. Patients who have glucose-6-phosphate de- the following is an appropriate treatment? hydrogenase (G6PD) are more sensitive to A. No surgical treatment excessive doses of local anesthetics and an- B. Figure-of-eight sutures to maintain in- tibiotics such as prilocaine and dapsone tegrity of the blood clot within the socket C. Requires a buccal fat pad flap advancement D. Tongue flap advancement Questions: 13-28 57

22. The following drugs act as an agonist on mu (␮) tent with a dental infection of an upper left receptors in the central nervous system (CNS) molar for the past 7 days. Her signs and symp- for analgesia, except: toms suggest cavernous sinus thrombosis. This complication involves which of the following A. Fentanyl cranial nerves? B. Ibuprofen C. Meperidine A. IV, V, VII, IX D. Tramadol B. II, III, IV, X C. IV, VI, VII, VIII 23. Benzodiazepines have all the following effects D. III, IV, V, VI except: 27. A 24-year-old male presents to the emergency A. Anxiolytic department very toxic and ill appearing. He is B. Amnestic having difficulty breathing, is febrile, and is C. Analgesic notably swollen below the jaw line. He com- D. Anticonvulsant plains of pain and difficulty swallowing and is 24. You are a first responder to a man found unre- also noted to be drooling. His mother mentions sponsive, lying supine with a pulse at the scene that he has had a toothache for the past 2 weeks. of an unwitnessed motor vehicle accident. He Initial management would include: is an unrestrained driver and was recovered A. Administration of antibiotics 40 yards from his car. His leg appears broken, B. Administration of intravenous fluids and and there are no obvious open wounds. You pain medications initiate cardiopulmonary resuscitation (CPR). C. Establishment and maintenance of an ade- How do you establish an airway? quate airway A. Head tilt D. Incise and drain the most fluctuant area of the swelling

B. Placement of an oropharyngeal airway Control Pain and Surgery

C. Perform a cricothyroidomy E. Extract offending tooth Maxillofacial and Oral D. Jaw thrust 28. A 43-year-old male presents with trismus and 25. You are observing an orthognathic surgery in pain localized to the right preauricular region. the operating room, when the healthy patient, On examination, he has limited range in jaw who is under general anesthesia, begins to de- movements, and it is noted that he has a right velop hypercarbia, rigidity, elevated tempera- posterior open bite. CT maxillofacial shows a ture, and tachycardia. The anesthetist has his calcified growth of his right condyle, which you hands full and asks you to get the medication suspect to be an osteochondroma. You plan that is used to treat this condition. to resect the right condyle and condylar neck. What muscle would be required to be stripped, A. This condition is benzodiazepine overdose according to your plan, to remove the condylar and should be treated with flumenazil segment successfully? B. This condition is an opioid overdose and should be treated with naloxone A. Temporalis C. This condition is neuroleptic malignant B. Masseter syndrome and should be treated with C. Lateral pterygoid dantrolene D. Medial pterygoid D. This condition is malignant hyperthermia and should be treated with dantrolene

26. A 60-year-old female comes to the emergency department with significant left periorbital edema, ptosis, mydriasis, and a history consis- 58 Chapter 3: Oral and Maxillofacial Surgery and Pain Control

29. You are a member of the craniofacial team at A. Relaxation the local hospital. As the team dentist, you are B. Increase in visual acuity approached by the mother of an 8-day-old baby C. Decreased blood pressure over time girl. She is curious about the timing of cleft D. Increased respiratory rate over time palate repair. What is the appropriate timing of treatment? 32. Which local anesthesia is associated with caus- ing methemoglobinemia? A. Rule of tens. When the patient is at least 10 lbs, has 10 mg /dL of hemoglobin, and is at A. Lidocaine least 10 weeks of age B. Bupivicaine B. The palate repair is usually performed be- C. Prilocaine tween 9 and 18 months of age D. Procaine C. As soon as possible. Usually between the first week of life and 6 months 33. After giving two cartridges of 2% lidocaine to D. It only has to be repaired if the baby devel- anesthetize the inferior alveolar nerve, the pa- ops problems with speech tient develops facial drooping on the side of the injection. This complication occurred because 30. A 35-year-old male presents to the emergency the needle was directed: department after an incident with his girlfriend. A. Too far lateral He was stabbed in the face with a box cut- B. Too far inferior ter, and upon your arrival, his vitals are sta- C. Too deep ble and the bleeding is controlled with direct D. Too far superior pressure. You assess the patient and note that portions of the parotid gland are herniating out 34. Which factor determines local anesthetic onset of the wound. Your examination would include time? which of the following? A. Lipid solubility of the anesthetic I. Facial nerve function B. The percentage of protein binding II. Mandibular nerve function C. The presence of a vasoconstrictor III. Maxillary nerve function D. The degree of ionization IV. Vitality of his teeth V. Patency of Stensen’s duct 35. A 5-month-old mentally retarded patient VI. Patency of Wharton’s duct presents to your office and on clinical examina- VII. Patency of nasolacrimal gland tion, you note brachycephaly, midface hypopla- A. I, II, IV sia, hypertelorism, and syndactyly of hands and B. VI, VII feet. These findings are consistent with the di- C. I, II, III, IV, V, VI agnosis of what syndrome?

Oral and Maxillofacial D. I, V A. Apert syndrome Surgery and Pain Control E. III, VII B. Treacher–Collins syndrome C. Eagle syndrome 31. A 10-year-old generally healthy male presents D. Beckwith–Wiedemann syndrome for endodontic treatment of #30, he is ex- tremely anxious and in acute pain. Because of 36. What is the initial force direction with forceps the acute nature of the disease process, it has placed on a tooth for a successful extraction? taken numerous carpules of local anesthesia to obtain any relief. Not soon after anesthesia is A. Apical achieved, the patient begins to become con- B. Buccal fused and systolic blood pressure increases by C. Palatal 50 torr from baseline. What other signs would D. Occlusally you also see the patient exhibit? Questions: 29-45 59

37. A 24-year-old male with history of 1-ppd smok- A. 225 ing and occasional alcohol use presents to your B. 375 office 4 days after a routine extraction of #30 C. 525 with complaints of severe throbbing pain and D. 750 a foul taste, but on examination, there is no drainage or erythema of the area. What is the 42. Which of the following suture materials is clas- most likely diagnosis? sified as nonresorbable? A. Inferior alveolar nerve injury A. Vicryl B. Alveolar osteitis B. Chromic gut C. Infection C. Plain gut D. Osteomyelitis D. Nylon

38. A 64-year-old female with history of atrial fibril- 43. What is the minimal safe distance for implant lation controlled medically with labetalol and placement from the mental foramen? taking coumadin presents for extraction of re- A. 2 mm maining lower teeth. Her current INR is 1.5. B. 0.5 mm What is the most appropriate method of treat- C. 2 cm ing this patient? D. 1 mm A. Remove the teeth and use local hemostatic E. 3 mm materials and sutures B. Remove teeth and start saline rinses day of 44. A 64-year-old female presents for removal of #2 procedure and #3 under local anesthesia. Her past medical C. Stop coumadin for 1 day and then remove history is significant for type II diabetes and a teeth history of rheumatic heart disease as a child. D. Stop coumadin for 3 days and then remove Medications: none, Allergies: Pen VK. What is the appropriate method of proceeding? Control Pain and Surgery teeth Maxillofacial and Oral A. Administer adequate local anesthesia and 39. To remove maxillary tori, the necessary nerve proceed with the removal of #2 and #3 blocks include which of the following? B. Give 2-g amoxicillin PO 1 hour prior to the A. Incisive procedure and remove #2 and #3 B. Incisive and posterior superior alveolar C. Give 1-g ampicillin IV 30 minutes prior to C. Greater palatine and incisive the procedure and remove #2 and #3 D. Greater palatine and posterior superior D. Give 600-mg clindamycin PO 1 hour prior alveolar to the removal of #2 and #3

40. The most difficult mandibular third molar im- 45. What are the most common teeth that may paction position is classified as: cause referred pain to the ear? A. Mesioangular A. Maxillary central incisors B. Soft tissue B. Mandibular central incisors C. Distoangular C. Mandibular molars D. Erupted D. Maxillary premolars E. Maxillary molars 41. A 75-kg male presents for extraction of several teeth. He has a noncontributory medical his- tory. What is the maximum amount of 2% li- docaine with 1:100 K epi patient can safely be administered (in mg)? 60 Chapter 3: Oral and Maxillofacial Surgery and Pain Control

46. A 54-year-old male presents with a 4-day history 51. Fentanyl, a synthetic derivative of morphine, of tooth pain and swelling of his right mandible. has the relative potency: Trismus of 10 mm, odynophagia, and dyspha- A. 10 times that of morphine gia findings are noted. PMH: HTN, DM II B. Equal to morphine Meds: HCTZ All: Pen VK. What is the nec- C. Half of that of morphine essary treatment for this patient? D. 100 times that of morphine A. Incision and drainage B. Antibiotic therapy with pen VK 52. Which of the following describes a graft mate- C. Antibiotic therapy with clindamycin rial derived from genetically unrelated mem- D. Incision and drainage followed by IV an- bers of the same species? tibiotics A. Autogenous B. Allogeneic 47. A 63-year-old male presents with pain and C. Xenogeneic swelling in the floor of mouth and superior D. Synthetic neck. PMH: Hyperlipidemia Meds: Lipitor All: NKDA. He notes that the pain is more evident 53. A 34-year-old female presents with a non- prior to eating. What is the most likely diagnosis painful 0.5 × 0.5 cm lesion on the underside based on this history and radiograph? of her tongue that was noted on routine exam- A. Sialolith ination (see photo). She states that it has been B. Lingual artery atherosclerosis present for approximately 1 year. She has no C. Normal findings on occlusal radiograph of past medical history and takes no medications. mandible What is the treatment for this lesion? D. Supernumerary tooth A. Incisional biopsy B. Excisional biopsy 48. A 30-year-old male presents for routine ex- C. Nothing amination, when a lesion is noted in the D. Resection left mandible. PMH: none, Meds: none, All: NKDA. What is the most likely diagnosis? 54. The panoramic radiograph depicts what type of A. Odontogenic keratocyst fracture in the left mandible? B. Pindborg tumor A. Pathologic fracture C. Calcifying odontogenic cyst B. Comminuted fracture D. Squamous cell carcinoma C. Closed fracture D. No fracture present 49. What medication reverses the effects of benzo- diazepines? 55. Bell’s palsy is caused by injury to what cranial Oral and Maxillofacial nerve? Surgery and Pain Control A. Narcan B. Naloxone A. CN V C. Flumazenil B. CN VII D. Neostigmine C. CN XII E. Decadron D. CN XI

50. Which is the first muscle incised in a floor of 56. Which peripheral nerve fibers are the thickest? mouth lowering procedure? A. Motor fibers A. Geniohyoid B. Pain fibers B. Genioglossus C. Autonomic fibers C. Buccinator D. Proprioception fibers D. Mylohyoid E. Touch fibers Questions: 46-64 61

57. What is the primary effect of local anesthetics the glenoid fossa, with medial rotation of the on the heart? condylar neck; the left parasymphyseal fracture is nondisplaced. Which of the following is the A. Decreases the maximum rate of depolariza- most appropriate management? tion of Purkinje fibers and myocardium B. They have dose-dependent negative in- A. Intermaxillary fixation for 4 weeks followed otropic effect on myocardium by passive/active physical therapy C. Depress spontaneous pacemaker activity in B. Closed reduction and intermaxillary fixa- the sinus node tion for 2 weeks and passive/active physical D. Blocks the intracellular release of calcium therapy C. Open reduction and internal fixation of 58. Which of the following would be most likely the right subcondylar fracture followed by encountered in the preoperative examination intermaxillary fixation for 6 weeks and of a patient with Treacher–Collins syndrome? passive/active physical therapy thereafter A. Mental retardation D. Open reduction and internal fixation of B. Pulmonary hypertension both the right subcondylar fracture and the C. Cleft palate left parasymphyseal fracture followed by in- D. Glossoptosis termaxillary fixation for 2 weeks and pas- sive/active physical therapy thereafter 59. Extraction wounds heal by which method? 63. A 40-year-old African-American female with a A. Primary intention chief complaint of needing “a cleaning.” B. Secondary intention C. Tertiary intention PMH: type II diabetes (diet controlled) D. None of the above Meds: none All: IV contrast 60. A 25-year-old female presents with past history Multiple areas of radiolucent lesions are seen Control Pain and Surgery of biting her lower lip for several weeks and in the panoramic radiograph, but the patient Maxillofacial and Oral now presents with a lesion that increases and does have any symptoms of pain and on exam- decreases in size (Photo 30). PMH: asthma, ination has no bony expansion or lesions asso- Meds: Albuterol, All: NKDA. On the basis of ciated with the overlying mucosa. Next step in the symptoms and clinical presentation, what treatment includes: is the treatment of choice? A. Resection of the mandible A. NS rinses four times daily B. Aspiration biopsy alone B. Pen VK 500 mg every 6 hours for 7 days C. Aspiration and excisional biopsy C. Excisional biopsy D. No treatment indicated D. Incisional biopsy 64. A 50-year-old male presents to your office for 61. A 16-year-old male presents with an unerupted evaluation and is noted to have a radiolucency maxillary upper left canine (Figure). He has no in his left posterior mandible underneath the past medical history. What is the diagnosis? inferior alveolar nerve and is asymptomatic. A. Mucous retention cyst The appropriate next step in treatment includes B. Antrolith which of the following? C. Condensing osteitis A. Resection of the mandibular angle D. Odontoma B. Aspiration biopsy C. Excisional biopsy 62. A 12-year-old girl sustains a traumatic right D. No treatment needed subcondylar fracture and a left parasymphy- seal fracture. Radiographs show that the right condylar head is displaced laterally out of 62 Chapter 3: Oral and Maxillofacial Surgery and Pain Control

65. On removal of #14, you have broken off a root 70. The mental nerve exits the mental foramen at tip. The tooth is missing a portion of its palatal which of the following sites? root. Unable to visualize the root tip, you take A. Below the canine halfway down the a radiograph and note its position to be: mandible A. In the infratemporal fossa B. Below the first premolar halfway down the B. In the nerve canal mandible C. In the maxillary sinus C. Below the first premolar, directed posteri- D. In the nasal floor orly D. Below the second premolar halfway down 66. The nerve injury classified as neurotmesis is the mandible described as? E. Below the lateral incisor A. A contusion of the nerve 71. During a LeFort I maxillary osteotomy, the B. Complete loss of nerve continuity tooth most likely to be injured by a low os- C. Loss of epineural sheath but intact axon teotomy line is the: D. A stretching of the nerve A. Central incisor 67. Dental elevators are used for which of the fol- B. Cuspid lowing purposes? C. First bicuspid A. To retract the gingival crest tissue D. Lateral incisor B. To reflect a full mucoperiosteal flap E. Second bicuspid C. To engage the tooth apical to the cemen- 72. The cervical branch of the facial nerve is tran- toenamel junction sected during parotidectomy. Which of the fol- D. To engage the tooth coronal to the cemen- lowing functions is most likely to be affected? toenamel junction A. Forward flexion of the neck 68. The levator veli palatini muscle is primarily in- B. Lateral flexion of the neck nervated by the: C. Pursing the lips A. V2 branch of the trigeminal (V) nerve D. Retraction and depression of the angle of B. V3 branch of the trigeminal (V) nerve the mouth C. Facial (VII) nerve E. Upward and downward movement of the D. Vagus (X) nerve hyoid bone E. Hypoglossal (XII) nerve 73. Which of the following glandular structures re- 69. A right unilateral cleft lip is most likely to result ceives innervation from the auriculotemporal from incomplete union between which of the nerve? Oral and Maxillofacial following prominences? Surgery and Pain Control A. Lacrimal A. The frontonasal prominence to the lateral B. Meibomian nasal prominence C. Parotid B. The frontonasal prominence to the medial D. Sublingual nasal prominence E. Submandibular C. The lateral nasal prominence to the maxil- 74. A 40-year-old woman develops Frey syndrome lary prominence after undergoing parotidectomy. The most D. The lateral nasal prominence to the medial likely cause is injury to branches of which of nasal prominence the following nerves? E. The medial nasal prominence to the max- illary nasal prominence A. Auriculotemporal B. Facial C. Great auricular Questions: 65-81 63

D. Posterior auricular D. Between the superficial and deep lobes of E. Vagus the parotid gland E. Within the buccal space 75. Which tooth cannot be removed by only rota- tion motion? 81. A 62-year-old man is 6 years status post right hemimaxillectomy for the treatment of a be- A. Maxillary central incisor nign tumor. He is otherwise in good physical B. Maxillary lateral incisor shape, with a noncontributory medical history. C. Maxillary canine He complains that when he speaks and eats, his D. Maxillary first premolar maxillary obturator becomes loose. He has had 76. Which of the following is the most common site a new obturator fabricated less than 4 months of squamous cell carcinoma of the oral cavity? ago, and the new one is just as ill fitting as his previous ones. He asks if there is anything A. Buccal mucosa that can be done to improve the retention of B. Floor of the mouth his obturator. Physical and radiographic exam- C. Mandibular gingivae inations (including computerized tomography D. Palate (CT) dentascan and panorex radiographs) show E. Tongue absence of the right hemimaxilla and complete maxillary edentulism with inadequate maxil- 77. In adults, the normal range of vertical mandibu- lary bone for placement of standard endosseous lar opening is from: implants. Which of the following is the best A. 20 to 30 mm possible treatment plan to improve retention B. 30 to 40 mm of this patient’s obturator? C. 40 to 50 mm A. Placement of bilateral zygomaticus im- D. 50 to 60 mm plants

B. Soft reline of the existing obturator Control Pain and Surgery

78. Each of the following is a muscle of mastication Maxillofacial and Oral C. Nothing can be done to improve this pa- except the: tient’s condition A. Buccinators D. Take new impressions to fabricate a new B. Lateral pterygoid obturator C. Masseter E. Placement of standard endosseous implants D. Medial pterygoid and a prescription for long-term prophylac- E. Temporalis tic oral antibiotics

79. Which of the following structures is a branch of the mandibular division of the trigeminal nerve? A. Infraorbital nerve B. Lingual nerve C. Nasopalatine nerve D. Posterosuperior alveolar nerve E. Posterosuperior nasal nerve

80. Stensen’s duct can be found at which of the following anatomic sites? A. Mandibular angle B. Preauricular border C. At the zygomatic arch 64 Chapter 3: Oral and Maxillofacial Surgery and Pain Control

82. A 15-year-old boy presents to your office with cleft site is deficient in bone. A decision is made his mother 2 days after sustaining facial trauma to graft the alveolar cleft site with autogenous after falling while riding his bicycle. He was iliac marrow. How long should the consolida- referred to you from the local hospital emer- tion period be for the alveolar cleft bone graft gency department, where he had a full workup. before the placement of an endosseous implant He suffered no bony injury; however, his per- to replace the missing lateral incisor? manent maxillary right central incisor (#8) has A. 4 weeks a small fracture of the crown (enamel only). B. 8 months The maxillary left central incisor (#9) was com- C. 4 months pletely avulsed and accidentally discarded by D. 1 year the emergency department team. The maxil- E. 2 years lary right central incisor is treated with good cosmetic result by smoothing down of the sharp 84. A 45-year-old female patient has three im- edges. The mother asks you if placement of an plant supported crowns in her right posterior endosseous implant is a possible treatment of mandible with three individual, nonconnected her child’s missing tooth. Which of the follow- implants replacing missing teeth #28, #29, and ing statements best answers the mother’s ques- #30. On clinical evaluation, the patient reports tion? pain in her jaw during percussion of the crown A. Because this patient is under 18, he is not a on the most mesial implant. Periapical radio- candidate for endosseous implant replace- graphs show a generalized radiolucency sur- ment of his missing tooth. He should wait rounding the implant in the #28 position, and at least 3 years prior to implant placement just less than one-half of the surrounding bone B. This patient is currently a candidate for en- has resorbed around the implant in the #29 dosseous implant replacement of his miss- position. No radiographic abnormalities are as- ing tooth sociated with the implant in the #30 position. C. Placement of endosseous implants in chil- There is no discharge, pus, erythema, or dis- dren will cause increased and faster erosion coloration of the peri-implant gingiva. Which of alveolar ridge height of the following implants meet the criteria for D. Placement of endosseous implants in chil- failure? dren is inherently unstable and will not A. #28 prevent supraeruption of opposing teeth or B. #29 help maintain a stable occlusion C. #30 E. Endosseous implants have not been re- D. #28 and #29 ported to migrate when placed in children. E. #28 and #30 The patient should wait until he is 21 to F. #29 and #30 have implants placed Oral and Maxillofacial

Surgery and Pain Control 85. A 45-year-old female patient has presented for 83. A 21-year-old female patient with a right-side restoration of a previously placed endosseous unilateral cleft lip and palate is referred by an implant in the right maxillary canine tooth #6 orthodontist to an oral surgeon for an alveo- position. She has a full maxillary and mandibu- lar cleft bone grafting procedure. Clinical ex- lar dentition, with the exception of tooth #6, amination reveals a missing permanent right which was extracted secondary to a failed root maxillary lateral incisor. All other permanent canal treatment. The patient has a slight distoc- teeth have erupted. Orthodontic appliances are clusion relationship between her maxillary and aligning the arches, and space has been main- mandibular dental arches, and the canine clas- tained for the replacement of the lateral incisor. sification directly correlates to the Angle molar A panorex radiograph confirms that all perma- relationship. Bilaterally, the mesiobuccal cusp nent teeth have erupted, no supernumerary or of her maxillary first permanent molars articu- impacted teeth are evident, and the alveolar lates mesial to the mesiobuccal groove of the Questions: 82-88 65

mandibular first permanent molar. Currently, B. This patient may undergo surgical place- her occlusion is balanced, stable, and repro- ment of endosseous implants using stan- ducible. She is without TMJ pain or symptoms. dard implant surgery techniques without How would you restore the canine tooth im- modification plant to preserve a functional and balanced oc- C. This patient should undergo a 2-month clusion? “drug holiday” prior to endosseous implant A. The maxillary canine tip should oppose the placement embrasure between the mandibular canine D. This patient should undergo a 2-month and first premolar “drug holiday” immediately after en- B. The maxillary canine tip should oc- dosseous implant placement clude distal to the embrasure between the E. This patient should undergo a 1-year “drug mandibular canine and first premolar holiday” prior to endosseous implant place- C. The maxillary canine tip should be flat- ment and is restricted from ever taking tened and taken out of occlusion bisphosphonate medications again in her D. The maxillary canine tip should occlude lifetime mesial to the embrasure between the 88. An 18-year-old male patient with anhidrotic ec- mandibular canine and first premolar todermal dysplasia presents to your office for E. It does not matter where the maxillary evaluation and treatment planning. He displays canine tip ends up. Normal wear of the hypodontia, hypohidrosis, and hypotrichosis. restoration will result in a functional and The patient’s existing dentition consists of bilat- stable occlusion eral maxillary canines, bilateral maxillary first 86. A 27-year-old female patient presents for a cos- molars, bilateral maxillary primary molars, bi- metic restoration of a previously placed en- lateral mandibular primary molars, bilateral dosseous implant in the left maxillary central mandibular first molars, and bilateral mandibu- lar second molars. All other permanent max- incisor position. Which of the following de- Control Pain and Surgery scriptive terms describes relationship in which illary and mandibular teeth are missing. His Maxillofacial and Oral the incisal edges of the maxillary teeth follow alveolar ridges are extremely narrow and are the curvature of the border of the lower lip in concave lingually. Which of the following will a posed, social smile? provide the most accurate information regard- ing the patient’s alveolar height and width for A. Smile arc implant treatment planning? B. Dental display C. Esthetic line of the dentition D. Curve of Spee E. Centric occlusion

87. A 65-year-old Caucasian female patient presents to your office because she is inter- ested in replacing her missing mandibular teeth with endosseous implants. Her medical history is significant for osteoporosis, for which she has taken alendronate 10 mg/d for the past 2 years. She does not report any other comorbidities A. Computed tomography with dental CT and has never taken any other medications. software (i.e., dentascan) Which of the follow statements is the most ap- B. Orthopantogram propriate treatment philosophy for this patient? C. Full-mouth periapical radiographs A. This patient’s history of bisphosphonate use D. Occlusal radiographs disqualifies her as a potential endosseous E. Bitewing radiographs implant recipient 66 Chapter 3: Oral and Maxillofacial Surgery and Pain Control

89. A 43-year-old female patient is scheduled for A. The patient should be administered clin- placement of endosseous implants to replace damycin 600 mg PO 1 hour prior to the her missing maxillary central incisor teeth #8 dental procedure, and clindamycin 300 mg and #9. Her medical history is significant for 1 hour after the dental procedure mitral valve prolapse with mild regurgitation. B. The patient should be administered amox- She tells you that she is allergic to peni- icillin 2 g PO 1 hour prior to the dental cillin. Which antimicrobial prophylaxis regi- procedure men should be followed for this patient? C. The patient should be administered cephra- dine 2 g PO 1 hour prior to the dental pro- A. The patient should be administered clin- cedure damycin 600 mg PO 1 hour prior to the D. The patient should be administered clin- dental procedure damycin 600 mg PO 1 hour prior to the B. The patient should be administered clin- dental procedure damycin 600 mg PO 1 hour prior to the E. No antimicrobial prophylaxis is indicated dental procedure, and clindamycin 300 mg at this time 1 hour after the dental procedure C. The patient should be administered amox- 92. Large postablative palatomaxillary defects are icillin 2 g PO 1 hour prior to the dental often treated prosthodontically with the fabri- procedure cation of a maxillary obturator. Which of the D. The patient should be administered van- following forces are properly matched with the comycin 2 g PO 1 hour prior to the dental destabilizing effect most likely encountered procedure with prosthetic obturation of the defect? E. Antimicrobial prophylaxis is not indicated at this time A. Cantilever forces causing the obturator to tip toward the defect 90. According to the original two-stage Brane- B. Rotational forces causing the obturator to mark method of endosseous implant place- spin uncontrollably with function ment, which of the following describes the C. Tensile forces causing the obturator to recommended healing time for maxillary and stretch mandibular bone in patients without bone D. Compressive forces causing compaction of grafts, soft bone, or oral parafunctional habits? the obturator

A. Maxilla: 5- to 6-month healing period. 93. Which of the following definitions describes Mandible: 3- to 4-month healing period the process by which new bone formation oc- B. Maxilla: 1- to 2-month healing period. curs from osteoprogenitor cells that are present Mandible: 1- to 2-month healing period in the graft, survive the transplant, and prolif- C. Maxilla: 1- to 2-month healing period. erate and differentiate to osteoblasts?

Oral and Maxillofacial Mandible: 3- to 4-month healing period

Surgery and Pain Control D. Maxilla: 3- to 4-month healing period. A. Osteoinduction Mandible: 1- to 2-month healing period B. Osteogenesis E. Maxilla: 5- to 6-month healing period. C. Osteoconduction Mandible: 1- to 2-month healing period D. Osseointegration E. Angiogenesis 91. A 57-year-old male patient is scheduled for sur- gical endosseous implant placement. He has 94. Which of the following is considered the “gold a history of right prosthetic hip total joint re- standard” in bone grafting? placement 4 months ago. He reports that he A. Autologous bone has a penicillin allergy. Which of the following B. Allograft antibiotic prophylaxis regimens is most appro- C. Xenograft priate for this patient? D. Alloplast E. Bone morphogenetic protein (BMP) Questions: 89-96 67

95. A 30-year-old female patient presents to your of- Panoramic and periapical radiographic ex- fice for reevaluation 1 week after surgical place- aminations of the posterior right mandible fail ment of a single endosseous implant in her pos- to show any abnormality or disruption of the terior right mandible. The implant was placed inferior alveolar canal. in a long-standing edentulous region to replace He is followed closely with serial sensory her missing second molar tooth (#31). The pa- examinations. During a repeat examination 4 tient complains of total “numbness” in her right weeks later, he reports improvement of symp- lip and chin that has persisted since the surgery. toms and returning sensation to his right men- A complete sensory evaluation is performed tal region. Sensory mapping at this examination including testing of the patient’s responses shows a decreased size of the area of altered sen- to light touch, pain, temperature, directional sation. movements, and two-point discrimination. Vi- tality testing of all mandibular teeth is also performed. Comparisons are made with the normal, contralateral side. Upon examination and mapping of the dis- tribution of her right mandibular nerve, an area of right-side mental region skin and oral mu- cosa displayed marked decreased perception of stimulation. Which of the following nerve in- jury terminologies best describes the sensations experienced by the patient? B A. Hypoesthesia B. Dysesthesia Which of the following of nerve injury classi- C. Hyperesthesia fications best describes the sensations experi- D. Allodynia Control Pain and Surgery

enced by the patient? Maxillofacial and Oral E. Hyperalgesia F. Hyperpathia A. Neurapraxia G. Anesthesia B. Axonotmesis C. Neurotmesis 96. A 33-year-old male patient is diagnosed with D. Hyperpathia a closed nerve injury 1 week after endosseous E. Allodynia implant placement in his posterior right mandible. Upon examination and mapping of the distribution of his right mandibular nerve, the following areas of hypoesthesia are noted.

A 68 Chapter 3: Oral and Maxillofacial Surgery and Pain Control

97. A 60-year-old male patient with chronic renal as there is less risk to the mother during this failure secondary to long-standing type I dia- time betes mellitus presents requesting dental im- D. Because of the presence of an impacted, plant replacement of his missing mandibu- ankylosed tooth, this patient is not a can- lar molar teeth. He reports that he undergoes didate for dental implant treatment in that hemodialysis three times weekly. What special region considerations must be taken in the care of this patient? 99. A 29-year-old female patient presents to your of- fice 1-week status postsurgical dental implant A. Consult with the patient’s physician regard- placement in her posterior right maxilla. She ing his current medical status. No antibiotic reports that since her last visit she has found prophylaxis is currently recommended for out that she is pregnant. She informs you that end-stage renal disease patients undergo- she is still in mild-to-moderate pain from the ing hemodialysis. Oral surgery procedures surgical procedure. The patient asks you what should be done on the same day as dialysis she can use for the pain. She has no known al- B. There is no need to consult with the pa- lergies to medications. Which of the following tient’s physician. He can undergo surgical pain medications is best suited for this pregnant dental implant placement without special patient? considerations C. Consult with the patient’s physician regard- A. None ing his current medical status, antibiotic B. Acetaminophen prophylaxis recommendations, and dialy- C. Aspirin sis schedule. It is recommended that oral D. Codeine surgery procedures be performed the day E. Hydromorphone after dialysis. The patient will require an- tibiotic prophylaxis 100. Which of the following diseases would con- D. This patient is not a candidate for surgical traindicate the placement of dental implants implant therapy in an elderly patient? A. None 98. A 31-year-old female patient is 9 weeks into her B. Dementia first pregnancy. She reports that while biting C. Hypertension into an apple, her retained left primary molar D. Hypercholesterolemia tooth “fell out.” There are no clinical signs of E. Atrial fibrillation infection. She is interested in dental implant re- placement of her missing maxillary left first pre- 101. When treatment planning the placement of os- molar tooth. A review of a panorex radiograph seointegrated endosseous implants, which of taken 1 year ago shows an impacted left maxil- the following is true? Oral and Maxillofacial

Surgery and Pain Control lary first premolar tooth apical to a retained de- ciduous molar. The patient would like to know A. As a general rule, always use the smallest when she will undergo dental implant replace- possible implant diameter ment of her missing tooth. You tell the patient: B. As a general rule, the diameter of the im- plant does not affect stability A. Elective surgical procedures such as im- C. As a general rule, always use the longest plantology should be delayed until after length implant available, even if there is po- delivery tential impingement of vital structures such B. This patient can undergo dental implant as the inferior alveolar nerve treatment immediately as there is no risk to D. As a general rule, always use the largest pos- the fetus at this time sible implant diameter C. It is best to wait until the third trimester to undergo surgical dental implant treatment 102. When treatment planning implant placement, what is the minimal mesiodistal distance from Questions: 97-109 69

the implant shoulder to the adjacent tooth at C. 60 minutes bone level? D. 120 minutes

A. A minimal distance of 1.5 mm is recom- 107. You are about to provide supplemental oxygen mended between the implant shoulder and via nasal cannula to a patient sitting in your den- the adjacent tooth at bone level tal chair with chronic obstructive pulmonary B. A minimal distance of 1.0 mm is recom- disease (COPD). Although the patient is not mended between the implant shoulder and in any significant distress, the patient’s pulse the adjacent tooth at bone level oximeter reading is 80%, which is lower than C. A minimal distance of 0.5 mm is recom- her usual value. Which of the following state- mended between the implant shoulder and ments is correct? the adjacent tooth at bone level D. A minimal distance of 3.0 mm is recom- A. Oxygen supplementation is never indicated mended between the implant shoulder and in patients with COPD the adjacent tooth at bone level B. One can provide whatever concentration of supplemental oxygen is required to main- 103. How much bone is recommended at a mini- tain oxygen saturation above 90%, as mea- mum on the facial and palatal surfaces of the sured by pulse oximeter implant to support the ideal orofacial implant C. One should administer 100% oxygen by position? non-rebreather mask to all COPD patients exhibiting even the slightest signs of dysp- A. 0.25 mm nea. B. 0.5 mm D. A nasal cannula is not an appropriate C. 1.0 mm method for administration of supplemental D. 2.0 mm oxygen in this case 104. What is the maximum torque that should not be

108. A man is diagnosed as having extremely in- Control Pain and Surgery

exceeded when inserting the implant fixture? Maxillofacial and Oral flamed retrodiscal tissue as a result of func- A. 25 Ncm tional loading of an internally deranged TMJ. B. 35 Ncm All conservative treatment has failed to resolve C. 45 Ncm the problem. What is the next course of action? D. 50 Ncm A. Arthrocentesis 105. Traditionally, what comprises the “Second B. Arthroscopic surgery Stage” implant surgical procedure? C. Prosthetic restoration D. Occlusal adjustment A. Placement of the healing abutment E. Orthognathic surgery B. Torque testing of the implant cover screw C. Placement of the closure screw 109. A patient presents to your office with symptoms D. Placement of the implant fixture that include stabbing, shocking pain lasting a few seconds to minutes. He says that the pain 106. You are about to provide supplemental oxygen is provoked by cold and only occurs on the left to a patient in your dental chair scheduled for side. What is the most likely diagnosis for this implant placement under IV sedation. The reg- patient? ulator gauge on the portable E-cylinder of oxy- gen reads 1,000 psi. You plan to use a flow of A. Myofascial pain dysfunction syndrome 10 L/min. Using this flow rate, approximately B. Trigeminal neuralgia how much time is remaining in this oxygen E- C. Osteoarthritis cylinder? D. TMJ ankylosis E. Disk displacement A. 30 minutes B. 45 minutes 70 Chapter 3: Oral and Maxillofacial Surgery and Pain Control

110. A 150-lb man presents for extraction of #30. C. Occlusal adjustment The patient has an abscess and severe swelling D. Extraction in the area around the tooth to be extracted. You have already administered six carpules of 112. A 55-year-old woman presents for extraction of 2% Lidocaine with 1:100,000 epinephrine to #28. She intends to have an implant placed the patient. How many more carpules can you in the extraction site. The woman has a medi- give without putting the patient at risk? cal history of controlled type II diabetes. She is currently taking oral bisphosphonates for mild A. 6 osteoporosis. #28 has a small abscess and +1 B. 8 mobility. There is 9 mm of vertical bone height C. 10 in the area. The abscess is drained and the pa- D. 14 tient notes immediate relief upon extraction. What, if any, is a contraindication for immedi- 111. A patient presents with tooth pain after being ate placement of an implant at the extraction punched in the mouth. Upon examination, the site? tooth causing the pain is fractured at the coro- nal portion of the root. There is no mobility or A. Not enough bone height displacement. What is the proper treatment for B. Bisphosphonate therapy this patient? C. Active infection D. Osteoporosis A. Removal of coronal segment and pulpec- E. type II diabetes tomy B. Rigid splint Oral and Maxillofacial Surgery and Pain Control Answers: 1-5 71

ANSWERS

1. The correct answer is A. The temporoman- 3. The correct answer is C. An elderly patient dibular joint is classified as a ginglymoarthro- with a history of atrial fibrillation and recent dial joint that consists of a translational (gliding) cardiac stent placement should alert you to the movement and a rotational (hinging) move- likely use of anticoagulants such as coumadin. ment. It is also a synovial joint. There are supe- International normalized ratio (INR) between rior and inferior synovial cavities that provide the ranges of 2 to 3 is therapeutic for patients an articular surface for the movement of diagnosed with atrial fibrillation. Recent car- the articular disk and the mandibular fossa, diac stent placements also require anticoagula- of the temporal bone, and also against the tion to prevent clot formation on newly placed mandibular condyle. stents. The removal of mandibular lingual bony Anterior dislocations are more common exostosis can lead to the development of post- than posterior TMJ dislocations because of operative bleeding, hematoma formation, and the posterior boundary that is the posterior airway compromise. A temporary acrylic stent glenoid process. The articular tubercle is rela- could be fabricated to assist in hemostasis at tively less prominent than the posterior glenoid the surgical site to prevent the development of process; in turn, less energy is required to cause a sublingual/submandibular hematoma. the mandibular condyle to be dislocated and Infection should be a concern; however, entrapped by the articular tubercle. Anterior it would not be suspected in the immediate dislocations are more common than posterior postoperative course. Although the removal of dislocations. Posterior TMJ dislocations are not extensive bony tori may predispose one to a as common as anterior dislocations. pathologic fracture, the potential for postoper- ative bleeding is more likely in this case presen- 2. The correct answer is B. Trigeminal neu- tation. Difficulty eating would not be suspected ugr n anControl Pain and Surgery

ralgia is commonly described as stabbing, in the immediate postoperative course. Maxillofacial and Oral burning, shocking pain that lasts for seconds to minutes. It generally presents unilaterally. 4. The correct answer is C. Alpha hemolytic Medications that are used to treat trigemi- Streptococcus spp. are aerobic Gram-positive nal neuralgia include: antiepileptics, tricyclic cocci in pairs. Aerobic bacteria roughly com- antidepressants, muscle relaxants, and ben- pose 25% of oral infections. Neisseria spp. are zodiazepines. Carbamazepine, tegretol, is a anaerobic Gram-negative cocci. Bacteroides is preferred treatment for trigeminal neuralgia. an anaerobic Gram-negative rod. Odontogenic Amitriptyline can be tried, but the success rate infections are from indigenous bacteria and is lower. Other antiepileptics that can be used polymicrobial in nature, composed of both aer- include: lamotrigine, gabapentin, and pheny- obic and anaerobic bacteria. toin. Amitriptyline is a tricyclic antidepressant that is also used for neuropathic pain. 5. The correct answer is A. An abscess is charac- Carbamazepine is an antiepileptic drug that terized by a disease process of chronic duration. has a mechanism of sodium channel blockade. Abscesses generally develop from cellulitic pro- It is a versatile drug that has multiple indica- cesses and form 2 to 3 days after the onset of tions such as for seizures disorders, mood disor- symptoms of pain and swelling. They may per- ders, and also for neuropathic pain. sist for long periods of time since purulence Trigeminal neuralgia is generally unilateral that is not drained will remain until the body and is commonly reported by patients to be removes the necrotic tissue, which can be a triggered by relatively benign events such as lengthy process dependent on the extent of the light touch, thermal stimulation, and wind. abscess formation. As the infection progresses into a mature abscess, the purulence produced 72 Chapter 3: Oral and Maxillofacial Surgery and Pain Control

will compress the surrounding tissue to create the deficiency of alveolar bone and can be de- granulation tissue, which helps to contain the fined by Parel’s classification system. infection from spreading. This containment of the infection produces a well-circumscribed 8. The correct answer is E. Clark’s rule is (weight border. of child in lb/150) (maximum adult dose in Abscesses are composed of predominantly mg). This rule helps define the maximum pedi- anaerobic bacteria with a well-circumscribed atric dose and is not affected by the presence of border. Cellulitic processes are composed of a epinephrine. It is a simple formula that should predominance of aerobic bacteria with a diffuse be calculated prior to pediatric procedures. border. There is a lack of a well-circumscribed border in cellulitis because of the lack of pu- 9. The correct answer is C. Esters are metabo- rulence. Abscesses are predominantly anaero- lized by pseudocholinesterase in the plasma. bic bacteria with a well-circumscribed border. Amides are metabolized by the microsomal They have a chronic duration, pain is gener- P450 enzymes of the liver via N-dealkylation ally localized, and the borders are well circum- and hydroxylation. Ester local anesthetics in- scribed. On palpation, they are fluctuant be- clude procaine, benzocaine, and tetracaine. cause of the presence of purulence. Amide local anesthetics generally have the let- ter “I” plus “-caine” in their drug names (li- 6. The correct answer is C. Necrotizing fasciitis docaine, mepivacaine, bupivacaine). All local is a rapidly progressing infection of the skin and anesthetics comprised a lipophilic aromatic fascia. It has a high mortality rate (30%–50%) ring linked to a hydrophilic amino group. The even with antibiotic administration. In many link is either an ester or an amide bond, and this cases, surgical debridement is required for ad- determines its classification. Children and the equate treatment and can lead to disfiguring elderly are at greatest risk for local anesthetic results. toxicity. Necrotizing fasciitis is a rapidly progress- ing infection of the skin and fascia. Etiol- 10. The correct answer is A. Amnesia and analge- ogy is attributed to Group A strep, mixed sia; delirium and excitement; surgical anesthe- aerobic–anaerobic bacteria or Clostridium per- sia; medullary paralysis. fringens. It has a high mortality rate (30%–50%) Amnesia and analgesia is the stage in which and without prompt surgical debridement of induction occurs; preservation of protective re- necrotic/infected tissue can lead to death and flexes, and this ends with loss of consciousness. significant disfigurement. The muscle that is af- Delirium and excitement is the second stage fected by necrotizing fasciitis occurs secondary and consists of the presentation of involuntary to the progression via the skin and fascia. movements, obtunded reflexes, and ends with the onset of total anesthesia. Nausea and vom-

Oral and Maxillofacial 7. The correct answer is C. An autograft is the iting are common in this stage. Surgical anes- Surgery and Pain Control transplantation of tissue from one region to an- thesia is the third stage and is composed of other within the same individual. Donor sites three planes—light, medium (ideal for surgical include the mandible (chin, ramus, maxillary procedures), and deep. Medullary paralysis is tuberosity), iliac crest (anterior/posterior hip), very deep anesthesia with loss of cardiovascular tibia, and even the cranium. An allograft is the function and imminent death. transplantation of tissue from one individual to another genetically nonidentical individual of 11. The correct answer is D. The patient is demon- the same species. Xenograft is the transplan- strating initial signs of local anesthetic toxicity. tation of tissue from one species to another. These include circumoral numbness, tachycar- Augmentation of an atrophic mandible prior dia, hypertension, drowsiness, confusion, tinni- to implant placement is indicated in a patient tus, and metallic taste. with less than 8 mm of bone height. The eden- Full-mouth extractions generally require a tulous maxilla can be classified depending on lot of local anesthetic, and at times if one is Answers: 6-16 73

not careful, the threshold for toxicity may be which is not effective at traversing a neuron’s exceeded. The assumption of local anesthetic cell membrane. There is a decrease in the non- toxicity should be ruled out before considering ionized form of the local anesthetic. A shift to- a diagnosis of anxiety. Drowsiness is unlikely ward more ionized local anesthetic molecules with anxious patients. prevents the local anesthetic from exerting their Although it is correct to discontinue the pro- effect on the sodium channels. cedure, one should monitor the patient for pro- gression of symptoms. Treatment for the initial 14. The correct answer is D. The percentage of signs of toxicity are supportive measures; how- local anesthetic is measured in grams per 100 ever, if later signs of CNS compromise (e.g., mL (i.e., 1% is 1 g/100 mL or 1,000 mg/100 mL tremor, hallucinations) appear, in conjunction or 10 mg/mL). Therefore, the concentration with cardiovascular collapse, immediate refer- in a 3% solution is 30 mg/mL. In 2 ml of this ral to the hospital is required. solution, there would be 60 mg of local anes- Patients who are in pain are unlikely to ap- thetic. Answer choice A is incorrect because pear drowsy. Most will wince or complain of 60 mg/mL is the concentration, and not the inadequate anesthesia. The presentation with amount, in a cartridge with 6% local anesthetic. CNS and cardiovascular signs/symptoms sug- In answer choice B, there is 30 mg/mL in a gest local anesthetic toxicity. 3% local anesthetic. Since there is a volume of 2 mL, the total amount of local anesthetic is 12. The correct answer is C. During the depo- 60 mg. Answer choice C is incorrect because larization of the neuron, there is an influx of this is the amount of local anesthetic in 1 mL sodium. Local anesthetics act by increasing the of a 3% local anesthetic. threshold for the opening of sodium channels. An inactive nerve cell has a resting mem- 15. The correct answer is B. 1:100,000 means brane potential of −50 to −70 mV. The cy- 1 g/100,000 mL. Converting into milligrams, a toplasm of a resting nerve cell has a high 1:100,000 solution contains 1,000 mg/100,000 ugr n anControl Pain and Surgery

concentration of potassium ions and a low mL or 0.01 mg/mL. The concentration 0.01 Maxillofacial and Oral concentration of sodium ions. This gradient is mg/mL in the vial is then multiplied by 5 mL maintained by a Na/K ion pump. (the volume in the vial) for a total of 0.05-mg During the repolarization of the peripheral epinephrine. If the vial had a volume of 1 mL, nerve, potassium ions pass out of the cell to re- 0.01 mg would be correct. store the resting cell to resting membrane po- tential. 16. The correct answer is C. Methemoglobin lev- Non-ionized molecules enter through nerve els may be obtained by both arterial and ve- cell membranes more readily than ionized nous blood draws. Hemoglobin is oxidized molecules. The local anesthetics lipophilic aro- to methemoglobin. Elevated levels of methe- matic ring facilitates passage through the nerve moglobin in the blood are caused when the sheath and membrane. mechanisms that defend against oxidative stress within the red blood cell are overwhelmed and + 13. The correct answer is D. The acidic environ- the oxygen carrying ferrous ion (Fe2 ) of the ment of infected tissue has a tendency to shift heme group of the hemoglobin molecule is + the balance regarding a local anesthetic’s equi- oxidized to the ferric state (Fe3 ). This con- librium toward more of the ionized form of the verts hemoglobin to methemoglobin, which local anesthetic. In turn, there is a decrease in is a non-oxygen binding form of hemoglobin the non-ionized form of local anesthetic and that binds a water molecule instead of oxy- less penetration of local anesthetic through the gen. Patients with G6PD deficiency lack the cell membrane of the neuron. Non-ionized production of sufficient amounts of NADPH, molecules readily diffuse across cell mem- an oxidizing agent. Normally, the produc- branes. Infected tissue is acidic; thereby, read- tion of methemoglobin is reduced back to a ily ionizing local anesthetic to an ionized form, normal state by protective enzymes such as 74 Chapter 3: Oral and Maxillofacial Surgery and Pain Control

cytochrome b5 reductase (major pathway), and 20. The correct answer is D. Alveolar osteitis (dry to a lesser extent NADPH methemoglobin re- socket) is a self-limiting condition that will im- ductase, glutathione, and ascorbic acid. prove and resolve with time. Antibiotics are not indicated without the signs of infection. It gen- 17. The correct answer is C. Replant and splint the erally presents after extraction of teeth. Alveo- avulsed tooth for 7 to 10 days (3–4 weeks for a lar osteitis is a painful phenomenon that most tooth with an immature/open apex). Primary commonly occurs a few days following the re- teeth are not reimplanted because this action moval of mandibular (lower) wisdom teeth.It could jeopardize the underlying adult denti- occurs when the blood clot within the healing tion. A space maintainer may be required if loss tooth extraction site is disrupted. In rare cases, of dental arch space is an issue. Avulsed teeth the removal of the upper wisdom teeth can also should not be splinted for greater than 7 to 10 result in possible alveolar osteitis. days, except in the case of an avulsed tooth with Signs of alveolar osteitis include worsening, an immature apex (3–4 weeks). Rigid splint throbbing pain 2 to 5 days after extraction of a placement is recommended for alveolar pro- tooth. Radiation of pain from the socket up and cess fractures, but not isolated tooth avulsions. down the head and neck. Fetid odor and bad Minimal contact with a tooth, especially the taste are common complaints. root, should be practiced to limit any loss of Management is palliative with interventions PDL. Immediate pulpectomy is generally re- that decrease pain during an episode of dry served for root fractures where less than one- socket. These treatments consists of a gentle third of the root is involved. rinsing of the inflamed socket followed by the direct placement within the socket of a seda- 18. The correct answer is D. With dental root tive dressing, which soothes the inflamed bone fractures of the apical or mid-root, the tooth for a period of time and promotes tissue growth. should be reimplanted and a rigid splint placed This is usually done without anesthesia. The ac- for 2 to 3 months. Subluxation is trauma to tive ingredients in these sedative dressings usu- a tooth that causes mobility without displace- ally include natural substances like zinc oxide, ment from the tooth socket. Avulsion of a tooth eugenol, and oil of cloves. Additional analgesics from a socket less than 30 minutes should be are sometimes prescribed. placed in a nonrigid splint after reimplanta- tion. Compared with rigid splints, this will have 21. The correct answer is B. Oroantral commu- less chance of causing ankylosis. Thinner wire nications 2 to 6 mm in size can be repaired will allow for physiologic dental movement, with a figure-of-eight suture to maintain the whereas thicker wire will act more like a rigid integrity of the blood clot within the socket. splint and predispose tooth to ankylosis. Additional measures such as the placement of gelfoam can further assist in blood clot stabil-

Oral and Maxillofacial 19. The correct answer is B. The quoted frequen- ity and formation. Oroantral communications Surgery and Pain Control cies of mandible fractures are: Symphyseal/ less than 2 mm can be monitored. No acute sur- parasymphyseal 22%; condyle 24.5%; alveolar gical treatment required. Oroantral communi- 3.1%; coronoid 1.3%; body 16%; angle 24.5%; cations greater than 7 mm should be repaired ramus 1.7%. Fractures of the condyle will re- with flap closure such as a buccal fat pad flap ad- sult in deviation on opening to the fractured vancement. A tongue advancement flap should side because of unopposed muscle action of the be reserved for situations where failure of local- contralateral medial pterygoid muscle. There- ized advancement flaps. fore, fracture of the right condyle will cause deviation of the mandible to the right upon 22. The correct answer is D. Ibuprofen is a nons- mouth opening. When considering the entire teroidal anti-inflammatory drug. It is believed facial complex, nasal bone fractures are the to work through inhibition of cyclooxygenase most common facial fracture. (COX), thus inhibiting prostaglandin synthe- sis. Fentanyl is a synthetic primary ␮ agonist Answers: 17-26 75

opioid, currently the most widely used syn- triggered by halogenated general anesthetics thetic opioid analgesic worldwide, with a pri- and/or succinylcholine. Susceptibility to ma- mary potency approximately 81 times that of lignant hyperthermia is often inherited as an morphine. Tramadol is a monoamine uptake autosomal dominant disorder. The potential for inhibitor and centrally acting analgesic, used malignant hyperthermia is caused in a large for treating moderate-to-severe pain. It is a syn- proportion (50%–70%) of cases by a mutation thetic agent, and it appears to have actions at the of the ryanodine receptor (type 1), located on ␮-opioid receptor as well as the noradrenergic the sarcoplasmic reticulum (SR), the organelle and serotonergic systems. Meperidine acts as an within skeletal muscle cells that store calcium. agonist at the ␮-opioid receptor. In addition to It results from the massive release of calcium its strong opioidergic and anticholinergic ef- from the sarcoplasmic reticulum into the skele- fects, it has local anesthetic activity related to tal muscle cell to cause persistent muscle con- its interactions with sodium ion channels. traction. Signs and symptoms include rigidity, fever, tachycardia, metabolic acidosis, hyper- 23. The correct answer is C. Benzodiazepines do carbia, and hypoxia. Considering that the pa- not act as an analgesic. Additional medication tient is ventilated and monitored by anesthesia, is required locally or systemically to produce overdose is generally not a concern and can be an analgesic effect. Benzodiazepines enhance controlled symptomatically. the binding of GABA to the GABA receptor Neuroleptic malignant syndrome (NMS) is complex; in turn, binding increases the fre- a life-threatening neurological disorder most of- quency of chloride channel opening to cause ten caused by an adverse reaction to neurolep- less neuronal stimulation. They can cause an- tic or antipsychotic drugs. It generally presents terograde amnesia and are recommended as with muscle rigidity, fever, autonomic instabil- an anticonvulsant for the treatment of status ity, and cognitive changes such as delirium and epilepticus. is associated with elevated creatine phosphoki-

nase (CPK). The setting and history of this pre- Control Pain and Surgery

24. The correct answer is D. A jaw thrust is the sentation is unlikely to be caused by NMS. Maxillofacial and Oral least traumatic manipulation of the head and neck to establish an airway when no adjunctive 26. The correct answer is D. The cavernous measures are readily available. One should be sinus exists in the middle cranial fossa and concerned of a cervical spine fracture and min- receives tributaries from the surrounding du- imal manipulation of the neck should be prac- ral sinuses and venous plexuses from the face. ticed. A head tilt would not be advised since the These veins are valveless and are susceptible fact this man was found ejected from his vehi- to retrograde flow; in turn, this can be a path- cle, unresponsive, and with a fractured leg, it way for septic emboli from facial infectious to is very likely that he may have a fractured neck travel into the cavernous sinus. Patients gener- as well. Placement of an oropharyngeal airway ally have sinusitis or a midface infection for may be conducted; however, the first attempt 5 to 10 days. The cavernous sinus contains to establish an airway should be a jaw thrust. If cranial nerves III, IV, V (V1, V2), and VI, in an airway cannot be obtained with a jaw thrust, addition to the internal carotid artery and as- one should use an oropharyngeal airway to as- sociated sympathetic nerve branches. Lateral sist in establishing an airway. A cervical collar gaze palsy (isolated cranial nerve VI) is usually should be placed to limit unnecessary manipu- seen first since CN VI lies freely within the si- lation while placing the oropharyngeal airway. nus in contrast to CN III and IV that lie within A cricothyrotomy should be reserved for cases the lateral walls of the sinus. Without effective in which more conservative measures are un- therapy, signs appear in the contralateral eye by successful. spreading through the communicating veins to the contralateral cavernous sinus. Eye swelling 25. The correct answer is D. This patient has begins as a unilateral process and spreads to malignant hyperthermia. It is believed to be the other eye within 24 to 48 hours via the 76 Chapter 3: Oral and Maxillofacial Surgery and Pain Control

intercavernous sinuses. This is pathognomonic greater wing of the sphenoid and an inferior for cavernous sinus thrombosis. head that originates from the lateral surface of the lateral pterygoid plate. These will insert into 27. The correct answer is C. Initial management the anterior portion of the condylar neck and should be the establishment and maintenance TMJ capsule. The other muscles are not specif- of an adequate airway is the sine qua non of ically located around the condyle or condylar therapy. Death is most likely from the acute neck. phase of cellulitis by airway obstruction. Lud- Temporalis: wig’s angina results from the involvement of Origin: floor of temporal fossa, deep surface infection/cellulitis of the bilateral submandibu- of temporal fascia; lar, sublingual, and submental spaces. In light Insertion: coronoid process, anterior border of signs and symptoms of an impending loss of of mandibular ramus. the patient’s airway (difficulty breathing, pain on swallow, uncontrolled secretions, edema Masseter: noted in the submandibular region), the acute Origin: superficial portion—anterior two- problems need to be addressed first; in turn, thirds of lower border of zygomatic arch; establishment of a secure airway is required as deep portion—medial surface of zygo- the initial management. Following the estab- matic arch. lishment of a secure airway, intravenous an- Insertion: lateral surface of ramus, coronoid tibiotics, and incision and drainage with the process, and angle of mandible. removal of the offending tooth/teeth is gener- Medial pterygoid: ally curative. Without an airway, the patient Origin: deep head—medial surface of lateral cannot ventilate. Antibiotics should be started pterygoid plate; pyramidal process of pala- upon establishment of an airway to treat the un- tine bone; superficial head—maxillary derlying infection. Generally, an empiric broad tuberosity. spectrum antibiotic is used to cover for Gram- Insertion: medial surface of ramus inferior positive and anaerobic bacteria. Administration to mandibular foramen. of intravenous fluids and pain medications are indicated since the patient is likely dehydrated 29. The correct answer is B. The palate repair is from his difficulty in swallowing and general usually performed between 9 and 18 months malaise and discomfort. Caution is used with of age to promote proper speech development. regard to pain control prior to a secure airway The babies’ first words are usually noted on since it could further depress the patient’s res- average at 12 months of age. Cleft lip repair piratory drive. Incision and drainage is an effec- is recommended to occur when the infant is tive treatment if the patient did not show signs at least 10 pounds in weight, has 10 mg/dL of of respiratory distress (difficulty breathing, pain hemoglobin, and is at least 10 weeks of age. The

Oral and Maxillofacial on swallow, uncontrolled secretions). This pro-

Surgery and Pain Control recommended period of 9 to 18 months is sug- cedure will likely follow the establishment of an gested because any further delay could affect airway to irrigate and debride the areas of infec- the patient’s speech development. Cleft palate tion. Drains are usually left in the drainage site repair attempts to prevent delays in speech de- to prevent reaccumulation of purulence. The velopment. extraction of the source of infection is gener- ally curative; however, with the extension of 30. The correct answer is D. The extra cranial the infection into multiple fascial spaces, fur- segment of the facial nerve exits from the sty- ther treatment with incision and drainage is re- lomastoid foramen to dive between the deep quired. and superficial layers of the parotid gland to branch and form the five branches of the sen- 28. The correct answer is C. The lateral ptery- sory segment of the facial nerve: temporal, goid is composed of a superior head that orig- zygomatic, buccal, marginal mandibular, and inates from the infratemporal surface of the Answers: 27-36 77

cervical branches. Injury to these branches 33. The correct answer is C. Having the needle di- will manifest as paralysis of the muscles they rected to high (superior) can cause anesthetic innervate. Damage to the temporal branch will to move through the sigmoid notch into the cause paralysis of the forehead and ptosis of the parotid gland, anesthetizing the facial nerve. ipsilateral side. An injection to inferior would cause lingual Also traversing within the parotid gland is nerve anesthesia but will miss the main branch Stensen’s duct that delivers salivary fluid to the of the IAN. Too far lateral will place the nee- oral cavity. The ampulla of the duct is gener- dle into the buccinator. Too deep a needle ally located buccal to the second maxillary mo- placement may cause complications such as lar. Expression of saliva upon manipulation of hematoma but will not likely cause CNVII the parotid gland demonstrates intact patency nerve dysfunction. of Stensen’s duct. Also, the duct may be cannu- lated with a lacrimal probe to assess for any per- 34. The correct answer is D. Answer choice D is forations visible through the external wound. correct because the closer the pKa (the pH at With respect to trauma, the mandibular which the ionized and unionized forms exist in nerve is associated with injury to the mandible. equal concentrations) of the local anesthetic is Local injury to an area of dentition or injury to to tissue pH, the more rapid the onset time. the ipsilateral mandibular nerve would lead to Answer choice A is incorrect; the lipid solubil- loss of vitality in teeth. ity affects the potency; the higher the solubil- Damage to the floor of the mouth or sub- ity, the greater the potency. Answer choice B mandibular gland/ducts would be associated is incorrect because the protein binding influ- with Wharton’s duct. Injury to the medial as- ences the duration of action not the timing of pect of the eye can damage the nasolacrimal onset. Answer choice C is incorrect because a duct. Damage to the maxillary nerve is associ- vasoconstrictor influences length of anesthetic ated with injuries to the midface and not the duration.

parotid area. Control Pain and Surgery

35. The correct answer is A. Apert syndrome is a Maxillofacial and Oral 31. The correct answer is C. The patient is ex- rare autosomal dominant disorder in patients hibiting signs of local anesthetic overdose. Tox- has the traits described above as well as relative icity occurs when too high levels enter the mandibular prognathism and cleft soft palate systemic blood stream because of either in- or uvula. Answer choice B is incorrect because travascular injection or a large dose. Answer Treacher–Collins syndrome is characterized choice A is wrong because the patient would by narrow, depressed cheeks, downward slant- become increasingly agitated. Answer choice ing palpebral fissures, and hypoplastic condy- B is incorrect because with LA toxicity visual lar and coronoid processes. Answer choice C, and auditory disturbances can occur, not im- Eagle syndrome, is incorrect because it is a cal- prove. Other neurologic signs and symptoms cification of the stylohyoid ligament and has no would include tremors, loss of consciousness, other craniofacial deformities associated with it. and seizures. Answer choice C is correct be- Beckwith–Wiedemann syndrome, answer D, cause after an initial chorionic villus sampling is not correct because it is a syndrome asso- (CVS) stimulation, including increase in heart ciated with gigantism, macroglossia, and vis- rate, blood pressure, and respiratory rate (an- ceromegaly. swer D), there is rapid fall of all of these param- eters. 36. The correct answer is A. The initial force is api- cally directed to wedge the crestal bone and also 32. The correct answer is C. Prilocaine is metabo- moves the fulcrum toward the apex of the tooth lized in the liver to O-toluidine, capable of ox- reducing possibility of root fracture. Buccal and idizing hemoglobin to methemoglobin. They palatal movements are only secondary to the will clinically appear cyanotic. It is reversed by initial apical force. Occlusal force is never used the administration of methylene blue IV. in the extraction process. 78 Chapter 3: Oral and Maxillofacial Surgery and Pain Control

37. The correct answer is B. Alveolar osteitis or dry 42. The correct answer is D. Nylon is a monofil- socket is delayed healing but is not associated ament nonresorbable suture material that will with infection (answer C) because of the lack need to be removed within 5 to 7 days of place- of the usual signs and symptoms of infection ment. Choices A, B, and C are incorrect be- including swelling, erythema, and fever. Most cause they are each types of resorbable suture common site for a dry socket is the mandible materials. Plain Gut resorbs quickly (approx- and can be associated with smokers. Answer imately 3–5 days), chromic gut holds tensile choice A is incorrect because IAN injury would strength for 7 to 14 days, and vicryl can hold present with neuropathic-type pain or paresthe- for up to 3 to 4 weeks. sia. Answer choice C is incorrect because of the symptoms and signs mentioned above. Answer 43. The correct answer is A. 2 mm is the recom- choice D is incorrect because the presentation mended distance from the mental foramen to of this chronic bony infection process would the inferior aspect of the implant. not present within the 3-day time frame. 44. The correct answer is D. 600 mg of clin- 38. The correct answer is A. Minor surgical proce- damycin PO 1 hour prior to the procedure is dures can be performed with an INR less than 3 the recommended dose and antibiotic for a pa- with control of bleeding achieved with hemo- tient who requires premedication and has an static packing and suture placement. Answer allergy to the class of penicillins. choice B is incorrect. Removing the teeth with- out sutures and starting rinses will precipitate 45. The correct answer is C. Mandibular molars bleeding. Answer choices C and D are incor- frequently refer pain to the ear because of cross rect because the INR is in the low range; if innervation of the inferior alveolar nerve. greater than 3, consulting the PCP and with- holding for 3 days would be appropriate. 46. The correct answer is D. The most appropriate treatment for a patient with a deep fascial space 39. The correct answer is C. Bilateral greater pala- infection is incision and drainage followed by tine and incisive nerve blocks to properly anes- antibiotics. thetize a patient for length-wise incision with anterior and posterior releases and reflection of 47. The correct answer is A. The radiograph and a full mucoperiosteal flap in the palate. Answer clinical correlation of increased pain prior to choice A is incorrect. This alone would not pro- salivation are pathognomonic for the diagno- vide anesthesia for the posterior palate. Answer sis of a sialolith (a calcification that is formed choices B and D are incorrect. The posterior within a salivary duct, most common being superior alveolar nerves do not innervate the found within the submandibular duct because palate. of it being a primary mucous secretions gland and anatomic position of the duct). Answer Oral and Maxillofacial

Surgery and Pain Control 40. The correct answer is C. Impactions in the choice B is incorrect, although calcifications of mandible have several classification systems; arteries can be noted on radiographic examina- the most common is positional relation. Dis- tion, this does not correlate with the pts present- toangular impactions in the mandible have ing symptoms. Answer C is incorrect because more bony coverage. these are not normal radiographic findings on an occlusal view of the anterior mandible. An- 41. The correct answer is C. The maximum swer D is incorrect because a supernumerary dosage of 2% xylocaine with 1:100 K epi is tooth would not be found outside of the bony 7 mg/kg; therefore, answers A and B would be confinements of the maxilla or mandible. acceptable dosages but not the max. Choice D is incorrect because it exceeds the 7 mg/kg 48. The correct answer is A. This radiograph de- limit. picts a radiolucent lesion in the left posterior mandible. The only pathology consistent with Answers: 37-60 79

this radiograph is an OKC; the others, a to the subcondylar and condylar regions in a COC and Pindborg, have radio-opaque find- dentate patient. ings within the radiolucent lesion, and in SCC one would see bony destruction. 55. The correct answer is B. Bell’s palsy is weak- ness or severance of the seventh cranial nerve 49. The correct answer is C. Flumazenil is a com- that controls motor function of the muscles of petitive benzodiazepine receptor antagonist at facial expression. CN V is involved in sensory binding sites in the central nervous system. An- innervation of the face and motor function of swers A and B are the same medication and are the muscles of mastication. CN XII controls used to reverse the effects of narcotics. Answer motor function of the tongue. CN IX controls D is incorrect because neostigmine is an acetyl- motor function of the trapezius and sternoclei- cholinesterase inhibitor. Decadron, answer E, domastoid. is incorrect because it is a steroid and has no effect on the mechanism of benzodiazepines. 56. The correct answer is A. Local anesthetics block nerve fibers at different rates. Thinner 50. The correct answer is D. The mylohyoid mus- fibers are easier to block than thicker fibers. cle is the most superficial muscle of the floor Accordingly, the blockade of peripheral nerves of the mouth followed by the genioglossus and usually follows the following order: (1) periph- finally by the geniohyoid. The buccinator is lo- eral vasodilation and skin temp, (2) pain and cated on the lateral aspect of the mandible. temperature, (3) proprioception, (4) touch and pressure sensation, and (5) motor paralysis. 51. The correct answer is D. Fentanyl is 100 times stronger than morphine. 57. The correct answer is A. All the answers are partially true. but the primary effect is decreas- 52. The correct answer is B. An allogeneic graft de- ing the maximum rate of depolarization in the scribes material derived from genetically unre- Purkinje fibers and myocardium. ugr n anControl Pain and Surgery

lated members of the same species. Autogeneic Maxillofacial and Oral is a material derived from the same person re- 58. The correct answer is C. Cleft palate is ceiving the graft. A xenogeneic graft is the one present in these patients approximately 30% derived from a different species than that into of the time. Mental retardation is uncom- which it is placed. Synthetic grafts are chemi- mon in patients with Treacher–Collins syn- cally made. drome. Pulmonary hypertension usually oc- curs in those patients who have chronic airway 53. The correct answer is B. An asymmetric raised obstruction. white lesion that persists longer than 2 weeks should be biopsied since the lesion is less than 59. The correct answer is B. Extraction sites heal 2 cm that appears clinically benign. Incisional by secondary intention, a healing process that biopsies are performed for lesions greater than entails clot formation, inflammatory phase and 2 cm. Resection is not an applicable treatment soft tissue coverage, and subsequent bone for- option based on the history and clinical appear- mation by osteoblasts surrounding the fibrin ance of this lesion. network. Primary intention is the process in wounds healing when adjacent structures are 54. The correct answer is A. Pathology that evolves reapproximated. There is no healing process enough to involve the inferior border can cause termed tertiary. weakness and eventual fracture, thus the term pathologic fracture. A comminuted fracture is 60. The correct answer is C. Excisional biopsy of one involving multiple fragmented segments. a mucocele is the treatment of choice in this A closed fracture is a term used to describe a case on the basis of its size and location. NS fracture that does not have exposure to the out- rinses would not provide any beneficial compo- side world; in the mandible, these are confined nent. Antibiotics are not indicated since there 80 Chapter 3: Oral and Maxillofacial Surgery and Pain Control

is no infectious process involved. An incisional period (>4 weeks) may result in postoperative biopsy would only be indicated if the lesion was ankylosis. large and did not have the classic presentation of a mucocele noted by patient report. 63. Correct answer is D. The radiographic and clinical findings are consistent with the diagno- 61. The correct answer is D. The pathology noted sis of florid cemento-osseous dysplasia. These around the permanent impacted canine is an lesions are commonly found in the African- odontoma. Odontomas are the most common American female population in their middle types of odontogenic tumors. They can be age, and the lesions showed marked areas of further classified as compound and complex radiolucency and many times symmetric in- forms. The compound form is composed of volvement. These lesions require no surgical multiple toothlike structures, whereas the com- intervention and can be watched unless areas plex subset consists of a conglomerate mass of become symptomatic. enamel and dentin and bears no resemblance to a tooth. Answer A is incorrect because a mu- 64. The correct answer is D. The lesion seen and cous retention cyst is a dome-like structure on described is consistent with the diagnosis of a the floor of the maxillary sinus and would not Stafne defect. This is an asymptomatic radiolu- prevent tooth eruption. Answer B is incorrect cency below mandibular canal in the posterior because an antrolith is a mass in the maxillary mandible and no treatment is needed. sinus that has undergone dystrophic calcifica- tion. Answer C is incorrect because it describes 65. The correct answer is C. The most likely lo- localized areas of bone sclerosis associated with cation to displace a root tip in a maxillary first teeth with pulpitis or large carious lesions not molar is into the maxillary sinus. seen here. 66. The correct answer is B. Neurotmesis is the 62. The correct response is D. Open reduction of a most severe nerve injury and involves loss of fracture of the mandibular condyle is not com- nerve continuity. Neurapraxia is the least se- monly performed because the procedure may vere and involves a contusion or stretching of be complicated and closed reduction is usu- the nerve, but there is no disruption of the ally sufficient. However, open reduction of the epineural or axonal segments. In axonotmesis, condyle is absolutely indicated in the follow- the epineural sheath is disrupted, but there is ing four situations: (1) displacement into the an intact axon. middle cranial fossa; (2) impossibility of obtain- ing adequate dental occlusion by closed reduc- 67. The correct answer is C. The single blade den- tion; (3) lateral extracapsular displacement of tal elevator is used to luxate a tooth apical to the the condyle; and (4) invasion by a foreign body cementoenamel junction prior to placement of forceps. Oral and Maxillofacial (e.g., a bullet from a gunshot wound). Because Surgery and Pain Control the condylar head is displaced in this patient, open reduction and internal fixation of the right 68. The correct response is D. The levator veli subcondylar fracture and the left parasymphy- palatini muscle comprises a muscular sling in seal fracture must be performed. Intermaxillary the posterior palate that is essential for effec- fixation should be applied before the proce- tive palatal closure. It arises embryologically dure and should remain in place for 2 weeks from the fourth pharyngeal arch and, there- after the surgery. Subsequently, active and pas- fore, is innervated by the pharyngeal plexus, sive physical therapy of the mandible should a derivative of the vagus (X) nerve. This nerve be performed to work the mandible and re- also innervates several other muscles that fig- mold the subcondylar union. Closed reduction ure prominently in speech development: the is contraindicated in this patient because of pharyngeal constrictors, the musculus uvu- the displacement of the condylar head. Inter- lae, the palatoglossus, and palatopharyngeus. maxillary fixation applied for an extended time The trigeminal (V) nerve has three separate Answers: 61-72 81

divisions. The first branch, the ophthalmic divi- in this region. Moreover, knowing the mental sion, is purely sensory as is the second division. nerve’s exact location can be of use in avoiding The second, or maxillary, division is interme- inadvertent injury during osseous genioplasty. diate in size between the first and third divi- sions. The mandibular branch of the trigeminal 71. The correct answer is B. The cuspids, or ca- nerve, the third division, is the largest branch nine teeth, have the longest roots in both the and contains both sensory and motor nerves. maxilla and mandible. The average length of a Developmentally, the third division is a nerve cuspid tooth from the tip of the root to the tip of the first branchial (or mandibular) arch and of the crown is 27 mm. It is vital to know the provides motor innervation to the muscular length and position of the dental roots to avoid derivatives of this arch, including the temporal, injuring the apices of the cuspid teeth during masseter, pterygoids, mylohyoid, tensor tym- Le Fort I osteotomy and placement of internal pani, and the anterior belly of the digastric. The fixation (interosseous miniplates or microplates facial (VII) nerve provides motor innervation to and screws). The dentition may also be injured the muscles of facial animation, including the during stabilization of maxillary or mandibu- buccinators. The hypoglossal (XII) nerve pro- lar fractures. The bicuspid, central and lateral vides motor innervation to the tongue. incisor, and molar teeth all have roots that are shorter than the roots of the cuspid teeth; the 69. The correct response is E. Five facial promi- average length of a molar tooth from the tip nences border the developing mouth. These of the root to the tip of the crown, for exam- include the bilateral mandibular prominences, ple, is 24 mm. Consequently, these teeth are the bilateral maxillary prominences, and the much less likely to be injured during LeFort I frontonasal prominence. The lateral nasal and osteotomy. medial nasal prominences are derived from the frontonasal prominence. A unilateral cleft lip 72. The correct answer is D. Transection of the results from incomplete union between the me- cervical branch of the facial nerve would re- ugr n anControl Pain and Surgery

dial nasal prominence and the maxillary promi- sult in loss of function of the platysma muscle, Maxillofacial and Oral nence on the affected side. The lateral and which acts to retract and depress the mandible. medial nasal prominences are derived from the This muscle lies in a superficial position within frontonasal prominence. Clefting would not re- the anterior neck and attaches to the superfi- sult from incomplete union of these structures. cial fascia of the pectoralis major and deltoid The lateral nasal prominence unites with the muscles as well as to the mandible and the maxillary prominence. A patient with a unilat- skin and subcutaneous tissue of the lower face. eral cleft lip will have continuity between the With these multiple attachments, it acts in a lateral alar base and the lateral segment of the synchronous motion with the other muscles of upper lip. The lateral nasal prominence does the lower lip to draw the oral commissure and not normally unite with the medial nasal promi- lower lip downward. The marginal mandibu- nence. A lack of union between the lateral and lar branch of the facial nerve innervates the medial nasal prominences will not result in a lip depressor muscle. Because the lip depressor cleft lip. muscle compensates for the loss of function of the platysma muscle, weakened depression of 70. The correct answer D. The mental nerve the lower lip would only be a temporary find- exits the mental foramen below the second pre- ing. Forward flexion of the neck involves re- molar, halfway down the mandible. The men- laxation of the posterior muscles of the neck tal nerve is the terminal branch of the inferior with the patient in the upright position or ac- alveolar nerve, a major branch of the third di- tion of the sternocleidomastoid muscles bilat- vision of the trigeminal (V) nerve. It provides erally with the patient in the supine position. sensation to the lower lip and skin overlying Muscles that contribute to lateral neck flexion the chin. As such, its location must be known include the sternocleidomastoid, splenius, and before using local anesthetic block techniques inferior obliquus capitis. Upward movement of 82 Chapter 3: Oral and Maxillofacial Surgery and Pain Control

the hyoid bone is accomplished through the the postganglionic fibers travel via the auricu- action of the digastric, stylohyoid, mylohyoid, lotemporal nerve to the parotid gland. and geniohyoid muscles; the sternohyoid, ster- Following parotidectomy, the dermal sweat nothyroid, thyrohyoid, and omohyoid muscles glands may be reinnervated abnormally by the are used for downward movement. Pursing the parasympathetic fibers, resulting in innervation lips is a function of the orbicularis oris muscle. of the skin in the preauricular or temporal area. Patients with this abnormal innervation 73. The correct response is Option C. The parotid may have localized erythema and diapho- gland receives its innervation from the auricu- resis instead of saliva production from the lotemporal nerve, which is a branch of the glos- parotid gland. Appropriate management for pa- sopharyngeal nerve that communicates with tients who develop this condition includes re- the trigeminal and facial nerves. Because of elevating the skin flap and using interposition its relationship with the parotid gland, the au- grafts and flaps. riculotemporal nerve has been shown to be a Frey syndrome does not develop following cause of Frey syndrome (gustatory sweating) injury to motor nerves. Therefore, the facial occurring after parotidectomy; division of the (VII) nerve, which provides innervation for nerve has been recommended as a potential most of the facial muscles, cannot be injured treatment option. The sympathetic plexus of in this patient. the external carotid artery, the facial nerve, and The great auricular nerve arises from the sec- the great auricular nerve also supply innerva- ond and third cervical nerves and emerges from tion to the parotid gland. The meibomian and the posterior border of the sternocleidomastoid lacrimal glands receive innervation from the muscle, then travels anterosuperiorly between lacrimal nerve, which is a branch of the oph- the sternocleidomastoid and platysma muscles thalmic division of the trigeminal nerve (V1). and divides into auricular, facial, and mastoid The sublingual gland is innervated by the lin- branches. The auricular branch provides sen- gual nerve, which is a branch of the mandibu- sation to the earlobe and posterior two-thirds of lar division of the trigeminal nerve (V3). The the ear and is prone to injury during surgery of submandibular gland is innervated by multiple the upper lateral neck. sources, including the chorda tympani, which The posterior auricular nerve arises from the is a component of the facial nerve, the sub- facial nerve at the stylomastoid foramen, re- mandibular ganglion, which is a branch of the ceives a contribution from the auricular branch lingual nerve, and the mylohyoid branch of the of the vagus nerve, and supplies two nerve inferior alveolar nerve. Each of these structures branches, one of which joins with the mas- is derived from the trigeminal nerve. toid branch of the great auricular nerve and an- other that joins with the lesser occipital nerve. 74. The correct answer is A. This 40-year-old Both branches provide sensibility to the pos-

Oral and Maxillofacial woman has Frey syndrome, which occurs as terior side of the pinna and the concha. The Surgery and Pain Control a result of peripheral autonomic dysfunction posterior auricular nerve supplies motor inner- because of surgical injury. In patients who de- vation to the posterior auricular and occipitalis velop Frey syndrome following parotidectomy, muscles. the most likely cause is injury to branches of The auricular branch of the vagus (X) nerve, the auriculotemporal nerve, which is a branch also known as Arnold’s nerve, arises from the su- of the mandibular division of the trigemi- perior ganglion, receives a contribution from nal nerve. The auriculotemporal nerve links the glossopharyngeal nerve, and travels along the parasympathetic secretory fibers to the the temporal bone, emerging through the au- parotid gland. The preganglionic parasympa- ricular fissure between the mastoid process and thetic fibers course down the tympanic branch external auditory meatus. It supplies sensation of the glossopharyngeal nerve and the lesser to the posterior aspect of the ear and external petrosal nerve to the otic ganglion; from there, auditory meatus. Answers: 73-78 83

75. The correct answer is D. A rotational move- Patients who may potentially have internal de- ment of the forceps can only be used on teeth rangement of the TMJ may also experience with a single root. painless clicking during attempted opening of the mandible. 76. The correct response is E. The tongue is the most common site of squamous cell carcinoma 78. The correct answer is A. The lateral and me- of the oral cavity. Tongue carcinoma occurs dial pterygoids, masseter, and temporalis mus- most often in elderly persons but has been seen cles are muscles of mastication, capable of ex- in adolescents and young adults. Excess alco- erting force on the mandible. These muscles hol and tobacco use are thought to be associ- are innervated by the mandibular division of ated. Approximately 36% of all squamous cell the trigeminal nerve (V3). The lateral pterygoid carcinomas of the oral cavity affect the tongue. muscle has two heads that arise from the in- In contrast, 30% of all oral squamous cell fratemporal surface and infratemporal crest of carcinomas involve the floor of the mouth, 16% the greater wing of the sphenoid bone and the involve the mandibular gingiva, and 10% in- lateral surface of the lateral pterygoid plate, and volve the buccal mucosa, while only 3% in- insert on the mandibular neck and the articu- volve the palate. Sites most likely to be affected lar capsule and disk of the temporomandibular by tongue carcinomas include the anterior two- joint. This muscle pulls the condylar process of thirds, lateral borders, and ventral surface of the the mandible and articular disk forward, open- tongue; lesions affecting the midline or dorsal ing the mouth. The medial pterygoid muscle surface are rare. Approximately 33% of these arises from the medial surface of the lateral tumors arise behind the circumvallate papilla. pterygoid plate and the pyramidal process of Because tongue carcinomas are typically pain- the palatine bone and inserts on the medial less, affected persons often do not seek treat- surface of the mandibular ramus and angle. It ment until the tumor becomes advanced and acts to elevate and protrude the mandible and begins to ulcerate and/or cause pain or difficul- produce side-to-side movements. ugr n anControl Pain and Surgery

ties with speech or swallowing. Tumor progres- The masseter muscle arises from the zygo- Maxillofacial and Oral sion correlates directly with the size and extent matic arch and inserts on the mandibular ra- of invasion of the tumor into the tongue mus- mus and coronoid process; during mastication, cles. Patients with extensive muscle infiltration it elevates the mandible to occlude the teeth. typically have restricted tongue movement. The temporalis muscle arises from the tem- Approximately 30% of patients with tongue poral fossa, passes deep to the zygomatic arch, carcinomas have lymph node metastases at and inserts on the coronoid process and an- initial presentation. The midjugular, subdigas- terior border of the mandibular ramus. This tric, and submandibular lymph nodes are most muscle elevates the mandible and contributes likely to be affected, while the lower jugu- to side-to-side grinding movements. The poste- lar, submental, and posterior cervical triangle rior fibers of the temporalis aid in retraction of nodes are rarely involved. the mandible once it has been protruded. The buccinator muscle arises from the alveolar pro- 77. The correct answer is C. In adults, vertical cess of the maxilla, from the mandible opposite mandibular opening measured from maxillary the molars, and from the anterior border of the incisal edge to mandibular incisal edge (inter- pterygoid mandibular raphe; it inserts into the incisal distance) typically ranges from 40 to submucosa of the cheeks and lips and is used 50 mm. In addition, normal range of motion to compress the cheeks against the teeth and of the mandible includes lateral jaw excursion gums. It is innervated by the facial (VII) nerve. (measured at the midline incisor) to 10 mm on Although it is not a muscle of mastication, it as- each side. Decreased mandibular opening may sists the tongue in directing food between the indicate dysfunction of the temporomandibu- molars during mastication. lar joint (TMJ) or surrounding soft tissues. 84 Chapter 3: Oral and Maxillofacial Surgery and Pain Control

79. The correct answer is B. The lingual nerve, The transverse facial artery is found at the zy- which supplies sensation to the anterior two gomatic arch. These locations do not lie within thirds of the tongue, is a branch of the mandi- the typical course of Stensen’s duct. bular division of the trigeminal nerve (V3). Other structures that arise from this nerve in- 81. The correct answer is A. Something, in fact, clude the inferior alveolar nerve, which sup- can be done for this patient as zygomaticus im- plies sensation to the mandibular teeth; the plants are a plausible option to enhance the long buccal branch, which supplies sensation retention of a maxillary obturator. Zygomati- to the buccal mucosa; and the mental nerve, cus implants can be used to retain and support which supplies sensation to the skin of the chin a maxillary obturator following extensive resec- and lower lip and the mucosa of the lip and tion of the maxilla. Zygomaticus implants are adjacent gingiva. In addition, the auriculotem- placed in the zygomatic bone, engaging the poral nerve divides from the posterior border of junction of the temporal and frontal process. V3 immediately after exiting the foramen ovale, A soft reline of the patient’s existing obturator passes around the middle meningeal artery as would not offer long-term retention and stabil- two units, and then courses between the ex- ity. It is likely that the reline procedure would ternal auditory canal and temporomandibular have to be repeated countless times, causing the joint (TMJ). This nerve supplies sensory inner- patient and clinician much frustration. Zygo- vation to the anterior auricle, a large portion maticus implants are a more appropriate option of the temporal region, and part of the external in this otherwise healthy patient. Taking new auditory canal and gives off a branch to supply impressions to fabricate a new obturator is not the TMJ. an acceptable treatment plan for this patient, The infraorbital nerve, nasopalatine nerve, who has had multiple obturators fabricated posterosuperior alveolar nerve, and posterosu- previously, all with poor retention. A better perior nasal nerve are branches of the maxillary choice would be to place bilateral zygomaticus division of the trigeminal nerve (V2). implants. Placement of standard endosseous implants and a prescription for long-term pro- 80. The correct answer is E. Stensen’s duct can be phylactic oral antibiotics is not an appropriate found within the buccal space, which is bor- treatment plan for this patient who has insuf- dered anteriorly by the orbicularis oris mus- ficient maxillary alveolar bone for placement cle, posteriorly by the edge of the masseter of standard endosseous implants. A better op- muscle, superiorly by the zygomaticus major tion would be to place bilateral zygomaticus muscle, and inferiorly by the fascial attachment implants. of the buccinator muscle to the mandible. The duct develops deep within the parotid gland 82. The correct answer is B. The ideal replace- and emerges from the superior third of the ment of missing teeth in children is a fixed pros-

Oral and Maxillofacial gland at its anterior border, then courses below thesis restoration of osseointegrated implants. Surgery and Pain Control the zygomatic arch and enters the buccal space, By age 12, vertical alveolar growth of the jaws inserting into the buccinator and then entering is nearly complete. Implants placed after erup- the oral cavity opposite the upper second mo- tion of the permanent dentition have been lar. The facial artery and vein, buccal branches shown to be successful in children. Vertical of the facial nerve, and buccal fat pad can also alveolar growth of the jaws is generally com- be found within the buccal space. Although pleted with the eruption of the 12-year molars. Stensen’s duct and branches of the facial nerve The patient is 15 years old and is a candidate for are vulnerable to inadvertent dissection in the endosseous implant placement at this time. He area of the parotid gland, there are no major does not have to wait until the age of 18 for sur- arterial branches in this region. The external gical implant placement. Loaded endosseous carotid artery can be found within the angle implants help to maintain alveolar ridge height. of the mandible, while the superficial temporal Osseointegrated implants placed into alveolar artery is located within the preauricular border. bone do not cause excessive resorption of bone. Answers: 79-85 85

r Placement of endosseous implants in children Rapid progressive bone loss refractory to has been shown to be stable. As children are of- stress reduction and peri-implant treatment r ten noncompliant with removable appliances, Pain during function or percussion r endosseous implants offer a preferred method Continued uncontrolled exudate refractory for the replacement of missing teeth in se- to surgical attempts at correction r lect children. Endosseous implants placed in Generalized radiolucency around the im- a child’s growing bone have been reported to plant r migrate. Vertical growth of alveolar bone can Greater than half of the surrounding bone is cause submersion of endosseous implants. The resorbed around the implant r patient in this vignette is 15 years old, and ver- Implants placed in nonrestorable position tical growth of his alveolus should have neared (Adapted from Misch, CE. Contemporary completion by the age of 12. Implants would Implant Dentistry, 2nd Edition. Mosby, St. be a viable option in this 15-year-old child for Louis; 1999.) replacement of his missing maxillary central in- The implant-supported prosthesis causes cisor. He does not have to wait until he is 21 for the patient pain upon percussion. The implant surgical placement of implants. in the position of missing tooth #28 has a gen- eralized radiolucency surrounding it, satisfying 83. The correct answer is C. Replacement of the the criteria for implant failure. Just less than missing lateral incisor with an osseointegrated one-half of the surrounding bone has resorbed implant depends on adequate volume and qual- around the implant in the position of miss- ity of bone. Studies have shown that the longer ing tooth #29. This does not meet the crite- the interval beyond 4 months, the less likely ria for implant failure, in which greater than there would be adequate bone volume to ac- one-half the bone has resorbed. As no radio- cept an implant in the bone grafted cleft site. graphic abnormalities are associated with the Four weeks would be too short for a consoli- implant in the position of tooth #30, it does dation period in the alveolar cleft site. Place- not meet the criteria for implant failure. While Control Pain and Surgery ment of an endosseous implant in the grafted Maxillofacial and Oral implant #28 does meet the criteria for implant alveolar cleft should occur after a 4-month con- failure, implant #29 does not. While implant solidation period. Eight months would be too #28 does meet the criteria for implant failure, long for a consolidation period in the alveolar implant #30 does not. Neither implants #29 cleft site. Placement of an endosseous implant nor #30 meet the criteria for implant failure as in the grafted alveolar cleft should occur after a described by Misch. 4-month consolidation period. One year would be too long for a consolidation period in the 85. The correct answer is D. The patient has a alveolar cleft site. Placement of an endosseous preexisting Angle Class II malocclusion as evi- implant in the grafted alveolar cleft should oc- dence by her Class II molar relationship. The cur after a 4-month consolidation period. Two question stem explains that the canine relation- years would be too long for a consolidation pe- ship correlates to the molar relationship. To riod in the alveolar cleft site. Placement of an preserve the patient’s existing, functional rela- endosseous implant in the grafted alveolar cleft tionship, the right canine should be restored should occur after a 4-month consolidation pe- to a Class II canine relationship. The Class riod. II canine relationship is best described as: the maxillary canine tip should occlude mesial to 84. The correct answer is A. Failure of an implant the embrasure between the mandibular canine is clinically and radiographically apparent by and first premolar. If the maxillary canine tip the following signs and symptoms: r opposes the embrasure between the mandibu- Horizontal mobility greater than 0.5 mm lar canine and first premolar, the patient is in or any clinically observed vertical movement a Class I canine relationship. This patient has with less than 500 g applied force a Class II canine classification that should be 86 Chapter 3: Oral and Maxillofacial Surgery and Pain Control

preserved with restoration of the maxillary right incidence is 0.7 per 100,000 person years of canine tooth in its characteristic position. If the exposure. This patient’s 2-year history of oral maxillary canine tip occludes distal to the em- bisphosphonate use makes her at risk for the de- brasure between the mandibular canine and velopment of BRONJ. Patients who have taken first premolar, the patient is in a Class III ca- oral bisphosphonates for less than 3 years with nine relationship. This patient has a Class II no clinical risk factors require no alteration canine classification that should be preserved or delay in planned oral surgery. Therefore, with restoration of the maxillary right canine this patient may undergo surgical placement tooth in its characteristic position. Flattening of endosseous implants using standard implant the maxillary canine tooth and taking it out surgery techniques without modification. The of occlusion would not result in an esthetic or history of oral bisphosphonate use does not ex- functional outcome. The patient’s right maxil- clude this patient as a potential endosseous im- lary canine tooth should be restored to a Class plant recipient. This patient has been taking an II canine position. Restoring the maxillary ca- oral bisphosphonate (Alendronate) for 2 years. nine without planning of the position of the tip Patients taking oral bisphosphonates are at a would lead to a poor functional and esthetic considerable lower risk for BRONJ than pa- result. It is likely that the restoration would tients taking IV bisphosphonates. Patients who cause the patient malocclusion, dysfunction, have taken oral bisphosphonates for less than and pain. The best answer is to restore the pa- 3 years with no clinical risk factors require no tient’s maxillary right canine into a Class II ca- alteration or delay in planned oral surgery. nine position. Placement of dental implants should be avoided in patients with a history of taking the 86. The correct answer is A. Smile arc is the an- more potent IV bisphosphonate medications terior arc of the esthetic line of the dentition. on a frequent dosing schedule. It is defined as the relationship of the curva- The following bisphosphonates are avail- ture of the incisal edges of the maxillary teeth able for use in the United States: to the curvature of the lower lip in the posed social smile. It can be classified as consonant Generic Name Brand Name Route (parallel), reverse, or flat. Dental display is mea- sured from the upper lip vermilion to the cen- Etidronate Didronel Oral tral incisal edge. The position of the upper lip Tiludronate Skelid Oral in repose and smiling determines the amount Alendronate Fosamax Oral of tooth show, which is also known as “dental Risedronate Actonel Oral display.” The esthetic line of the dentition de- Ibandronate Boniva Oral scribes the facial surfaces of the maxillary teeth Pamidronate Areda IV Zoledronate Zometa IV in relation to the smile. The curve of Spee is Clodronate Bonefos IV

Oral and Maxillofacial the curvature of the mandibular occlusal plane Surgery and Pain Control beginning at the tip of the mandibular canine and following the buccal cusps of the mandibu- The patient does not need to undergo a lar posterior teeth. Centric occlusion describes 2-month drug holiday prior to implant place- the relationship of the maxillary and mandibu- ment. A drug holiday is recommended for pa- lar jaws in which there is maximal contact of tients treated with oral bisphosphonates for 3 the inclined planes of the opposing cusps of the years or more, or patients that have taken con- maxillary and mandibular teeth. comitant steroid medication. The drug holiday should be arranged by the prescribing provider 87. The correct answer is B. Estimated incidence of the bisphosphonate and only be considered if of bisphosphonate-related osteonecrosis of the systemic conditions permit it to be undertaken jaws (BRONJ) ranges from 0.8% to 12% for pa- safely. Standard drug holidays are a discontin- tients receiving intravenous formulations. For uation of bisphosphonate for at least 3 months patients taking oral medications, the estimated prior to oral surgery. Answers: 86-89 87

The patient does not need to undergo a ment planning for implant placement would 2-month drug holiday following implant place- be provided by CT. Bitewing radiographs show ment. A drug holiday is recommended for the crowns of posterior teeth and the alveolar patients treated with oral bisphosphonates for bone height at the cementoenamel junction. 3 years or more, or patients that have taken con- Bitewing radiographs more accurately display comitant steroid medication. The drug holiday bone levels than do periapical radiographs. should be arranged by the prescribing provider However, the most accurate representation of of the bisphosphonate, and only be consid- alveolar bone height and width for implant ered if systemic conditions permit it to be un- treatment planning purposes would be pro- dertaken safely. Standard drug holidays are a vided by CT. discontinuation of bisphosphonate for at least 3 months prior to oral surgery. 89. The correct answer is E. This patient does not This asymptomatic patient does not require require antibiotic prophylaxis. According to the a 1-year drug holiday and is not restricted from 2007–2008 recommendations of the American taking bisphosphonates for life. This patient Heart Association (AHA), only an extremely may resume bisphosphonate use after osseous small number of cases of infective endocarditis healing has occurred. The patient should be might be prevented by antibiotic prophylaxis placed on a regular recall schedule to monitor for dental procedures, even if such prophy- for the development of BRONJ, and the patient lactic therapy were 100% effective. Therefore, should be given appropriate informed consent only those patients with underlying cardiac regarding potential implant failure and possi- conditions associated with the highest risk war- ble osteonecrosis of the jaws if she continues to rant antibiotic prophylaxis. take oral bisphosphonates. Cardiac Conditions Associated with the Highest 88. The correct answer is A. A CT Dentascan is Risk of Adverse Outcome from Endocarditis for a dental computed tomography (CT) imag- which Prophylaxis with Dental Procedures Is Reasonable Control Pain and Surgery ing program that provides an image in true Maxillofacial and Oral life-size format with vertical scale in millime- Prosthetic cardiac valve or prosthetic material ters. It provides for a cross-sectional view of used for cardiac valve repair the dental arch. The multiaxial reconstruction Previous infective endocarditis Congenital heart disease (CHD) of the image allows for measurement of the r alveolus, location of the inferior alveolar nerve, Unrepaired cyanotic CHD, including palliative shunts and conduits r and accurate treatment planning for surgi- Completely repaired congenital heart defect cal implant placement. An orthopantogram with prosthetic material or device, whether or “panorex” is a scanning dental x-ray show- placed by surgery or by catheter intervention, during the first 6 months after the procedure ing a two-dimensional view of the maxilla and r mandible. Magnification of 15% to 30% is Repaired CHD with residual defects at the site inherent in the panorex radiograph system. or adjacent to the site of a prosthetic patch or prosthetic device (which inhibit While an orthopantogram will provide impor- endothelialization) tant information for surgical implant treatment Cardiac transplantation recipients who develop planning, a CT Dentascan will offer more ac- cardiac valvulopathy curate imaging. Periapical radiographs show the tip of the roots of the teeth. While peri- Modified with permission. Circulation. 2007;116: 1736–1754. apical radiographs offer important information about the patient’s dentition, the most accu- rate information about the patient’s alveolar It is recommended that patients who meet height and width would be provided by CT. these criteria receive antibiotic prophylaxis for The occlusal radiograph can provide informa- all dental procedures that involve manipula- tion about the floor of the mouth or palate. tion of gingival tissue or the periapical region The most accurate information regarding treat- of teeth or perforation of the oral mucosa. 88 Chapter 3: Oral and Maxillofacial Surgery and Pain Control

The regimen of clindamycin 600 mg PO should only be considered when a patient has 30 minutes to 1 hour prior to the procedure not received the preprocedure dose. would be appropriate in a patient requiring The regimen of2gofamoxicillin PO 1 hour prophylaxis who is also allergic to peni- prior to the procedure would be appropriate cillins. An antibiotic for prophylaxis should be for an adult patient not allergic to penicillins. administered as a single dose prior to the proce- Vancomycin is not in the AHA recommended dure. Administration of a post-procedure dose regimens for a dental procedure.

AHA Recommended Antibiotic Prophylaxis Regimens for Dental Procedures

Regimen: Single Dose 30–60 min Before Procedure Situation Agent Adults Children Oral Amoxicillin 2 g 50 mg/kg Unable to Take Oral Ampicillin 2gIMorIV 50 mg/kg IM or IV Medication OR Cefazolin or 1gIMorIV 50 mg/kg IM or IV Ceftriaxone Allergic to Penicillins or Cephalexin 2g 50 mg/kg Ampicillin Oral OR Clindamycin 600 mg 20 mg/kg OR Azithromycin or 500 mg 15 mg/kg clarithromycin Allergic to Penicillins or Cefazolin or 1gIMorIV 50 mg/kg IM or IV Ampicillin and Unable ceftriaxone to Take Oral OR Medication Clindamycin 600 mg IM or IV 20 mg/kg IM or IV Modified with permission. Circulation. 2007;116:1736–1754.

90. The correct answer is A. According to Brane- illa is correct; however, a healing period of 1 mark’s original two-stage technique, the rec- to 2 months in the mandible was not originally ommended healing time was described as 3 described by Branemark to ensure adequate os-

Oral and Maxillofacial to 4 months for mandibular bone and 5 to seointegration.

Surgery and Pain Control 6 months for the more cancellous bone of the maxilla. A healing period of 1 to 2 months for 91. The correct answer is D. This patient, who is both the maxilla and mandible was not orig- allergic to penicillin, should be administered inally described by Branemark to ensure ade- clindamycin 600 mg 1 hour prior to the dental quate osseointegration. A healing period of 1 procedure. to 2 months for the maxilla was not originally The following are the American Dental described by Branemark. However, a 3- to 4- Association/American Academy of Orthopedic month healing period in the mandible was de- Surgeons suggested antibiotic prophylaxis reg- scribed as appropriate time for osseointegration imens: to occur. A healing period of 3 to 4 months for Clindamycin is a bacteriostatic antibiotic the maxilla and 1 to 2 months for the mandible that acts to inhibit bacterial protein synthe- was not originally described by Branemark. A sis. It targets the 50S bacterial ribosomal sub- healing period of 5 to 6 months for the max- unit and is effective against anaerobes and Answers: 90-93 89

Recommended Single-Dose New guidelines proposed by the AAOS recom- Patient Type Drug Regimen mend that clinicians consider antibiotic pro- phylaxis in ALL total joint replacement patients Patients not Amoxicillin, 2 g PO 1 hour prior to any invasive procedure that may cause penicillin cephalexin, prior to allergic or cephradine dental bacteremia. procedure It is appropriate to consider antibiotic pre- Patients not Ampicillin or Ampicillin 2 g medication for these patients. Therefore, the penicillin cefazolin IM or IV or statement “No antimicrobial prophylaxis is in- allergic, cefazolin dicated at this time” is incorrect. unable to 1 g 1 hour take oral prior to 92. The correct answer is A. Prosthetic obtura- medications dental tion of postablative palatomaxillary defects can procedure be associated with instability and poor reten- Penicillin- Clindamycin 600 mg PO tion. Prosthetic instability is characterized by allergic 1 hour prior patients to dental a breakdown in the oronasal prosthetic-tissue procedure seal, oro-nasal regurgitation, and compromised Penicillin- Clindamycin 600 mg IV speech. Destabilizing cantilever forces in large allergic 1 hour prior palatomaxillary defects often lead the recon- patients, to dental structive team to consider reconstructive max- unable to procedure illofacial surgery including the utilization of take oral free-flaps or bone grafts and later placement medications of osseointegrated implants. The most com- Modified with permission. Stroke. 1997;96:358– mon destabilizing forces affecting obturators 366. are cantilever forces causing the prosthesis to tip toward the defect. Rotational forces during

function are not as common. A tensile force is a Control Pain and Surgery

Gram-positive bacteria. As it is highly effective stretching force pulling at both ends of a struc- Maxillofacial and Oral against oral anaerobes, and it is not a ␤-lactam ture along its length. Tensile forces are not the antibiotic, clindamycin is appropriate for use most common destabilizing forces affecting ob- in patients who are allergic to penicillin. turators. A compressive force applied to a mate- The recommended antibiotic prophylaxis rial would result in its compaction or decrease regimens are single-dose schedules adminis- in volume. Compressive forces are not the most tered 1 hour prior to the procedure. No sec- common destabilizing forces affecting obtura- ond doses are currently recommended for tors. any of the suggested regimens. This patient is allergic to penicillin, and therefore admin- 93. The correct answer is B. Autologous bone, istering another ␤-lactam antibiotic such as the gold standard in bone grafting, is the only amoxicillin at this time would be inappro- graft material that contains all four qualities priate. This patient has a documented peni- of osteogenesis, osteoinduction, osteoconduc- cillin allergy. Administration of cephalosporin tion, and osseointegration. Osteogenesis is the would not be advisable as there is potential process of laying down new bone material by cross-hypersensitivity between penicillins and osteoblasts. Osteoinduction is the active pro- cephalosporins. The patient has undergone cess by which osteogenesis is induced. Osteoin- total joint replacement 4 months ago. The duction involves the recruitment of immature American Dental Association and the Ameri- cells, and the stimulation of these cells to can Academy of Orthopedic Surgeons (AAOS) develop into preosteoblasts through chemical used to suggest that all patients during the first signaling. Osteoconduction is the (passive) pro- 2 years following total joint replacement are cess by which bone grows on a scaffolding, ma- considered at potential increased risk of expe- trix, or surface. Osseointegration, as originally riencing hematogenous total joint infection. described by Branemark, is the apparent direct 90 Chapter 3: Oral and Maxillofacial Surgery and Pain Control

connection of osseous tissue to an inert alloplas- acterized by an exaggerated subjective painful tic material without intervening connective tis- response to any stimulus. Anesthesia is the ab- sue. Angiogenesis is the physiological process sence of perception of stimulation by a noxious by which new blood vessels grow from preex- or non-noxious stimulus. isting blood vessels. 96. The correct answer is A. Neurapraxia is a be- 94. The correct answer is A. Autologous bone, nign injury, usually caused by compression or the gold standard in bone grafting, is the only prolonged retraction. Neurapraxia consists of a graft material that contains all four qualities temporary failure of conduction stimuli. Spon- of osteogenesis, osteoinduction, osteoconduc- taneous recovery from neurapraxia usually oc- tion, and osseointegration. An autologous graft curs within 4 weeks. Axonotmesis is caused by (autograft) is composed of cells or tissues that partial severance or crushing of the nerve. It is are reimplanted into the same individual from a more severe injury than neurapraxia. While whom the graft was harvested. An allograft is the general structure of the nerve remains in- composed of cells or tissues that are trans- tact, there is a loss of continuity of some axons, planted into a different individual from whom which then undergo Wallerian degeneration. the graft was harvested. These grafts are also Conduction failure is prolonged. Initial symp- known as homologous grafts. A xenograft is toms of returning sensation occur 5 to 11 weeks composed of cells or tissues that are harvested after injury. Patients often experience hypoes- from one species and transplanted into another. thesia, which may be accompanied by persis- An alloplast is an inert material originating tent dysesthesias. Neurotmesis is the complete from a nonliving source that is surgically im- severance or internal physiologic disruption of planted. An alloplast may be a metal, polymer, all the nerve layers (epineurium, perineurium, (plastic, silicone, etc.), or other nonbiologic endoneurium). Neurotmesis is characterized material. Recombinant human bone morpho- by total conduction block of all impulses. To- genetic protein-2 (rhBMP-2) is a growth factor tal numbness for 12 weeks or longer is likely produced using recombinant DNA technology because of neurotmesis. Hyperpathia is char- that has been used to promote bone growth. acterized by an exaggerated subjective painful The osteoinductive agent (BMP) is combined response to any stimulus. Allodynia, a sub- with an osteoconductive matrix (collagen) to class of hyperesthesia, describes pain because promote osteogenesis. This material is not con- of a stimulus that normally does not provoke sidered to be the gold standard of bone grafting. pain.

95. The correct answer is A. “Numbness” is an 97. The correct answer is C. Antibiotic prophy- ill-defined, subjective lay term that can de- laxis is recommended for end-stage renal dis- scribe any altered or abnormal sensation after ease patients undergoing hemodialysis. Typical

Oral and Maxillofacial nerve injury. It is important for the clinician streptococcal and staphylococcal infections as- Surgery and Pain Control to investigate the patient’s symptoms and inter- sociated with dental bacteremia can contami- pret them into scientific terms. Hypoesthesia nate dialysis access sites (shunts or fistulas). It is is the decreased perception of stimulation by a recommended that oral surgery procedures be noxious or non-noxious stimulus in an area of performed the day after dialysis. Therefore, it is skin or mucosa. Dysesthesia describes an un- best to coordinate treatment with the patient’s pleasant abnormal sensation, spontaneous or physician. End-stage renal disease patients do evoked. Hyperesthesia is increased sensitivity require special considerations to their care. It to any noxious or non-noxious stimulus. Allody- is correct to consult with a patient’s physician. nia, a subclass of hyperesthesia, describes pain Current recommendations are for oral surgery because of a stimulus that normally does not procedures to be performed on the day after provoke pain. Hyperalgesia, a subclass of hyper- dialysis, and antibiotic prophylaxis will likely esthesia, is an increased response to a stimulus be required. This patient is a candidate for sur- that is normally painful. Hyperpathia is char- gical implant therapy, so long as the patient’s Answers: 94-99 91

physician is consulted, and special therapeutic during the second trimester. However, inva- conditions are met to minimize risk. sive elective procedures such as implantology should be delayed until after delivery. It is im- 98. The correct answer is A. Elective surgical pro- portant to remember that dental treatment for cedures such as implantology should be de- a pregnant patient who is experiencing pain layed until after delivery when the mother’s or infection should not be delayed until after health is at her normal state. It is preferred that delivery. Maxillary and mandibular third molar the mother not be breastfeeding because of the teeth are the most often impacted teeth. Next possible effects of anesthesia and medications in frequency are the maxillary canine teeth, on the infant. This patient is 9-weeks pregnant. mandibular bicuspid teeth, maxillary bicuspid Therefore, she is in her first trimester of preg- teeth, and maxillary second molar teeth. Im- nancy. The first trimester begins at conception pactions of first molars and incisors are rela- and continues through the 11th week or day tively uncommon. This patient’s surgical plan 70. The second trimester is from day 70 to 154. will most likely involve surgical removal of the The third trimester is from day 154 to deliv- impacted tooth, bone grafting procedures, and ery. The period between days 18 and 60 is the dental implant placement. It is best to wait until time where the human embryo is most vul- after delivery to undergo such extensive, elec- nerable to teratogenic insult. Therefore, risk to tive dental treatment. the fetus is greatest during this period. Many physiologic changes occur in pregnancy. By 99. The correct answer is B. Acetaminophen is the third trimester, the vena cava and aorta can the analgesic and antipyretic of choice in preg- be compressed by the uterus with the patient nancy. Acetaminophen is considered a preg- in the supine position. This can result in the nancy category B drug. supine hypotensive syndrome. Invasive dental The following Food and Drug Administra- therapy during the third trimester poses the tion (FDA) categories of prescription drugs can greatest risk to the mother. Elective restorative be used as a reference: ugr n anControl Pain and Surgery

and periodontal treatment can be performed Maxillofacial and Oral

Pregnancy Adequate and well-controlled human studies have failed to demonstrate a Category A risk to the fetus in the first trimester. (And there is no evidence of risk in later trimesters.) The probability of fetal harm seems remote Pregnancy Animal studies indicate no fetal risk, and there are no adequate, Category B well-controlled human studies to demonstrate risk. OR Animal studies show an adverse effect on the fetus, but adequate, well-controlled human studies do not show a risk to the fetus in any trimester Pregnancy Teratogenic or embryocidal effects are shown in animals. No controlled Category C studies have been performed in either animals or humans. Potential benefits versus risks must be assessed to warrant use of the drug in pregnant women Pregnancy Positive evidence of human fetal risk exists based on adverse reaction data Category D from investigational or marketing experience or studies, but potential benefits may outweigh potential risks in certain situations Pregnancy Studies or experience have shown fetal risk that clearly outweighs any Category X possible benefit for use in pregnant women Adapted from Pregnancy categories for prescription. FDA Drug Bull 1979;9:23–24, www.fda.gov.

Aspirin is a pregnancy category D drug. Its Codeine is considered a pregnancy category use is considered unsafe during pregnancy. Ac- C drug. Codeine use during the first trimester etaminophen is a better choice for treatment of has been associated with congenital abnormali- mild-to-moderate pain in this patient. ties such as cleft lip and palate. Acetaminophen 92 Chapter 3: Oral and Maxillofacial Surgery and Pain Control

is a better choice for the treatment of this pa- Atrial fibrillation is the most common car- tient’s mild-to-moderate postoperative pain. diac arrhythmia. During atrial fibrillation, the Hydromorphone is a pregnancy category C atrial muscles fibrillate rather than contract drug. However, this patient’s mild-to-moderate with coordination. Many patients with atrial pain will likely respond to acetaminophen. fibrillation will be treated with anticoagulants This patient is pregnant, and appropriate by their primary medical doctors or cardiolo- pain medication can be used. The safest med- gist. Careful coordination and communication ication should be chosen for the clinical sit- is necessary between the implant surgeon and uation. This patient is experiencing mild-to- prescribing physician. The patient may be ad- moderate pain 1 week after dental implant vised to discontinue anticoagulant use prior to placement. She will likely respond well to ac- implant placement and resume the anticoag- etaminophen. ulant after surgery. Local measures should be taken to ensure adequate hemostasis at the sur- 100. The correct answer is B. The geriatric patient gical site. Atrial fibrillation is not considered group can be divided into three categories: the a contraindication for the placement of dental elderly (ages 65–74 years), the aged (ages 75–84 implants in the elderly. years), and the very old (ages 85 and older). Diseases or conditions contraindicating den- 101. The correct answer is D. As a general rule, tal implants in the elderly patient include: one should always use the largest possible im- r plant diameter. Larger diameter implants offer Dementia r greater surface area for osseointegration, and Diabetes mellitus (poorly controlled) r thus provide greater implant stability. Because Cardiac infarction r of their reduced mechanical stability, small- Hemorrhage syndromes r diameter implants are used only in cases with a Psychosis r low mechanical load. The diameter of the im- Cerebral stroke r plant does, in fact, affect the stability of the Epilepsy r implant fixture. The wider (larger) diameter Extreme dyskinesia r implants provide greater surface area for os- Hepatic cirrhosis r seointegration, and therefore provide greater Cortisone-medication effect r stability. An implant must never be placed into Radiation effects r (or too close to) vital structures such as the in- Alcoholism r ferior alveolar nerve. Cardiovascular transplant Hypertension is not considered a contraindi- 102. The correct answer is A. Because the implant cation for the placement of dental implants shoulder is the largest part of the implant, it in the elderly. Normal blood pressure (BP) is is used as the point of reference for measuring

Oral and Maxillofacial defined as systolic BP less than 120 mm Hg the mesiodistal distance. A minimal distance Surgery and Pain Control and diastolic BP less than 80 mm Hg. Pre- of 1.5 mm is recommended between the im- hypertension is defined as systolic BP 120 to plant shoulder and the adjacent tooth at bone 139 mm Hg, or diastolic BP 80 to 89 mm Hg. level. A distance of 1.0 mm is too small between Stage 1 hypertension is defined as systolic BP the implant shoulder and the adjacent tooth at 140 to 159 mm Hg or diastolic BP 90 to bone level. A distance of 0.5 mm is too small 99 mm Hg. Stage 2 hypertension is classified between the implant shoulder and the adjacent as a systolic BP ≥ 160 mm Hg or diastolic tooth at bone level. A distance of 3.0 mm will BP ≥ 100 mm Hg. not damage the implant or the adjacent tooth Hypercholesterolemia increases a patient’s because this value is greater than the recom- risk of cardiovascular disease (i.e., atheroscle- mended minimal distance of 1.5 mm at bone rosis). Hypercholesterolemia is not considered level. a contraindication for the placement of dental implants in the elderly. Answers: 100-108 93

103. The correct answer is C. The recommended cover the implant, remove the cover screw, and orofacial implant position is contingent upon place the healing abutment. The healing abut- a minimum of 1.0-mm thickness of facial and ment will extend transmucosally, allowing for palatal bone. 1.0 mm of facial or palatal bone soft tissue sculpturing. Torque testing of the is recommended to ensure stability of the hard implant fixture can help determine if the im- and soft tissues. If the facial or palatal bone plant has achieved stability in bone. It can be wall width is less than 1-mm thick, augmenta- done during the second-stage surgical proce- tion procedures are indicated. Ideally, the axis dure. However, torque testing of the implant of the implant should be angulated so that the cover screw is not a part of the second-stage sur- screw channel of a screw retained restoration gical procedure. Placement of the cover screw is located behind (palatal) to the incisal edge would occur during the first stage of a tradi- of an implant placed in the anterior region. tional, second-stage implant surgery model. In 0.5 mm is too thin amount of bone to ade- a single-stage procedure, the implant healing quately support the implant. To ensure pre- abutment would be placed immediately, and dictability of hard and soft tissue stability, a no cover screw would be utilized. Placement minimum of 1.0-mm thickness of bone is rec- of the implant fixture occurs during the first ommended on the facial and palatal aspects surgery of a second-stage implant. of the implant. 0.25 mm is definitely too thin amount of bone to adequately support the im- 106. The correct answer is A. Estimates of the time plant fixture. Such a thickness is likely to result remaining in oxygen E-cylinders can be ap- in fenestration. proximated using the following formula: 2.0 mm is greater than the 1.0 mm mini- (psi × F)/L/min = Time remaining in mum thickness required facial and palatal to minutes the implant fixture. While 2.0 mm would ad- equately support the implant fixture, the ques- Where F for E-cylinders = 0.3

tion asks for the minimum thickness of bone. Control Pain and Surgery In this case, (1,000 × 0.3)/10 = 30 min Therefore, the correct answer is 1.0 mm. Maxillofacial and Oral Portable E-cylinders of oxygen should be re- 104. The correct answer is B. It is recommended placed when their content is below 1,000 psi. that implant insertion torque should not ex- ceed 35 Ncm. 45 and 50 Ncm exceed the 107. The correct answer is B. Supplemental oxy- recommended maximum torque for implant gen can be safely administered to patients with insertion. 25 Ncm is not the maximum torque COPD. The current thinking is that one can allowed for implant placement. Implant inser- provide whatever concentration of supplemen- tion can occur using a torque up to 35 Ncm, tal oxygen that is required to maintain the and a maximum of 15 rpm using the implant patient’s oxygen saturation above 90% as mea- handpiece. Always follow the manufacturer’s sured by pulse oximeter. Generally, this can be recommendations for implant placement. accomplished by nasal cannula.

105. The correct answer is A. The “second-stage” 108. The correct answer is A. Arthrocentesis. This implant surgical procedure traditionally in- is the indicated therapy for recent onset of uni- volves the placement of the healing abutment lateral failure of translation because of abnor- (healing cap). This is the second surgical proce- mal relationship of the articular disk to the dure in a second-stage implant surgery model. mandibular condyle because of inflammation During the first implant surgery, the implant from functional loading. Arthroscopic surgery fixture is placed into the bone, and a cover (answer choice B) is used to visualize intra- screw is inserted. The full thickness mucope- articular pathology such as adhesions, synovitis, riosteal flaps are sutured closed, and submu- osteoarthritis, disk displacement, and disk per- cosal healing occurs. After the healing period, foration but is not the treatment once a diagno- a second surgical procedure is required to un- sis is made. All other answers involve occlusal 94 Chapter 3: Oral and Maxillofacial Surgery and Pain Control

adjustment, which has not been proven to be 111. The correct answer is A. For coronal frac- the etiology is any type of TMJ disorder. ture of a root, the correct treatment is the re- moval of coronal segment and pulpectomy. 109. The correct answer is B. This is a nerve-related Rigid splint (B) is indicated for 2 to 3 months in TMJ disorder that is usually described as stab- patients who fracture apical and mid-root por- bing, burning, shocking pain that is provoked tions of the root. Occlusal adjustment (C) is by wind, tactile, or thermal stimulation. My- only indicated if needed in cases of subluxation, ofascial pain dysfunction or MPD primarily af- luxation, avulsion, and alveolar process frac- fects muscles of mastication and is often stress ture. Extraction (D) is indicated if greater than related. Osteoarthritis is a degenerative joint one-third of the root is fractured and pulp is disease and does not present as described in exposed. the question. TMJ ankylosis is most commonly caused by trauma and results in fusion of the 112. The correct answer is C. It is contraindicated condyle, disk, and TMJ fossa. Disk displace- to place implants immediately after extraction ment would most likely cause the patient to of teeth in a region of active infection. At least have trouble opening and/or closing, which is 8 weeks should pass before placing the im- not apparent in this situation. plant. Oral bisphosphonates (answer choice B) are not a contraindication to implant place- 110. The correct answer is A. The maximum ment, although IV bisphosphonates may be. amount of 2% lidocaine with 1:100,000 Not enough bone height (answer choice A) is epinephrine that can be administered to a incorrect because the patient has 9 mm of ver- healthy 150-lb man is 477 mg or roughly 13 tical height and needs 8 mm at least. Osteo- dental cartridges. Therefore, this patient would porosis (answer choice D) and type II diabetes be able to receive eight or more carpules in ad- (answer choice E) are not contraindications as dition to the six he has already received. There- long as they are controlled. fore, six carpules is the correct answer. Oral and Maxillofacial Surgery and Pain Control CHAPTER 4 Orthodontics

95 96 Chapter 4: Orthodontics

QUESTIONS

1. A 12-year-old boy was classified as having C. Bring the mandible forward with strain so class II division 2 malocclusion. What position that the mandibular incisors touch the max- would you suspect the patient’s maxillary cen- illary incisors trals and laterals are in? D. Bring the mandible back without strain so that the mandibular incisors touch the max- A. Maxillary centrals tipped palatally and in illary incisors retruded position; maxillary lateral tipped labially and mesially 4. This is the most innermost point on contour of B. Maxillary centrals tipped palatally and in mandible between incisor tooth and bony chin. retruded position; maxillary lateral tipped This landmark is known as: palatally C. Maxillary centrals tipped labially and in protruded position; maxillary lateral tipped labially and mesially D. Maxillary centrals tipped labially and in protruded position; maxillary lateral tipped palatally

2. How would you classify the occlusion below?

A. #11—point B B. #11—point A C. #10—point A D. #13—pogonion A. Class III malocclusion, also known as dis- E. #12—menton toclusion B. Class II malocclusion, also known as mesio- 5. When the lines connecting sella–nasion and clusion gonion–menton meet, they create an angle that C. Class III malocclusion, also known as helps to determine the malocclusion. What mesioclusion type of anterior vertical dimension and maloc- D. Class II malocclusion, also known as disto- clusion correlates with this steep angle? clusion E. Class I malocclusion A. Long anterior vertical dimension and a deep-bite malocclusion. 3. In a pseudo–class III malocclusion, the patient B. Short anterior vertical dimension and an has the ability to: open-bite malocclusion. C. Short anterior vertical dimension and a A. Bring the mandible back with strain so that deep-bite malocclusion. the mandibular incisors touch the maxillary D. Long anterior vertical dimension and an incisors open bite malocclusion. B. Bring the mandible forward without strain so that the mandibular incisors touch the 6. A 35-year-old man presents for an evaluation.

Orthodontics maxillary incisors Upon examination, you notice that the patient’s Questions: 1-11 97

buccal cusps of the lower posterior teeth oc- used in skeletal class III malocclusions to pro- clude buccal to the buccal cusps of the upper tract the maxilla? posterior teeth on the left and the right side. A. Cervical-pull headgear What treatment should be rendered to correct B. Straight-pull headgear this malocculsion? C. High-pull headgear D. Reverse-pull headgear

10. Functional appliances have dental and skeletal effect and can be tissue-borne or tooth-borne. Which of the following appliances is tissue- borne? A. Bionator B. Frankel functional appliance C. Herbst D. Edgewise appliance

11. A 14-year-old girl demonstrates a deviation in maxillary and mandibular midlines as the pa- tient closes and does not have smooth closure to centric occlusion. What type of crossbite, if A. Surgery any, does the patient demonstrate? B. Rapid maxillary expander C. Quad helix expander D. W-arch E. Do nothing, it will eventually rectify by itself

7. This has been adapted as the best horizontal orientation that assesses lateral representation of the skull. Which of the following cephalo- metric landmarks make up this plane when a line is drawn to connect them? A. Sella and nasion B. Gonion and gnathion A C. Porion and orbitale D. Spheno-occipital synchondrosis and ante- rior nasal spine (ANS)

8. A 37-year-old Caucasian man has SNA angle of 88 degrees. What does SNA angle of 88 degrees indicate regarding the patient’s occlusion? A. Maxillary retrusion B. Maxillary protrusion Orthodontics C. Class I skeletal pattern B D. Mandibular prognathism E. Mandibular retrognathism A. Skeletal crossbite B. Dental crossbite 9. Which headgear has an extraoral component C. Functional crossbite that is supported by chin and forehead and is D. This is not an example of a crossbite. 98 Chapter 4: Orthodontics

12. A 15-year-old patient presents for orthodontic treatment with the cephalometric radiograph shown below. Upon examination, the patient does not have an open bite. What is the most likely cause for the malocclusion?

A. Disengage and remove the archwire and send the patient for endodontic evaluation. B. Continue with orthodontic treatment and send the patient for an endodontic evalu- ation after the comprehensive treatment is complete. C. Do not disengage the wire from the brack- ets, leave as is, and send the patient for en- dodontic evaluation. D. Continue with the orthodontic treatment, but disengage tooth #9 and send patient for A. Thumb sucking endodontic evaluation. B. Tongue thrusting C. Lower lip sucking 14. A 10-year-old girl has teeth #12, #13, #14, and D. Skeletal in origin #15 in crossbite. The orthodontist has placed a palatal expander as part of her treatment. 13. A 14-year-old boy is ongoing a comprehensive Which of the following statements are correct? orthodontic treatment and presents for a regu- lar orthodontic visit. Upon intraoral examina- A. After activation is complete, the expander tion, the physician finds a fistula above tooth #9 remains in the mouth for at least 3 months. and takes a radiograph, which is shown below. B. After activation is complete, the expander What is the next best step of treatment for this can be taken off right away. patient? C. The expander is activated once a day by the patient. D. The expander is never activated by the pa- tient, only by the physician. E. The expander is activated 0.5 mm each turn.

Orthodontics F. The expander is activated 1 mm each turn. Questions: 12-21 99

15. A 20-year-old man is finishing his compre- D. Reposition occurring at the anterior surface hensive orthodontic treatment that involved a of the ramus and apposition of bone oc- rotated tooth #7. What is the next step of treat- curring along the posterior surface of the ment? ramus

A. Perform circumferential supracrestal fibro- 18. What is the most common cause of an anterior tomy prior to removal of orthodontic appli- crossbite of maxillary teeth in young children? ances. B. Perform circumferential supracrestal fibro- A. Oral habits tomy after removal of orthodontic appli- B. Trauma to the incisors ances. C. Jaw size discrepancies C. Perform circumferential subcrestal fibro- D. Prolonged retention of primary teeth tomy prior to removal of orthodontic ap- E. All of the above pliances. D. Perform circumferential subcrestal fibro- 19. In orthodontic therapy, adult patients in com- tomy after removal of orthodontic appli- parison with adolescent patients: ances. A. Need less periodontal maintenance during E. Circumferential fibrotomy should not be comprehensive orthodontic treatment performed. B. Are more compliant C. Are less concerned with esthetic 16. What site is responsible for mandibular vertical D. Are more prone to decalcification stains on growth and how will this site affect the facial enamel height during puberty? 20. A 7-year-old boy was previously diagnosed with A. Coronoid process, increase in posterior fa- marked “bowing of the femurs” and decreased cial height radiographic bone opacity. What disease does B. Coronoid process, no change in posterior this patient have? What would you notice upon facial height the examination of this patient’s oral cavity and C. Body of mandible, no change in posterior intraoral radiographs? facial height D. Condylar cartilage, increase in posterior fa- I. Hyperthyroidism cial height II. Rickets E. Condylar cartilage, no change in posterior III. Scurvy facial height IV. Missing or delayed tooth eruption F. Ramus of the mandible, increase in poste- V. Premature exfoliation of primary teeth rior facial height VI. Normal eruption pattern A. III and VI 17. Which process does mandibular growth follow? B. III and V A. Reposition occurring at the posterior sur- C. I and VI face of the ramus and apposition of bone D. I and V occurring along the posterior surface of the E. II and IV ramus as well F. II and V B. Reposition occurring at the anterior surface 21. All these conditions are associated with super- of the ramus and apposition of bone occur-

numerary teeth except: Orthodontics ring along the anterior surface of the ramus as well A. Gardner syndrome C. Reposition occurring at the posterior sur- B. Down syndrome face of the ramus and apposition of bone C. Ectodermal dysplasia occurring along the anterior surface of the D. Cleidocranial dysplasia ramus E. Sturge–Weber syndrome 100 Chapter 4: Orthodontics

22. A 10-year-old boy presents with severe crowd- A. Anterior open bite ing. The patient has a bilateral class I malocclu- B. Class I sion and discrepancy in all four quadrants of 5 C. Class II mm. As a part of the comprehensive orthodon- D. Unilateral crossbite tic treatment, this patient needs several teeth extracted. Permanent incisors and the first per- 24. What is the most efficient way to move teeth, manent molars are the only permanent teeth reinforce anchorage, and avoid friction in the erupted. Which teeth need to be extracted to appliance system? correct the malocclusion? A. Apply gentle constant force. A. Permanent lateral incisor → primary ca- B. Incorporate springs into archwire. nine → primary first molars C. Incorporate interarch elastic. B. Primary first molars → permanent first pre- D. Bond magnets to individual teeth. molars → permanent second premolars E. All of the above C. Primary canines → primary first molars → primary second molars 25. There are multiple forces being applied to teeth D. Primary first molars → primary second #8 and #9 by the step bend between two teeth. molars → permanent first premolars What type of force is being applied to each E. Primary canines → primary first molars → tooth? permanent first premolars

23. Interarch elastics are sometimes used as an aid tool in treatment of malocclusions. What mal- occlusion is being treated with the help of the interarch elastic mentioned below?

A

A. Teeth #8 and #9 are being intruded. B. Teeth #8 and #9 are being extruded. C. Tooth #8 is being intruded and #9 is being extruded. D. Tooth #8 is being extruded and #9 is being intruded. E. Teeth #8 and #9 are being rotated. B Orthodontics Questions: 22-30 101

26. A 5-year-old girl has her molar occlusion in A. Carpal bones of the wrist flush terminal plane. What is most likely to be B. Metacarpals of the hands her occlusion at age 10 with proper adequate C. Phalanges of the fingers space management? D. All of the above

28. What type of tooth movement does the diagram below represent?

A. Class I B. Class II division 1 C. Class II division 2 D. Class III

27. Physiologic age of the patient can be deter- mined by examining the ossification pattern of:

A. Rotation B. Translation C. Tipping D. Intrusion E. Extrusion

29. What is the condition that may complicate mo- lar uprighting? A. Open bite B. Presence of periodontal disease C. Extruded maxillary and mandibular molars D. Molar having short roots E. All of the above

30. Which of the following malocclusions is the

easiest to maintain after orthodontic treatment? Orthodontics A. Expansion B. Rotation C. Anterior crossbite D. Generalized spacing E. Generalized crowding 102 Chapter 4: Orthodontics

31. Which is a true statement about edgewise clusion. The patient does not want “a mouth brackets? full of metal.” What is the best treatment option for this patient? A. The magnitude of the forces generated in the faciolingual and occlusogingival direc- tion is dependent on the bracket slot size. B. Twin bracket is used to gain more rotational control. C. Single-wing bracket is used to gain more rotational control. D. All statements above are true.

32. Why would an orthodontist place a nonrigid archwire at the beginning of treatment? A. The appliance may break if a rigid archwire is placed. B. The nonrigid archwire is more pliable, which is needed in the initial stages of treat- ment of malocclusions. C. The rigid archwire may become perma- nently deformed if placed at the beginning of treatment. D. All of the above are correct.

33. Which statement is true regarding Invisalign treatment? A. Aligners are worn 24 hours a day and taken off for eating, drinking, and oral hygiene. B. Change set of new aligners every two weeks. C. Focus is esthetic and functional alignment without utilization of complex orthodontic auxiliary treatments. D. Treatment is focused on straightforward cases (20 aligners or less). E. All of the above are true.

34. Which case would satisfy the Invisalign crite- ria? A. More than 2 mm crowding between any two anterior teeth A. Place brackets only on anterior teeth. B. Class III bilateral molar relationship B. Place brackets on the lingual surface. C. Patient with moderate to severe periodontal C. Treat her mild spacing with the Invisalign disease product. D. A 9-year-old patient D. Refer her to a specialist for possible place- E. Patient must have fully erupted second ment of laminates. molars. 36. This is a stress–strain graph. Each letter is an 35. A 24-year-old woman presented to an orthodon- important part of the elasticity curve. What do tist’s office to fix the spaces between her teeth. the letters X, Y, and Z represent, pertaining to

Orthodontics Intraoral examination shows class I molar oc- the graph below? Questions: 31-43 103

Refer to this picture for questions 40–44. X

Stress Z Y

0 0 Strain

I. X = Plastic limit II. X = Elastic limit III. Y = Module of plasticity IV. Y = Module of elasticity V. Z = Plastic range VI. Z = Elasticity range A. I, III, and VI 40. What type of movement is shown above? B. I, IV, and V A. Rotation C. I, IV, and VI B. Translation D. II, III, and V C. Tipping E. II, III, and VI D. Intrusion F. II, IV, and V E. Extrusion 37. What are the determining factors of the load– 41. What will be detected in the dark zones after deflection rates of wires? light continuous force is applied for a few min- A. Modules of elasticity utes? B. Length of wire A. Osteoclasts C. Cross-sectional area B. Osteoblasts D. All of the above C. Both osteoblasts and osteoclasts 38. What type of tooth movement are open-coil D. Neither osteoblasts nor osteoclasts springs used for? E. Cyclic adenosine monophosphate (cAMP) A. Close spaces between teeth 42. What will be detected in the dark zones with ap- B. Upright teeth plication of light continuous force for 4 hours? C. Intrusion A. Osteoclasts D. Translation B. Osteoblasts 39. When is the best time to refer a patient to an or- C. Both osteoblasts and osteoclasts thodontist for the maximum improvement with D. Neither osteoblasts nor osteoclasts minimum treating time? E. cAMP A. At approximately age 3, when all the pri- 43. What will be detected in the dark zones with mary teeth are present application of light continuous force for 2 days? Orthodontics B. At approximately age 7, when there is mixed A. Osteoclasts dentition B. Osteoblasts C. At approximately age 13, when all the per- C. Both osteoblasts and osteoclasts manent dentition is present D. Neither osteoblasts nor osteoclasts D. All of the above are correct. E. cAMP 104 Chapter 4: Orthodontics

44. What will be detected in the dark zones with application of heavy continuous force for 4 hours? A. Increase in cAMP levels B. Cell differentiation C. Cell death D. Release of prostaglandins and cytokines

45. Methods employed in orthodontic treatment to align a crowded dentition include: A. Extraction of teeth B. Expansion of arches C. Distalization of teeth D. Interproximal reduction of teeth E. All of the above A. Perform a frenectomy prior to orthodon- tic treatment and after maxillary canines 46. Which of the following will require the most erupt. force to move tooth #27? B. Perform a frenectomy prior to orthodon- A. Tipping tic treatment and before maxillary canines B. Translation erupt. C. Root uprighting C. Perform a frenectomy after orthodontic D. Rotation treatment and after maxillary canines erupt. E. Extrusion D. Perform a frenectomy after orthodontic F. Intrusion treatment and before maxillary canines erupt. 47. Below are the indications for using bands in- E. Do not perform a frenectomy, and proceed stead of brackets, except for: with the orthodontic treatment. A. Patient with amelogenesis imperfecta 50. Which one of the following statements is false B. Teeth with long clinical crowns about headgear? C. Patient with cuspal interference A. Headgear is typically used in skeletal class D. Teeth that will need both lingual and labial II. attachment B. Headgear needs to be worn 10 to 14 hours per day to be effective. 48. Which one of the following statements is true C. Treatment length is 6 to 18 months. about primate space in the primary dentition? D. Headgear can be used in adult patients. A. Primate space is located between ca- E. All of the above statements are false. nine and first molar in the mandibular F. All of the above statements are true. arch. B. Primate space is located between canine 51. The wrist–hand radiograph is used by or- and first molar in the maxillary arch. thodontists to predict the time of the pubertal C. It should be present in permanent teeth. growth spurt and thus jaw growth. What is ex- D. It should be present in permanent teeth and amined in the wrist–hand radiograph? deciduous teeth. A. Carpal bones of the wrist B. Metacarpal bones of the hand 49. A 10-year-old girl presents to the office request- C. Phalanges of the fingers ing to close her diastema. What is the correct D. All of the above treatment for this patient? Orthodontics Questions: 44-62 105

52. What is the most common site to find a super- A. Class I numerary tooth? B. Class II division 1 A. Distal to the third molars C. Class II division 2 B. Between the central incisors D. Class III C. Between the central and lateral incisors 59. In class II occlusion, in what relationship is the D. Inferior border of the mandible maxillary canine to the mandibular teeth? 53. Which of the following is not considered a A. Mesial to the embrasure between mandibu- method of closing a diastema? lar canine and mandibular first premolar A. Posterior bite plate B. Between the mandibular canine and man- B. Lingual arch with finger spring dibular first premolar C. Hawley appliance with finger spring C. Distal to the embrasure between the D. Cemented orthodontic bands with inter- mandibular canine and mandibular first tooth traction premolar D. Lingual to the mandibular canine 54. Which of the following is used to maintain the space of one prematurely missing primary molar? 60. The left molar is in a class II relationship, whereas the right molar is in a class I relation- A. Lingual holding arch ship. B. Distal shoe C. Band and loop A. Class II division 1 subdivision left D. A and B B. Class II division 1 subdivision right E. B and C C. Class II division 1 subdivision 1 D. Class I division 2 subdivision left 55. A pediatric patient has premature loss of tooth #A, but tooth #B is present, and #3 has not 61. What is the sequence of treatment for an im- erupted yet. What is the best space maintainer pacted maxillary canine? for this situation A. Extract overretained primary tooth, surgi- A. Hawley with finger spring cally expose canine, place bracket, pull into B. Hawley without finger spring arch through keratinized tissue. C. Distal shoe B. Surgically expose canine, place bracket, ex- D. Nance appliance tract overretained primary tooth once ca- nine has been pulled into arch as far as 56. When is a Nance appliance indicated? possible. A. Bilateral loss of primary maxillary molars C. Extract overretained primary tooth, surgi- B. Bilateral loss of primary mandibular molars cally expose canine, place bracket, pull into C. Loss of one primary molar arch through alveolar mucosa. D. To control thumb-sucking behavior D. Wait for canine to descend so that it is vis- ible through palatal tissue, place bracket, 57. What percent of the population is in class I extract overretained primary tooth, pull ca- occlusion? nine into arch through keratinized tissue.

A. Less than 5% 62. What is the name given to the line connecting B. 25% the upper border of the external auditory canal C. 50% and the lower orbital rim? Orthodontics D. 70% A. Frankel plane 58. The mesiobuccal cusp of the maxillary first mo- B. Frankfort plane lar is between the mandibular first molar and C. Posselt envelope of motion mandibular second molar. What classification D. Sagittal plane of occlusion is this? E. Angle plane 106 Chapter 4: Orthodontics

63. Identify pogonion on the lateral cephalometric tracing.

A. 2 B. 14 C. 6 D. 7 A. 1 67. Which is represented by #9 on the lateral B. 11 cephalometric radiograph? C. 12 D. 13

64. Spacing is normal throughout the anterior sec- tion of the primary dentition. The spaces are called primate spaces and are found: A. Between the lateral incisors and canines in the maxilla B. Between the canines and first molars in the mandible C. Between the lateral incisors and canines in the mandible D. A and B E. B and C

65. On the lateral cephalometric radiograph, the A. ANS craniometric point orbitale refers to: B. Point A A. Lowest point on the inferior margin of the C. Point B orbit D. Menton B. Most anterior point on anterior wall of the E. Pogonion orbit C. Highest point on the superior margin of the 68. Where is nasion found on the lateral cephalo- orbit metric radiograph? D. Most posterior point on posterior wall of A. The point of intersection between the orbit shadow of the zygomatic arch and the pos- terior border of the ramus 66. Identify basion on the lateral cephalometric B. Anterior point of the intersection between tracing. the nasal and frontal bones Orthodontics Questions: 63-79 107

C. Intersection between the occipital and ba- 74. A patient’s maxillary and mandibular midlines sisphenoid bones are not coincident, and there is a mandibu- D. Between the menton and pogonion lar shift noted as the patient closes his mouth. What type of crossbite is likely in this patient? 69. Which is the goal of molar uprighting? A. Functional A. Better lifetime of restorations due to im- B. Skeletal proved direction and distribution of oc- clusal forces 75. Which of the following are types of removable B. Better periodontal prognosis retainers? C. Easier access should endodontic/prosthetic A. Lingual bonded retainer treatments become necessary B. Hawley retainer D. Better alveolar contour C. Essix retainer E. All of the above D. A and B 70. What are the properties of an ideal archwire? E. B and C F. All of the above A. High strength B. Low stiffness 76. What is the drawback of a palatal bonded re- C. High range tainer? D. High formability A. Occlusal interference E. All of the above B. Aesthetics 71. Which of the following alloys is not currently C. Patient compliance used in archwires? D. Ability to prevent relapse of midline di- astemas A. Nickel and titanium B. Copper, tin, and titanium 77. What are the properties of the alveolar ridge? C. Beta titanium (titanium and molybdenum) A. It exists to support teeth. D. Stainless steel and cobalt chromium B. It does not form if a tooth fails to erupt. 72. Which of the following is not a possible nega- C. It resorbs after a tooth is extracted. tive sequella associated with orthodontic treat- D. It grows in height and length. ment? E. All of the above A. Root resorption with fast tooth movement 78. When is postorthodontic circumferential sup- or prolonged treatment racrestal fibrotomy indicated? B. Development of gingivitis A. To prevent mesial drift of the mandibular C. Xerostomia incisors D. Decalcification of enamel around brackets/ B. To prevent maxillary lateral incisors from bands relapsing into their previously rotated posi- 73. Why is a palate expander kept in the patient’s tion mouth 3 to 6 months after expansion is com- C. To maintain the buccal plate after move- pleted? ment of an impacted canine D. To control thumb-sucking behavior A. To give patients time to improve oral hy- giene 79. Which is the normal relationship between pri- Orthodontics B. To allow for bone fill/reorganization in the mary molars? midpalatal suture region A. Flush terminal plane C. To allow third molars to erupt B. Mesial step D. To prevent derangement of the temporo- C. Distal step mandibular joint (TMJ) 108 Chapter 4: Orthodontics

80. The distal step relationship in primary molars A. Loss of primary incisors due to early child- often translates to which classification of occlu- hood caries sion in the permanent molars? B. Severe class I malocclusion in mixed den- tition stage, with insufficient arch length A. Class I C. Congenitally missing maxillary lateral in- B. Class II cisors C. Class III D. To prevent mesial drift and subsequent crowding of mandibular incisors after all 81. Which of the following are considered systemic permanent teeth have erupted causes for delayed tooth eruption? A. Hereditary gingival fibromatosis 87. Which of the following conditions is not asso- B. Down syndrome ciated with supernumerary teeth? C. Rickets A. Gardner syndrome D. All of the above B. Cleidocranial dysplasia C. Hyperthyroidism 82. An orthodontist has moved the crown of tooth D. Down syndrome #8 in one direction, whereas the roots were dis- E. Sturge–Weber syndrome placed in the opposite direction about an axis of rotation. This is the definition of: 88. Serial extraction refers to the orderly removal A. Tipping of selected primary and permanent teeth in B. Bodily movement patients with severe arch length discrepancy. C. Rotation Which teeth are extracted, and in what order? D. Extrusion A. Primary canines, primary first molars, per- manent first premolars 83. Which orthodontic force moves the tooth into B. Primary canines, primary first molars, pri- the socket along the long axis of the tooth? mary second molars A. Extrusion C. Primary first molars, primary second mo- B. Intrusion lars, permanent first premolars C. Torque D. Primary first molars, permanent first pre- D. Tipping molars 89. What is not included in a routine set of or- 84. Which cells are pivotal for the process of tooth thodontic diagnostic records? movement through bone? A. Intraoral photographs A. Red blood cells and white blood cells B. Computed tomography of the temporo- B. Osteoclasts and osteoblasts mandibular joint C. Neutrophils C. Panoramic radiograph D. Oligodendroglia D. Impressions for dental casts of both arches E. Wax or polyvinyl siloxane record of occlu- 85. What are the average distances of mandibular sion leeway space and maxillary leeway space per F. Lateral cephalometric radiograph side, respectively? A. 2.5 mm, 1.5 mm 90. Which of the following bones is formed by in- B. 3 mm, 4 mm tramembranous ossification? C. 3 to 4 mm, 2 to 2.5 mm A. Maxilla D.5to6mm,4to5mm B. Mandible C. Femur 86. Which of the following scenario is a serial ex- D. Ethmoid

Orthodontics traction indicated in? E. Temporal Questions: 80-100 109

F. A and B 96. What is the purpose of mixed dentition G. C, D, and E analysis? H. A and E A. To predict the number of hours per day I. All of the above the patients must wear their headgear ap- 91. Bone grows by the addition of new layers on pliance. previously formed layers. What is this process B. To predict the amount of spacing or crowd- called? ing that will be present after the permanent teeth erupt. A. Endochondral growth C. To predict the need for a space maintainer B. Appositional growth if the primary second molar is lost prema- C. Intramembranous growth turely. D. Interstitial growth D. To determine the type of retainer necessary for the patient. 92. What is the normal overjet in class I occlusion? A. –2 mm 97. What is the mechanism of action of headgear? B. 0 mm A. Used in skeletal class II patients to hold the C.1to2mm growth of the maxilla back while allowing D. 4 mm growth of the mandible to catch up. B. Used in skeletal class III patients to protract 93. Bimaxillary protrusion is a profile type that pro- the maxilla vides more space to accommodate teeth and C. Used in class I malocclusion patients alleviate crowding. The clinical picture of bi- D. A and B maxillary protrusion is: E. B and C A. The mandibular teeth lean buccally F. All of the above B. The maxillary teeth lean buccally C. The teeth in both arches lean buccally 98. The norm for SNA is 82 ± 2 degrees in the Cau- D. The teeth in both arches lean lingually casian population. What does an SNA value of 85 in a Caucasian patient indicate? 94. When is it preferable to use bands instead of A. Maxillary prognathism brackets in orthodontic treatment? B. Maxillary retrognathism A. To resist breakage in areas of heavy masti- C. Mandibular prognathism cation or occlusal interference D. Mandibular retrognathism B. When both labial and lingual attachment are needed 99. What does an ANB angle of 2 to 4 degrees in- C. For teeth with short clinical crowns dicate? D. For tooth surfaces that are incompatible A. Class I skeletal pattern with bonding B. Class II skeletal pattern E. All of the above C. Class III skeletal pattern 95. How is the edgewise appliance different from 100. ANB is a measurement gathered from a lateral the Begg appliance? cephalometric radiograph, used to determine A. The slot is horizontally positioned in the the magnitude of the skeletal jaw discrepancy. edgewise appliance. How is ANB determined? Orthodontics B. No wires are necessary. A. SNA minus SNB C. The slots are vertically positioned in the B. SNA plus SNB edgewise appliance. C. ANS-Me minus SNB D. The edgewise appliance is made of porce- D. SN-MP minus SNB lain instead of metal. 110 Chapter 4: Orthodontics

101. All of the following are analyses commonly A. 70% used to study cephalometric radiographs B. 25% except one: C. Less than 5% D. Less than 1% A. Steiner B. Wits 107. Class II division 1 subdivision right means: C. Downs D. COGS A. The right molar is in a class II relationship, E. Richards whereas the left molar is in a class I rela- tionship. 102. If a person has a severe skeletal class II maloc- B. The right molar is in a class I relationship, clusion, one would typically expect to see this whereas the left molar is in a class II rela- manifest clinically with a: tionship. C. Both molars are in class I relationship, A. Severe underbite whereas maxillary incisors are in linguover- B. Severe overjet sion. C. A protuberant chin D. Both molars are in class II relationship, D. whereas maxillary incisors are in labiover- E. Excessive occlusal wear sion. 103. What is a TAD? 108. Each of the following statements is cor- A. Transmural advancement device rect about pseudo–class III malocclusion B. Temporary anchorage device except: C. Topical anesthetic distributor A. It describes a situation where the patient D. Training assessment for dentistry adopts a jaw position upon closure which is E. Torque application distributor forward to normal. B. A pseudo–class III patient typically presents 104. A “Bolton discrepancy” refers to: with an edge-to-edge bite. A. The amount of overjet C. This patient cannot bring the mandible B. The amount of overbite back without strain; and therefore, the C. The difference in size between the maxil- mandibular incisors will not touch the max- lary and mandibular teeth illary incisors. D. The difference in transverse width between D. This is a milder form of the true class III the maxilla and the mandible malocclusion. E. The difference in anteroposterior length be- tween the maxilla and the mandible 109. Crowding of the permanent incisors in the early mixed dentition typically results in crowding in 105. A rapid palatal expander (RPE) is not effective the permanent dentition. in creating skeletal expansion in adults. After The premature loss of the primary canines, par- the fusion of the mid-palatine suture during ticularly in the mandibular arch is indicative of adolescence, an RPE will produce dental tip- incipient malocclusion. ping rather than true skeletal expansion. A. Both statements are TRUE. A. Both statements are TRUE. B. Both statements are FALSE. B. Both statements are FALSE. C. The first statement is TRUE, the second is C. The first statement is TRUE, the second is FALSE. FALSE. D. The first statement is FALSE, the second is D. The first statement is FALSE, the second is TRUE. TRUE. 110. Each of the following speech sounds are cor- 106. What percentage of the population manifests rectly linked to their related malocclusion

Orthodontics class II malocclusion? except: Questions: 101-118 111

A. “s,” “z”: anterior open bite, large gap be- measured at the intersection formed by tween incisors mandibular plane and the sella–nasion line. B. “t,” “d”: lingually positioned maxillary in- A. Both the statement and the reason are cor- cisors rect and related. C. “f,” “v”: skeletal class II B. Both the statement and the reason are cor- D. “th,” “sh,” “ch”: anterior open bite rect but NOT related. C. The statement is correct, but the reason is 111. Each of the following is a contraindication to NOT. molar uprighting except one. Which one is this D. The statement is NOT correct, but the rea- EXCEPTION? son is correct. A. High mandibular plane E. NEITHER the statement NOR the reason B. Presence of root resorption is correct. C. Severe lingual inclination of the tooth in addition to mesial tipping 115. Porion is located: D. Occlusal plane discrepancy A. In the center of external auditory canal E. Presence of periodontal disease B. Below the external auditory canal F. Low mandibular plane C. In front of the external auditory canal D. Behind the external auditory canal 112. Adequate space should be provided in the arch E. On the outer upper margin of the external before attempting to pull an impacted tooth auditory canal into position. During surgical exposure, flaps should be re- 116. All of the following are indications for remov- flected so that the impacted tooth is pulled into able appliances except: unkeratinized tissue. A. Limited tipping movement A. Both statements are TRUE. B. Retention after comprehensive treatment B. Both statements are FALSE. C. Growth modification during mixed denti- C. The first statement is TRUE, the second is tion FALSE. D. Close extraction spaces fully D. The first statement is FALSE, the second is 117. Which one of the following is a disadvantage of TRUE. removable appliances versus fixed appliances? 113. Which of the following is an application of A. Cannot achieve two-point tooth contact, cephalometric radiographs? therefore, making bodily tooth movement impossible A. It is the only method to determine the Angle B. Improved hygiene classification. C. Increased patient comfort B. It can be used to analyze treatment re- D. Decreased chair time sults. C. It is the only reliable method to determine 118. Each of the following statements about head- which teeth to extract. gear is correct except? D. It is the only radiograph that determines the status of wisdom teeth. A. Headgear needs to be worn 10 to 14 hours/ E. It is used to determine the exact inclination day to be effective. and location of an impacted tooth. B. High-pull headgear places a distal and in- Orthodontics trusive force on maxillary molars. 114. A flat mandibular plane angle correlates with C. Cervical-pull headgear places a distal and a long anterior vertical dimension and an an- intrusive force on maxillary molars. terior open-bite malocclusion because cep- D. Reverse-pull headgear is used in class III halometrically, the mandibular plane angle is malocclusions to protract the maxilla. 112 Chapter 4: Orthodontics

119. An example of a tissue-borne functional appli- C. In teeth with short clinical crowns ance is: D. In amelogenesis imperfecta cases E. In patients with stainless steel crowns A. Bionator B. Herbst 124. All of the following statements regarding cross- C. Frankel bite are true except: D. Forsus A. A skeletal crossbite has smooth closure to 120. Functional appliances are designed to modify centric occlusion. growth during mixed dentition. B. A functional crossbite manifests as a devia- Functional appliances have a dental effect tion in upper and lower midline as the pa- only. tient closes. C. Posterior crossbites may be associated with A. Both statements are TRUE. mandibular shifts. B. Both statements are FALSE. D. Mild anterior crossbites can be corrected at C. The first statement is TRUE, the second is later stages of treatment. FALSE. E. After RPE is used, it must remain intraorally D. The first statement is FALSE, the second is for 3 to 6 months for bone to form in the TRUE. midpalatal suture region

121. Which of the following is NOT a negative 125. All of the following statements regarding open sequellae associated with orthodontic treat- bite are true except: ment? A. Maxillary constriction usually seen in open- A. Orthodontic appliances decrease the risk of bite cases results from increased pressure gingivitis. on the mylohyoid muscles from finger B. Prolonged orthodontic treatment is associ- sucking. ated with root resorption. B. The resultant maxillary constriction seen C. Prolonged orthodontic treatment is associ- in open-bite cases tends to cause bilateral ated with decalcification stains on enamel. crossbites. D. Orthodontic appliances may increase the C. If the finger-sucking habit involves the hand risk of gingivitis due to irritation to gingiva resting on the chin, mandibular growth can or mucosa. get retarded producing a class II retrog- nathic profile. 122. In straight-wire appliance: D. Compensatory tongue thrust habit is often A. Torque in bracket slots compensate for observed in patients with open bite. mesial and distal inclination of teeth. E. Combined orthodontic and surgical treat- B. Angulation of bracket slot relative to the ment is required in patients who show an ex- long axis of the tooth allow for facial or lin- cess of anterior vertical facial height, steep gual inclination of the teeth. mandibular plane angle, or long lower fa- C. Variation in bracket thickness compen- cial heights. sates for incisal or gingival positioning of teeth. 126. Which of the following is NOT a treatment D. The brackets are designed with a built-in modality for open-bite cases? prescription to eliminate wire bends. A. Tongue crib B. RPE 123. Each of the following statement is indication C. Posterior bite plates for using bands instead of brackets except: D. High-pull headgear A. In heavy mastication areas E. Straight-pull headgear B. In situations where only lingual attach-

Orthodontics ments are needed Questions: 119-133 113

127. The correct name of the appliance below is: 130. The correct name of the following appliance is:

A. Tongue crib B. Bluegrass appliance A. Nance C. Transpalatal bar B. Band and loop D. Lower lingual holding arch C. Distal shoe D. Transpalatal bar 128. All of the following statements regarding types of space maintainers are true except: 131. All of the following are examples of removable retainers except: A. Band and loop appliance prevents mesial migration of primary second molar after A. Wrap-around unilateral loss of primary first molar. B. Hawley B. Distal shoe appliance prevents mesial mi- C. Essix gration of primary second molar after uni- D. Bonded lingual wire lateral loss of primary first molar. 132. Which of the following statements is NOT a C. A Nance appliance is used to prevent mesial rationale for retention? rotation and mesial drift of permanent max- illary molars when primary maxillary mo- A. To minimize changes due to growth lars have been bilaterally lost. B. To maintain teeth in unstable conditions D. Premature loss of a primary canine may C. To allow for reorganization of the gingival be due to arch length deficiency, and it re- and periodontal tissues sults in lingual collapse of mandibular an- D. The occlusal result of hard tissue is modi- terior. fied and maintained by retention.

129. Early loss of primary second molar leads to 133. Retention can be accomplished with either mesial tipping and rotation of permanent first fixed or removable retainers. molars. Anterior crossbite is retained after orthodontic When primary second molars are lost, no space correction by the overjet achieved during treat- maintenance is necessary. ment. Orthodontics A. Both statements are TRUE. A. Both statements are TRUE. B. Both statements are FALSE. B. Both statements are FALSE. C. The first statement is TRUE, the second is C. The first statement is TRUE, the second is FALSE. FALSE. D. The first statement is FALSE, the second is D. The first statement is FALSE, the second is TRUE. TRUE. 114 Chapter 4: Orthodontics

134. Which of the following statements are cor- 138. All of the following are correct with regards to rect regarding postorthodontic circumferential treatment of a diastema except: supracrestal fibrotomy? A. Spaces will always close as permanent ca- A. It is indicated for rotated maxillary lateral nines erupt. incisors because the supraalveolar tissue is B. Diastema of 2 mm or less usually closes significantly responsible for relapse of or- spontaneously if lateral incisors are in good thodontically rotated teeth. position. B. The incision is made at the crest of the bone C. Teeth should be aligned first if the diastema where all the collagen fibers are inserted is caused by an abnormal frenum. into the root of the tooth. D. In such cases, a frenectomy should be per- C. This procedure eliminates potential for re- formed after canines have erupted. lapse due to collagen fiber retraction. D. This procedure allows new fibers to form 139. All of the following are methods to close a di- that will help retain the tooth in its new astema except: position. A. Lingual arch with finger spring E. All of the above are correct. B. Hawley appliance with finger spring C. Cemented orthodontic bands with inter- 135. A distal step means: tooth traction A. Edge-to-edge position of cusps of perma- D. Using an Essix appliance nent maxillary and mandibular first molars, leading to Angle class II malocclusion 140. All of the following statements are true B. Distal of maxillary first molars is mesial to except: the distal of mandibular molars, leading to A. Nonnutritive sucking habit leads to mal- Angle class II malocclusion occlusion only if it continues during the C. Distal of maxillary first molars is distal to mixed dentition stage. the distal of mandibular molars, leading to B. Adenoids are a definite etiologic agent in a Angle class II malocclusion “long-face” pattern of malocclusion D. Incisal edge of maxillary incisors is distal to C. Tongue trust swallow originates from the the incisal edge of mandibular incisors. need to attain an oral seal, especially in an- terior open bite cases. 136. The maxillary primate space is located between D. Tongue trust swallow is considered the re- lateral incisors and canines. sult of displaced incisors, not the cause. The mandibular primate space is located be- tween canines and first premolars. 141. All of the following statements are true A. Both statements are TRUE. except: B. Both statements are FALSE. A. Intramembranous ossification takes place C. The first statement is TRUE, the second is in membranes of connective tissue. FALSE. B. Osteoprogenitor cells in the membrane dif- D. The first statement is FALSE, the second is ferentiate into osteoblasts. TRUE. C. The maxilla and mandible are formed via endochondral ossification. 137. Which one of the following are causes of a di- D. Short and long bones and ethmoid, sphe- astema: noid, and temporal bones are formed via A. Tooth size discrepancy endochondral ossification. B. Mesiodens C. Abnormal frenum attachment 142. Growth of bone occurs via appositional growth D. Normal stage of development phenomenon and interstitial growth phe- nomenon.

Orthodontics E. All of the above Questions: 134-150 115

Growth of cartilage occurs via appositional 147. What causes disc displacement and the sound growth only. of “popping” after trauma to the joints? A. Both statements are TRUE. A. Fractured condylar head B. Both statements are FALSE. B. Stretched or torn ligaments opposing the C. The first statement is TRUE, the second is action of pterygoid muscle FALSE. C. A and B D. The first statement is FALSE, the second is D. None of the above TRUE. 148. All of the following are true in regards to growth 143. Minor imperfections in the occlusion can trig- of the mandible except: ger clenching and grinding and lead to tem- A. Resorption of bone occurs along the ante- poromandibular disease or disorder (TMD). rior surface of the ramus. Condylar cartilage a critical growth center of B. Apposition of bone occurs along the poste- the mandible. rior surface of the ramus. A. Both statements are TRUE. C. Growth at the mandibular condyle during B. Both statements are FALSE. puberty usually results in increased poste- C. The first statement is TRUE, the second is rior facial height. FALSE. D. The main growth thrust of the condyle is D. The first statement is FALSE, the second is upward and forward to fill in the resultant TRUE. space to maintain contact with the base of the skull. 144. What is the single most important indicator of joint function? 149. In both arches, the permanent incisor tooth buds lie buccal and apical to primary incisors. A. Right and left excursions The permanent mandibular canines erupt B. Protrusion more facially but most often in line compared C. The amount of maximum opening with primary mandibular canines. D. Absence of pain A. Both statements are TRUE. 145. Orthodontic extractions (premolars) cause B. Both statements are FALSE. TMD. C. The first statement is TRUE, the second is TMD symptoms can disappear during or- FALSE. thodontic treatment. D. The first statement is FALSE, the second is TRUE. A. Both statements are TRUE. B. Both statements are FALSE. 150. Which one of the following is a systemic cause C. The first statement is TRUE, the second is of failure or delayed tooth eruption? FALSE. D. The first statement is FALSE, the second is A. Abnormal position of the crest TRUE. B. Supernumerary teeth C. Dilacerated roots 146. What are the general TMD classifications? D. Hereditary gingival fibromatosis E. Congenital absence A. Internal joint pathology and disk displace-

ments Orthodontics B. Disc displacements and myofascial pain C. Disc fractures and muscle fatigue D. Internal joint pathology and myofascial pain 116 Chapter 4: Orthodontics

151. Which of the following is correct regarding ec- 154. All of the following statements regarding serial topic eruption? extraction are correct except: A. Most likely to occur in the eruption of A. The indication is in severe class I maloc- maxillary first molars and mandibular in- clusion in mixed dentition with insufficient cisors. arch length. B. Much more common in the maxilla. B. The extraction sequence is primary canine, C. Treatment of ectopically erupting molar en- primary first molar, and permanent first pre- tails placing a brass wire between primary molar. second molar and permanent first molar to C. Extract first premolars after permanent ca- tip it distally. nines erupt. D. Ectopic eruption of mandibular lateral in- D. One should always use a lingual arch in cisors may lead to transposition of lateral mandible and Hawley appliance in maxilla incisor and canine. for support and retention. E. All of the above 155. The maxillary leeway space is bigger than 152. What is the gender predilection for supernu- mandibular leeway space, this will help with merary teeth in the permanent dentition? achievement of class I occlusion via late mesial shift of permanent first molars. A. 2:1 male:female B. 2:1 female:male A. Both the statement and the reason are cor- C. 4:1 male:female rect and related. D. 4:1 female:male B. Both the statement and the reason are cor- rect but NOT related. 153. Which of the following conditions is associated C. The statement is correct, but the reason is with multiple supernumerary teeth? NOT. A. Diabetes D. The statement is NOT correct, but the rea- B. Hyperthyroidism son is correct. C. Gardner syndrome E. NEITHER the statement NOR the reason D. Rickets is correct. Orthodontics Answers: 1-8 117

ANSWERS

1. The correct answer is A. In class II division is flat, it signifies a short vertical dimension 2, it is typical for maxillary centrals to have with a deep-bite malocclusion. Although an- linguoversion, whereas maxillary laterals are swer choice C is a correct statement, the ques- tipped in the labial mesial direction. Class II di- tion was asking for a steep angle. vision 1 typically has maxillary centrals tipped palatally and in a retruded position. Maxillary 6. The correct answer is A. The patient presented laterals are also tipped palatally. with a bilateral posterior crossbite, and since the patient is 35 years of age, the sutures have 2. The correct answer is C. The image shows a closed and the maxilla and mandible stopped dentition in class III malocclusion, which is growing. Surgery is one of the best treatment known as mesioclusion. Class II, not class III, options to correct a crossbite. is known as distoclusion. While the statement A rapid maxillary expander is an appliance in answer choice D is true, to classify maloc- that is used for a skeletal expansion of the up- clusion as class II: the first mandibular molar per arch in the primary or mixed dentition, needs to be distal to the first maxillary molar. in cases where the upper arch is very narrow Class I malocclusion has the mesiobuccal cusp and/or crossbite exists. of the maxillary first molar interdigitating with The quad helix expander is a fixed, spring- the buccal groove of the first mandibular molar. loaded orthodontic appliance that uses four he- lical springs and is used primarily to expand the 3. The correct answer is D. Pseudo–class III maxillary dental arch, in cases where it is very patients adopt a closed-jaw position that is for- narrow and/or crossbite exists. It is utilized at ward to normal and typically presents with an early stage of development to guide growth. an edge-to-edge bite. Bringing the mandible The W-arch is a fixed appliance constructed back with strain so that the mandibular in- of steel wire soldered to molar bands. The W- cisors touch the maxillary incisors depicts a arch is a form of rapid palatal expander (RPE) patient who has class III malocclusion. Bring- that is used for a skeletal expansion of the upper ing the mandible forward without strain so that arch in the primary or very early mixed denti- the mandibular incisors touch the maxillary tion, in cases where the upper arch is very nar- incisors depicts a class I malocclusion. Bring- row and/or crossbite exists. It is utilized at an ing the mandible forward with strain so that early stage of development to guide growth. the mandibular incisors touch the maxillary in- Bilateral posterior crossbite is corrected only cisors depicts a class II malocclusion. with orthodontic intervention.

4. The correct answer is A. Point B is also known 7. The correct answer is C. This plane is known as supramentale; #10 is point A; #12 is pogo- as the Frankfort-horizontal plane, which is nion; #13 is menton. constructed by drawing a line connecting po- rion and orbitale. Sella–nasion line makes the 5. The correct answer is D. The mandibular mandibular plane angle when intersected with plane angle is a line connecting gonion – the mandibular plane and is also used to es- gnathion (menton), and the sella–nasion. tablish the vertical position of the mandible. Mandibular plane angle determines open- or Gonion and gnathion lines create the deep-bite malocclusion by its measurement. mandibular plane. There is no plane connect- Orthodontics A steep angle signifies a long vertical dimen- ing the spheno-occipital synchondrosis and an- sion with an open-bite malocclusion. A steep terior nasal spine (ANS). angle signifies a long vertical dimension with an open-bite malocclusion, not deep-bite mal- 8. The correct answer is B. SNA angle is the occlusion. When the mandibular plane angle inferior–posterior angle between S–N line to 118 Chapter 4: Orthodontics

N–A line. It shows the anterior–posterior posi- is a deviation in maxillary and mandibular mid- tion of the maxilla in relation to cranial base. lines as the patient closes and has a smooth The adult norm is 82 ± 4 degrees. SNA angle closure to centric occlusion. Skeletal crossbite greater than 86 degrees is indicative of a maxil- results from a discrepancy in the structure of lary prognathism. SNA angles less than 78 de- the mandible or maxilla. grees indicate maxillary retrusion. SNA angles Dental crossbite is observed when a maxil- of 2 to 4 degrees indicate a class I skeletal pat- lary anterior tooth is lingual to the mandibular tern. SNA angle is not indicative of mandibular anterior teeth. position. Angle SNB is used to evaluate the an- Answer choice D is an example of a func- teroposterior position of the mandible. tional crossbite.

9. The correct answer is D. Reverse-pull head- 12. The correct answer is C. Lower lip sucking gear (also known as Delaire-type facemask) is presents with closed bite in addition to features used to place forward traction against the max- of thumb sucking. In this case, one can clearly illa. Cervical-pull headgear consists of a neck see that the lower lip can fit in the space be- strap connected to a face bow and places a distal tween upper and lower incisors. and extrusive force against the maxillary molars Thumb sucking causes malocclusion char- and maxilla. Straight-pull headgear is similar to acterized by flared and spaced maxillary in- cervical-pull headgear but places the force in cisors, lingually positioned lower incisors, a straight distal direction. High-pull headgear anterior open bite, and a narrow upper arch. consists of a head cap connected to a face bow Tonguethrust swallow does not always imply and places a distal and intrusive force on the an altered rest position or a malocclusion and maxillary molars and maxilla. does not cause an open bite. The maxillary an- terior teeth are severely protruded in this case, 10. The correct answer is B. Frankel functional ap- which is typically not a result of a tongue thrust pliance is the only tissue-borne appliance that habit. alters mandibular posture and contour of fa- This patient has a normal maxilla to cial soft tissue. Bionator is a tooth-borne appli- mandible relationship; therefore, this maloc- ance and advances mandible to an edge-to-edge clusion is not skeletal in origin. position to stimulate mandibular growth for correction of class II malocclusion. The Herbst 13. The correct answer is C. Sometimes, there is appliance is a tooth-borne appliance and splints a loss of tooth vitality during orthodontic treat- maxillary and mandibular framework together ment. Usually, there is a history of trauma to via pin that holds the mandible forward. An the tooth; however, poor control of orthodon- edgewise appliance is a tooth-borne appliance tic force may also be the cause of the periapical with horizontally positioned slot. pathology in this case. Do not disengage the wire from the brackets, leave as is, and send 11. The correct answer is C. A crossbite is clin- the patient for endodontic evaluation is the ically observed when teeth are on the wrong correct choice because the wire stabilizes the side of the opposing dentition. If there is no ev- tooth. idence of a discrepancy in the upper and lower Disengagement and removal of the archwire midlines when the mandible is at rest but a de- may cause the teeth to relapse and move toward viation of the mandible toward the side of the their original position. crossbite is noted when teeth are brought into Orthodontic treatment should be paused, occlusion and do not have smooth closure to rather than continued or discontinued, because centric occlusion, the malocclusion should be any changes in pressure may lead to deleterious considered functional in origin. effects to tooth #8. An endodontic evaluation A crossbite is clinically observed when teeth should be obtained as soon as possible. Disen- are on the wrong side of the opposing denti- gagement of the wire from tooth #9 may cause

Orthodontics tion. Skeletal crossbite is observed when there the tooth to extrude and become unstable. Answers: 9-25 119

14. The correct answer is A. After the activation is from dietary deficiency of water-soluble vita- complete, the expander remains in the mouth min C. There may be gingival involvement, for at least 3 to 6 months for proper bone for- which includes swelling, friability, bleeding, mation in the midpalatal suture region. The secondary infection, and loosening of teeth. expander is activated twice a day by the patient Hyperthyroidism is the result of increased se- and is activated 0.25 mm each turn. cretion of thyroxine (T4) by the thyroid gland and usually presents with premature exfoliation 15. The correct answer is A. Supraalveolar tissue of primary teeth. is responsible for the relapse of orthodontically relapsed teeth. Circumferential supracrestal 21. The correct answer is C. Ectodermal dyspla- fibrotomy should be performed prior to re- sia is usually associated with oligodontia (con- moval of orthodontic appliances, which elimi- genitally missing one or more teeth) and other nates the potential for relapse due to collagen missing ectodermal derivatives such as hair and fiber retraction. New fibers will form that will nails. aid in retention of the tooth in its new location. Although supernumerary or extra teeth can incidentally be found in healthy children; 16. The correct answer is D. The major site of they are much more common in children mandibular vertical growth is condylar carti- with Gardner syndrome, Down syndrome, clei- lage, which provides space between the jaws docranial dysplasia, and Sturge–Weber syn- into which the teeth erupt. Growth of the drome. The most common site for these is condyle during puberty usually results in the between central incisors (mesiodens). increase in posterior facial height. 22. The correct answer is E. Serial extraction is the 17. The correct answer is D. The mandible grows orderly removal of selected primary and perma- by reposition and opposition processes. Reposi- nent teeth in a predetermined sequence, which tion occurs at the anterior surface of the ramus is as follows: primary canines → primary first and apposition of bone occurs along the poste- molars → permanent first premolars. rior surface of the ramus.

18. The correct answer is E. Anterior crossbite of 23. The correct answer is D. Interarch forces have one or more of the permanent incisors is often side effects, such as widening or constriction of associated with prolonged retention of primary the dental arches and alteration of the occlu- teeth, as well as poor oral habits and trauma to sion; therefore, it is best used in treatment of incisors. unilateral crossbite.

19. The correct answer is B. Adult patients tend 24. The correct answer is E. All of the above to be more compliant with treatment than statements are correct. Gentle constant force are adolescents. All patients need periodontal must be applied for the survival of cells in the maintenance during comprehensive orthodon- periodontal ligament (PDL). Incorporation of tic treatment because fixed appliance makes springs into the archwire eliminates the diffi- it more difficult to maintain the same oral culty in anchorage control caused by frictional health. There is no significant difference bet- resistance. Reinforcement of anchorage can be ween adult and adolescent patients with respect produced by using interarch elastic. Magnets in to their concern for esthetics. Adult and adoles- attraction or repulsion could generate forces of the magnitude needed to move teeth and would

cent patients are equally predisposed to decal- Orthodontics cification stains on enamel. have the advantage of providing predictable force levels without direct contact or friction. 20. The correct answer is E. The data presented in the question are indicative of rickets disease sec- 25. The correct answer is C. A step bend between ondary to vitamin D deficiency, which presents two teeth produces intrusive force on tooth #8 with missing or delayed eruption. Scurvy results and extrusive force on tooth #9. 120 Chapter 4: Orthodontics

26. The correct answer is A. Flush terminal plane When expansion has been completed, a 3- is the normal primary molar teeth relationship. month period of retention with the appliance This causes edge-to-edge position of cusps of in place is recommended. A fixed appliance permanent maxillary and mandibular first mo- can be removed, but a removable retainer that lars. When mandibular molars drift forward, covers the palate is often needed as further in- the flush terminal plane becomes class I occlu- surance against early relapse. sion. When rotation has been completed, cir- The primary molars have to be in distal step cumferential supracrestal fibrotomy should be relationship to classify for class II occlusion. performed prior to removal of orthodontic ap- Deviation I and II are determined by the ante- pliances, which eliminates the potential for rior maxillary to mandibular teeth only. relapse due to collagen fiber retraction. New The primary molars have to be in mesial step fibers will form that will aid in retention of the relationship to be classified as class III occlu- tooth in its new location. sion. When treatment for generalized crowding or spacing has been completed, either a fixed 27. The correct answer is D. All of the above or removable appliance is needed to prevent statements are correct. Hand–wrist radiographs relapse. are used by some investigators to judge the skeletal age and development of the patient 31. The correct answer is D. All of the statements because physical maturity and skeletal devel- are true regarding edgewise appliances. The opment correlate well with jaw growth. magnitude of the forces generated in the fa- ciolingual and occlusogingival direction is de- 28. The correct answer is B. White circles indi- pendent on the bracket slot size. Both bracket cate the center of resistance at the starting tooth types (twin bracket and single-wing bracket) are position. Shaded circles show the center of re- used to gain more rotational control. sistance moved in the direction of the force. A force through the center of resistance causes all 32. The correct answer is D. All of the above state- points of the tooth to move the same amount ments are true. There are many irregularities in the same direction. This type of movement in malocclusions, and a rigid appliance may is called translation or bodily movement. Ro- break or create permanent deformation in the tation is the movement of a tooth around its archwire. If a rigid wire is used, the patient may longitudinal axis. Tipping is the movement of be in a lot of pain. a tooth in any direction while its apex remains in almost the original position. Intrusion is the 33. The correct answer is E. All of the above state- movement of the tooth in an inward direction. ments are true. Aligners are worn 24 hours a Extrusion is the movement of teeth beyond the day and taken off for eating, drinking, and oral natural occlusal plane that may be accompa- hygiene. Change your set of new aligners every nied by a similar movement of investing tissues. 2 weeks. The focus of Invisalign treatment is on 29. The correct answer is E. All of the above state- aesthetic and functional alignment without uti- ments are correct. Slow progress in molar up- lization of complex orthodontic auxiliary treat- righting in an adult patient is most likely due to ments. Treatment is focused on straightforward occlusal interferences, such as extruded maxil- cases requiring 20 or fewer aligners. lary and mandibular molar with an open bite al- 34. The correct answer is E. Eruption is a change ready present. Presence of periodontal disease in vertical movement—aligners made before or short clinical crown is not favorable for or- all the teeth are fully erupted (excluding third thodontic treatment. molar) will not fit, because they are not made 30. The correct answer is C. Corrected anterior to accommodate the erupting molar. crossbite is best retained by the normal incisor To satisfy the Invisalign criteria, there must

Orthodontics relationship. be less than 2 mm crowding between any two Answers: 26-42 121

anterior teeth. There also must be ideal class I position. Springs are not able to make intru- molar relationship. sive tooth movement, are usually not used in Thin tissue that is prone to recession is not translation, and alone could not accomplish favorable for any regular orthodontic treat- this type of tooth movement. ment, including Invisalign. Mild horizontal bone loss does not mean that the patient is 39. The correct answer is B. The American As- a poor candidate for Invisalign. However, pa- sociation of Orthodontists recommends an or- tients should be warned of possible risks that thodontic screening examination by age 7 (may may occur during treatment. be performed by a general dentist). The patient must be at least 14 years of age Three-year-old children may not have all the and should not have a mixed dentition. The primary teeth yet, which makes them poor can- aligners will not fit in the process of primary didates for orthodontic treatment. Thirteen- teeth exfoliation and permanent teeth erup- year-old patients are not at their prime age for tion. an orthodontic treatment.

35. The correct answer is C. Although all choices 40. The correct answer is C. During tipping, the are acceptable, the best option is treatment with crown and the apex move in opposite direc- Invisalign (the patient does not want the con- tions, creating two areas of compression: the ventional braces, and she is very concerned cervical area on the side toward which the tooth about her smile). is tipping and the apical region on the side op- posite from which the tooth crown is moving. 36. The correct answer is F. Stress–strain curve is The tension areas are located on the opposite like a spring. X represents the elastic limit, the sides of where the compression occurs. Rota- point at which permanent deformation occurs. tion is the movement of a tooth around its lon- Y represents the stiffness (modulus of elastic- gitudinal axis. A force through the center of re- ity), the ration of load to deflection. Z repre- sistance causes all points of the tooth to move sents plastic range, the amount of deformation the same amount in the same direction. Intru- or strain a wire can undergo between the elas- sion is the movement of the tooth in an inward tic limit and the ultimate strength. Ultimate direction. Extrusion is the movement of teeth strength of the material is the maximum load beyond the natural occlusal plane that may be at which point the material will fracture. accompanied by a similar movement of invest- ing tissues. 37. The correct answer is D. All the choices are correct. The type of wire used is determined by 41. The correct answer is D. Neither osteoblasts its modulus of elasticity. The main two types of nor osteoclasts are located in the dark zones wires used are the nickel titanium and stainless after a force is applied for a few minutes. Blood steel. The length of wire is directly proportional flow is altered and oxygen tension begins to to the springiness and indirectly proportional to change, leading to release of prostaglandins and the strength. The cross-sectional area of wire is cytokines. Osteoclasts and osteoblasts remodel indirectly proportional to the springiness and bony socket, which induces tooth movement directly proportional to the strength. at approximately second day after a continu- ous light force is applied to a tooth. There 38. The correct answer is B. When an open spring is an increase in cyclic adenosine monophos- is compressed, it has a tendency to reopen, phate (cAMP) level along with cellular differ- Orthodontics causing an uprighting of the tooth that it is at- entiation within the periodontal ligament in tached to (e.g., making room for an implant on approximately 4 hours after an application of site #19, where tooth #18 has displaced mesially continuous light force to a tooth. and tooth # 20 distally). The closed coil spring is used to close spaces, given its natural recoil 42. The correct answer is E. There is an increase in tendency once it is placed on teeth in open cAMP level along with cellular differentiation 122 Chapter 4: Orthodontics

within the periodontal ligament in approxi- be lengthened to make room for proper erup- mately 4 hours after an application of contin- tion of teeth with palatal expander. Distaliza- uous light force to a tooth. Osteoclasts and tion is a process of tipping a crown of the tooth osteoblasts remodel the bony socket, which distally to create space. Interproximal reduc- induces tooth movement at approximately the tion is a process of removing a small portion of second day after a continuous light force is ap- tooth enamel interproximally to create space plied to a tooth. and align teeth.

43. The correct answer is A. Resorption of alveolar 46. The correct answer is B. Tooth movement bone is represented by the dark areas (known as such as translation requires the most force compression areas of the periodontal ligaments). (70–120 gm). Tooth movements in descending Osteoclasts are usually found in these areas. Os- order of requiring force are: teoclasts and osteoblasts remodel bony socket, I. Translation which induces tooth movement at around sec- II. Root uprighting ond day after a continuous light force is applied III. Tipping, rotation, and extrusion require to a tooth. Osteoblasts are present only on the similar force application opposite surface of a root that is undergoing IV. Intrusion bodily movement and appear on the second day of continuous light force application. Along the 47. The correct answer is B. Patients with short tension front, a different type of activity occurs clinical crowns have an indication to use a as the fibers are under tension and the bone band instead of a bracket. Patients with amel- quickly activates; here, bone is stimulated to be- ogenesis imperfecta have tooth surface that is gin formation. There is an increase in cAMP incompatible with successful bonding for the levels along with cellular differentiation within placement of brackets. Therefore, the use of the periodontal ligament, in approximately bands is indicated. Patients with cuspal in- 4 hours after an application of continuous light terference or who experience heavy mastica- force to a tooth. tory forces require band placement rather than brackets to better resist breakage. Patients with 44. The correct answer is C. The amount of force teeth requiring both lingual and labial attach- (heavy or light) determines the biological path- ment have an indication for a band place- way of tooth movement and the formation or ment. lack of formation of a hyalinized zone with un- dermining resorption. With an application of 48. The correct answer is A. Generalized spacing heavy continuous force, the cell death is ob- of primary teeth is required for proper align- served in the compressed area. Increased cAMP ment of the permanent incisors. The primate levels are detectable within 4 hours of continu- space is located between lateral incisor and ca- ous light force application to the tooth. There is nine in the maxillary arch, and between the cellular differentiation within periodontal lig- canine and first molar in the primary dentition ament in approximately 4 hours of continu- of the mandibular arch. It should be present in ous light force application to the tooth. Few deciduous teeth. The primate space should not minutes of light continuous force application be present in permanent teeth, and if there is usually leads to release of prostaglandins and spacing, then an orthodontic appliance should cytokines. be placed to close it.

45. The correct answer is E. All of the methods 49. The correct answer is C. In this case, a di- mentioned are employed in orthodontic treat- astema is caused by a low frenum attachment. ment to align a crowded dentition. The extrac- Teeth should be aligned orthodontically, fol- tion of first premolars to relieve the crowding lowed by a frenectomy after maxillary canines (in case of severe crowding) is known as se- erupt, because spaces tend to close as perma-

Orthodontics rial extraction. For example, palatal arch can nent canines erupt. Answers: 43-60 123

50. The correct answer is F. Headgear is typically molar. To control thumb-sucking behavior, a used in skeletal class II growing patients to hold crib should be used. the growth of the maxilla back and to allow the mandible to catch up. To be effective, it 57. The correct answer is D. Approximately 70% needs to be worn 10 to 14 hours per day for 6 to of the population is in class I occlusion. Ap- 18 months depending on the case. Adults can proximately 25% of the population is in class II wear orthodontic headgear for maintenance of occlusion, and less than 5% of the population anchorage. is in class III occlusion.

51. The correct answer is D. The hand–wrist radio- 58. The correct answer is D. In a class III occlu- graphs can help pinpoint a child’s skeletal age sion, the mesiobuccal cusp of the maxillary and the areas examined are the carpal bones of first molar is posterior to the buccal groove the wrist, metacarpal bones of the hand, and of the mandibular first molar. In class I oc- phalanges of the fingers. Orthodontists com- clusion, the mesiobuccal cusp of the maxillary pare a hand–wrist radiograph with a picture first molar is lined up with the buccal groove atlas of wrist bones at various radiographic de- of the mandibular first molar. In class II divi- velopmental stages. This is used to help predict sion 1, the mesiobuccal cusp of the maxillary the child’s next growth spurt. first molar falls between the mandibular first molar and the mandibular second premolar. 52. The correct answer is B. The most common (THINK: The whole maxilla is more forward supernumerary tooth is the mesiodens, which is of where it should be.) In class II division 2, a peg-like tooth that occurs between the central the relationship between the first molars is the incisor. This occurs in 0.15% to 1.9% of the same as in class II division 1. The difference population. here is that the maxillary central incisors are tipped palatally, and the maxillary lateral in- 53. The correct answer is A. Posterior bite plates cisors are tipped buccally and mesially. are used in correcting an anterior open bite. Lingual arch with finger spring appliance, 59. The correct answer is C. In a class II occlu- Hawley appliance with finger spring, and ce- sion, the maxillary canine would be mesial to mented orthodontic bands with intertooth trac- the embrasure between the mandibular canine tion are all methods that can be used to close a and the mandibular first premolar. In a class I diastema. occlusion, the maxillary canine is in the em- 54. The correct answer is E. Both distal shoe and brasure between the mandibular canine and band and loop are appliances used to maintain mandibular first premolar. In a class III occlu- the space of one prematurely missing primary sion, the maxillary canine is distal to the em- molar. Lingual holding arch is used to main- brasure between the mandibular canine and tain the spaces for bilateral or multiple missing mandibular first premolar. A maxillary canine primary teeth. that is lingual to the mandibular canine signi- fies skeletal arch discrepancy between the max- 55. The correct answer is C. A distal shoe is in- illa and mandible. dicated when the primary second molar is lost prior to eruption of the permanent first molar. 60. The correct answer is A. Subdivision is added to the classification when only one side of the 56. The correct answer is A. Nance appliance is dental arch has a malocclusion and according Orthodontics best used to maintain the space for bilateral loss to the side of the dental arch the malocclusion of primary maxillary molars. A lingual hold- is found on. Even though one side of the arch ing arch is best used to maintain the space is normal (class I), the classification is named when bilateral primary mandibular molars are for the abnormal (class II). The malocclusion lost. A distal shoe or band and loop can be (class II) is on the left, so it is subdivision left. used to maintain the space of one loss primary Subdivision is always followed by either left or 124 Chapter 4: Orthodontics

right, as it refers to the side of the arch the metric radiograph that is between menton and malocclusion is found on. pogonion.

61. The correct answer is A. A space should be es- 69. The correct answer is E. As per answers. tablished in the arch prior to pulling the canine into position. It is also best to pull an impacted 70. The correct answer is E. As per answers. canine into position through keratinized mu- 71. The correct answer is B. Copper, tin, and tita- cosa, not alveolar mucosa. And if treatment is nium alloy are not used for orthodontic arch- delayed, a canine is more likely to ankylose in- wires. Commonly used alloys for orthodontic stead of erupting on its own through the palate. wire fabrication are made out of nickel and tita- nium alloys, titanium and molybdenum alloys, 62. The correct answer is B. Frankfort plane is the and stainless steel and cobalt chromium alloys. line on the lateral cephalogram connecting the upper border of the external auditory meatus 72. The correct answer is C. Common causes of and the lower orbital rim. The posselt envelope xerostomia include medications, cancer ther- of motion is a diagram that shows the shape of apy, Sj¨ogren syndrome, stress nutritional de- the pathway the mandible makes during func- ficiencies, and nerve damage. Orthodontic tioning. treatment does not cause xerostomia but can have other common negative sequelae such as 63. The correct answer is C. Pogonion is the root resorption, development of gingivitis, and craniometric point that is the most forward- decalcification of enamel around orthodontic projecting point on the anterior surface of the appliances. chin. 73. The correct answer is B. A palate expander is 64. The correct answer is D. Primate space is kept in the patient’s mouth for 3 to 6 months found between the lateral incisors and canines after expansion to allow for bone formation in the maxilla and between the canines and first in the midpalatal suture region. Third molars molars in the mandible. erupt at approximately the age of 18 years. Tra- ditional palate expansion cannot be completed 65. The correct answer is A. Orbitale is the cran- at this age, unless it is surgically assisted, be- iometric point that is the lowest point on the cause the midpalatal suture is already closed. inferior orbital margin. 74. The correct answer is A. A skeletal crossbite has 66. The correct answer is A. Basion is the cranio- smooth closure to centric occlusion whereas metric point that is the midpoint of the anterior there tends to be shift when it is a functional border of the foramen magnum. crossbite.

67. The correct answer is A. ANS is the cranio- 75. The correct answer is E. Orthodontic retainers metric point that is the anterior extremity of are custom-made devices used to hold teeth in the intermaxillary suture. position after realignment or surgery and usu- ally made of wires or clear plastic. The Hawley 68. The correct answer is A. Nasion is the ante- retainer is a removable retainer made of a metal rior point of the intersection between the nasal wire that surrounds the teeth and is anchored and frontal bones on the lateral cephalometric in an acrylic arch that sits in the palate or lin- radiograph. Articulare is the point of intersec- gual walls of the mouth. The Essix retainer, a tion between the shadow of the zygomatic arch vacuum-formed retainer, is a clear retainer that and the posterior border of the ramus. Spheno- fits over an entire arch of teeth or only from the occipital synchondrosis is the intersection be- canine to canine and is removable. The lingual tween the occipital and basisphenoid bones. bonded retainer is fixed and consists of a passive Gnathion is the point on the lateral cephalo-

Orthodontics wire bonded to the tongue side of the incisors. Answers: 61-88 125

76. The correct answer is A. Benefits of the palatal 83. The correct answer is B. Intrusion is when the bonded retainer include aesthetics as it is hid- tooth is moved into the socket along the long den from sight in the palate, increased patient axis of the tooth, whereas extrusion is the op- compliance as it is fixed, and ability to prevent posite, in which the tooth is moved out of the relapse of the midline diastemas. socket along the long axis of the tooth. Torque movement is used to tip a tooth buccal lingually 77. The correct answer is E. An alveolar ridge is or mesial distally about an axis of rotation. one of the two ridges on the maxilla and on the mandible that contain the alveoli of the teeth. 84. The correct answer is B. Osteoclasts and It fails to form if a tooth fails to erupt, resorbs osteoblasts are vital to bone resorption and for- after a tooth is extracted, and can grow in height mation as a tooth is moved orthodontically. and length. Oligodendroglia is a type of glial cell found in the brain. 78. The correct answer is B. The supracrestal cir- cumferential fibrotomy is used to prevent max- 85. The correct answer is A. Leeway space is the illary lateral incisors from pure rotational re- space differential between the primary poste- lapse. rior teeth (i.e., primary canine to primary sec- ond molars) and their replacement teeth (i.e., 79. The correct answer is A. The normal relation- permanent canine to permanent second pre- ship of primary molars is one in which the molar). It spans approximately 2.5 mm per side maxillary and mandibular molars are in a flush in the mandibular arch and 1.5 mm per side in terminal plane. After the eruption of the per- the maxillary arch. manent first molars, there is an early mesial shift with the closing of the mandibular primate 86. The correct answer is B. Serial extractions are space, and after the exfoliation of the primary employed for selected patients with straight- molars, a late mesial shift develops and the lee- forward crowding such as moderate class I way space is closed. As a result of the early and malocclusion in mixed dentition stage due to late mesial shifts, a class I molar relationship insufficient arch length. It is a planned se- develops. quence of extractions that begins initially with 80. The correct answer is B. The primary denti- the deciduous canine and then the deciduous tion equivalent of Angle class II relationship is first molars. This is designed to allow crowded the distal step. A mesial step relationship cor- incisor segments to align spontaneously during responds to Angle class I. An equivalent to a the mixed dentition by shifting labial segment class III relationship is almost never seen in the crowding to the buccal segments where it could primary dentition and the normal pattern of be dealt with by premolar extractions. craniofacial growth in which the mandible lags 87. The correct answer is C. Supernumerary behind the maxilla. teeth are commonly associated with the fol- 81. The correct answer is D. Disturbances in tooth lowing: Gardner syndrome, cleidocranial dys- eruption are commonly due to mechanical plasia, Down syndrome, and Sturge–Weber interferences by supernumerary teeth, crowd- syndrome. Hyperthyroidism is associated with ing, odontogenic tumors and cysts, eruption early exfoliation of primary teeth. sequestrum, or local causes such as gingival 88. The correct answer is A. Serial extractions are

fibromatosis. A number of systemic disease and Orthodontics syndromes can also affect the delayed tooth employed for selected patients with straight- eruption and shedding of primary teeth, such as forward crowding such as moderate class I rickets, cleidocranial dysplasia and Down syn- malocclusion in mixed dentition stage due to drome. insufficient arch length. It is a planned se- quence of extractions that begins initially with 82. The correct answer is A. As per the question. the deciduous canine and then the deciduous 126 Chapter 4: Orthodontics

first molars. This is designed to allow crowded appliance that uses an attachment bracket slot incisor segments to align spontaneously during that receives a rectangular archwire horizon- the mixed dentition by shifting labial segment tally that can be used to control tooth move- crowding to the buccal segments where it could ment in all three planes of space. The Begg ap- be dealt with by premolar extractions. pliance is an orthodontic appliance based on a modified ribbonarch attachment, and the slots 89. The correct answer is B. Routine set of ortho- are positioned vertically. dontic diagnostic records include radiographs such as panoramic and lateral cephalometric 96. The correct answer is B. Mixed dentition space radiographs, impressions for dental casts of both analyses form an essential part of diagnostic pro- arches, and occlusion records. Computed to- cedure. It helps to assess the amount of space mography or magnetic resonance imaging of required for the alignment of unerupted per- the temporomandibular joints are not routinely manent teeth in a dental arch. Inappropriate indicated unless the patient has symptoms of and not valid analysis could lead to extractions dysfunction of that joint that may be related to that can negatively alter a patient’s soft tissue internal joint pathology. facial profile.

90. The correct answer is F. Maxilla and mandible 97. The correct answer is D. When used in skele- bones are formed by intramembranous ossifica- tal class II patients, high pull, cervical pull, tions. Femur, ethmoid, and temporal bones are and reverse straightpull headgears apply forces formed by endochondral ossification. distally to the maxilla/maxillary teeth to hold the growth of the maxilla back are used. When 91. The correct answer is B. Appositional bone used in a skeletal class III patient, a reverse-pull growth is when bone grows by the addition of headgear force is used. new layers on previously formed layers. Endo- chondral ossification is when a bone is formed 98. The correct answer is A. Maxillary prog- through replacement of a cartilage template by nathism with the norm of SNA 82 ± 2 a bone matrix. Intramembranous ossification is degrees has an SNA >84 degrees. Maxillary ret- the direct ossification of embryonic connective rognathism would have an SNA <80 degrees if tissue. the SNA is 82 ± 2 degrees. Mandibular prog- nathism would be an SNB >82 degrees and 92. The correct answer is C. Normal overjet in mandibular retrognathism would be an SNB class I occlusion is 1 to 2 mm. An overjet of <78 degrees if the SNB norm is 80 ± 2 degrees. 4 mm will tend to be class II occlusion. An overjet of –2 mm will tend to be class III occlu- 99. The correct answer is A. ANB has a norm of sion. 2 to 4 degrees. It tells us what the maxillary and mandibular skeletal base discrepancies are. A 93. The correct answer is C. Bimaxillary protru- large value, such as 8 degrees, would indicate a sion is a relatively forward position, or prog- large maxillary protrusion and a negative value nathism, of the maxillary and mandibular would indicate a mandibular prognathism. teeth, alveolar processes, or jaws. This pro- vides space to accommodate teeth and alleviate 100. The correct answer is A. ANB can be deter- crowding. mined by subtracting SNB from SNA or by straight measurements with the three cranio- 94. The correct answer is E. As per answer choices. metric points. Tooth surfaces such as a stainless steel crown or in cases of amelogenesis imperfecta are in- 101. The correct answer is E. There is no Richard compatible with bonding. analysis of cephalometry. Other analyses are 95. The correct answer is A. The edgewise ap- very commonly used by orthodontists to ana- lyze cephalometric radiographs. Orthodontics pliance is a fixed, multibanded orthodontic Answers: 89-111 127

102. The correct answer is B. Skeletal class II mal- pseudo–class III patients slide the mandible for- occlusions are frequently accompanied by ex- ward is to avoid interferences and achieve max- cessive overjet. imal intercuspation. These patients can often be guided back to a normal jaw position once 103. The correct answer is B. Temporary anchor- the interference is removed. age device. It is a small implant that can be used for additional anchorage to assist in tooth 109. The correct answer is A. The first statement is movement during orthodontic treatment. true because arch perimeter increases slightly after the eruption of the incisors, but after this 104. The correct answer is C. A Bolton discrepancy stage of dental development, arch length re- is typically referred to as a “tooth size discrep- duces as a result of the loss of E space. Hence, ancy” and measures the difference in tooth size any crowding of the permanent incisors in the between the maxillary and mandibular dental early mixed dentition typically results in crowd- arches. ing in the permanent dentition. The second statement is true because the 105. The correct answer is A. Both statements are premature loss of the primary canines is indica- true. RPEs are commonly used in children be- tive of insufficient arch size in the anterior re- cause the halves of the palate can still be sep- gion. During eruption of the lateral incisors, the arated with relative ease. RPEs are much less crown of the laterals impinges on the roots of effective in older teenagers and adults because the primary canines and causes them to resorb. of the fusion of the midpalatine suture. The When the canine is shed, the midline will shift halves of the palate cannot usually move apart, in the direction of the lost tooth. This causes so the pressure from the RPE has a greater ef- lateral and lingual migration of the mandibu- fect on the dentition instead. This can cause lar incisors. the tipping of posterior teeth rather than true skeletal expansion. 110. The correct answer is C. Answer C is a false statement because difficulty in pronouncing 106. The correct answer is B. On the basis of the re- sounds “f” and “v” are associated with skeletal cent National Health and Nutrition Estimates class III cases. An anterior crossbite of multi- Survey III of the U.S. population among white, ple teeth does not allow for maxillary incisors black, and Hispanic races, 25% of the popula- to touch the lower lip, which is a position tion manifests a class II malocclusion. 70% of needed to create these sounds. the population manifests class I malocclusion. Less than 5% of the population manifests class 111. The correct answer is F. Answer A is incorrect III malocclusion. because in patients with high mandibular plane angle, unsuccessful molar uprighting can lead 107. The correct answer is A. Answer B is incor- to an increased open bite and loss of anterior rect because the subdivision describes the side guidance. where the malocclusion occurs. Answer C is in- Answer B is incorrect because any orthodon- correct because both molars cannot be in class I tic movement in teeth with existing root resorp- when the question statement says it is class II. tion can lead to more resorption. Answer D is incorrect because subdivision Answer C is incorrect because correcting the means malocclusion has occurred only on one lingual inclination can lead to bite opening. side of the dental arch. Answer D is incorrect because occlusal Orthodontics plane discrepancy (i.e., extruded maxillary and 108. The correct answer is C. Answer C is a false mandibular molars) will only pose more prob- statement because pseudo–class III patients lems in terms of deficient occlusal clearance can bring the mandible back without strain needed to upright a molar. to a position where mandibular and maxil- Answer E is incorrect because any orthodon- lary incisors can touch. The typical reason that tic movement in teeth with existing periodontal 128 Chapter 4: Orthodontics

disease can lead to more compromised peri- 116. The correct answer is D. Answer D is false odontal conditions. because bodily tooth movement is required to Answer F is a false statement because pa- close extraction spaces fully for which full or- tients with low mandibular plane angle are the thodontic treatment with fixed appliances is best candidates in which to upright molars, necessary. since these patients usually present with a deep overbite. Molar uprighting can help “open up” 117. The correct answer is A. Answer A is correct and correct their occlusion. because removable appliances can accomplish only tooth tipping. 112. The correct answer is C. The first statement Answer B is incorrect because improved hy- is correct because without adequate space pro- giene is one of the advantages of removable vided, only crowding will be introduced into appliances the system, which will hinder the path of erup- Answer C is incorrect because increased pa- tion of the impacted tooth. tient comfort is one of the advantages of remov- The second statement is incorrect because able appliances. impacted teeth should be pulled into kera- Answer D is incorrect because decreased tinized tissue, and not alveolar mucosa, to en- chair time is one of the advantages of remov- sure adequate periodontal support. able appliances.

113. The correct answer is B. Superimposition of 118. The correct answer is C. Answer C is false be- cephalometric radiographs taken before, dur- cause cervical-pull headgear produces a distal ing, and after treatment will show changes in and extrusive force on the maxillary molars and jaw and tooth positions that have occurred maxilla. due to orthodontic therapy and orthognathic surgery. 119. The correct answer is C. Answer A is incor- Answer A is incorrect because Angle classi- rect because the Bionator is a tooth-borne func- fication can usually be determined from cast tional appliance. models, photographs, or clinical examination Answer B is incorrect because the Herbst is of the patient. a tooth-borne functional appliance. Answer C is incorrect because the decision Answer D is incorrect because the Forsus is to extract teeth is the result of a comprehensive a tooth-borne functional appliance. evaluation of the patient’s complete record in combination with clinical presentation of the 120. The correct answer is C. The first statement patient in addition to patient preferences. is correct. The second statement is incor- Answer D is incorrect because the status of rect because functional appliances used during the wisdom teeth can usually be determined mixed dentition have both dental and skeletal via Panorex radiogram or orthopantogram. effects. Answer E is incorrect because the exact lo- cation of an impacted tooth is best studied 121. The correct answer is A. Answer A is correct be- through Cone Beam CAT Scans. cause orthodontic appliances increase the risk of gingivitis due to irritation of gingiva or mu- 114. The correct answer is D. The statement is not cosa. correct because a steep mandibular plane angle correlates with a long anterior vertical dimen- 122. The correct answer is D. Answer A is incorrect sion and an anterior open bite. The reason is because torque in bracket slots compensate for also correct. facial and lingual inclination of the teeth. Answer B is incorrect because angulation 115. The correct answer is E. Porion is located on of bracket slot relative to the long axis of the the outer upper margin of the external auditory teeth allow for proper positioning of the tooth roots. Orthodontics canal. Answers: 112-127 129

Answer C is incorrect because variation in 127. The correct answer is D. The image pictured bracket thickness compensates for the varying is that of a lower lingual holding arch. thickness of individual teeth. Correct picture of tongue crib:

123. The correct answer is B. Answer B is a false statement because bands are used in cases where both lingual AND buccal attachments are needed. Bands are preferably used in heavy mastication areas or areas with cuspal inter- ference to reduce chance of breakage. Bands are preferably used in cases with short clinical crown. Bands are preferably used in patients with amelogenesis imperfect since tooth sur- faces are incompatible with successful bond- ing. Bands are preferably used in patients with stainless steel crowns since crown surfaces are incompatible with successful bonding.

124. The correct answer is D. Answer D is a false statement because mild anterior crossbites need to be corrected in the first stage of treat- Correct picture of Bluegrass appliance: ment (as soon as possible), as the transverse di- mension is the first to stop growth.

125. The correct answer is A. Answer A is a false statement because maxillary constriction usu- ally seen in open-bite cases result from in- creased pressure on the buccinators muscles from finger sucking.

126. The correct answer is E. Answer A is incorrect because habit-breaking appliances can help with treatment of open-bite cases. Answer B is incorrect because RPE can help with treatment of open-bite cases to correct nar- row maxillas. Correct picture of transpalatal bar: Answer C is incorrect because posterior bite plates can help with treatment of open- bite cases by discouraging posterior teeth from erupting further and encouraging anterior teeth to erupt more and close the bite. Answer D is incorrect because high-pull headgear places a distal and intrusive force on the molars. The intrusive force will discourage Orthodontics molars from erupting more. Answer E is correct because straight-pull headgear places a horizontal force on the mo- lars, neither intruding nor extruding the mo- lars. This does not help with the correction of open-bite in any way. 130 Chapter 4: Orthodontics

128. The correct answer is B. Answer B is a false Correct picture of wrap-around retainer: statement because distal shoe appliance is usu- ally used when primary second molar is lost prior to the eruption of the permanent first molar.

129. The correct answer is C. Early loss of primary second molars leads to the most rapid loss of arch perimeter due to mesial tipping and ro- tation of the permanent first molar; therefore, space maintenance is always needed until ar- rival of the second premolar.

130. The correct answer is A. Nance orthodontic appliance. Correct picture of band and loop: Correct picture of Hawley:

Correct picture of distal shoe:

Correct picture of Essix:

131. The correct answer is D. A bonded lingual re-

Orthodontics tainer is fixed. Answers: 128-141 131

Correct picture of Bonded lingual wire: correction by the overbite achieved during treatment.

134. The correct answer is E. All of the above are true statements about the postorthodontic cir- cumferential supracrestal fibrotomy.

135. The correct answer is B. Answer A is incorrect because an edge-to-edge position is the flush terminal plane, which is the normal relation- ship of maxillary and mandibular first molars. This leads to class I Angle classification due to early mesial shift of mandibular molars in relation to maxillary molars. Answer B is correct because the more for- ward position of the maxillary first molar in re- lation to mandibular molar often leads to Angle class II malocclusion. 132. The correct answer is D. Answer D is a false Answer C is incorrect because the more statement because the occlusal result of hard backward position of the maxillary first molar tissue is modified by orthodontic treatment. in relation to mandibular molar often leads to Angle class III malocclusion. 133. The correct answer is C. The first statement Answer D is incorrect because distal step is a is true. The second statement is false because term involving primary molars and has nothing anterior crossbite is retained after orthodontic to do with incisors.

Flush Distal Mesial Terminal Step Step Plane

1. Flush terminal plane: The distal surfaces of maxillary and mandibular primary second molars lie in the same vertical plane. 2. Distal step: The distal surface of the mandibular primary second molar is distal to that of the maxillary primary second molar. 3. Mesial step: The distal surface of the mandibular primary second molar is mesial to that of the maxillary primary second molar.

136. The correct answer is A. Both statements are 139. The correct answer is D. Answer D is a false true. statement because Essix is a passive appliance.

137. The correct answer is E. All of the above are 140. The correct answer is B. Answer B is a false

causes of diastemas. statement because studies show that the major- Orthodontics ity of the “long-face” population has no nasal 138. The correct answer is A. Answer A is a false obstruction. statement because not all spaces always close. The greater the spacing, the less likely the di- 141. The correct answer is C. Answer C is a false astema space will close on its own. statement because the maxilla and mandible are formed via intramembranous ossification. 132 Chapter 4: Orthodontics

142. The correct answer is B. Both statements are is all but impossible to overwork the jaw mus- false. cles to this extent during normal eating and Growth of bone occurs only via apposi- chewing. To produce myofascial pain, the pa- tional growth phenomenon where new layers tient must be clenching or grinding the teeth for are added to those previously formed. Carti- many hours per day, presumably as a response lage grows via both appositional and intramem- to stress. Great variations are seen in the way branous growth phenomena. In appositional different individuals respond to stress, both in growth, recruitment of fresh cells, chondrob- the organ system that feels the strain (those who lasts, perichondral stem cells, and the addition develop an ulcerated colon rarely have TMD) of new matrix to the surface takes place. In and in the amount of stress that can be toler- interstitial growth, mitotic division of and de- ated before symptoms appear (tense individuals position of more matrix around the existing develop stress-related symptoms before their re- chondrocytes already established in the car- laxed colleagues do). For this reason, it is im- tilage occur. This does not occur in bone possible to say that occlusal discrepancies of growth. any given degree will lead to TMD symptoms. It is possible to demonstrate that some types of 143. The correct answer is B. The first statement is occlusal discrepancies predispose patients who false because if this were true, it would mean clench or grind their teeth to the development that there is an indication to treat everyone’s of TMD symptoms. It must be kept in mind occlusion to perfection to avoid possibility of that it takes two factors to produce myofascial developing facial muscle pain. The number pain: an occlusal discrepancy and a patient who of people with moderate degrees of malocclu- clenches or grinds the teeth. sion (50–75% of the population) is far more The second statement is false because in than the number of people with TMD (5– some difficult births, the use of forceps to the 30% depending on which symptoms are ex- head to assist in delivery might damage ei- amined). Therefore, it seems unlikely that oc- ther or both joints. At least in theory, heavy clusion alone is enough to cause hyperactivity pressure in the area of the joints could cause of the oral musculature. Usually, a reaction to internal hemorrhage, loss of tissue, and a sub- stress is involved. There are some people with sequent underdevelopment of the mandible. At very poor occlusion who have no muscle pain. one time, this was a common explanation for Some types of malocclusion (especially poste- mandibular deficiency. If the cartilage of the rior crossbite with a shift on closure) correlate mandibular condyle were an important growth positively with TMD, whereas others do not. center, the risk from damage to a presumably So, for the majority of people, there is no associ- critical area would seem much greater. In light ation between malocclusion and TMD. On the of the current understanding that the condylar other hand, if a patient does respond to stress cartilage is not critical for proper growth of the by increased oral muscle activity, improper oc- mandible, it is not as easy to blame underdevel- clusal relationships may make the problem opment of the mandible on birth injuries. It is more severe and difficult to control. Therefore, interesting to note that although the use of for- malocclusion coupled with pain and spasm in ceps in deliveries has decreased considerably the muscles of mastication may indicate a need over the last 50 years, the prevalence of class for orthodontic treatment as an adjunct to other II malocclusion due to mandibular deficiency treatment for the muscle pain. But orthodontics has not decreased. as a primary treatment process is almost never indicated. If the problem is a pathologic process 144. The correct answer is C. Answer C is correct within the joint itself, improving the dental oc- because jaw function is more than temporo- clusion may or may not help the patient adapt mandibular (TM) joint function, but evalua- to the necessarily altered joint function. tion of the TM joints is an important aspect of Myofascial pain develops when muscles are the diagnostic workup. As a general guideline,

Orthodontics overly fatigued and tend to go into spasm. It if the mandible moves normally, its function is Answers: 142-148 133

not severely impaired, and by the same token, or destruction of the intraarticular disk, and restricted movement usually indicates a func- those with symptoms primarily of muscle ori- tional problem. For this reason, the most impor- gin, caused by spasm and fatigue of the mus- tant indicator of joint function is the amount cles that position the jaw and head. Because of maximum opening. muscle spasm and joint pathology can coex- ist, the distinction in many patients is difficult. 145. The correct answer is D. The first statement is Nevertheless, the distinction is important when false because it is possible that functional prob- orthodontics will relieve TMD symptoms in lems related to malocclusion would appear as a patient who has internal joint problems or TMD. Little or no data support the idea that other nonmuscular sources of pain. Those who orthodontic treatment is needed at any age to have myofascial pain/dysfunction, on the other prevent the development of TMD. hand, may benefit from improved occlusal re- The relationship, if any, between TMD and lationships. Almost all people develop some premolar extraction is difficult to assess because symptoms of degenerative joint disease as they data from well-controlled studies are not avail- grow older, and it is not surprising that the able. There is simply no evidence to support jaw joints are involved sometimes. Arthritic in- the allegation that premolar extraction causes volvement of the TMJ is most likely to be the TMD. cause of TMD symptoms in patients who have In the late 1980s, it was claimed by some arthritic changes in other joints of the body. A dentists that extraction of upper first premolars component of muscle spasm and muscle pain would later lead to TMD problems. The theory should be suspected in individuals whose only was that retracting the upper incisors would in- symptoms are in the TMJ, even if radiographs evitably lead to incisor interferences, and this show moderate arthritic degeneration of the would cause TMD. The claim was never sup- joint. ported by any evidence, and research data have refuted it. 147. The correct answer is B. Answer B is correct The second statement is true because the ex- because displacement of the disk can arise from tent to which TMD symptoms in many adults a number of causes. One possibility is trauma disappear when comprehensive orthodontic to the joint, such that the ligaments that op- treatment begins can be surprising. TMD pose the action of the lateral pterygoid mus- symptoms disappear long before the occlusal cle are stretched or torn. In this circumstance, relationships have been corrected. The expla- muscle contraction moves the disk forward as nation is that orthodontic treatment makes the the mandibular condyles translate forward on teeth sore, grinding or clenching sensitive teeth wide opening, but the ligaments do not restore as a means of handling stress does not produce the disk to its proper position when the jaw the same subconscious gratification, the para- is closed. The result is a click upon opening functional activity stops, and symptoms vanish. and closing, as the disk pops into place over the The changing occlusal relationships also con- condylar head as the patient opens, but is dis- tribute to breaking up the habit patterns that placed anteriorly on closure. The click and the contributed to the muscle fatigue and pain. symptoms associated with it can be corrected At the end of orthodontic treatment, however, if an occlusal splint is used to prevent the pa- even if the occlusal relationships have been tient from closing beyond the point at which significantly improved, clenching and grinding displacement occurs.

tend to recur. Use of interocclusal splints may Orthodontics be helpful in these situations. 148. The correct answer is D. Answer D is a false statement because the main growth thrust of 146. The correct answer is D. Answer D is cor- the condyle is upward and backward to fill rect because patients with TMD can be di- in the resultant space to maintain contact with vided into two large groups: those with in- the base of the skull. ternal joint pathology, including displacement 134 Chapter 4: Orthodontics

149. The correct answer is D. Answer D is correct 153. The correct answer is C. Gardner syndrome, because in both arches, the permanent incisor also known as familial colorectal polyposis,is tooth buds lie lingual and apical to primary an autosomal dominant disease characterized incisors. by gastrointestinal polyps, multiple osteomas, skin and soft tissue tumors, and dental abnor- 150. The correct answer is D. Hereditary gingival malities such as multiple supernumerary teeth. fibromatosis is a generalized condition. Answers A, B, C, and E are incorrect because 154. The correct answer is C. Answer C is a false they are the localized causes of local delayed statement because the whole point of serial ex- tooth eruption. traction is to extract the first premolar before the permanent canine erupts. The crowding is, therefore, corrected when the canine is 151. The correct answer is E. All of the above are guided to erupt into the premolar space. true statements. 155. The correct answer is D. The maxillary lee- 152. The correct answer is A. There is no signif- way space is smaller than mandibular leeway icant sex distribution in primary supernumer- space; therefore, the mandibular permanent ary teeth. However, males are affected approx- first molars shift mesially more than maxillary imately twice as frequently as females in the permanent first molars, which in turn helps permanent dentition. with achievement of a class I occlusion. Orthodontics CHAPTER 5 Pediatric Dentistry

135 136 Chapter 5: Pediatric Dentistry

QUESTIONS

1. A 20-month-old female presents with her mother D. Perform pulpectomy, place stainless steel to your office for her first check-up. Her mother crown

Pediatric Dentistry is concerned because her daughter only has a E. Extract tooth, place space maintainer few erupted teeth. Upon examining the patient, which teeth should you expect to find? 5. A disease process is characterized by painful, bleeding gingival tissue, punched out erosions A. Primary centrals and laterals only covered by gray pseudomembrane, blunting of B. Primary centrals, laterals, canines interproximal papillae, and a fetid odor. Treat- C. Primary centrals, laterals, first molars, second ment usually comprises debridement, mouth molars rinses, and antibiotics. This description is associ- D. Primary centrals, laterals, first molars ated with which of the following conditions that E. Primary centrals, laterals, first molars, ca- may exist in teenagers? nines A. Aggressive periodontitis 2. A 5-year-old girl lives in an area with 0.4 ppm F in B. Acute necrotizing ulcerative gingivitis the city drinking water. How much supplemen- C. Primary herpetic gingivostomatitis tal fluoride should you prescribe for the patient D. Severe gingivitis to consume the optimal amount of fluoride? E. Early childhood caries A. 0 B. 0.25 mg 6. How much fluoride, in ppm, is contained in C. 0.33 mg commonly used toothpastes? D. 0.5 mg A. 1,000 ppm E. 1.0 mg B. 5,000 ppm C. 9,000 ppm 3. A child presents to your office after a fall from a D. 12,300 ppm tree, during which his primary maxillary central incisor was avulsed. The mother has brought the 7. Fordyce granules are: tooth in a cup of milk and says the fall occurred 2 hours ago. What is the best treatment for this A. Epithelial inclusion cysts patient? B. Found on the buccal and lingual aspects of the alveolus A. Replant the tooth, stabilize the tooth for 1 to C. Ectopic sebaceous glands 2 weeks at which time a pulpotomy should D. Remnants of the dental lamina be performed B. Replant the tooth, stabilize for 1 to 2 weeks, 8. As compared with permanent teeth, primary and then perform a pulpectomy teeth have: C. Replant tooth, stabilize for 1 to 2 weeks, and begin apexification if pulp necrosis is evident A. Pulp horns further away from the tooth D. Take a radiograph, irrigate socket, do not re- surface plant tooth B. Smaller pulp relative to crown size C. Thicker and shorter roots 4. A 6-year-old child presents with a Class I fracture D. Increased number of accessory canals in to a tooth with an immature apex. What is the pulpal floor treatment of choice for this patient? 9. The calcification of maxillary and mandibular A. Restore tooth permanent first molars occurs at what age? B. Place calcium hydroxide to exposed dentin, restore tooth A. 3 to 4 months C. Perform pulpotomy, then temporarily restore B. Birth eiti Dentistry Pediatric Questions: 1-18 137

C. 10 to 12 months A. Overuse of the instrument D. 1.5 to 2 years B. Excessive force C. Filing a dry canal 10. The definition of primate space is: D. Manufacturing defects A. The space mesial to the mandibular primary E. Using larger files before smaller ones fit canines and distal to the maxillary primary loosely canines B. The space distal to the mandibular primary 15. Which of the following is true of apexogenesis? canines and distal to the maxillary primary A. It is nonvital pulp therapy canines B. Its objective is to artificially close the root C. The space mesial to the maxillary primary apex canines and distal to the mandibular primary C. It is indicated for a tooth with damaged radic- canines ular pulp D. The space mesial to the maxillary primary D. It is indicated for an immature avulsed tooth canines only E. Endodontic therapy can be performed more effectively following apexogenesis 11. What amount of fluoride should be prescribed to a 4-year old child who lives in a community 16. Fluoride varnish (5% sodium fluoride in a resin with 0.4 ppm fluoridated water? vehicle) is the preferred choice of pediatric den- A. 0 tistry for in-office fluoride delivery for which of B. 0.25 the following reasons? C. .5 A. Ease of delivery D. 1.0 B. Measured maximum dose/exposure to fluo- E. 0.85 ride ion 12. A tooth has a periapical radiolucency with nor- C. Stays on enamel surfaces longer than any mal periodontal probing depths except at the other fluoride product/longer contact time mesiobuccal (MB) line angle it probes to the D. Can be used more frequently for high-risk apex. The tooth tests nonvital. What is the most patients likely classification of the lesion? E. All of the above A. Periodontal 17. Which of the following antimicrobial therapies B. Endodontic can be utilized for high-caries-exposed patients? C. Primarily periodontal with secondary en- dodontic involvement A. Xylitol gums, lozenges, mints, rinses, sprays D. Primarily endodontic with secondary peri- B. ART restorative techniques odontal involvement C. Stannous fluoride E. True combined lesion D. Fluoride varnish therapy at high-frequency intervals 13. Which of the following is a contraindication to E. All of the above the use of an electric pulp tester? 18. According to the American Academy of Pediatric A. A pediatric patient Dentistry Guidelines, the following restorative B. A patient with a recent MI materials for a high-caries experience patient are C. A patient with a cardiac pacemaker recommended: D. A patient with recent trauma E. A patient with a large metal restoration A. Composite resin B. Glass ionomer or resin modified GI 14. Which of the following factors is the least likely C. Stainless steel crown to cause an instrument separating inside a root D. B and C only canal? E. All of the above 138 Chapter 5: Pediatric Dentistry

19. Which of the following combinations is cor- A. Class I rect regarding local anesthesia and pediatric B. Class II dentistry? C. Class III D. Class IV A. Short-acting local anesthetics, that is, ones E. There is no classification for bilateral cleft without epinephrine should be avoided Pediatric Dentistry lip B. Articaine can be used to locally infiltrate the mandibular primary posterior teeth 24. A child was brought to your office after tooth #E C. Increasing the epinephrine dose above fell out when he tripped and fell playing out- 1:100,000 should be avoided side. Mom says the tooth has been out for only D. A and C only 15 minutes and she put it in milk right away and E. All of the above came to your office. The tooth looks viable and clean. You should: 20. Avulsed teeth, pimary-permanent should be treated as follows: A. Reimplant the tooth immediately since it has been less than 1/2 hour A. Primary: reimplanted/Permanent: reimplan- B. Clean the tooth with normal saline then ted reimplant the tooth since it has been less than B. Primary: reimplanted and splinted/Perma- 1/2 hour nent: reimplanted and splinted C. Reimplant the tooth and splint it. Have the C. Primary: no reimplantation/Permanent: re- child follow-up in 2 weeks implanted and splinted rigidly D. Reimplant the tooth, perform a pulpotomy, D. Primary: no reimplantation/Permanent: re- splint, and follow-up in 1 week implanted and splinted flexibly for 14 days E. Do not reimplant tooth maximum E. Primary: no reimplantation/Permanent: re- 25. A 5-year old who lives in a nonfluoridated area implanted and splinted flexibly should receive how much fluoride supplemen- tation? 21. A mother brings her son to your office for his 1-year-old dental appointment. You notice that A. This child does not need fluoride supple- there is a problem with the shape of his teeth. mentation The problem probably occurred in what stage of B. 0.25 mg development? C. 0.5 mg D. 1.0 mg A. Initiation B. Proliferation 26. At what age can one expect eruption of a maxil- C. Differentiation lary permanent canine in an individual with an D. Apposition ideal eruption sequence? E. Calcification A. 7 to 8 years 22. You are counting the teeth of a 5-year old, and B. 8 to 9 years there are 20 crowns, but when you look at the C. 9 to 10 years radiograph, there are only 19 roots. This is most D. 10 to 11 years likely: E. 11 to 12 years A. Fusion 27. When recommending a storage medium for B. Concrescence avulsed teeth, order the following from most fa- C. Gemination vorable to least favorable. D. Attrition E. There is no problem A. Cold milk, Hank’s balanced salt solution, saliva, water 23. A child comes to your office and has a bilateral B. Hank’s balanced salt solution, cold milk, cleft lip. This is classified as what class? saliva, water eiti Dentistry Pediatric Questions: 19-30 139

C. Hank’s balanced salt solution, cold milk, 29. Basic pediatric behavior management tech- water, saliva niques include all of the following except: D. Cold milk, Hank’s balanced salt solution, A. Voice control water, saliva B. Tell-show-do E. Saliva, Hank’s balanced salt solution, cold C. Hand over mouth exercise (HOME) milk, water D. Positive reinforcement E. Distraction 28. The amount of articaine in a 1.7-mL cartridge of articaine HCL 4% with epinephrine 1:100,000 30. Which of the following is not a factor in a caries- is: risk assessment tool? A. 34 mg A. Previous caries experience B. 36 mg B. Socioeconomic status of caregiver C. 54 mg C. Exposure to fluoride D. 68 mg D. Eruption sequence E. 72 mg E. Level of mutans streptococci 140 Chapter 5: Pediatric Dentistry

ANSWERS

1. The correct answer is E. By 19 months, a child 4. The correct answer is A. Class I fracture involves should have mandibular and maxillary central only the enamel of the tooth. Smoothing out the

Pediatric Dentistry and lateral incisors, all first molars, and all ca- rough edges of the enamel and restoring with nines. According to answer choice A, primary a permanent restoration is the only treatment central and laterals should be found by 7 months necessary for this tooth. In Class II fracture, this of age. Answer B is incorrect also in terms of may be covered with calcium hydroxide or glass sequence—if the canines were present, the first ionomer and restored as there will be much more molars should also be present. By the time the substantial dentin exposure. No pulpotomy is second molars erupt, the canines should already necessary since there was no pulp exposure (as be present in the mouth. Primary centrals, later- in a Class III fracture) and the pulp is still vital. als, and first molars should erupt by 15 months As in answer choice C, the pulp is still healthy, of age. so a pulpectomy is not necessary. An stainless steel crown (SSC) is also not necessary if only 2. The correct answer is B. In a population with a minimal amount of enamel has been lost. In 0.3- to 0.6-ppm fluoride in the drinking water, Class IV fracture, a pulpectomy and an SSC are a 0.25-mg supplement is the recommended both necessary as the entire crown has been lost. amount. According to answer choice A, no fluo- There is no indication for extraction of a tooth ride supplements are recommended from birth that has a healthy pulp, no fracture present, and to 6 months, in populations where 0.6 ppm or may be restored with a permanent restorative more is present, or for a patient aged 6 months material. to 3 years in a 0.3- to 0.6-ppm fluoridated area. 0.33 mg is not included as a recommended 5. The correct answer is B. Acute necrotizing ul- amount of fluoride supplementation. 0.5 mg is cerative gingivitis may occur in both adults and the recommended dose for a child aged 3 to teenagers and is also known as Vincent’s infec- 6 years in an area of less than 0.3-ppm fluorida- tion, Vincent’s angina, or trench mouth. This tion, and for 6- to 16-years old in a 0.3- to 0.6-ppm is a painful condition that reacts well to de- area. 1.0 mg is recommended for a child aged bridement, hydrogen peroxide rinses, and an- 6 to 16 years in a population with less than tibiotics and is caused by fusiform, spirochetes, 0.3-ppm water fluoridation. and Prevotella intermedia. Aggressive periodon- titis may be localized or general and is asso- 3. The correct answer is D. A primary tooth has ciated with Actinobacillus actinomycetemcomi- a poor prognosis if replantation is attempted; tans. It is marked by rapid loss of attachment therefore, primary teeth are very rarely re- and bone (in the localized form, on the first per- planted. A radiograph should be taken to visu- manent molars and permanent incisors) and in- alize if any fragments of the tooth remain. If creased plaque and calculus. Treatment usually the tooth was a permanent tooth, the tooth may includes surgery and antibiotics for the perma- be replanted, splinted, and then root canal ther- nent dentition. Primary herpetic gingivostom- apy (RCT) performed after 7 to 10 days. If the atitis is caused by the Herpes simplex virus, and apices of the tooth are open, the tooth should be the primary form usually involves children un- monitored and apexification procedures should der the age of 3. The primary form is usually begin if there is evidence of an infected pulp. subclinical. Gingivitis in children is very com- As explained before, if the tooth was permanent mon and may be treated with oral hygiene in- with closed apices, the steps indicated in answer struction and more parental supervision. There choice B should be followed. This answer choice is no loss of attachment associated with gingivi- is the best treatment if a permanent tooth was tis, though there may be bleeding on probing. avulsed that had open apices and evidence of an Early childhood caries is rampant decay that is infected pulp. usually associated with letting a child sleep with eiti Dentistry Pediatric Answers: 1-14 141

a bottle filled with sugary liquids and/or milk. 6 years, 0.5 mg/day of fluoride supplement; and The teeth most usually affected are the maxil- for 6 to 16 years, 1.0 mg/day of fluoride sup- lary incisors. Parents should be educated as to plement is recommended. When fluoride is be- encouraging their children to drink from a cup tween 0.3 and 0.6 ppm, for age 0 to 6 months, before their first birthday and to not allow them no supplement is needed; for age 6 months to to constantly be drinking from a bottle or to sleep 3 years, no supplement is needed; for age 3 to with a bottle. 6 years, 0.25 mg/day of fluoride supplement is needed; and for age 6 to 16 years, 0.5 mg/day 6. The correct answer is A. Answer B is incorrect; of fluoride is needed. When fluoride is above 5,000 ppm is the amount contained in 1.1% NaF 0.6 ppm, no supplements are needed for all ages. foam that is delivered in a tray. Answer C is in- correct; 9,000 ppm is the amount contained in 12. The correct answer is D. The inflammatory pro- 2% NaF that is delivered in gel form. Answer cess of an endodontic lesion may create a si- D is incorrect; 12,300 ppm is the amount con- nus tract along the periodontal ligament (PDL) tained in 1.23% APF that is delivered in gel or space, which clinically appears as a narrow, deep foam form. pocket. Therefore, choice D is correct. Choice A is wrong because periodontal lesions usually 7. The correct answer is C. Answer A is the defini- have broad-based pocket formation, not narrow, tion of Epstein’s Pearls. Answer B is descriptive and usually have a history of periodontal disease of Bohn’s Nodules. Answer D is descriptive of a and are vital. Choice B is wrong because there dental lamina cyst. is a probing depth to the apex; therefore, there must be some type of periodontal involvement. 8. The correct answer is D. Answer A is incorrect. Choice C is wrong because there is no broad- The pulp horns of primary teeth are closer to based pocketing and no history of extensive pe- the outer surface of the tooth than those in per- riodontal disease. Choice E is wrong because manent teeth. Answer B is incorrect. Primary true combined lesions are very rare and would teeth have larger pulp to crown ratios than per- have signs of periodontal involvement, that is, manent teeth. Answer C is incorrect. Primary wide-based pocketing. molars have longer and thinner roots than those of permanent molars. 13. The correct answer is C. An electric pulp tester 9. The correct answer is B. Answer A is the may interfere with some cardiac pacemakers and calcification time of maxillary and mandibu- is therefore contraindicated in a patient with a lar canines. Answer C is the calcification time pacemaker. Answer choice A is wrong because of maxillary and mandibular lateral incisors. electric pulp testing is not a reliable test in pedi- Answer D is the calcification time of maxillary atric patients but is not contraindicated. Answer and mandibular first premolars. choice B is wrong because there is no contraindi- cation to the use of an electric pulp tester in a 10. The correct answer is C. Answer A is incorrect. patient with a recent MI, although endodontic The space mesial to the mandibular canine is not therapy is contraindicated. Answer choice D is included in the primate space. Answer B is in- wrong because recent trauma may cause a false correct. The space distal to the maxillary canine negative response to electric pulp testing but is is not included in the primate space. Answer D not a contraindication. Choice E is wrong be- is incorrect. The space distal to the mandibu- cause a metal restoration may give a false posi- lar canine is also included in the definition of tive response to electric pulp testing but is not primate space. contraindicated.

11. The correct answer is B. When fluoride in 14. The correct answer is D. Although manufactur- water is below 0.3 ppm, for age 0 to 6 months, no ing defects can cause instruments to separate in supplement is needed; for 6 months to 3 years, a canal, they are very rare and much less likely to 0.25 mg/day of fluoride supplement; for 3 to cause separation than the other answer choices. 142 Chapter 5: Pediatric Dentistry

15. The correct answer is E. One of the objectives of 19. The correct answer is E. Pediatric anesthesia is apexogenesis is to allow for safer and more effec- important to master. Understanding that dosage tive endodontic therapy. Choice A is wrong be- and toxicity are different in children as com- cause apexogenesis is vital pulp therapy. Choice pared with adults is paramount. Dosage and tox- B is wrong because apexogenesis encourages nat- icity are directly related to epinephrine (EPI)

Pediatric Dentistry ural root lengthening, root wall thickening, and concentration as increasing EPI decreases the apical closure. Choice C is wrong because apex- speed of systemic uptake. To avoid local anes- ogenesis is indicated in teeth with damaged coro- thesia (LA) toxicity in children, a concentration nal pulp but healthy radicular pulp. Choice D of 1:100,000 EPI should not be exceeded. Avoid is wrong because apexogenesis is indicated for using 1:50,000 concentrations. Another poten- an immature tooth but contraindicated for an tially fatal mistake is to use LA without any EPI. avulsed tooth. This can cause too rapid systemic uptake of LA. Children have increased heart rates inherently; 16. The correct answer is E. All of the above is the thus, faster systemic uptake of drugs and LA with- correct answer as fluoride varnish is quickly be- out EPI will be released too fast into the sys- coming the choice of pediatric dentists because temic circulation. Articaine has been shown in it is a premeasured dose, so overdose is impos- the literature and extensive European and Cana- sible; the fluoride is delivered via a resin carrier dian use to provide pulpal anesthesia following so ingestion is greatly reduced and adhesion is mandibular infiltrations in the posterior, includ- greatly increased; because of these benefits, it is ing second primary molars. Articaine can poten- perfect for high-caries risk patients and should be tially eliminate the dreaded “Missed” block. used safely on more frequent intervals to reduce enamel damage, enhance enamel remineraliza- 20. The correct answer is D. Avulsed primary teeth tion, and decrease pathogenic bacteria. should not be reimplanted as it can result in damaging the permanent tooth bud or follicle. 17. The correct answer is E. All of the above is the Permanent teeth need to be reimplanted as soon correct answer because xylitol has been exten- as possible with best results if reimplanted within sively shown to affect pathogenic bacteria cell 30 minutes. Flexible splinting is preferred for walls, thus reducing pathogenic bacteria viabil- 14 days or less; rigid splinting seems to not mimic ity; ART techniques can be used as a therapy the flexibility of the PDL and inhibits proper to enhance enamel/dentin before final restora- healing. tions; stannous fluoride also inhibits proper cell wall biology of pathogenic bacteria; and fluo- 21. The correct answer is B. Proliferation (cap ride varnish also can enhance enamel and cause stage) is the formation of shape of tooth and pathogenic bacteria reductions. enamel organ. Bud stage is the interaction of oral epithelium and dental lamina. Bell stage is 18. The correct answer is D. The AAPD guide- the differentiation into specific tooth and tissue lines suggest avoiding resin-based composite types, histodifferentiation. Apposition cells be- material in high-caries patients. Resin compos- gin depositing their corresponding tissues. Pri- ite has significant shrinkage and is not suited mary teeth begin calcification during second to pediatric restorations in high-risk patients. trimester. Glass ionomer and resin-based GI has fluoride release and bonds chemically to enamel and 22. The correct answer is C. Gemination is when dentin without significant shrinkage. Stainless two crowns are on a single root; therefore, the steel crowns provide 360-degree coverage from root count would be one less than the crown microbial contamination when properly fit and count. Fusion appears as a large crown; usually, cemented using glass ionomer cement. Stainless there are two roots, so the root count would be steel crowns are the gold standard for any high- normal and crown count would be one less. This risk pediatric patient with multiple cavitated is a form of fusion; the cementum is in contact. surfaces. Attrition is physiologic wear of the incisal and eiti Dentistry Pediatric Answers: 15-30 143

occlusal surfaces. There is a problem with the 26. The correct answer is E. Eleven to 12 years is count. the age when one can expect the eruption of a maxillary permanent canine in an individual 23. The correct answer is D. Class I—unilateral with an ideal eruption sequence. The most favor- notching of the vermilion not extending to lip. able eruption sequence in the permanent denti- Class II—same as Class I but extending to lip tion is: Maxilla: 61245378 Mandible: 61234578. but not to floor of the nose. Class III—Class II Note the difference in the eruption sequence and extending to floor of the nose. Class IV—any with regards to the permanent canines (3s). bilateral cleft of the lip. 27. The correct answer is B. Hank’s balanced salt 24. The correct answer is E. Never reimplant pri- solution, cold milk, saliva, water. Extra-oral dry mary teeth—risk of ankylosis and affecting per- time and root development are major determi- manent tooth eruption. A to D—these are wrong nants of reimplantation prognosis. Preservation choices because you never want to reimplant pri- of PDL fibers is key to success. The best medium mary teeth no matter how soon or clean the tooth is one that does not destroy tissue. Water is hy- is. For a permanent tooth that has avulsed, you potonic and causes cell lysis. want to reimplant as soon as possible and splint for 1 to 2 weeks. 28. The correct answer is D. Sixty-eight milligram. 4% = 4,000 mg/100 mL = 40 mg/1 mL. 25. The correct answer is C. When fluoride in 68 mg = 1.7 mL. water is below 0.3 ppm, for age 0 to 6 months, no supplement is needed; for 6 months to 3 years, 29. The correct answer is C. Hand over mouth ex- 0.25 mg/day of fluoride supplement; for 3 to ercise. According the AAPD Reference Manual, 6 years, 0.5 mg/day of fluoride supplement; and voice control, tell-show-do, positive reinforce- for 6 to 16 years, 1.0 mg/day of fluoride sup- ment, and distraction are all acceptable basic plement is recommended. When fluoride is be- behavior management techniques. Hand over tween 0.3 and 0.6 ppm, for age 0 to 6 months, mouth, while previously utilized with success, no supplement is needed; for age 6 months to is no longer mentioned in the AAPD Clinical 3 years, no supplement is needed; for age 3 to Guidelines. This technique has fallen out of fa- 6 years, 0.25 mg/day of fluoride supplement is vor because of lack of parental acceptance. needed; and for age 6 to 16 years, 0.5 mg/day of fluoride is needed. When fluoride is above 30. The correct answer is D. Eruption sequence. All 0.6 ppm, no supplements are needed for all of the other answers have been shown to have a ages. direct impact on caries risk. This page intentionally left blank CHAPTER 6 Endodontics

145 146 Chapter 6: Endodontics

QUESTIONS

1. A patient received a large MOD composite D. Necrotic pulp, chronic apical periodontitis restoration 1 week ago. She is now experiencing E. Necrotic pulp, acute apical periodontitis intense, spontaneous pain, with exacerbation of symptoms occurring when she applies heat or 5. You have been treating a patient in your practice cold or when she eats sweets. The pulpal diag- for 20 years. As your patient has aged, numerous nosis is: changes have occurred in his pulp tissues. All of the following can be associated with age-related A. Pulp necrosis changes to the dental pulp except: B. Acute periapical periodontitis C. Reversible pulpitis A. Decreased cellular elements D. Traumatic occlusion B. Pulp stone formation E. Irreversible pulpitis C. Radiographic obliteration of the pulp space D. Increased response to electric pulp testing 2. A 12-year-old boy has arrived in your office after E. Decreased vascularity a fall that fractured tooth #9 up to the gingival margin on the mesial aspect with a pulp expo- 6. A patient presents to your office with a chief sure. What is the appropriate treatment? complaint of dull, diffuse pain in the lower right Endodontics quadrant. The nerve fibers responsible for this A. Extraction sensation are: B. Pulpotomy C. Direct pulp cap A. Myelinated A fibers D. Root canal therapy B. Unmyelinated A fibers E. Apexogenesis C. Myelinated C fibers D. Unmyelinated C fibers 3. Root canal therapy was completed on a nonvital E. Subodontoblastic plexus of Raschkow tooth that suffered trauma 5 years prior. At the time of obturation, the tooth exhibited a periapi- 7. On physical and radiographic examination, your cal radiolucency on radiographic examination. patient presents with DO decay and a gingival Radiographically, healing should be visualized swelling at tooth #28. The conical defect on the in: tooth probes more than 12 mm on the buccal aspect and does not respond to electrical pulp A. 1 week testing. There is no mobility, and this condition B. 6 weeks is localized to the affected tooth. The periapical C. 1 month radiograph shows destruction of the periodon- D. 6 months tium from the level of the gingival sulcus to the E. 1 year apex of the tooth. Proper treatment of this con- dition includes: 4. A patient presents with a chief complaint of pain in the upper right quadrant. Cold test produces A. Endodontic treatment only a response lingering for 1 minute on tooth #4. B. Endodontic treatment followed by periodon- In addition, tooth #4 is sensitive to percussion tal treatment with the blunt end of an instrument. What is the C. Periodontal treatment only pulpal and periapical diagnosis? D. Periodontal treatment followed by endodon- tic treatment A. Irreversible pulpitis, normal periapex E. Extraction B. Irreversible pulpitis, acute apical perio- dontitis 8. Your patient presents with diffuse pain in her up- C. Reversible pulpitis, acute apical perio- per right quadrant. She is unable to determine dontitis the offending tooth. She states that the pain is Questions: 1-15 147

exacerbated when she drinks cold liquids but not 12. An Asian patient presents to your office with pain with mastication. Tooth#4 has an existing MOD bilaterally in her lower second premolars. Both amalgam restoration, as does tooth #5. All other teeth are sensitive to percussion and show peri- teeth in the quadrant are free of restorations and apical radiolucencies on radiographic examina- caries. What is the appropriate first-line diagnos- tion. The teeth do not respond to either cold or tic pulp test? electric pulp tests. There is an irregular bulge on the occlusal surfaces of each tooth. The most A. Electric pulp test likely diagnosis is: B. Heat test C. Cold test A. Irreversible pulpitis D. Test cavity B. Dens invaginatus E. Percussion test C. Dens evaginatus D. Pulp stones 9. All of the following factors may affect endodontic E. Internal resorption anesthesia except Endodontics 13. When instrumenting and subsequently obturat- A. Fatigue ing a root canal, the length should be determined B. Anxiety by the: C. Tissue inflammation D. Tooth type A. Anatomic apex E. Previous unsuccessful anesthesia B. Apical foramen C. Apical constriction 10. You have placed files in all three canals located in tooth #19 during a root canal procedure to 14. The endodontic access form is triangular in obtain measurement. Upon taking the first ra- shape for which of the following teeth? diograph, the two files in the distal canal are A. Maxillary central incisor, mandibular central superimposed. For your second radiograph, you incisor, and maxillary lateral incisor move the cone to the mesial. The resulting im- B. Mandibular central incisor, maxillary first age shows both files in the distal canal. The file molar, and maxillary first premolar that has moved to the mesial is positioned: C. Maxillary canine, maxillary first premolar, A. Mesially and maxillary lateral incisor B. Distally D. Maxillary second molar, mandibular first C. Buccally molar, and maxillary lateral incisor D. Lingually E. Maxillary central incisor, maxillary first mo- E. Cannot be determined lar, and maxillary second molar

11. You are instrumenting a canal with a size 30, 15. Root canal therapy is completed on a mandibu- 25 mm k-type file. What does each of the sizes lar first molar and closed temporarily with a cot- denote, respectively? ton pellet and a temporary sealing material, with plans for a definitive restoration to be placed as A. The length of the file and the diameter of the soon as possible. What is the most important fac- tip of the file tor in ensuring the success of the procedure? B. The diameter of the tip of the file and the length of the file A. Type of definitive restoration C. The taper of the file and the length of the B. Marginal integrity of definitive restoration file C. Type of temporary sealing material D. The diameter of the tip of the file and the D. Placement of a post taper of the file E. Type of sealer used during obturation E. The length of the file and the taper of the file 148 Chapter 6: Endodontics

16. A patient presents to your office for an emer- cedural error, you attempt to bypass and remove gency visit with a chief complaint of constant, se- the instrument. If you are unable to remove the vere throbbing pain in the upper right quadrant instrument, what should your next step be? for 2 days that has kept him awake at night and A. Extract the tooth is not relieved by over-the-counter pain medica- B. Perform a root amputation tions. He is unable to discriminate which tooth C. Perform a bicuspidation of the tooth is causing the pain and is visiting your office for D. Prepare and obturate up to the separated in- the first time on the basis of the referral of an- strument other one of your patients. What is the correct E. Obturate the distal canals only sequence for your initial diagnosis? 1. Extraoral examination 20. One of the most serious procedural errors that 2. Elicit details about the history of the chief can occur during root canal therapy is instru- complaint ment aspiration. What is the most important 3. Pulp vitality testing precaution an operator can take to prevent in- 4. Obtain a full medical and dental history strument aspiration? 5. Radiographic interpretation A. Proper rubber dam isolation A. 1, 2, 3, 4, 5 B. Only use rotary files B. 5, 1, 2, 3, 4 C. Use adequate lubrication during instrumen- Endodontics C. 4, 1, 2, 5, 3 tation D. 4, 2, 1, 3, 5 D. Recapitulate between each file E. 1, 4, 2, 5, 3 E. Irrigate often with sodium hypochlorite

17. You have initiated root canal therapy on a pa- 21. A patient presents to your office for initial exam- tient when he suddenly experiences sudden pain ination. A root canal procedure had been com- during working length determination, begins pleted on tooth #3 a year prior. Upon clinical to hemorrhage, and detects a burning sensa- examination, you observe a narrow periodontal tion when you attempt to irrigate with sodium pocket measuring the full length of your probe hypochlorite. What is the most likely cause of in the area of the mesial root. Upon radiographic these symptoms? examination, you detect a J-shaped radiolucency A. Inadequate straight line access surrounding the mesial root. Your initial diagno- B. Root canal contamination sis is: C. Root perforation A. Incomplete debridement of the mesial root D. Incomplete canal debridement B. Ledging of the mesial root E. Ledging C. Underobturation of the mesial root D. Periodontal abscess 18. During instrumentation of tooth #19, you real- E. Vertical fracture of the mesial root ize that you are unable to negotiate your file to the complete working length. The procedural 22. You completed a root canal procedure on tooth error that has occurred is most likely: #9 approximately 6 months ago. Your patient is A. Ledging still reporting persistent symptoms of acute api- B. Instrument separation cal pathosis including sensitivity on mastication C. Vertical root fracture and a dull ache, and the periradicular radiolu- D. Inadequate straight line access cency that was visible on your obturation radio- E. Furcation perforation graph has not appeared to decrease in size. The most likely reason for the persistence of these 19. During instrumentation of the mesial root of symptoms is: tooth #30, your file separates and remains stuck A. Phantom tooth pain in the canal. In an attempt to remedy this pro- B. Root canal failure Questions: 16-29 149

C. Trigeminal neuralgia 26. A 4-year-old child has fallen and hit his central D. Traumatic occlusion incisor. Over time, the tooth has become increas- E. Myofascial pain ingly more discolored and does not resolve. The most likely cause of the discoloration is: 23. After completion of root canal therapy, you recall your patient to evaluate the treatment outcome. A. Endemic fluorosis What are the criteria for successful root canal B. Systemic drugs therapy? C. Enamel hypocalcification D. Intrapulpal hemorrhage 1. Absence of pain E. Amelogenesis imperfecta 2. Absence of swelling 3. Sinus tract healing 27. Following obturation, sealer was left in the 4. No residual probing defects coronal pulp chamber of tooth #9, and the con- 5. Resolution or healing of periapical lesions servative access was filled with a composite A. 1, 3, 5 restoration. This could most likely result in: Endodontics B. 1, 2, 4, 5 A. Root canal failure C. 2, 3, 4 B. Discoloration of the tooth D. 1, 2, 3, 4, 5 C. Vertical root fracture E. 1, 2, 3, 4 D. Inadequate coronal seal E. Bacterial leakage 24. Retrograde (apical resection surgery) treatment is considered over orthograde treatment (root 28. A patient presents to your office with a discolored canal retreatment) for patients by their dentist tooth #24 that was treated with root canal 4 years after their root canal therapy is considered to before. The material of choice for an internal have failed. When should retrograde treatment bleaching procedure is: be performed over orthograde treatment? A. Hydrogen peroxide A. When an expensive, yet coronally sealed B. Carbamide peroxide restoration would have to be refabricated to C. Sodium perporate accommodate orthograde treatment B. When a patient is anxious about traditional 29. Incision for drainage will release exudates from retreatment procedures a soft tissue swelling, reducing irritants and pain C. If the goal of treatment is to eliminate mi- from pressure buildup. What are the ideal con- croorganisms from the root canal system ditions under which to perform an incision for D. When the root canal filling materials are easy drainage? to remove A. A tooth with reversible pulpitis and pain on E. If the treating dentist prefers surgical inter- mastication vention B. A necrotic tooth with an indurated swelling 25. An 8-year-old patient presents to your office for at the apex an emergency visit with a traumatic exposure of C. A necrotic tooth with a fluctuant swelling at tooth #9. The treatment of choice for this patient the apex is: D. A tooth with irreversible pulpitis and pain on percussion A. Root-end closure procedure/apexification E. A necrotic tooth with spontaneous pain but B. Traditional root canal therapy no swelling C. Vital pulp therapy/apexogenesis D. Partial pulpectomy E. Temporization and reevaluation in 1 week 150 Chapter 6: Endodontics

30. A patient presents to your office for an initial B. Severe pain that disappears 1 to 2 seconds maintenance visit and you prescribe a full se- after removing the cold ries of radiographs. During interpretation, you C. Mild to moderate pain that disappears 1 to note as an incidental finding a periapical radiolu- 2 seconds after removing the cold cency on the mesial root of tooth #19. The tooth D. Moderate to severe pain that lingers after is restored with an intact amalgam MO restora- removing the cold tion with intact margins and no signs of leakage or recurrent caries. The tooth is asymptomatic 34. You are doing a deep occlusal preparation on and responds normally to all vitality testing. Your tooth #30. The tooth was asymptomatic prior to patient has indicated a history of cancer in his treatment. All caries have been removed, but you medical history. The proper course of action in notice a pinpoint mechanical pulpal exposure. this case is: Hemorrhage is easily stopped. The treatment of choice is: A. Pulpotomy B. Biopsy the lesion A. Pulpotomy C. Root canal therapy B. Indirect pulp cap D. Extraction C. Root canal therapy D. Direct pulp cap 31. When making a diagnosis, the primary goal of E. Amalgam restoration

Endodontics your diagnostic tests is to reproduce the chief complaint. You test the suspected tooth for per- 35. The outline form of the access cavity of which cussion sensitivity and palpation sensitivity. Your of the following teeth is trapezoid in shape? positive percussion findings can be interpreted A. Maxillary first molar and mandibular first as follows: molar A. The tooth is nonvital and should be treated B. Mandibular first molar and maxillary second with root canal therapy. molar B. There is inflammation in the PDL. C. Maxillary first molar and maxillary second C. There is inflammation in the PDL and the premolar surrounding periodontium. D. Mandibular second premolar and maxillary D. There is a root fracture present. second molar E. The tooth is necrotic. E. Mandibular first molar and mandibular sec- ond molar 32. When making a diagnosis as to the vitality of a tooth, you employ the electric pulp tester. You 36. A patient presents to your office with a fractured obtain a measurement that suggests the tooth is tooth #9. The fracture involves enamel, dentin, necrotic. With this information alone, you can and pulp. This fracture can be classified as: determine that: A. Root fracture A. The tooth is necrotic. B. Crown–root fracture B. Further testing is necessary to make a defini- C. Complicated crown fracture tive diagnosis. D. Uncomplicated crown fracture C. The tooth needs root canal therapy. D. The tooth has an inflamed PDL. 37. You are halfway through the root canal treat- E. The tooth has calcified canals. ment on tooth #30. To prevent bacterial growth in the canal between appointments, you decide 33. A patient presents to your office with pain in to use an intracanal medication. The interap- tooth #12. You perform a cold test with Endo Ice pointment medicament of choice is: to determine vitality. If the tooth has irreversible A. Sodium hypochlorite pulpitis, the cold test will result in: B. Ethylenediaminetetraacetate A. No response to the cold test C. Chlorhexidine Questions: 30-43 151

D. Calcium hydroxide C. Tracing the fistula with a gutta percha point E. Gutta-percha in conjunction with the radiograph D. Periodontal probing of all teeth in the area 38. A 9-year-old patient has avulsed tooth #8 in a E. Take radiograph from two different angles playground accident. His mother has recovered the tooth and has called to ask how it should be 42. A 35-year-old woman was in a horse back rid- stored while she gets her son to your office. The ing accident less than 1 hour ago. On clinical best way to store an avulsed tooth for the best examination, the tooth is painful to palpation prognosis is: and has slight mobility. The tooth is fractured in the occlusal third and there appears to be no A. Dry exposure. A periapical radiograph reveals frac- B. Tap water ture above the pulpal space and no periapical C. Saline radiolucency. The treatment of choice for the D. Saliva

asymptomatic maxillary central incisor is: Endodontics E. Milk A. Root canal treatment in the occlusal segment 39. A 20-year-old male patient presents to your office B. RCT in the occlusal segment and 2 weeks of with tooth #9 in his hand. It had been avulsed passive splinting the day before during a camping trip and was C. RCT in both segments stored dry in a plastic bag. Your treatment plan D. RCT in the occlusal segment and surgical should be: removal of the apical segment A. Perform root canal therapy and replant the E. No treatment at this time and continued ob- tooth servation B. Give your patient all of his options to replace 43. You are playing a soft ball game in Central Park. the tooth Your good friend is the catcher. He is not wearing C. Clean the tooth and socket and replant the a mask. A foul tip hits him in the mouth and the tooth left central incisor is avulsed and lands in the dirt D. Clean the tooth and socket, replant the tooth, behind the home plate. Your office is 10 minutes and splint it for a week away. The best treatment for the tooth is: 40. A primary tooth has an exposed vital pulp. The A. Scrape off all the debris and remove the tooth has less than two-third of its root remain- contaminated periodontal ligament; then re- ing and caries perforating the furcation. There plant immediately is a succedaneous tooth forming normally api- B. Gently clean the tooth of debris and replant cal to the tooth. The treatment of choice for this C. Gently clean the tooth and carefully remove tooth is: the periodontal ligament and initiate en- A. Indirect pulp cap dodontic therapy B. Pulp cap D. Gently clean the tooth of debris with saline; C. Pulpotomy carefully remove several millimeters of the D. Extraction apex so as not to disturb the remaining peri- E. Root canal therapy odontal ligament

41. The most effective method of diagnosing the origin of fistula is: A. Visually locating the closest tooth to the fis- tula B. Percussing all of the teeth in the area of the fistula 152 Chapter 6: Endodontics

44. A new patient comes to your office. He has no A. Both the statement and the reason are correct adverse symptoms. On routine radiographic ex- and related amination, you notice apical radiolucency on a B. Both the statement and reason are correct root-canal–treated lateral incisor, which was ad- but NOT related equately restored with a post and a PFM crown. C. The statement is correct, but the reason is The RCT was completed 2 years ago. The radio- NOT graph shows an adequate widening and filling of D. The statement is NOT correct, but the rea- the canal. The patient has no contributory med- son is correct ical history. The tooth is asymptomatic. What is E. NEITHER the statement NOR the reason is the likely diagnosis? correct

A. Chronic apical periodontitis 48. During an intracoronal bleaching procedure the B. Foreign body reaction surface to which bleaching agent is applied is C. Apical radicular cyst the? D. Scar tissue E. Irreversible pulpitis A. Mesial B. Distal 45. What should the treatment plan be? C. Facial D. Lingual A. Replace the crown; retreat the canal. Endodontics B. Perform another surgery and place another 49. A Hedstrom file is made by twisting a tapered or root end material. square wire into elevated cutting edges. C. Place the patient on antibiotics to resolve the lesion. It produces its cutting effect on pulling strokes D. No treatment is needed. only. E. Extraction A. Both statements are TRUE B. Both statements are FALSE 46. When performing an endodontic re-treat pro- C. The first statement is TRUE, the second is cedure, gutta percha may be plasticized using FALSE each of the following except, which one is the D. The first statement is FALSE, the second is exception? TRUE. A. Xylol B. Sodium Hypochlorite 50. Which of the following best illustrates the rea- C. Eucalyptol son why calcium hydroxide is not used endodon- D. Chloroform tic procedures involving the primary denti- tion? 47. Transportation in the apical portion of canal A. Can cause external resorption walls may occur on the inner curve BECAUSE B. Promotes the formation of reparative dentin files have the tendency to return to their linear C. Has an alkaline pH shape. D. Produces no anti-microbial effect Answers: 1-6 153

ANSWERS

1. The correct answer is E. The classic signs of ir- and endothelial cells infiltrate, bone is replaced, reversible pulpitis include pain that is intense in cementum and dentin are repaired by cellular nature that occurs spontaneously without a spe- cementum, and the PDL is restored. Answer E is cific stimulus. Answer A is incorrect. A necrotic incorrect. Success is defined as elimination (or pulp indicates that the affected tooth is non- no development of) a radiolucency for at least vital and would therefore have no response to 1 year following treatment. heat, cold, or sweets. Answer B is incorrect. Acute apical periodontitis is a periapical diagno- 4. The correct answer is B. Irreversible pulpitis is sis that is characterized by percussion sensitivity. characterized by intense, spontaneous pain that Answer C is incorrect. Reversible pulpitis is usu- lingers with cold stimuli. Acute apical periodon- ally asymptomatic, but when there are symp- titis is characterized by sensitivity to percussion. Endodontics toms, they include sharp, transient pain with hot Answer A is incorrect. The pulpal diagnosis of or cold that disappears when the stimulus is re- irreversible pulpitis is correct; however, a nor- moved. Answer D is incorrect. Traumatic oc- mal periapex would not respond to percussion clusion refers to tissue damage due to occlusal with pain. Answer C is incorrect. A tooth with forces, and symptoms include sensitivity to per- reversible pulpitis would not exhibit lingering cussion and tooth mobility. pain to cold. Answer D is incorrect. A nonvi- tal, necrotic tooth would not respond to thermal 2. The correct answer is D. At 12 years of age, the tests, and chronic apical periodontitis would not maxillary central incisors should be fully devel- respond positively to percussion tests. Answer E oped with closed apices. Root canal therapy is is incorrect. As explained earlier, the symptoms the treatment of choice as it is necessary to ac- do not describe a necrotic tooth. commodate the post, core, and crown needed to restore the tooth to form and function. Answer 5. The correct answer is D. The number of nerves A is incorrect. Extraction is the last resort treat- and blood vessels in the pulp decreases with age, ment for a tooth that is nonrestorable. Answer B and this loss of sensory innervation leads to a is incorrect. Pulpotomy is performed on a trau- decreased response to electric pulp testing with matic exposure to preserve vital pulp tissue in age. Answer A is incorrect. There is a reduc- an immature tooth. Since the fractured tooth is tion in all cell types as the pulp ages, result- mature, pulpotomy is not indicated. Answer C is ing from deposition of secondary and tertiary incorrect. Direct pulp caps are usually reserved dentin. Answer B is incorrect. Pulp stone for- for small (<0.5 mm), atraumatic mechanical ex- mation increases as the pulp ages. Answer C is posures during caries removal. Answer E is incor- incorrect. Root canals decrease in diameter as a rect. Apexogenesis is a type of vital pulp therapy result of dentin deposition and pulp stone for- in which the pulp of an immature tooth is main- mation. Answer E is incorrect. Blood vessels de- tained to allow for continued dentin formation crease in number and undergo arteriosclerosis and root end closure. The tooth in this case is changes with age. already fully developed. 6. The correct answer is D. Unmyelinated noci- 3. The correct answer is D. Root canal therapy ceptive C fibers with a diameter less than 1 ␮m is considered successful in the absence of a pe- produce a pain with slow onset and a dull, dif- riapical radiolucency. Rate and completion of fuse quality. They are found mainly in the core of healing vary depending on the size of the ini- the pulp and are the most numerous nerve fibers. tial lesion but should show improvement radio- Answer A is incorrect. When stimulated, myeli- graphically in 6 months. Answers A, B, and C nated A fibers produce a fast, localized sharp are incorrect. During this time, if the bacterial pain sensation. Answer B is incorrect. A fibers are load has been decreased sufficiently, fibroblasts myelinated. Answer C is incorrect. C fibers are 154 Chapter 6: Endodontics

unmyelinated. Answer E is incorrect. The sub- 9. The correct answer is D. Tooth type should not odontoblastic plexus of Raschkow is composed affect ability to achieve profound local anesthe- of the terminal portions of the A fibers as they sia. Answer A is incorrect. Often, patients receiv- branch off, lose their myelination, and rise to the ing root canal therapy have been in severe pain coronal portion of the pulp around the level of for a few days, preventing them from getting ad- the odontoblasts. equate sleep. This can result in a decreased pain threshold. Answer B is incorrect. Many patients 7. The correct answer is B. This is the classic are fearful of root canal therapy, either as a result presentation of a true endo–perio lesion. If the of a previous bad experience or because a friend periodontal prognosis of the tooth is favorable, or family member had a bad experience. As a re- endodontics therapy should be initiated first. sult, their pain threshold is decreased. Answer C This will prevent drainage from the periapical is incorrect. If the periodontium is inflamed, pa- lesion from interfering with the healing of the tients may experience hyperalgesia or a painful periodontal lesion. Once definitive root canal reaction to a normal stimulus. In addition, it has therapy has been completed, periodontal treat- been hypothesized that because the inflamed tis- ment should be initiated. Answer A is incorrect. sue is more acidic than normal, there is less local If the endodontic lesion is treated without peri- anesthetic available in its basic form to penetrate odontal therapy following, the lesion will heal the nerve. Answer E is incorrect. Patients who up to the base of the periodontal lesion, with- have had previous difficulty achieving anesthe- Endodontics out further resolution. Answer C is incorrect. If sia are more likely to have anxiety and, as a result, the periodontal lesion is treated without the en- are more likely to have problems in the future dodontic therapy, the endodontic lesion will per- with anesthesia. sist, as will the periodontal defect, as the drainage from the periapical area will prevent regenera- 10. The correct answer is D. We are able to de- tion from occurring. Answer D is incorrect. If the termine that the mesially shifted canal is posi- endodontic treatment does not precede the pe- tioned lingually by using the SLOB rule. As the riodontal treatment, the periodontal treatment x-ray cone moves in the horizontal direction, will not be successful. Answer E is incorrect. Ex- the object that moves in the same direction is traction is the last resort treatment to be used positioned lingually (Same, Lingual), and the only if the prognosis of the tooth is hopeless. object that moves in the opposite direction is Since this condition is localized and there is no positioned buccally (Opposite, Buccal). Answer mobility, the tooth should be treated. A is incorrect. The two canals in the distal root of a mandibular molar are positioned buccally 8. The correct answer is C. It is helpful to attempt and lingually. Answer B is incorrect. The two to elicit the painful symptom during the diagnos- canals in the distal root of a mandibular molar tic process. Since cold drinks elicit pain in the are positioned buccally and lingually. Answer C patient, a cold test should be conducted prior to is incorrect. If the canal was positioned buccally, other vitality tests. Answer A is incorrect. Elec- it would have moved distally, in the opposite di- tric pulp testing may be an appropriate first test rection of the cone. Answer E is incorrect. As in another clinical situation. Answer B is incor- was stated earlier, the SLOB rule can be used rect. Since pain is not elicited by hot drinks or to determine the orientation of the files in the foods, the cold test is more appropriate. Answer distal canal. D is incorrect. A test cavity is never a first-line diagnostic pulp vitality test. It is to be used only if 11. The correct answer is B. The first number is other tests are inconclusive and a necrotic pulp a measure of the diameter of the tip of the is suspected. Answer E is incorrect. The patient k-file (30 = 0.30 mm). The second number is does not report sensitivity to mastication, so per- a measure of the length of the k-file (25 mm). cussion testing is not likely to elicit the symptom. The file diameter (taper) increases at a regular In addition, it is not a test for pulp vitality, rather rate of 0.02 mm up the shaft of the instrument it is used to make a periapical diagnosis. in k-files. Answer A is incorrect. The value 30 is Answers: 7-16 155

the measure of the diameter of the tip of the file, 0.5 mm short of the apical foramen at the apical not the length, and 25 mm is the measure of the constriction. length of the file, not the diameter of the tip. An- swer C is incorrect. The taper of k-files is 0.02, 14. The correct answer is E. The access form is not 30, although the measure of the length is triangular for the maxillary central incisor, the 25 mm. Answer D is incorrect. While 30 is the maxillary first molar, and the maxillary second measure of the diameter of the tip of the file, molar. Answer A is incorrect. The access form the taper of k-files is 0.02. Answer E is incorrect. for the maxillary central incisor is triangular and The value 30 is a measure of the diameter of the the access forms for both mandibular central in- tip of the file, and 0.02 would represent the taper cisor and maxillary lateral incisor are ovoid. An- of the file, not 25 mm. swer B is incorrect. The access forms for both mandibular central incisor and the maxillary first 12. The correct answer is C. Dens evaginatus usu- premolar are ovoid, and the access form for the

ally presents as a tubercle, most commonly in maxillary first molar is triangular. Answer C is Endodontics the mandibular premolars of Asian individuals. incorrect. The access forms for the maxillary ca- They often contain pulp tissue, as when they nine, the maxillary first premolar, and the max- fracture off or wear down as a result of attrition, illary lateral incisor are all ovoid. Answer D is the pulp may become exposed. Answer A is in- incorrect. The access form for the maxillary sec- correct. This correct pulpal diagnosis for this pa- ond molar is triangle, the access form for the tient is necrotic pulp, as there is no response to mandibular first molar is trapezoidal, and the pulp vitality tests, and there is destruction of the access form for the maxillary lateral incisor is periodontium. Answer B is incorrect. Dens in- ovoid. vaginatus most commonly occurs in the lingual of maxillary lateral incisors and is an infolding 15. The correct answer is B. A restoration that has a rather than an outgrowth of the enamel organ. It proper seal will prevent recurrent caries and bac- can also result in communication between the terial recontamination of the root canal system, oral cavity and the pulp and would be treated ensuring success of the root canal therapy. An- with root canal therapy as would dens evagi- swer A is incorrect. The type of final restoration natus. Answer D is incorrect. Pulp stones form placed after endodontic therapy is unimportant within the pulp, possibly as a result of irritation as long as the marginal integrity is maintained and are often seen as radiopaque on radiographs. and proper cuspal coverage is achieved where Answer E is incorrect. Internal resorption occurs needed. Answer C is incorrect. The type of tem- as a response to irritation, and it may be small porary sealing material is not important, as long and almost undetectable or so destructive that it as the seal is tight and it is not left in for more causes root perforation. than approximately 3 months. Answer D is in- correct. Placement of a post is used strictly as a 13. The correct answer is C. The apical constric- means for retaining core material in a tooth that tion, assumed to be at the junction of the ce- has lost significant coronal structure. It does not mentum and the dentin, is reliably measured by provide a coronal seal until a final restoration is apex locators and is usually 0.5 mm from the api- placed. Answer E is incorrect. The type of sealer cal foramen. It is the accepted determinant for used during obturation does not affect the suc- working length determination. Answer A is in- cess of the procedure, the fit of the gutta percha correct. The anatomic apex, which may also be is a more important factor in sealing the apex. called the radiographic apex, is the most apical portion of the root. We do not work to this length, 16. The correct answer is D. The first step in all pa- as the apical foramen rarely coincides with the tients’ visits should be a complete review of their anatomic apex. Answer B is incorrect. The api- medical and dental history, whether it is their cal foramen is usually 0.5 mm from the radio- first visit or not. Following this, it is important graphic/anatomic apex. Usual working length is to ascertain the details of their chief complaint including information about the duration and 156 Chapter 6: Endodontics

the quality of pain and what, if any, stimuli exac- blunted tip, signaling that an error has occurred. erbate it. Next, an extraoral examination should Answer C is incorrect. A vertical root fracture be completed, noting any areas of swelling, ten- most often occurs during postcementation or derness, or lymphadenopathy. After the extrao- condensation during obturation. It would not ral examination, a complete intraoral examina- present with a sudden inability to reach work- tion should be completed, including appropriate ing length and usually would not occur during pulp vitality tests and percussion and palpation routine instrumentation. Answer D is incorrect. tests. Finally, a determination should be made Inadequate straight line access may be one of the as to what radiographs are needed, followed by causes of ledging but would not directly cause a interpretation. Answer A is incorrect, answer B sudden loss of working length. Answer E is in- is incorrect, answer C is incorrect, answer E is correct. A furcation perforation might prevent incorrect; see the earlier reasons. an operator from accurately measuring working length if a file were inserted through the perfora- 17. The correct answer is D. Failure to direct the tion, but it would not result in a sudden inability bur parallel to the long axis of the tooth dur- to reach working length. ing access preparation can result in crown or root perforation. Perforation into the PDL usu- 19. The correct answer is D. The initial strategy for ally results in hemorrhage that is difficult to con- a separated instrument is to try and bypass the trol. Pain may occur as the operator attempts instrument in the same way you would bypass Endodontics to place an instrument through the perforation a ledge. If this is successful, broaches or Hed- and enters the PDL. A burning sensation or a strom files can be used in attempt to remove bad taste can be detected during irrigation with the separated segment. If this is possible, the sodium hypochlorite as the solution exits the root canal is cleaned, shaped, and obturated to work- canal system. Answer A is incorrect. Inadequate ing length. If it is not possible to remove the straight line access may be a cause of root per- segment, the canal is cleaned, shaped, and ob- foration and many other procedural errors that turated to the new working length at the most may occur during root canal therapy, but it is coronal portion of the separated instrument. An- not the direct cause of the listed symptoms. An- swer A is incorrect. Extraction is the last resort of swer B is incorrect. Root canal contamination treatment, usually reserved for failed root canal would not result in the immediate symptoms but therapy or nonrestorable teeth. The prognosis of would cause more long-term problems such as a tooth with a separated instrument left in the intervisit emergencies or, ultimately, root canal canal depends on how much debridement had failure. Answer D is incorrect. Incomplete root been finished prior to the separation, and the canal debridement would result in problems procedure should be deemed a failure before similar to root canal contamination. Answer E extraction. Answer B is incorrect. Similar to ex- is incorrect. Ledging can result in incomplete traction, root amputation is usually reserved for root canal debridement and inadequate obtu- cases in which root canal therapy has failed or a ration, none of which would cause the listed portion of a tooth is periodontally involved. An- symptoms. swer C is incorrect. Bicuspidization of a tooth, similar to root amputation, is usually not a first- 18. The correct answer is A. To maintain the pa- line treatment for instrument separation. Answer tency of the canal and to maintain the working E is incorrect. It is not acceptable to leave any length, it is necessary to recapitulate frequently canals in a root canal treated to unobturated. and use irrigation and lubrication. Other causes The treatment would be incomplete and would of ledging include inadequate straight line ac- result in definite failure. cess, retention of debris in the apical end of the canal, and excessive enlargement of curved 20. The correct answer is A. Aspiration of instru- canals. Answer B is incorrect. An instrument sep- ments can be avoided by judicious use of rubber aration is possible but is not the best answer. dam isolation, which is the standard of care dur- An instrument that has been broken will show a ing root canal therapy. Answer B is incorrect. Answers: 17-24 157

If a tooth is properly isolated, there is no risk presenting symptoms and will typically present of instrument aspiration. Answer C is incorrect. as worse during particular times of the day, upon Using proper lubrication may help to avoid in- waking in the morning, for example. strument separation, but it will not prevent as- piration. Answer D is incorrect. Recapitulation 23. The correct answer is D. A root canal is con- is important in maintaining working length, not sidered successful if there is an absence of signs in preventing instrument aspiration. Answer E and symptoms including all of those mentioned. is incorrect. Frequent irrigation is necessary for Answer A is incorrect. If a patient is experienc- complete debridement of the canals, not for the ing swelling or periodontal pocketing, the root prevention of aspiration. canal procedure is deemed a failure. Answer B is incorrect. If a sinus tract fails to heal, the root 21. The correct answer is E. Narrow pockets to the canal therapy was not successful. Answer C is in- level of the fracture and J-shaped or teardrop- correct. Pain and periapical lesions that do not

shaped radiolucencies are indicative of vertical decrease in size are signs of root canal failure. Endodontics root fractures. The only way to definitively diag- Answer E is incorrect. In order for a root canal nose a vertical root fracture is to see it during to be deemed a success, periapical lesions must an exploratory surgical procedure. The progno- decrease in size and/or resolve. sis of a root with a vertical fracture is hopeless, and it should be amputated or the tooth must be 24. The correct answer is A. The decision to per- extracted. Answer A is incorrect. Incomplete de- form retrograde or orthograde retreatment is bridement of canals can lead to root canal failure highly dependent on the circumstances of each but will not present with the narrow pocketing individual patient. In certain circumstances, if it or J-shaped lesions of a fractured root. Answer B would be more costly and time consuming to fab- is incorrect. Ledging can also lead to failure if ricate a new restoration following conventional the canal has not been fully debrided or filled retreatment, it is acceptable to consider apicoec- to original working length. Answer C is incor- tomy procedures as initial retreatment modali- rect. Underobturation can result in failure if de- ties. Answer B is incorrect. It is necessary to help bris remains in the apical portion of the canal patients deal with their anxiety, and avoidance or the apical seal is compromised. Answer D is of a procedure due to patient anxiety is not an incorrect. A periodontal abscess does not usu- acceptable treatment option. It is quite common ally present as a narrow defect to the apex of a for patients to have higher anxiety surrounding a tooth. retreatment procedure than an initial treatment, but it is not a reason to choose one treatment 22. The correct answer is B. The persistence of over another. Answer C is incorrect. Retrograde swelling, sinus tracts, spontaneous or dull pain, treatment does not function to remove microor- or pain of biting, in addition to lack of resolution ganisms from the root canal system, orthograde of periapical lesions, all indicate root canal fail- treatment accomplishes this. Rather, the goal of ure. Answer A is incorrect. The more likely rea- retrograde treatment is to prevent microorgan- son for these symptoms to remain is a failed root isms from leaving the root canal system. Answer canal procedure. Answer C is incorrect. Trigemi- D is incorrect. When root canal filling mate- nal neuralgia typically presents with intense pain rials are easy to remove, it makes orthograde that is more intense than applied stimuli, re- retreatment a better option than retrograde re- ferred pain, and trigger points. These are not treatment. Answer E is incorrect. Treatment is typical presentations for failed root canal ther- not decided on the bass of the preference of the apy. Answer D is incorrect. This is possible if the dentist, it should be based on the best progno- treated tooth was restored to high occlusion and sis for the patient. In addition, if practitioners the PDL was irritated; however, the more likely feel they are unable to properly perform the best answer, considering the radiographic evidence, procedure for the patient, the case should be re- is root canal failure. Answer E is incorrect. ferred to another practitioner who can. Myofacial pain is usually not as localized as the 158 Chapter 6: Endodontics

25. The correct answer is C. An 8-year-old child will Answer A is incorrect. As long as the coronal seal have maxillary central incisors with immature was adequate, sealer remnants in the pulp cham- apices. Because this is a vital exposure, vital pulp ber would not lead to root canal failure. Answer therapy including a shallow pulpotomy should C is incorrect. Vertical root fracture is most often be the treatment of choice. This will result in caused during the placement of a post. Leaving the remaining vital pulp to continue to allow the sealer in the chamber would not increase the apex to develop normally. Once the apex has the chance of vertical root fracture. Answer D formed, traditional root canal therapy should be is incorrect. The sealer remaining in the cham- completed. Answer A is incorrect. Root-end clo- ber should not prevent an adequate coronal seal. sure should be used only to treat nonvital teeth Answer E is incorrect. Because it is possible to with immature apices because there is no possi- obtain a coronal seal even if the sealer is left in bility of further growth of the tooth structure. the chamber, it would not be the direct cause of Answer B is incorrect. Traditional root canal bacterial leakage. therapy is not possible on a tooth with an open apex as it is hard to form an acceptable apical 28. The correct answer is C. Sodium perborate is seal. Answer D is incorrect. A partial pulpec- a powder that decomposes into hydrogen perox- tomy would remove vital pulp tissue from the ide and oxygen in the presence of acid or water. root canal system, making continued root for- It is safer and less damaging to tissues than hy- mation less likely. Answer E is incorrect. Tem- drogen peroxide and is therefore the material of Endodontics porization without a shallow pulpotomy would choice for internal bleaching. Answer A is incor- potentially allow for bacterial contamination of rect. Hydrogen peroxide is an oxidizer that is un- the pulp and root canal system. stable in high concentrations and tends to burn tissue. Therefore, sodium perborate is a better 26. The correct answer is D. Intrapulpal hemor- choice for the bleaching procedure. Answer B rhage occurs as the result of injury to coronal is incorrect. Carbamide peroxide is used for ex- blood vessels, usually suffered during impact ternal bleaching and not for internal bleaching to a tooth. The hemorrhaged red blood cells procedures. and their disintegration products stain the sur- rounding dentin, and it tends to increase over 29. The correct answer is C. When a fluctuant time. Answer A is incorrect. Endemic fluorosis is swelling is incised, the purulence is released im- caused by ingestion of too much fluoride during mediately and a relief is felt right away. Answer tooth formation. Teeth may erupt with a chalky A is incorrect. A tooth with reversible pulpitis appearance, getting discolored only after they and pain on mastication does not need incision absorb stains from the oral cavity. Answer B is in- for drainage or root canal therapy. Rather, the correct. Discoloration from systemic drugs, most offending stimulus must be removed to relieve often tetracycline, is usually bilateral and can the symptoms. Answer B is incorrect. A necrotic range from yellow to dark gray depending on tooth with an indurated swelling may be incised the exposure. Answer C is incorrect. Enamel for drainage; however, the outcome is less pre- hypocalcification usually appears as a distinct dictable. Relief of pressure will most likely be brown or white area on the crown of a tooth. felt; however, it is possible that only blood or Answer E is incorrect. Amelogenesis imperfecta serous fluid will be released. Answer D is incor- may result in yellow or brown discolorations, rect. A tooth without a swelling is a contraindi- usually affecting more than one tooth. cated for incision for drainage. Answer E is in- correct. A tooth without swelling is a contraindi- 27. The correct answer is B. Incompletely remov- cated for drainage. ing the sealer or gutta percha after obturation is a common cause of tooth discoloration following 30. The correct answer is B. All vitality testing and root canal therapy. This dark discoloration can the lack of symptoms should lead you to deter- be prevented by maintaining pulp chamber with mine that the tooth is vital; therefore, a peri- clean walls prior to final restorative treatment. radicular lesion should not be attributed to an Answers: 25-35 159

infection originating in the root canal system. 33. The correct answer is D. A tooth with irre- In addition, the patient’s history of malignancy versible pulpitis will react with severe pain, re- makes a biopsy the correct treatment for the un- producing the chief complaint of the patient that defined lesion. Answer A is incorrect. Because lingers after the stimulus is removed. Answer A the tooth is vital, with no symptoms and an intact is incorrect. No response to Endo Ice would in- restoration, a pulpotomy should not be per- dicate a necrotic pulp. Answer B is incorrect. formed. Answer C is incorrect. Root canal ther- Severe pain that does not linger after removal of apy should not be the treatment of choice for this the stimulus is indicative of reversible pulpitis. tooth for the same reasons as pulpotomy is not Answer C is incorrect. Mild to moderate pain the treatment of choice. Answer D is incorrect. that does not linger after removal of the stimulus The tooth should not be extracted except as the is indicative of a normal pulp. last resort if the biopsy is positive for malignancy and it was determined that all of the malignant 34. The correct answer is D. Indications for direct

could not be removed without extraction. pulp cap therapy include permanent teeth in Endodontics which there has been a pinpoint mechanical ex- 31. The correct answer is B. The percussion test posure of a vital, asymptomatic tooth. Bleeding determines whether or not there is inflamma- must be controlled, and the pulp cap material tion in the periodontal ligament. Answer A is must make direct contact with the exposure. An- incorrect. The percussion test cannot be used swer A is incorrect. A pulpotomy is indicated to determine whether a tooth is vital or not and in primary teeth with exposed vital pulps or ir- should not be used as a test to determine whether reversible pulpitis. It is indicated in permanent root canal therapy is indicated as a treatment op- teeth as a temporary emergency procedure to tion. Answer C is incorrect. The palpation test alleviate symptoms prior to conventional root would be used to determine whether inflamma- canal therapy or in permanent teeth with im- tion has spread from the PDL to the surrounding mature apices to encourage further root devel- periodontium. Answer D is incorrect. Although opment and closure. Answer B is incorrect. An percussion sensitivity is common in the presence indirect pulp cap is indicated on permanent of a root fracture, it is not diagnostic. Answer E teeth with immature apices if there is a cari- is incorrect. To determine the vitality of a tooth, ous lesion, which, if removed, will result in a other tests would be employed, including elec- pulpal exposure as long as the tooth is asymp- tric pulp testing and cold testing. tomatic with no periapical pathology. Answer C is incorrect. Root canal therapy would be indi- 32. The correct answer is B. Electric pulp testing cated in this case if the direct pulp capping pro- produces a high incidence of both false-positives cedure failed, resulting in symptoms. Answer E and false-negatives. As a result, it is not a defini- is incorrect. A conventional amalgam restora- tive test, and additional diagnostic aids should tion may be placed only if there is no pulpal be used. Answer A is incorrect. Because elec- exposure. tric pulp testing is not definitive, a diagnosis of necrotic at this point would be premature. An- 35. The correct answer is E. The outline of the swer C is incorrect. A definitive treatment plan access cavities for both the first and second cannot be made until a definitive diagnosis is mandibular molars is trapezoid. Answer A is in- made. Answer D is incorrect. Electric pulp test- correct. The outline of the access cavity for the ing measures the level of stimulation for sensory maxillary first molar is triangular, and the out- nerves within the pulp, giving no information line of the access cavity for the mandibular first about inflammation. The proper test to diag- molar is trapezoid. Answer B is incorrect. The nose inflammation in the PDL would be per- outline of the access cavity for the mandibular cussion. Answer E is incorrect. Calcified canals first molar is trapezoid, and the outline of the may lead to an electric pulp test reading that access cavity for the maxillary second molar is suggests necrosis but can be diagnosed only by triangular. Answer C is incorrect. The outline of radiographic or clinical examination. the access cavity for the maxillary first molar is 160 Chapter 6: Endodontics

triangular, and the outline of access cavity for the able, saline would be a good storage option. maxillary second premolar is ovoid. Answer D is Answer D is incorrect. Saliva could be used in incorrect. The outline of the access cavity for place of saline if milk or Hank solution were not the mandibular second premolar is ovoid, and available. the outline of the access cavity for the maxillary second molar is triangular. 39. The correct answer is B. A tooth that had been avulsed more than an hour earlier and not stored 36. The correct answer is C. A complicated crown in an appropriate liquid medium is generally not fracture involves the enamel, dentin, and pulp. replanted as the prognosis is very poor. Answer Answer A is incorrect. A root fracture involves A is incorrect. If the tooth had been avulsed less cementum, dentin, and pulp. Answer B is in- than an hour prior and had been stored properly, correct. A crown–root fracture involves enamel, root canal therapy would have been completed dentin, and cementum and may or may not in- only after a week of splinting to reduce mobil- volve the pulp. Answer D is incorrect. An un- ity. Answer C is incorrect. If the tooth had been complicated crown fracture involves the enamel avulsed less than an hour prior and had been and dentin but not the pulp. stored properly, this would be a correct proce- dure, followed by flexible splinting for a week 37. The correct answer is D. Calcium is a white and definitive root canal therapy. Answer D is powder with a high pH and is applied to incorrect. This would have been the proper treat- Endodontics the canals between appointments as a cream ment plan for a tooth avulsed less than an hour mixed with sterile water. It is antibacterial and prior that had been stored in the proper liquid prevents bacterial growth between treatments. medium. Calcium hydroxide is also used to encourage calcification in pulp capping, apexogenesis, and 40. The correct answer is D. Pulpal therapy is not apexification procedures. Answer A is incorrect. indicated for this tooth, so extraction is the only Sodium hypochlorite is an acidic liquid that acts viable treatment option. Answer A is incorrect. as a solvent on organic debris and also as an an- The pulp is already exposed in this case, so indi- tibacterial agent. It is the irrigation solution of rect pulp capping in no longer an option. Answer choice but is very caustic and can cause severe B is incorrect. Direct pulp caps are not indicated reactions if extruded past the apex. Answer B for primary teeth as the low pH of calcium hy- is incorrect. Ethylenediaminetetraacetate dem- droxide will irritate the pulp and cause internal ineralizes dentin and removes inorganic mate- resorption. Answer C is incorrect. Pulpotomy is rial. It is used to remove the dentin smear layer contraindicated on primary teeth if more than and is the main ingredient in many canal lu- one-third of the root has already resorbed, if there bricants. Answer C is incorrect. Chlorhexidine is internal resorption, if the furcation is perfo- is an antiseptic used to inhibit bacteria but is rated, or if there is a periradicular lesion that not used as an interappointment canal medica- may damage the succedaneous tooth. Answer E tion. Answer E is incorrect. Gutta percha is an is incorrect. Root canal therapy is indicated only obturating material that would be used only to for primary teeth when there is no evidence of a definitively seal a clean root canal. succedaneous tooth, when there is no evidence of periradicular lesions, and when treatment will 38. The correct answer is E. Milk and Hank bal- not damage the succedaneous tooth. anced salt solution provide the best conditions in which to store an avulsed tooth due to their 41. The correct answer is C. osmolality. Answer A is incorrect. Storing an A. This would be inaccurate because the fistula avulsed tooth dry would give it the worst progno- may be located between the teeth. sis. Answer B is incorrect. Along with dry storage, B. This is wrong because the patient may have storage in tap water provides the worst progno- referred pain on adjacent teeth. sis for an avulsed tooth. Answer C is incorrect. If neither milk nor Hank solution were avail- Answers: 36-49 161

C. Although this is a good diagnostic tool to This resolves following successful treatment of identify vertical fracture, it is not sufficient the condition with RCT. Answer B is wrong be- to identify the source of the fistula. cause foreign-body reaction would present with D. A fistulous tract cannot be seen on a radio- swelling, irritation, and redness in the area. An- graph. swer C is wrong because apical radicular cyst is a fluid-filled sac at the apex of the tooth following 42. The correct answer is E. The fact that the patient necrosis of the pulp, which usually resolves with sustained trauma to her tooth is not an indication RCT. Answer E is wrong because patient cannot to initiate root canal treatment. Clinically, the have irreversible pulpitis AFTER the root canal tooth is not exposed. The accident happened therapy. only 1 hour ago, which is not a sufficient time for radiographic changes. Splinting is optional but 45. The correct answer is D. A complete resolu- not necessary since there is only slight mobility. tion of the infection does not necessarily coin-

The luxation caused inflammation around the cide with completion of root canal therapy. This Endodontics periodontal ligament, but there is a good chance condition will probably resolve on its own once that it will resolve on its own. Observation and the host immune system completely eliminates radiographic monitoring is the right treatment at the infection. No treatment is indicated for this this time. asymptomatic presentation. All other choices recommend a treatment much too aggressive for A. Toothis not exposed; therefore, no root canal an asymptomatic tooth. treatment is indicated. B. Treatment for luxated teeth is initiated with 46. The correct answer is B. NaOCl is an irrigating 2 weeks of passive splinting to allow for solution. Answer A is incorrect because xylol is reattachment of periodontal ligament and/or an aromatic isomer used as a solvent. Answers ankylosis of the tooth. RCT is initiated after C and D are incorrect because they are organic confirming unsuccessful reattachment and solvents used in gutta percha removal. pulpal necrosis. C. See reasons presented earlier. 47. The correct answer is D. Transportation will D. Surgical removal of the apical segment is occur on the OUTER curve of canal walls, not usually the treatment of choice in cases when the inner due to files returning to their linear infection develops or persists AFTER root form throughout treatment. Answers A, B, and canal therapy. C are incorrect because transportation occurs is the outer curve of the canal wall. Answer E is 43. The correct answer is B. All the other answers incorrect because files due have the tendency to call for removal of periodontal ligament, which return to their linear form. is wrong, since it contains viable cells neces- sary for successful implantation. It is necessary 48. The correct answer is C. Bleaching material is to only gently remove gross debris. acting up facial/esthetic portion of the tooth. A, B, and D are incorrect. 44. The correct answer is D. There may be extensive bone destruction in the vicinity of the infected 49. The correct answer is D. Hedstrom files are tooth, such as perforation of both labial and lin- made from a tapered wire only, K-files and gual cortical plates; sometimes, it takes years Reamers are made from various shaped wires for the bone to fill the defect, and sometimes and use a twist-and-pull method. Answer A is in- this does not happen at all. Instead, the area correct because Hedstrom wires are made from fills with scar tissue. Answer A is wrong because a tapered wire only. Answer B is incorrect be- chronic apical periodontitis is generally a condi- cause Hedstrom wires are made from tapered tion in which the apical portion of a tooth’s root wire only; second part is correct. Answer C is in- is chronically inflamed. There may be drainage correct because Hedstrom wires are made from through the gums from around the tooth’s root. 162 Chapter 6: Endodontics

tapered wire only and DO produces cutting A, B, and D are incorrect because calcium hy- effect only with pulling strokes. droxide causes internal resorption in primary dentition, reparative dentition formation in per- 50. The correct answer is C. Alkaline pH causes in- manent dentition, and produces an antimicro- ternal resorption in primary dentition. Answers bial effect, respectively. Endodontics CHAPTER 7 Periodontics

163 164 Chapter 7: Periodontics

QUESTIONS

1. Which sign or symptom is most common with a A. Miller class II periodontal abscess? B. Miller class III C. Miller class IV A. Severe throbbing pain D. Miller class I B. Sinus tract in mucosa C. Vital tooth 4. What is the source of the final tissue in a free D. Deep caries gingival graft? 2. What Miller classification would the first and A. Stratum basal of the epithelium second lower premolars be classified as? B. Adipose tissue C. Lamina propria of the connective tissue D. Stratum spinosum of epithelium

5. What is a normal hemoglobin A1c (Hb A1c) value in a nondiabetic person? A. 8–9% B. 3–4% C. 4–6% D. 2–3%

6. Why use a palatal approach for osseous peri- odontal surgery? A. Palatal embrasures are wider, allowing for better access to osseous surgery B. Less sensitivity for the patient C. More subgingival calculus on the palatal sur- faces D. Less bleeding A. Miller class II B. Miller class III 7. Which enzyme and/or cytokine is secreted by a C. Miller class IV macrophage?

Periodontics D. Miller class I A. Tumor necrosis factor (TNF) B. Transforming growth factor (TGF) 3. What Miller classification would the lower left C. Proteases central incisor be classified as? D. All of the above

8. What is the typical dose of nitroglycerin for a patient experiencing anginal discomfort? A. 0.8 mg B. 5.0 mg C. 0.3 mg D. 3.0 mg Questions 1-19 165

9. Which of the following bone grafting materials A. Nance appliance are taken from another animal species? B. Distal shoe C. Band-and-loop A. Autografts D. Lower lingual arch B. Xenografts C. Alloplasts 15. Which of the following is not a feature of primary D. Allografts resistance form in amalgam preparation? 10. Which of the following materials is not used for A. Maintain as much sound structure as prefabricated posts? possible B. Rounded axiopulpal line angle in class II A. Stainless steel preparation B. Gold-pleated brass C. Vertical walls that converge occlusally C. Silver D. Adequate thickness of amalgam D. Fiber-reinforced polymers E. Titanium 16. Which appliance is not used to correct class II malocclusion? 11. According to Miller’s index, a tooth has class II mobility when A. Reverse-pull headgear B. Herbst appliance A. Movement of greater than 1 mm in any di- C. Twin block appliance rection D. Cervical-pull headgear B. Movement of greater than 1 mm in any di- rection and it can be depressed vertically 17. Which medicament can be used during pulpo- C. Movement of less than 0.5 mm in any direc- tomy procedure? tion D. Movement of less than 1 mm in any direction A. Calcium hydroxide B. Ethylenediaminetetraacetic acid (EDTA) 12. Which of the following is not a common oral C. Mineral trioxide aggregate manifestation of Crohn disease? D. Flowable composite Periodontics A. Hypertrophy of the lips 18. A patient presents with teeth #2, 3, 4, and 14 B. Hypertrophy of the gingival tissue missing. Which Kennedy classification does this C. Aphthous ulcer partially edentulous arch belong to? D. Cobblestone appearance of the buccal mu- cosa and palate A. Kennedy class III mod 1 E. Dry mouth B. Kennedy class III C. Kennedy class I 13. During placement of a post, what is the criterion D. Kennedy class II mod 1 for the post width? 19. Allografts have the following characteristics 1/ A. It should not exceed 3 of the root width at except: its narrowest location. B. It should not exceed 1/2 of the root width at A. Osteoinductive its narrowest location. B. Osteogenic C. No criteria since post width does not affect C. Osteoconductive the retention of the post. D. Demineralized

14. If a 7-year-old patient presents with early loss of Tooth A, maxillary right second primary molar, what is the appropriate space maintainer that can be used? 166 Chapter 7: Periodontics

20. A membrane is utilized in guided tissue regen- A. Class I eration to promote repair of intraosseous defect B. Class II BECAUSE it prevents epithelial cells from con- C. Class III tacting the root surface and forming a long junc- D. Class IV tional epithelium. E. Class V

A. Both the statement and the reason are correct 25. It is important to have adequate space between and related. a crown margin and the crest of bone to avoid B. Both the statement and the reason are correct impingement of the biologic width. Which of but NOT related. the following is NOT a limiting factor? C. The statement is correct, but the reason is NOT. A. Mobility of tooth D. The statement is NOT correct, but the rea- B. Location of furcation son is correct. C. Crown-to-root ratio E. NEITHER the statement NOR the reason is D. Age of the tooth correct. E. Tooth–arch relationship

21. Periodontitis can be associated with all of the 26. A 44-year-old patient presents a third of the den- following except: tition with 5 mm of clinical attachment loss, bleeding upon probing, none of the teeth being A. Bleeding upon probing mobile, and probing depths ranging between 4 B. Increased gingival exudate and 6 mm. What is the most probable diagnosis? C. Elevated sulcular temperature D. Attachment and bone loss that is not pro- A. Localized chronic moderate periodontitis gressing B. Generalized chronic severe periodontitis E. Enlarged gingival contours due to edema C. Localized chronic severe periodontitis and fibrosis D. Generalized chronic moderate periodontitis E. None of the above 22. Indications for crown lengthening include the following except: 27. Which of the following treatment modalities will not assist in pocket reduction? A. Esthetics B. Furcation involvement A. Osseous surgery C. Delayed passive eruption B. Gingivectomy D. Crown fracture C. Apically positioned flap E. Post or pin perforations D. Guided tissue regeneration Periodontics E. Connective tissue graft 23. Aggressive periodontitis is always associated with poor oral hygiene. Chronic periodontitis is a 28. Which of the following is NOT a predictable condition found exclusively in older patients. objective of periodontal surgery? A. Both statements are TRUE. A. Remove or eliminate the lesion B. Both statements are FALSE. B. Regenerate interdental papillae C. The first statement is TRUE, the second is C. Promote gingival reattachment FALSE. D. Pocket reduction D. The first statement is FALSE, the second is E. Treat and/or control the etiology TRUE. 29. Biofilm found on tooth surface is termed as: 24. A patient presents with mandibular molar A. Enamel with furcal involvement where the bone loss B. Dental plaque is through-and-through and visible clinically. C. Saliva What is the Glickman classification? D. Dental caries Questions 20-33 167

30. Periodontitis that does not resolve with treatment 32. Increased alkaline phosphatase levels are seen is termed as: with: A. Aggressive periodontitis A. Paget disease B. Chronic periodontitis B. Hypophosphatasia C. Refractory periodontitis C. Cherubism D. Juvenile periodontitis D. Hyperparathyroidism

31. Radiographically, driven snow appearance is 33. Contrast on a radiograph is: seen with: A. Uneven density on a radiograph A. Adenomatoid odontogenic cyst B. Range of densities on a radiograph B. Calcifying odontogenic cyst C. Overall appearance of the radiograph C. Calcifying epithelial odontogenic tumor D. Overall degree of darkening of the radio- D. Keratocyst graph. Periodontics 168 Chapter 7: Periodontics

ANSWERS

1. The correct answer is C.

Pulpal Lesions Periodontal Lesions (Endodontic Abscess) (Periodontal Abscess) Vitality Nonvital Vital Usual area of swelling Vestibule Attached gingiva Pain Intermittent throbbing Dull, continuous Probing Narrow, isolated defect Generalized defect Sinus tract location Mucosa Attached gingiva, sulcus Radiograph Localized bone loss Generalized bone loss of area Local factors Variable Calculus Etiology Deep caries or restoration Possibly nonrestored

Figure 7-3. Before a free gingival graft #26. 2. The correct answer is D. The recession does not extend to the mucogingival junction (MGJ) and no bone loss exists. Miller class I defects are the most predictable for root coverage. Review the chart below.

Miller 1 Recession not extending to the MGJ and no bone loss Miller 2 Recession extending to or beyond the MGJ and no bone loss Miller 3 Recession extending to or beyond the MGJ and some bone loss Miller 4 Recession extending to or beyond the MGJ and extensive bone loss or tooth malposition

Figure 7-4. After a free gingival graft #26. Periodontics 3. The correct answer is B. The recession extends to the MGJ and there is some bone loss. Com- plete (100%) coverage of this root would not be anticipated.

4. The correct answer is C. The underlying lamina propria of the connective tissue of the donor graft will be the source of the final tissue result in a free gingival graft. Epithelium is 0.75 mm on average, hence it is better to harvest grafts over 0.75 mm to ensure an underlying connective tissue base. Answers: 1-14 169

The source of attached tissue was derived deproteinized, sterilized bovine bone and is cate- from the underlying connective tissue. gorized as a calcium deficient carbonate apatite). Autograft: Tissue transferred from one position 5. The correct answer is C. Normal values for a to another within the same individual (iliac crest nondiabetic person are 4% to 6%. Glycosylated graft, mandibular block graft). Alloplast: A syn- hemoglobin is a test that indicates how much thetic or inert foreign body implanted into tissue sugar has been in a person’s blood during the past (hydroxyapatite). Allograft: A graft between ge- 2 to 4 months. It is used to monitor the effective- netically dissimilar members of the same species ness of diabetes treatment. The amount bound (demineralized freeze-dried bone allograft, reflects how much glucose has been in the blood DFDBA; freeze-dried bone allograft, FDBA). during the past average 120-day lifespan of red cells. Diabetes treatment should achieve glyco- 10. The correct answer is C. Stainless steel has been sylated hemoglobin levels of less than 7.0%. Un- used for a long time in prefabricated posts. How- controlled diabetes is a risk factor for periodontitis. ever, it contains nickel that can cause sensitivity in some people. Titanium posts are biocompat- 6. The correct answer is A. When treating a pa- ible and least corrosive but have low fracture tient with osseous surgery, the surgeon should strength. Also, it is difficult to detect on radio- ramp the bone toward the palate. The follow- graphs because of their similar density as gutta ing are reasons for a palatal approach to osseous percha material. Fiber-reinforced polymers are surgery: Palatal embrasures are wider, allowing esthetic and do not corrode and their removal is for better access to osseous surgery. More can- easier than that of other material if endodontic cellous bone palatally, therefore better healing. failure occurs. More keratinized tissue. Tongueallows for better hygiene palatally. Better esthetics. 11. The correct answer is A. Class I: Up to 1 mm of movement in horizontal direction. Class II: 7. The correct answer is D. Numerous enzymes Greater than 1 mm of movement in horizontal and cytokines are secreted by the macrophage. direction. Class III: Excessive horizontal move- These include collagenases, which debride the ment and vertical movement. wound; interleukins and tumor necrosis fac-

tor (TNF), which stimulate fibroblasts (pro- 12. The correct answer is E. Crohn disease can af- Periodontics duce collagen) and promote angiogenesis; and fect all areas of the digestive tract. transforming growth factor (TGF), which stimu- lates keratinocytes. They also produce hydrolytic 13. The correct answer is A. Although the post enzymes (lysozyme, proteases), small cationic diameter affects the post retention minimally, peptides called defensins, enzymes that create when it is increased, root fracture is more likely reactive forms of oxygen, and enzymes that cre- to occur. Therefore, the post width should not 1/ ate reactive forms of nitrogen. exceed 3 of the root, with a minimum of 1 mm of dentin being retained circumferentially in the 8. The correct answer is C. Average dose is 0.3 apical area. mg given sublingually, and may be repeated at 5-minute intervals three times. Nitroglycerin is 14. The correct answer is C. Early loss of a primary employed sublingually in the management of second molar can result in mesial migration of anginal discomfort. Indications: angina, acute permanent molars. If the first permanent molar myocardial infarction, and congestive heart fail- has already erupted (usually erupts between 6 ure with pulmonary edema. Contraindications: and 7 years), band-and-loop is recommended. A myocardial infarction with hypotension, hy- distal shoe is an appropriate choice if permanent potension, and glaucoma. molar has not fully erupted. Nance appliance is a removable appliance that can be used to prevent 9. The correct answer is B. Xenograft: Tissue trans- mesial movement or tipping of the permanent ferred from one animal species to another (Bio- first molar but has a main disadvantage that it is Ossr bone grafting material is composed of removable. 170 Chapter 7: Periodontics

15. The correct answer is C. Primary resistance – Osteoconductive: Provide a lattice or scaffold form features should prevent restoration and that promotes bone formation tooth from fracturing against occlusal forces. – Demineralized/decalcified: Allografts are of- Other features include removing unsupported ten demineralized to expose components of enamel, box-like preparation, and marginal bone matrix bone morphogenetic proteins amalgam of 90 degrees or higher. Preparation (BMPs) to facilitate bone formation. An ex- of converging vertical walls is one of the features ample is DFDBA. that help amalgam to be mechanically retained Autografts are not osteogenic because they do in the tooth structure (primary retention form). not have live osteocytes to produce bone. Only autografts, which are grafts from the same pa- 16. The correct answer is A. Cervical pull head- tient, are osteogenic. gear is used to correct class II malocclusions with deep bite and produces an extrusive and 20. The correct answer is D. Guided tissue regen- distal force on maxillary molars. Herbst appli- eration is a surgical procedure that specifically ance is a fixed functional appliance that pro- aims to regenerate lost periodontal tissues. Re- duces growth of mandible in a forward direc- generation is reproduction of lost attachment tion. Twin block appliance also treats class II comprising bone, cementum, and periodontal malocclusion through interaction between max- ligament (PDL). Repair is healing of a wound illa and mandible. It is more easily tolerated by that does not fully restore original periodontal patients. Reverse-pull (or protraction headgear, tissue architecture and attachment. The mem- face mask) is used for patients with class III mal- brane prevents fast growing gingival epithelial occlusion by encouraging growth of maxilla by cells from contacting the root to allow regenera- pulling it forward. It has two soft pads that fit tion of periodontal attachment rather than repair onto patient’s forehead and chin, and they are with a long junctional epithelium. connected by an adjustable framework with an- terior wire with hooks that can be used to adjust 21. The correct answer is D. Periodontitis is defined the movement. as progressive destruction of bone and periodon- 17. The correct answer is C. Mineral trioxide aggre- tal ligament. All other signs are included in gin- gate is a medicament that can be used appropri- gival inflammation that may be associated with ately in pulpotomy procedure in replacement of periodontitis. Attachment and bone loss that is formocresol. However, because of its high cost, not progressing could be diagnosed as gingivi- it is not often used. tis on reduced periodontium. Some signs of pe- riodontitis are increased probing depths, radio- 18. The correct answer is A. Kennedy classification graphic bone loss, increasing tooth mobility, and Periodontics system—class I is bilateral edentulous area lo- presence of biochemical markers of tissue break- cated posterior to the remaining teeth. Class II down. is unilateral edentulous area located posterior to the present, natural teeth. Class III is unilateral 22. The correct answer is B. Crown lengthening edentulous area with natural teeth both anterior would cause increased furcation involvement; and posterior to the remaining teeth. Class IV is therefore, it would not be an indication for treat- bilateral-bounded anterior partially edentulous ment. Crown lengthening would be indicated area crossing the midline. to improve esthetics in cases where there is ex- cess gingiva such as drug-induced gingival over- 19. The correct answer is B. Allografts are grafts ob- growth. It could also be of esthetic value in tained from other humans and are stored in tis- cases of delayed passive eruption where natu- sue banks. They show the following characteris- ral recession has not occurred. Crown lengthen- tics: ing is also indicated in cases where restorative – Osteoinductive: Induce bone formation by re- or crown margins violate biological width be- cruiting differentiation of host mesenchymal cause of caries, crown fracture, or post/pin per- cells forations. Answers: 15-33 171

23. The correct answer is B. Answer as per question. the outermost layer of a tooth structure. Answer C is incorrect because saliva is a secretion from 24. The correct answer is D. A class IV furcation salivary glands that moistens and helps in lubri- is a through-and-through furcation that is visible cating the food. Answer D is incorrect because clinically. When a patient has a through-and- dental caries is an infectious microbiologic dis- through furcation, it is more advantageous to ease of the teeth that results in localized dissolu- have it clinically visible because the patient has tion and destruction of the calcified tissues. access to keep the area clean. Good hygiene is possible with a class IV furcation, but not with a 30. The correct answer is C. Refractory periodon- class III furcation. A Proxabrush is a good aid in titis. Answer A is incorrect because aggressive cleaning a furcation. A class III furcation leaves periodontitis manifests before age 35 and leads a plaque, debris, and food trap. There are several to rapid deterioration of tissues. Answer B is in- ways to treat a class III furcation. Successful treat- correct because chronic periodontitis is a pro- ment of a class III furcation can lead to a class gressive loss of bone and soft tissues. Answer D IV furcation. This would be a clinical success. is incorrect because juvenile periodontitis usu- ally occurs in children and young adult, healthy 25. The correct answer is D. The age of the tooth has individuals. no prognostic value in determining if a tooth is a candidate for crown lengthening. If the tooth 31. The correct answer is C. The calcifying epithe- has no mobility, sufficient attached gingiva, and lial odontogenic tumor (CEOT), or Pindborg favorable crown-to-root ratio, and the decay has tumor, radiographically shows a cyst like radiolu- not entered the furcation, the tooth in question cency, and it may contain very small foci of cal- is a viable candidate for crown lengthening. The cified material as white flecks. The CEOT has goal of functional crown lengthening is to cre- also been described radiographically to have a ate a longer clinical crown to fabricate a restora- “driven snow” appearance. The calcifying odon- tion that will rest on sound tooth structure that togenic cyst and the adenomatoid odontogenic is more than 2 mm away from the crestal bone. cyst radiographically would usually show a well- defined unilocular radiolucency associated with 26. The correct answer is B. A patient who presents an unerupted tooth. Answer D is incorrect be- 30% or more of attachment loss is classified as Periodontics cause radiographically, a keratocyst has a uni- or “generalized.” Anything less than 30% is classi- multi-locular appearance and is hazy because of fied as “localized.” The severity is characterized the keratin-filled lumen. on the basis of the amount of clinical attachment loss—slight: 1 to 2 mm, moderate: 3 to 4 mm, 32. The correct answer is A. Paget disease has in- and severe: anything 5 mm or greater. creased alkaline phosphatase levels during os- 27. The correct answer is E. A connective tissue teoblastic phases. Answer B is incorrect because graft is an autogenous subepithelial graft that is hypophosphatasia has low level of serum alka- used to gain root coverage to treat gingival re- line phosphatase activity. Answer C is incorrect cession. This is the most predictable treatment because cherubism has no systemic abnormal- modality for root coverage. It does not aid in ities. Answer D is incorrect because in hyper- pocket reduction. parathyroidism, levels of alkaline phosphatase are normal. 28. The correct answer is B. The most unpre- dictable regenerative procedure in periodontal 33. The correct answer is B. Range of densities on surgery is the regeneration of the interdental a radiograph. Uneven density on a radiograph is papillae. The other four choices are predictable incorrect because it is defined as radiographic objectives of periodontal surgery. mottle. Overall appearance of the radiographic is defined as image clarity. Overall degree of 29. The correct answer is B. Dental plaque. Answer darkening of the radiograph is defined as radio- A is incorrect because enamel is the hardest and graphic density. This page intentionally left blank CHAPTER 8 Radiology

173 174 Chapter 8: Radiology

QUESTIONS

1. As digital radiography has becoming more preva- 5. Most of the incident radiation during x-ray imag- lent for use in the dental settings, the common ing is absorbed by the soft tissue regions of the digital detectors used have changed. What is the face. most common digital detector used currently in A. True dental digital radiography? B. False A. Charged-couple device (CCD) B. Intensifying screen phosphors (ISP) 6. A dental student would like to obtain a film that C. Photostimulable phosphor (PSP) is diagnostic for identifying interproximal caries. After much searching on the internet, he comes 2. A general dentist has been practicing dentistry to the conclusion that interproximal caries are for 36 years and is currently in the midst of re- best diagnosed with films that are low in con- modeling his office. He would like to switch trast as it allows the clinician to see the subtle to digital radiography. The following are advan- differences in tooth density. tages of digital radiology technique in compari- A. True son to traditional techniques except for: B. False A. Radiation dose reduction up to 60% B. Low cost of sensors 7. A first-year dental student helping out in the C. Increased image resolution and contrast as clinic for the first time repeatedly obtains x-rays compared to D speed film of tooth #14 with a zigzagged pattern. What D. Delayed display of images could have prevented the zigzagged pattern on E. Increased ability to detect proximal carries the radiographic image? in noncavitated teeth A. Placing the film incorrectly with the lead por- tion facing the cone 3. What are the components of a dental x-ray B. Telling the patient not to move during the tube? exposure A. Glass envelope C. Bending the film less when placing in posi- B. Cathode tion C. Anode D. Using the film only once instead of exposing D. Tungsten target it multiple times E. All of the above E. Decreasing the kilovoltage

4. What are the common rules to follow to have 8. A periapical film of tooth #9 was obtained in an accurate radiographic image formation when preparation for a root canal and an initial mea- taking x-rays? surement for length of the canal was performed. After initial debridement of the canal was per- A. Using the correct focal spot size formed, another x-ray was captured with the ini- B. Placing the film close enough to prevent dis- tial root length measured and it appears that the tortion file is approximately 4 mm from the apex. How C. Cutting down the amount of divergent could we have obtained a more accurate initial x-rays reaching the object by using the length measurement? longest target-film distance D. 90-degree angle of the central ray of the x-ray A. Increasing the vertical angulation of the cone in relation to the film in relation to the film E. Film and object should be as parallel as pos- B. Decreasing the vertical angulation of the sible cone in relation to the film Radiology F. All of the above Questions: 1-15 175

C. Decreasing the horizontal angulation of the 12. What are the double arrows pointing to in Figure cone in relation to the film 8-1? D. Increasing the horizontal angulation of the A. Floor of nasal fossa cone in relation to the film B. Maxillary zygomatic arch 9. What is the name of the radiographic technique C. Floor of maxillary sinus when the central ray of the cone is directly per- D. Zygomatic bone pendicular to the object? E. Schneiderian membrane A. Paralleling technique 13. What is the single arrow pointing to in Figure B. Bisecting technique 8-1? C. Submentovertex technique A. Floor of nasal fossa 10. A 17-year-old male was referred to an oral sur- B. Maxillary zygomatic arch geon for extraction of an impacted tooth #11 that C. Zygomatic bone is located in area apical to teeth #10–#12, which D. Inferior border of maxillary sinus was found on routine x-rays. How would the oral E. Sigmoid notch surgeon decide as to where the impacted tooth 14. What is the single arrow pointing to in Figure lay in respect to being either buccal or palatal to 8-2? teeth #10 and #12? A. Lamina dura A. Application of the SLOB rule B. Periodontal ligament B. Use of panoramic radiograph C. Cementum C. Towne’s view x-ray D. Enamel D. Water’s view x-ray E. Dentin E. Lateral head radiograph

11. What are the triple arrows pointing to in Figure 8-1? A. Floor of nasal fossa B. Floor of maxillary sinus C. Maxillary zygomatic arch D. Sigmoid notch E. Zygomatic bone

15. What are the double arrows pointing to in Figure 8-2?

A. Periodontal ligament Radiology B. Lamina dura C. Cementum D. Enamel E. Dentin 176 Chapter 8: Radiology

16. What structures are contained within the ra- A. Floor of maxillary sinus diopaque structure surrounded by the three ar- B. Infraorbital margin rows in Figure 8-3? C. Zygomatic arch D. Pterygomaxillary fissure E. Floor of nasal fossa F. Hard palate

18. #2 is referring to what structure in Figure 8-4? A. Floor of maxillary sinus B. Pterygomaxillary fissure C. Zygomatic arch D. Infraorbital margin E. Floor of nasal fossa F. Hard palate

19. #6 is referring to what structure in Figure 8-4? A. Floor of maxillary sinus B. Zygomatic arch C. Hard palate D. Pterygomaxillary fissure E. Floor of nasal fossa F. Nasal septum

20. #12 is referring to what structure? A. Styloid process B. Mandibular condyle C. Occipital bone D. Coronoid notch A. Inferior alveolar neurovascular bundle E. Sphenoid bone B. Sphenopalatine neurovascular bundle F. Cervical vertebrae C. Infraorbital nerve 21. What is the main ingredient of the developing D. Facial nerve solution that serves to convert the exposed silver E. Nasopalatine neurovascular bundle salts on the emulsion film into silver metallic 17. #4 is referring to what structure in Figure 8-4? ions? Radiology Questions: 16-31 177

A. Sodium carbonate 26. Which positioning error would most likely cause B. Potassium bromide a reverse occlusal plane curve on a Panorex? C. Sodium sulfite A. Chin tilted too far upward D. Hydroquinone B. Chin tilted too far downward E. C. Head turned slightly laterally 22. What are the components of a developing solu- 27. Which projection is best for examining zygo- tion? matic arch fractures? A. Clearing agent, antioxidant preservative, A. Waters projection acidifier, hardener B. Submentovertex projection B. Developing agent, acidifier, antioxidant pre- C. Reverse Towne projection servative, hardener D. Lateral cephalometric projection C. Developing agent, accelerator, restrainer D. Clearing agent restrainer, accelerator, acidi- 28. All of the following are advantages of a pano- fier ramic radiograph except: 23. In trying to reduce the amount of low-quality, A. It shows areas that may not be visible on a long-wavelength x-rays from exiting the cone, bitewing or periapical what is commonly added to the cone for added B. It shows both arches on the same film filtration? C. It gives better detail and definition than A. Aluminum disks PAs B. Tin disks D. It is more comfortable for the patient because C. Thicker glass it eliminates the gagging reflex D. Different color plastics E. It requires less time than a full mouth series 24. All of the following cells in the body are relatively resistant to damage by radiation except: 29. A phenomenon caused by a relatively lower x- ray absorption on the mesial or distal of a tooth A. Muscle cells between the edge of the enamel and the adjacent B. Mucosal cells crest of the alveolar ridge is called C. Nerve cells D. Mature bone A. Apical burnout B. Cervical burnout 25. A 55-year-old female with a history of laryngeal C. Coronal burnout cancer who underwent a prolong course of ra- D. Root burnout diation and chemotherapy presents with pain to her left mandible for 3 weeks. Patient has also 30. Which intra-oral radiographs are the most useful noticed a white material appearing through her in detecting interproximal caries? mucosa. On examination, exposed alveolar bone A. Periapical x-rays is noted in teeth #19–#21 area and no erythema B. Bitewing x-rays or purulence noted in the area. Teeth #19–#21 C. Occlusal x-rays are without decay. What is likely the first choice in treatment? 31. The period between radiation exposure and the A. Antibiotics onset of symptoms is called the B. Daily sodium peroxide rinses A. Latent period Radiology C. Hyperbaric oxygen therapy B. Period of cell injury D. Extraction of teeth #19–#21 C. Recovery period E. Resection of left mandible with reconstruc- tion F. Saline rinses 178 Chapter 8: Radiology

32. Removing parts of the x-ray spectrum by using A. Too much vertical angulation absorbing materials in the x-ray beam is called: B. Too little vertical angulation C. Incorrect horizontal angulation A. Elimination D. Beam not aimed at the center of the film B. Filtration C. Collimation 39. Foreshorting where the teeth appear too short is D. Reduction caused by: 33. X-ray fixer contains all of the following A. Too much vertical angulation except: B. Too little vertical angulation C. Incorrect horizontal angulation A. A clearing agent D. Beam not aimed at the center of the film B. An antioxidant preservative C. An accelerator 40. Elongation (the most common error) can be D. An acidifier caused by: E. A hardener A. Too little vertical angulation 34. All are advantages of digital radiography B. The film is not parallel to the long axis except: C. The occlusal plane is not parallel to the floor D. All of the above A. Digital subtraction B. The ability to enhance the image 41. What is the major disadvantage of the paralleling C. Size of the intraoral sensor technique? D. Patient education A. The image formed on the film will not have dimensional accuracy 35. The process of obtaining a digital image similar B. Because of the amount of distortion, peri- to scanning a photograph to a computer screen odontal bone height cannot be accurately is called: diagnosed A. Indirect digital imaging C. An increase in exposure time is necessary B. Direct digital imaging because of the use of a long cone C. Storage phosphor imaging D. An increase in exposure time is necessary D. CMOS/APS because of the use of a short cone

36. Which solid-state detector(s) is/are necessary to 42. Which cells are considered the most radioresis- capture direct digital x-rays? tant? A. CCD (charged-coupled device) A. Immature reproductive cells B. CMOS/APS (complementary metal oxide B. Young bone cells semiconductor/active pixel sensor) C. Mature bone cells C. CID (charge injection device) D. Epithelial cells D. All of the above 43. What is the recommended distance (in feet) that 37. Digital x-rays require less radiation than conven- the operator should stand from the patient while tional x-rays because: taking radiographs? A. The sensor is larger A. 2 feet B. The exposure time is increased B. 4 feet C. The sensor is more sensitive to x-rays C. 6 feet D. The pixels sense transmitted light quickly D. 8 feet

38. Which of the following errors in technique 44. The larger the focal spot the: would be the most likely reason for an image A. The greater the loss of the definition and Radiology to appear elongated? sharpness of the image Questions: 32-56 179

B. The lesser the loss of the definition and sharp- to discriminate against radiation from unwanted ness of the image directions such as scattered radiation is called: C. There is no effect A. Discrimination 45. Copper is used to house the anode because: B. Collimation C. Filtration A. It is a good thermal conductor D. Coning B. It dissipates heat from the tungsten target C. It reduces the risk of melting the target 52. Filtration: D. All of the above A. Reduces patient dose 46. The radiation generated at the anode of the x-ray B. Decreases contrast tube is called: C. Decreases the density of the film D. All of the above A. Primary radiation B. Scattered radiation 53. The herringbone effect will appear on the pro- C. Potential radiation cessed film when: D. Reverse radiation A. The film is bent 47. X-rays have: B. The film is placed backwards in the mouth C. An improper vertical angulation is used A. More energy than light D. An improper horizontal angulation is used B. Less energy than light C. The same energy as light 54. Image magnification may be minimized by:

48. The yearly MPD (maximum permissible dose) A. Using a short cone for a nonoccupationally exposed person is: B. Placing the film as far from the tooth as pos- sible A. 0.1 REM C. Using a long cone B. 0.5 REM D. Shortening the exposure time C. 1.0 REM D. 2.0 REM 55. The rules for producing the most accurate image formation when taking x-rays are the following 49. The yearly MPD (maximum permissible dose) except: for someone who works near radiation is: A. Using the largest focal spot A. 1.0 REM B. Using the longest source–film distance B. 2.0 REM C. Placing the film as close as possible to the C. 3.5 REM object D. 5.0 REM D. Directing the central ray as close to a right angle to the film as anatomical structures will 50. The most effective means in reducing the time of allow: exposure, the amount of radiation reaching the E. Keeping the film as parallel as possible to the patient and the amount of radiation scattered to object the dentist is: A. A lead apron 56. The size of the focal spot influences radiographic B. Ekta-speed film definition and sharpness because they are:

C. Lead diaphragms A. Inversely proportional Radiology D. Increasing target-film distance B. Directly proportional C. Not related 51. The use of metal plates, slots, etc., to confine and direct radiation to a specific region and/or 180 Chapter 8: Radiology

57. The one aspect of radiography the operator can- C. Waters projection not control is: D. Submentovertex projection A. The source–film distance 64. Which of the following projections is the best for B. The object–film distance evaluating midface fractures? C. The size of the focal spot D. The angulation of the cone beam A. Lateral jaw projection B. Reverse Towne projection 58. A short cone (20 cm/8 inches): C. Waters projection D. Submentovertex projection A. Exposes more tissue by producing a more divergent beam 65. Which of the following projections is the best for B. Exposes less tissue by producing a less diver- evaluating the condyles? gent beam C. Produces a sharper image A. Waters projection B. Transcranial projection 59. A long cone (41 cm/16 inches): C. Townes projection D. Submentovertex projection A. Exposes more tissue by producing a more divergent beam 66. Foreshortening and elongation are produced by: B. Exposes less tissue by producing a less diver- A. Incorrect horizontal angulation gent beam B. Incorrect vertical angulation C. Produces a sharper image C. Either of the above D. Both B and C 67. X-ray developer contains all of the following 60. Osteoradionecrosis is more common: except: A. In the mandible A. A developing agent B. In the maxilla B. An antioxidant preservative 61. The image of the coronoid process of the C. A clearing agent mandible often appears in periapical x-rays D. An accelerator of: E. A restrainer A. The incisor region of the mandible 68. The functions of the developer and the fixer are: B. The molar region of the mandible A. The developing solution reduces the silver C. The incisor region of the maxilla halide crystals to black metallic silver while D. The molar region of the maxilla the fixing solution stops the development 62. Which of the following is not a disadvantage of and removes the remaining unexposed crys- the bisecting technique? tals B. The developing solution stops the develop- A. Image on x-ray film may be dimensionally ment and removes the remaining unexposed distorted crystals while the fixing solution reduces the B. Increased exposure time silver halide crystals to black metallic silver C. If using a short cone, the image may be dis- torted 69. After your film has been processed, you notice D. May not be able to determine correct alveo- it appears brown in color. The most likely cause lar bone height is: A. Solutions are too strong 63. Which of the following projections is the best for B. Solutions are too weak evaluating the maxillary sinus? C. Fixing time was not long enough A. Lateral jaw projection D. Fixing time was too long Radiology B. Reverse Towne projection E. Film was underdeveloped Questions: 57-75 181

70. After your film has been processed, you notice C. Roentgen it appears too dark in color. The least likely D. QF cause is: 73. The unit used to measure the equivalent dose A. The film is overdeveloped that relates the absorbed dose of the human tis- B. The temperature of the solutions was too hot sue to the effect of the biological damage caused C. The temperature of the solutions was too by the radiation is called: cold A. REM 71. After your film has been processed, you notice B. RAD it appears too light in color. The least likely C. Roentgen cause is: D. QF

A. The film was left in the solution for too short 74. Increasing Peak kilovoltage (kVp) causes the x- of a time ray to have: B. The temperature of the solutions was too cold A. Decreased density C. The solution was too weak B. More latitude D. The solution was too old C. A shorter scale of contrast E. None of the above D. A longer scale of contrast

72. The unit for measuring the absorption of x-rays 75. It is best to retain dental x-rays for: is called: A. 2 years A. REM B. 4 years B. RAD C. 6 years D. Indefinitely Radiology 182 Chapter 8: Radiology

ANSWERS

1. The correct answer is A. The digital receptor is When the electrons strike the tungsten target, the device that intercepts the x-ray beam after it X-rays are produced. has passed through the patient’s body and pro- duces an image in digital form, a matrix of pixels, 4. The correct answer is F. All of the above and as each with a numerical value. Charged-couple per the answers. device detectors, the most common digital re- ceptor used in dentistry, consist of a chip of pure 5. The correct answer is B. Subject contrast is the silicon with an active area that is divided into a range of characteristics of the subject that influ- two-dimensional array of elements called pixels. ences the radiographic contrast. Dense regions CCD detectors generally have a smaller active of the face such as the bone and teeth and bones surface area which approximate intraoral film absorb most of the incident radiation. Less dense in size. Intensifying screen phosphor detectors regions of the face such as the soft tissue profile produce an image via the creation of an optical transmit most of the radiation. Subject contrast is signal when a light is emitted from the incident also influenced by the energy of the x-ray beam, x-radiation against the phosphor screen. Photo- such as when the kVp is increased, the overall stimulable phosphor receptors (PSP) work dif- density of the image also increases, and when ferently from ISP. X-ray photons excite the elec- it is decreased, the subject contrast increases. A trons in the phosphors and light is produced but a fine balance has to be struck with the kVp and significant amount of electrons are trapped. The exposure time to obtain an image where subtle image is produced when a laser in the read-out changes can be observed and be diagnostic. unit stimulates the trapped electrons to produce an emitted blue light that is detected by a pho- 6. The correct answer is B. Film contrast describes tomultiplier tube and the output of the tube is the ability of a radiographic film to reveal dif- digitized to form the image. They are the most ferences in the subject’s contrast, meaning the widely used detectors in medical radiology. variations in the intensity of the remnant beam that strikes the film. A high contrast film re- 2. The correct answer is B. Advantages of digital veals small differences in the subject’s contrast technique include radiation dose reduction up more clearly than a low-contrast film. Proper ex- to 60% compared with D-speed films, imme- posure of a film will have more contrast than diate display of images, and ability to enhance underexposed, light, films. Proper processing is image (changes in contrast, gray scale, and another factor as mishandling of the film will brightness). High initial cost of sensors is a disad- result in incomplete or excessive development vantage. Digital-image resolution and contrast is will result in diminished contrast of the anatomic decreased compared with D-speed film. Digital structures. radiology allows us to have immediate display of images. Ability to detect proximal carries in 7. The correct answer is A. The zigzagged pat- noncavitated teeth via digital radiology is only tern is the result of incorrectly placing the film comparable and not increased with the newer with the exposed portion facing away from the direct digital-imaging techniques. cone and the lead portion facing the cone caus- ing the herringbone effect. A blurred image is 3. The correct answer is E. Dental x-ray tube con- caused when a patient moves during exposure. sists of a glass that houses a vacuum. Inside this Overbent films will appear as black semilunar vacuum environment is the cathode, anode, and radiopacities or cracks in the film image. Mul- a tungsten target. X-ray is produced when the tiple images will appear on the same film if it cathode is heated up to emit electrons, an electri- is exposed more than once. Poor contrast results cal potential is then created between the cathode when the kilovoltage is set too high and not a Radiology and anode with tungsten target in the middle. herringbone image. Answers: 1-21 183

8. The correct answer is B. Foreshortening is char- bone, foramen magnum, mandibular condyles acterized by the film image appearing squashed and the midfacial skeleton. The Water’s view is or shortened in the vertical, caused by too much an angled PA radiograph of the skull that shows vertical angulation of the cone in relation to the the orbits and maxillary sinuses. film. Elongation is the opposite in that the im- age appears stretched in the vertical caused by 11. The correct answer is C. The maxillary zy- too little angulation of the cone to the object. gomatic process forms a radiopaque U-shaped Changing the horizontal angulation of the cone structure from which the zygoma extends poste- in relation to the film will not change the length riorly. of the image. 12. The correct answer is A. The thin horizontal 9. The correct answer is A. For the paralleling tech- radiopaque line represents the floor of the nasal nique to be accurate in the image capture, the fossae. film must be parallel to the long axis of the tooth 13. The correct answer is D. The inferior border of and the target to film distance must be optimum. the maxillary sinus projects downwards. Bisecting technique is the technique when a film is placed against the tooth with the central ray 14. The correct answer is A. The bone lining the perpendicular to the bisecting line made from tooth socket is the lamina dura. the angle of the film to the tooth. Disadvantages of this technique include distortion when the 15. The correct answer is A. The roots of the tooth angulation is increased and the necessity of a are separated from the lamina dura of the socket short cone that increases the amount of diver- by the periodontal ligament. gent x-rays. Submentovertex technique is used to provide information on the zygoma, zygo- 16. The correct answer is E. The radiopaque struc- matic arches, and mandible. The occlusal film is ture being demarcated by the arrows is the placed on the occlusal plane with the emulsion incisive foramen. Contained within the na- facing the chin and the central x-ray placed per- sopalatine foramen is the nasopalatine nerve, pendicular to the film. artery, and vein.

10. The correct answer is A. The SLOB rule, or the 17. The correct answer is B. As per answer. ‘Same side lingual, opposite side buccal’ rule can be applied when multiple periapical x-rays 18. The correct answer is B. As per answer. are taken in an area. To do this, a reference point 19. The correct answer is C. As per answer. must first be assigned with a single PA (such as the root tip of tooth #12). Then, another periapi- 20. The correct answer is A. As per answer. cal x-ray is taken, except, this time, the central ray is off-set mesially by 60 degrees. If the im- 21. The correct answer is D. Hydroquinone is the pacted tooth #11 appears to move in the same main ingredient in the developing agent. It con- direction of the x-ray tube, then the object (tooth verts the exposed silver salts on the emulsion #11) is lingual to the reference point. If the im- film into silver metallic ions while unexposed pacted tooth #11 appears to move opposit to the silver salts are left unaffected. Sodium Carbon- cone, then the object (tooth #11) is buccal to ate is an alkali salt in the developing solution the reference point. The panoramic radiograph that maintains the alkaline pH of the solution can be used as a survey for initial screening for as well as provides ideal conditions for reactions

pathology of the maxilla and mandible and of- to occur. Potassium bromide controls the action Radiology fers views of the condylar head and ramus of the of the developing agent such that the unexposed mandible. The lateral cephalometric radiograph silver salts on the emulsion film are not removed. is used in cephalometric analysis and evaluation Sodium sulfite serves to prevent the fixer from of craniofacial growth. The Towne’s view is a re- spontaneous oxidative processes. Acetic Acid is verse tilted AP projection that shows the occipital in the fixer solution and serves to neutralize any 184 Chapter 8: Radiology

alkaline developing agent carried over and also may appear flat or inverted. If the chin is tilted serves to buffer the solution. too far downward, the teeth become severely overlapped and the symphyseal region of the 22. The correct answer is C. Components of a de- mandible may be cut off the film. Head slightly veloping solution include the developing agent, turned laterally will cause either depending on accelerator, restrainer, and antioxidant preserva- the direction of the turned head to be unequally tive. The developing agent converts the exposed magnified. silver salts on the emulsion film into silver metal- lic ions. The accelerator maintains the alkali pH 27. The correct answer is B. The submentovertex of the solution to provide the ideal condition for view demonstrates the base of the skull, the sphe- the reaction to occur. The restrainer prevents the noid sinuses, the lateral wall of the maxillary si- unexposed silver salts from being removed from nuses, the curvature of the mandible, and the the emulsion film. The clearing agent, antiox- curvature of the zygomatic arches. The water’s idant preservative, acidifier, and hardener are view is a slight variation of the PA view and it components of the fixer solution. is useful for evaluating the maxillary sinuses, the frontal and ethmoid sinuses, the orbits, the zygo- 23. The correct answer is A. For added filtration, ad- maticofrontal suture, and the nasal cavity. The ditional aluminum disks are placed in the cone reverse Towne’s projection is used to evaluate to filter the lower grade radiation. Inherent fil- for a condylar fracture, especially a medially dis- tration of the beam is accomplished by the glass placed condyle. The lateral cephalometric pro- or plastic cones. jection is used to survey the skull and bones of the face for abnormalities. 24. The correct answer is B. Radiosensitive cells are cells in the body that undergo rapid mitosis or 28. The correct answer is C. Panoramic radiographs meiosis in the body and include cells such as are useful for evaluating problems that require mucosal cells, hair cells, reproductive cells, lym- broad coverage of the mandible and maxilla. It is phocytes, and bone marrow. Radioresistant cells useful as the initial survey film that will provide typically do not undergo rapid mitosis or meio- insight into the need for other more specific films sis and are less susceptible to the damage caused such as intraoral periapical films. by radiation and include cells such as muscle, nerves, and mature bone. 29. The correct answer is B. This phenomenon may cause intact mesial or distal tooth surfaces to 25. The correct answer is F. Patients with previous appear carious on the radiograph when it is not. exposure to radiation therapy to head and neck A true carious lesion can be differentiated by the area can develop osteoradionecrosis of the bone. absence of an image of the tooth edge and by The mandible is the most common site as it is less the appearance of a diffuse inner border where vascular than the maxilla. The period in between the tooth substance has been lost. the initial exposure and the first clinical sign of damage is known as the “latency period.” Treat- 30. The correct answer is B. Posterior bitewings are ment for osteoradionecrosis is primarily support- useful for evaluating for caries in the distal third ive and includes superficial debridement of the of the canine, and the interproximal and oc- wound, saline rinses, and nutritional support. clusal surfaces of the premolars and molars. Pe- Antibiotics are only used if there is definite sec- riapical x-rays are useful for detecting changes ondary infection. Hyperbaric oxygen therapy has in the periapical and interradicular bone. Oc- been advocated but is controversial and results clusal radiographs display a large area of a den- have been inconclusive. Unless the teeth are tal arch and may include either the palate or unrestorable, they should not be extracted. the floor of the mouth. This is useful in examin- ing for precise locations of roots, supernumerary 26. The correct answer is B. If the chin is tilted too or unerupted teeth, foreign objects in floor of Radiology upward, the occlusal plane on the radiograph the mouth, evaluating maxillary sinuses and for Answers: 22-39 185

patients with trismus that can only open a few phor screens are exposed to ionizing radiation, millimeters. which excites the BaFBR/EU+2 crystals in the screen storing the image. A computer-assisted 31. The correct answer is A. Latent period is the laser then promotes the release of energy from time between exposure to radiation and the ap- the crystals in the form of blue light that is sub- pearance of a delayed effect. Period of cell injury sequently scanned to reconstruct the image dig- is when a cell is exposed to radiation and is af- itally. CMOS stands for complementary metal fected by it either through direction interaction oxide semiconductor, which is one of the detec- where a cell’s macromolecule such as protein tors used for direct digital imaging. or DNA are hit by the ionizing radiation which affects the cell as a whole or through indirect in- 36. The correct answer is D. Charged-couple de- teraction when the radiation energy is deposited vice consists of a chip or pure silicon with an in the cell and the radiation interacts with the active area called pixels. After exposure to radia- cellular water rather than with macromolecules tion, charges stored by the pixels are sequentially within the cell. Recovery period is used to refer removed electronically and an output signal to the period after radiation therapy when the is created. Charge injection device is an image body replaces the malignant cells with normal sensor in that the image points are accessed in cells. reference to their horizontal and vertical coor- dinates. Complementary metal oxide semicon- 32. The correct answer is B. Filtration is the mech- ductor/active pixel sensor is an image sensor anism where the low-quality, long-wavelength consisting of an integrated circuit containing an x-rays are absorbed by the exiting beam. Colli- array of pixel sensors, each pixel containing a mation refers to the control of the size and shape photodetector and amplifier. of the emitted beam through the use of shaped cones to reduce the total area exposed to radi- 37. The correct answer is C. Sensors used in dig- ation. Elimination and reduction are terms not ital radiography are more efficient than film in used in radiation physics. detecting radiation. Sensors used in digital radio- graphy tend to be larger and bulkier compared 33. The correct answer is C. X-ray fixer consists of to plain films but size is not the reason as to clearing agent, antioxidant preservative, acidi- why it requires less radiation. Exposure time in fier, and hardener. digital radiography is decreased compared with plain films as the sensors are more sensitive. Pix- 34. The correct answer is C. The size of the in- els sense transmitted electromagnetic energy in traoral sensor tends to be three to four times the range of x-rays and not visible light. the size of an intraoral film. Advantages to using digital radiography include the ability for digital 38. The correct answer is B. When the beam of the subtraction, immediate display of images, abil- x-ray is not perpendicular to the bisector plane, ity to enhance images, and radiation dose of up the length of the projected image of the tooth to 60%. It is easier for patient education as you can change. If the central beam is directed at can enlarge the image on the computer screen too much vertical angulation, the tooth can ap- to demonstrate the findings to the patient. pear foreshortened and when there is too little vertical angulation, it appears elongated. Hori- 35. The correct answer is A. Radiographic film is zontal angulation does not cause elongation or used as the image receiver and the image is foreshortening of the image on film.

subsequently digitized from signals created by Radiology a video device or scanner that views the ra- 39. The correct answer is A. When the beam of the diograph. Radiographic image is acquired by x-ray is not perpendicular to the bisector plane, a CCD or complementary metal oxide semi- the length of the projected image of the tooth conductor that is sensitive to electromagnetic can change. If the central beam is directed at too radiation. In storage phosphor imaging, phos- much vertical angulation, the tooth can appear 186 Chapter 8: Radiology

foreshortened and when there is too little verti- per unit target area becomes decreased as the cal angulation, it appears elongated. Horizontal focal spot increases in size. angulation does not cause elongation or fore- shortening of the image on film. 45. The correct answer is D. The anode consist of a tungsten target embedded in a copper stem 40. The correct answer is D. If the central beam is and the purpose of the target in an x-ray tube directed at too little vertical angulation, the resul- is to convert the kinetic energy of the electrons tant radiographic image will appear elongated. generated from the filament into x-ray photons. Elongation of a radiographic image occurs when The thermal conductivity of the target is rel- the central ray is perpendicular to the object but atively low and as a result is typically embed- not the film. When the occlusal plane is not par- ded in a large copper block. Copper is a good allel to the floor, the tooth and film is likely not thermal conductor that dissipates heat from the perpendicular to the central x-ray causing elon- tungsten target to reduce the risk of melting the gation. target.

41. The correct answer is C. The paralleling tech- 46. The correct answer is A. Primary radiation is nique uses a long cone to increase the focal emitted directly from the anode in the x-ray tube spot-to-object distance and as a result of the in- toward the object. Scattered radiation arises from creased distance, an increase in exposure time the interactions of the primary radiation beam is necessary. The paralleling technique is the with the atoms in the object being image and preferred method for intraoral radiographs as it deviates from the straight line between the x-ray allows for minimal image distortion and more focus and the image receptor. accurate imaging, such as assessing periodontal bone height. There is slight image magnifica- 47. The correct answer is A. X-radiation is a form tion because to achieve the parallel orientation, of electromagnetic radiation with wavelengths the practitioner must often place the film to- in the range of 10–0.01 nm with frequencies in ward the middle of the oral cavity away from the the range of 30 PHz to 30 EHz and energies in teeth. the range from 12 eV to 120 KeV. Visible light consists of photons with energies from 1.6 eV 42. The correct answer is C. The most radiosen- (red light) up to 3.4 eV (violet light). sitive cells are cells with high mitotic rates, undergo many future mitoses, and are most 48. The correct answer is B. The maximum permis- primitive in differentiation such as immature sible dose of radiation for a nonoccupationally reproductive cells, young cells, and epithelial exposed worker in a calendar year is 0.5 REM. cells. 49. The correct answer is D. The maximum per- 43. The correct answer is C. The radiation received missible dose of radiation for radiation workers by the person taking the x-ray comes from the in a calendar year is 5 REM. scatter. The strength of the radiation hitting any unit area falls off geometrically depending on 50. The correct answer is B. Film speed can be an the distance from the source of scatter. A per- important aspect in determining the amount of son standing 6 feet away from the target receives radiation exposure received by a patient. The one-ninth as much scatter radiation as person greater the film speed, the lesser the radiation standing 2 feet away from the target. exposure received by the patient.

44. The correct answer is A. The focal spot is the 51. The correct answer is B. A collimator is a de- area on the target where the focusing cup directs vice for use to narrow waves or particles toward a the electrons from the filament. The sharpness specific region. Filtration in radiology is used to and definition of the image decreases as the size absorb the lower-energy photons emitted by the Radiology of the focal spot increases. The heat generated tube before they reach the targets. Use of filter Answers: 40-63 187

produces a cleaner image by absorbing the lower niques. The target-film distance is usually 8 energy x-ray photons that tend to scatter more. inches and the beam is more divergent causing more tissue to be exposed. The resulting image 52. The correct answer is D. Filtration in radiology x-ray is also somewhat larger than the long cone. is used to absorb the lower-energy photons emit- Image is also less sharp compared to a long cone. ted by the tube before they reach the targets. Use of filter produces a cleaner image by absorbing 59. The correct answer is D. The x-ray source to the lower energy x-ray photons that tend to scat- object distance should be as long as possible. ter more. This will enable the x-ray photons to emerge in a straighter line in a less divergent beam, therefore 53. The correct answer is B. When the film is placed producing more accurate shadow. The straighter backwards in the mouth, the image of the foil the x-ray photon line, the less divergent the beam which is in a herringbone fashion would ap- with a resulting image that is more accurate. pear on the processed film. Overlapped contacts of interproximal surfaces of teeth will superim- 60. The correct answer is A. Osteoradionecrosis is pose over each other if the direction of the x-rays a complication of surgery or trauma in a pre- is misdirected in the horizontal plane. A short- viously irradiated bone. Radiation-induced vas- ened image or elongated image will appear if too cular insufficiency rather than infection causes much vertical angulation is used or not enough bone death and occurs most commonly in the vertical angulation is used, respectively. mandible after head and neck irradiation. Con- dition is painful, debilitating, and may result in 54. The correct answer is C. A long cone results significant bone loss. The recommended treat- in greater definition and less image magnifica- ment guidelines include irrigation, antibiotics, tion. A short cone is used to take x-rays with hyperbaric oxygen therapy, and surgical tech- the bisecting-angle technique. The resultant im- niques, including hemimandibulectomy and age is somewhat larger than when using the graft placements. long cone. Placing the film away from the tooth results in distortion and image magnification. 61. The correct answer is D. In the periapical x-rays Shortening the exposure time will not change of the maxilla, the image of the coronoid process size of the image but clarity of film. of the mandible often appears.

55. The correct answer is A. The increase in the 62. The correct answer is B. Exposure time is simi- source–film distance increases the sharpness of lar between the bisecting technique and parallel the image. To get the most accurate image, the technique determined by film speed. But images smallest focal spot should be used. The distance resulting from the bisecting technique maybe between the film and object should be as close dimensionally distorted and may not be able to as possible and the film should be parallel. To determine the correct alveolar bone height de- decrease the distortion, the x-ray should be as pending on the angulation of the x-ray. close to a right angle to the film as possible. 63. The correct answer is C. Waters projection is an 56. The correct answer is A. As the size of the focal x-ray technique used to show the maxilla, max- spot decreases, the radiographic definition, and illary sinuses, and zygomatic bones. Lateral Jaw sharpness increases. projection is useful to examine the posterior re- gion of the mandible and this radiographic pro-

57. The correct answer is C. Size of the focal spot is jection is also called the lateral oblique view. Radiology set on the anode of the x-ray by the manufacturer Reverse Townes projection is a tilted mandibular and cannot be changed by the operator. posterior anterior projection to shows the frontal view of those portions of the mandible that 58. The correct answer is A. A short cone is used to can only be visualized laterally in the panora- take x-rays with bisecting angle exposure tech- mic film. Submentovertex projection helps to 188 Chapter 8: Radiology

identify the position of the condyle, visualize opment and removes the remaining unexposed base of the skull, and evaluate fractures of the zy- crystals. gomatic arch. This projection also demonstrates the sphenoid and the ethmoid sinuses and lateral 69. The correct answer is C. When solutions are too wall of maxillary sinus. weak or if it is excessively fixed or when the film is underdeveloped, a light radiograph results. If 64. The correct answer is C. it appears brown in color, it can be because of multiple causes such as fixing time that is not 65. The correct answer is C. Townes projection is long enough, depleted fixer, insufficient wash- a reverse tilted anterior–posterior technique of ings, or contaminated solutions. When solu- the skull used for the visualization of the foramen tions are too strong, the radiograph becomes very magnum, petrous ridges, internal auditory canal, dark. mandibular condyle, and articulating fossa. Wa- ters projection is an x-ray technique used to show 70. The correct answer is C. When a dark radio- the maxilla, maxillary sinuses, and zygomatic graph results it is because of processing errors bones. Transcranial projection is a technique such as overdevelopment with too high a tem- used to evaluate the condylar head anatomy and perature or too long a time in the developer, in- position in relation to the fossa from a lateral as- adequate fixation, accidental exposure to light, pect. Submentovertex projection helps to iden- or improper safelighting. A dark radiograph can tify the position of the condyle, visualize base also result due to overexposure from excessive of the skull, and evaluate fractures of the zygo- milliamperage, excessive peak kilovoltage exces- matic arch. This projection also demonstrates sive time, or too short a film to source distance. the sphenoid and the ethmoid sinuses and lat- When a solution is to cold, underdevelopment eral wall of maxillary sinus. of the radiograph results and a light radiograph is the result. 66. The correct answer is B. A shortened image or elongated image will appear if too much vertical angulation is used or not enough vertical angu- 71. The correct answer is E. A light radiograph lation is used, respectively. Overlapped contacts is the result of processing errors and underex- of interproximal surfaces of teeth will superim- posure. Processing errors such as underdevel- pose over each other if the direction of the x-rays opment with too low a processing temperature is misdirected in the horizontal plane. or too short a time, depleted developer diluted, or contaminated developer or excessive fixation 67. The correct answer is C. Components of a de- will cause a light radiograph. Underexposure veloping solution include the developing agent, from insufficient milliamperage, insufficient accelerator, restrainer, and antioxidant preserva- peak kilovoltage, insufficient time, too great a tive. The developing agent converts the exposed fill to source distance or reversed film packet in silver salts on the emulsion film into silver metal- mouth are also causes of a light radiograph. lic ions. The accelerator maintains the alkali pH of the solution to provide the ideal condition for 72. The correct answer is B. Absorbed dose is a the reaction to occur. The restrainer prevents measure of the amount of energy absorbed by the unexposed silver salts from being removed any ionizing radiation per unit mass of any type from the emulsion. Clearing agent, antioxidant of matter. RAD, radiation absorbed dose, is the preservative, acidifier, and hardener are compo- traditional unit. The SI unit is gray, Gy, and nents of the fixer solution. 1 Gy is equal to 1 J/kg. Roentgen equivalent man, REM, is the traditional unit of equivalent dose. 68. The correct answer is A. As per answer. The The equivalent dose is used to compare the bi- role of the developing solution is to reduce ologic effects of different types of radiation on a the silver halide crystals to black metallic sil- tissue. Roentgen, R, is the traditional unit used Radiology ver while the fixing solution stops the devel- to measure radiation quantity, the capacity of Answers: 64-75 189

radiation to ionize air. Quality factor, QF, is a 74. The correct answer is D. As the kilovoltage is variable that takes into account the different de- increased, the energy of the x-ray beam is in- grees of biological damage produced by equal creased and the scale of contrast is decreased. A doses of different types of radiation. low contrast image allows for the visualization of smaller differences in density within an ob- 73. The correct answer is A. Roentgen equivalent ject and a longer scale of contrast results. Higher man, REM, is the traditional unit of equivalent kilovoltage images are useful for periodontal di- dose. The equivalent dose is used to compare agnosis where minute changes in bone can be the biologic effects of different types of radiation detected. Lower kVp images are useful when an- on a tissue. Absorbed dose is a measure of the alyzing large differences in density within an ob- amount of energy absorbed by any ionizing radi- ject such as caries or soft tissue calcifications. ation per unit mass of any type of matter. RAD, radiation absorbed dose, is the traditional unit. 75. The correct answer is D. Ideally, records and The SI unit is gray, Gy, and 1 Gy is equal to x-rays should be maintained and kept accessi- 1 J/kg. Roentgen, R, is the traditional unit used ble indefinitely as it is the patient’s treatment to measure radiation quantity, the capacity of ra- history. But, regulations for record retention do diation to ionize air. Quality factor, QF, is a vari- exist and are mostly regulated by individual state able that takes into account the different degrees regulations. HIPAA mandates a 6-year require- of biological damage produced by equal doses of ment to keep records, but offices should abide different types of radiation. by the state requirements. Radiology This page intentionally left blank CHAPTER 9 Pathology

191 192 Chapter 9: Pathology

QUESTIONS

1. An otherwise healthy 19-year-old female was ad- What is the most common clinically significant Pathology mitted to the pediatric ward with longstanding, odontogenic tumor? multiple painful oral ulcers on her buccal mu- A. Adenomatoid odontogenic tumor (AOT) cosa and ventral tongue. She was underweight B. Cementoblastoma and reported severe oral pain. An examination C. Ameloblastoma revealed lesions on her skin similar to those in D. Calcifying epithelial odontogenic tumor her mouth. She was placed on acyclovir. The le- (CEOT) sions persisted and showed no signs of improve- E. Hemangioma ment. Ophthalmologic and pelvic examination F. Odontogenic myxoma revealed no lesions. Which of the following is the most likely diagnosis? 4. A routine panoramic examination revealed the A. Pemphigus vulgaris following finding in 17-year-old male. All of the B. Benign mucous membrane pemphigoid teeth in the quadrant were vital and the patient (BMMP) was asymptomatic. Intraorally, there was obliter- C. Erythema multiforme (EM) ation of the vestibule on the affected side. Which D. Recurrent aphthous stomatitis of the following is the most likely diagnosis? E. Herpes simplex

2. A 45-year-old male smoker presents with a slightly elevated, asymptomatic 2 mm × 1cm white plaque on the posterior lateral tongue ex- tending onto the ventral surface. The lesion was picked up on routine head and neck examina- tion and the patient was unaware of its existence. Adjacent to this area, a molar with a fractured distolingual cusp is present. What is the most appropriate action to be taken? A. Inform the patient that this is a high-risk area for oral cancer and biopsy immediately. A. Dentigerous cyst B. Make a note of the lesion in the chart describ- B. Osteosarcoma ing its size and appearance and re-evaluate C. Periapical cyst at a 3-month recall visit. D. Buccal bifurcation cyst C. Inform the patient that this is due to trauma E. Ameloblastoma from biting his tongue and is nothing to be concerned about. 5. A 45-year-old male of Southeast Asian descent D. Remove any suspected etiology and biopsy presents to the dentist with a chief complaint of in 2 weeks if still present and not improving. staining of his teeth and difficulty opening his mouth to chew. Upon clinical examination, it 3. There are many types of odontogenic tumors, is noted that the patient has a maximum open- both benign and malignant. Treatment for these ing of 27 mm. Diffuse white lesions are seen neoplasms varies from simple procedures such bilaterally on his buccal mucosa, lateral borders as curettage to more extensive surgery, including of tongue, and labial mucosa. Further inspec- resections. Knowing the prevalence, prognosis, tion of his buccal mucosa reveals what is seen and associated features of these lesions will help in this picture. Subsequent manipulation of this with both patient education and in determining mucosa did not change the appearance of the a more predictable treatment plan and outcome. Questions: 1-8 193 Pathology

lesion. He reports no pain associated with any of D. Neisseria gonorrhoeae infection the intraoral findings. His social history is signif- E. Treponema pallidum infection icant for 20 pack-years smoking and occasional alcohol drinking. What is the most likely diag- 8. A 43-year-old male presents for routine dental nosis of the lesions in this patient’s mouth? care. He is asymptomatic and intraoral and ex- traoral examinations are all within normal limits. All teeth in the lower right quadrant were vital. A panoramic radiograph was taken. What is the most likely diagnosis?

A. Smoker’s melanosis B. Oral submucous fibrosis C. Morsicatio buccarum D. Oral hairy leukoplakia (OHL) E. Leukoedema A. Dentigerous cyst 6. A 38-year-old otherwise healthy male presents B. Metastatic lesion with a crusted-over ulcer on his left upper lip. C. Stafne bone cyst He states that he noticed a tingling 9 days ago D. Calcifying odontogenic cyst and little “blisters” 8 days ago. He reports having E. Periapical cyst had lesions like this before and is positive that he has never had any intraoral lesions in the past. What is the best plan of treatment for his current lesion? A. Re-evaluate in 2 weeks B. Topical antivirals C. Systemic antivirals D. Biopsy the lesion E. Refer to ophthalmologist

7. A 33-year-old otherwise healthy male presents to the clinic with a 5-mm diameter painless ulcer on his left upper lip that has been present for 3 weeks. He reports sexual encounters with multi- ple partners over the past 3 months and has no other symptoms. What is the most likely diagno- sis of this lesion? A. Histoplasma capsulatum infection B. Herpes simplex infection C. Cryptococcus neoformans infection 194 Chapter 9: Pathology

9. A 40-year-old female presents to the dental clinic A. Burkitt’s lymphoma, African type with the following appearance of her lateral B. Burkitt’s lymphoma, North American type tongue. She is asymptomatic and a former intra- C. Multiple myeloma

Pathology venous (IV) drug user. She also reports a 25-pack- D. Ewing’s sarcoma year history of smoking. A cytologic smear was E. Hodgkin’s lymphoma positive for candida organisms and the lesion was treated with antifungals. A follow-up examina- 11. A 14-year-old male presents with diffuse, tion 2 weeks later showed some resolution of the boggy, nontender enlargement of maxillary and lesion, but white patches were still present and mandibular gingiva. His parents report no sig- were subsequently biopsied. The biopsy demon- nificant prior medical history and that he has strated chromatin beading of the superficial ep- never been on any kind of long-term medica- ithelial cells that was consistent with Oral hairy tion. Scaling was performed and no resolution leukoplakia (OHL). She reports no lesions else- was noted at a 2-week re-evaluation. A biopsy where and that she is in good health. What is the was performed and the tissue showed infiltration next step in the management of this patient? of normal tissue by sheets of poorly differenti- ated cells with myelomonocytic characteristics. These findings are most consistent with a diag- nosis of which of the following disorders? A. Leukemia B. Thrombocytopenia C. Neutropenia D. Multiple myeloma E. Anemia

12. A 63-year-old nonsmoking African American fe- male presents with a 3-cm preauricular single nodular mass. She states that the lesion has been slowly growing over the past 13 years. She is asymptomatic and wants it removed for cosmetic reasons. Examination reveals that the nodule is freely mobile and not fixed to underlying tissue. A. Watch the lesion at subsequent follow-up The most likely diagnosis for this lesion is which visits and biopsy if there is a change. of the following? B. Referral to her primary care physician for A. Warthin’s tumor HIV testing. B. Hodgkin’s lymphoma C. Do nothing and continue routine dental. C. Adenoid cystic carcinoma D. Excise the lesion with margins. D. Pleomorphic adenoma E. Treat the lesion with topical steroids. E. None of the above

10. A 65-year-old man presents with painless cervi- 13. A 24-year-old male presents at a clinic for routine cal lymphadenopathy. He notes gradual enlarge- dental care. A panoramic radiograph is taken ment over the past year. A routine radiographic and a 2 × 2 cm multilocular radiolucency is examination revealed no abnormalities in any noticed in the right posterior mandible. The pa- bones. He is asymptomatic and the rest of his tient is asymptomatic, and there is no clinical physical examination is unremarkable. Biopsy expansion noted. An incisional biopsy is per- of the neck mass demonstrated binucleated formed and the specimen shows focal areas of gi- (“owl-eye”) and multinucleated cells with promi- ant cells clustered around areas of hemorrhage. nent nucleoli. These findings are consistent with The background stroma consists of plump, a diagnosis of which of the following? Questions: 9-19 195 Pathology

ovoid-shaped cells. What is the next step in the 17. A mass in the right mandibular buccal vestibule management of this patient? of a 60-year-old female has been slowly enlarging A. Assess circulating platelet levels over the past few months. Upon biopsy, sheets of B. Assess serum calcium levels amorphous, glassy pink material are noted sub- C. Enucleation of the lesion jacent to the surface epithelium. Which of the D. Intralesional steroid injections following disorders is consistent with this find- E. Partial mandibular resection ing? A. Hodgkin’s lymphoma 14. A 12-year-old healthy male presents for routine B. Multiple myeloma dental care during December vacation from C. African-type Burkitt’s lymphoma school. Upon examination, you notice that he D. Ewing’s sarcoma has coryza, cough, and red, watery, photopho- bic eyes. He is slightly febrile and has signifi- 18. A 27-year-old male presents for evaluation of gin- cant lymphadenopathy. His mother reports that gival discoloration. The discoloration appears as he has been symptomatic for the past 2 days. a bluish line along the marginal gingiva. Gray Intraorally, he has multiple areas of erythema areas are also noted on the buccal mucosa and bilaterally on his buccal mucosa. Within these tongue. Upon protrusion of his tongue, a visible erythematous areas, numerous small, painless, tremor is noted. Radiographs reveal advanced blue-white macules are noted. No other lesions periodontal disease. Salivary flow tests indicate were noted elsewhere on his body. What is the excessive salivation. These findings are most most likely diagnosis? consistent with which of the following? A. Rubeola A. Plumbism B. Mumps B. Acrodynia C. Secondary herpes simplex C. Argyria D. Hand-foot-and-mouth disease D. Arsenic poisoning E. None of the above 19. A 15-year-old male with a history of epilepsy and 15. A 45-year-old female presents with multiple os- grand mal seizures presents for routine dental teomas, odontomas, and supernumerary teeth. care. His medical history is otherwise unremark- She reports a history of intestinal polyps. What able. Upon examination, he is noted to have gen- syndrome does this patient most likely have? eralized moderate gingival hyperplasia. Which A. Gorlin syndrome of the following drugs is the most likely cause of B. Ascher syndrome his gingival hyperplasia? C. Gardner syndrome A. Sodium valproate D. Klinefelter’s syndrome B. Cyclosporine C. Verapamil 16. A 49-year-old HIV+ male with a CD4+ count D. Nifedipine of 120 presents with a long-standing indurated, ulcerated lesion on the right lateral border of his tongue. A biopsy was performed and a fun- gal organism was detected. A special stain (mu- cicarmine) was positive and provided positive identification of the fungal organism. Which of these fungal organisms has a mucopolysaccha- ride capsule that is mucicarmine-positive? A. Blastomyces dermatitidis B. Paracoccidioides brasiliensis C. Histoplasma capsulatum D. Cryptococcus neoformans 196 Chapter 9: Questions: 9-19

20. An otherwise healthy 10-year-old male presents 23. A 10-year-old male presents for evaluation of a for routine dental care. Upon examination, mul- missing mandibular right permanent first mo- tiple teeth (both deciduous and permanent) are lar. A panoramic radiograph is taken and the

Pathology noted to be mobile. Bitewings and periapical tooth is seen displaced apically toward the in- films are taken. Moderate to severe bone loss ferior border of the mandible. Coronally, there is noted in all four quadrants. No other radio- is a well-circumscribed, unilocular radiolucent graphic changes are noted and the patient is defect with a significant amount of calcified ma- normocephalic. Examination of the ventral sur- terial with the radiodensity of tooth structure. face of the hands reveals palmar keratosis. While The crown of the tooth is present within the de- questioning, the patient reveals similar findings fect. Which of the following is the most likely on the soles of his feet. No other skin lesions are diagnosis? noted. On the basis of this information, what is A. Odontogenic keratocyst (OKC) the most likely diagnosis of this patient? B. Cementoblastoma A. Nevoid basal cell carcinoma syndrome C. Radicular cyst B. Apert syndrome D. Ameloblastic fibro-odontoma C. Ascher syndrome D. Papillon–Lef`evre syndrome 24. An incisional biopsy of a palatal swelling in a 30-year-old male patient reveals necrosis of sali- 21. A 50-year-old female with no significant medi- vary gland acini with preservation of overall lob- cal history presents for evaluation of “sores on ular architecture, squamous metaplasia of the her gums.” Upon inspection, several areas of salivary ducts, and overlying pseudoepithe- Desquamative gingivitis are noted. There are liomatous hyperplasia. What is the appropriate also some blistering areas on the buccal mu- management of this patient? cosa. Applying firm lateral pressure on an unaf- fected area induces a bulla and pain. A biopsy is A. Refer this patient to a head and neck surgeon. taken and sent for direct immunofluorescence. B. Excise the remaining portion of this lesion The pattern of deposition of antibodies shows down to bone. a “chicken wire” appearance. The diagnosis C. Assure the patient he has nothing to worry and appropriate management of this patient are about and monitor the lesion every 2 weeks. which of the following? Biopsy again in 6 weeks if no improvement. D. Refer the patient to begin chemotherapy A. BMMP and referral to ophthalmologist. and/or radiation therapy. B. BMMP and treatment with topical steroids. E. Refer the patient to ophthalmology to rule C. Pemphigus vulgaris and referral to a vesicu- out ocular lesions. lobullous specialist. D. Pemphigus vulgaris and treatment with top- 25. A 30-year-old female presents with a nonexpan- ical steroids. sile multilocular radiolucency in the left poste- 22. A 60-year-old female presents with a firm, mov- rior body of the mandible. The lesion is apical able, 1-cm swelling in his right upper lip. He re- to the teeth in the area (they are not involved) ports that it has been slow growing “for a while.” and extends from the distal of second molar to Other than occasionally biting on it and trauma- the distal of the ipsilateral canine. Inferiorly, the tizing it, the lesion is asymptomatic. The over- lesion extended to just superiorly to the inferior lying mucosa is normal in color. Which of the alveolar nerve canal. Prior to biopsy, aspiration following is the most likely diagnosis? of the lesion is performed. A white, cheese-like material is obtained upon first aspiration. The A. Monomorphic adenoma second aspiration returns more of the same ma- B. Mucocele terial, this time with a slight reddish hue. Which C. Peripheral ameloblastoma of the following is the most likely diagnosis? D. Ranula E. Stafne cyst Questions: 20-30 197 Pathology

A. Dentigerous cyst A. Perform an incisional biopsy that samples B. Static bone cyst both the ulcerated component and the ad- C. Traumatic bone cyst jacent normal mucosa. D. Odontogenic keratocyst (OKC) B. Perform an excisional biopsy. C. Perform an incisional biopsy on tissue that is 26. A 50-year-old man is referred by his primary care taken from the central, ulcerated portion of physician for “multiple blood-filled blisters” in the lesion. his mouth. Bilaterally on his buccal mucosa, you D. Sample tissue from normal mucosa adjacent note large dome-shaped, red-blue raised lesions to the lesion. that are fluctuant. Multiple smaller petechiae are noted on the dorsal surface of his tongue and 29. A 24-year-old Caucasian female dental student gingiva. Medium-sized lesions are noted on his recently visited Africa to participate in a den- lateral tongue. The patient reports that they wax tal outreach program. After spending 4 weeks and wane and he sometimes tastes blood. He performing routine dental procedures, she re- reports that he is a heavy drinker and frequently turned home. While volunteering to allow one takes acetaminophen for headaches. Which of of her professors to demonstrate a procedure to the following is the most likely cause of these the class on her, it was noticed that she had a lesions? diffuse, symmetrical, brown-black pigmentation covering most of her palate. At an examination A. Kidney disease just prior to leaving, she had no pigmentation at B. Liver disease all. She is asymptomatic. What is the most likely C. Crohn’s disease cause of this dental student’s palatal pigmenta- D. Hodgkin’s disease tion? 27. A 3-year-old female presents with Stickler syn- A. Amalgam tattoo drome. Upon examination, she is noted to have B. Racial pigmentation the following abnormalities: posterior displace- C. Melanoma ment of her tongue, cleft palate, and mandibu- D. Drug-related pigmentation lar micrognathia. This triad of anomalies is well recognized and is seen isolated and in associa- 30. A 50-year-old man presents for evaluation of a tion with other syndromes. What is this triad of darkly pigmented area of the facial gingiva apical anomalies more commonly known as? to his maxillary left lateral incisor. He reports A. Klinefelter sequence having been to the dentist regularly before and B. Apert sequence that the spot was a lot smaller when he started. It C. Pierre Robin sequence has gradually increased in size and gotten darker. D. Gorlin sequence The only restoration in the patient’s mouth is E. Gardner sequence an all-ceramic crown on his mandibular right second molar. He is asymptomatic and otherwise 28. A 60-year-old male presents for evaluation of a healthy. What is the proper management for this nonhealing ulcer (>2 months) on the lateral patient? border of his tongue. He reports a 60-pack-year A. Remove the lesion purely for cosmetic rea- history of smoking and is a heavy drinker. The le- sons and perform a periodontal procedure sion is 1 cm by 1 cm. It is painless and indurated. for esthetics. Adjacent to the tooth is a fractured tooth. In or- B. Biopsy the lesion and submit for histopatho- der to rule out a traumatic etiology, the tooth is logical examination. smoothed out and the patient is re-evaluated in C. Have the patient return in 3 months for re- 2 weeks. The lesion persists and the decision is evaluation. made to perform a biopsy. Which of the follow- D. Inform the patient he has nothing to worry ing is the most appropriate method of perform- about, but if it gets bigger he should return. ing the procedure? 198 Chapter 9: Pathology

31. A 56-year-old woman presents with a chief com- C. Multiple giant cell lesions plaint of pain in her mandibular right second D. Multiple supernumerary teeth molar. Her medical history is significant for

Pathology breast cancer (6 years ago) and she subsequently 34. A 20-year-old male presents for initial examina- had a total left-sided mastectomy. The tooth that tion. Although his overall hygiene is good, you was bothering her was deemed nonrestorable notice generalized loss of tooth structure. This and requires extraction. Which of the following loss includes incisal edges and occlusal surfaces. should the dentist be most concerned about re- The lingual of the anterior maxillary teeth and garding this patient’s extraction? the labial of the anterior mandibular teeth are also affected. Large, flat wear facets are seen. He A. Osteoradionecrosis reports that he has been grinding his teeth for B. Xerostomia secondary to chemotherapeutics years. What type of tooth wear is this patient suf- C. Hypoglycemia fering from? D. Potential IV bisphosphonate therapy A. Attrition 32. A 25-year-old male is given a broad-spectrum B. Abrasion antibiotic to combat a streptococcal pharyngitis. C. Abfraction After a few days of taking the antibiotic, he de- D. Erosion velops several erythematous patches on his pos- terior hard palate that burn. The patient informs 35. A 30-year-old female lawyer presents with hyper- you that aside from the strep throat, he is un- elasticity of the skin and cutaneous fragility. In aware of any other medical conditions he may areas of prior trauma (i.e., knees), she has scars have. How should this patient be managed? that resemble crumpled cigarette paper. She also has hypermobility of the joints and bruises easily. A. Discontinue the antibiotic immediately. Pre- Other than the scarring, she has no other lesions scribe a liquid steroid such as dexamethasone on her skin. She reports no other medical prob- to swish and spit. lems. These findings are most consistent with B. Treat the patient with an antifungal and con- which of the following? tinue until antibiotic therapy is complete. C. Refer to oral surgeon to biopsy as soon as A. Osler–Weber–Rendu syndrome possible. B. Tuberous sclerosis D. Explain to the patient that it is probably un- C. Ehlers–Danlos syndrome related and to come back in 3 months for D. Sturge–Weber angiomatosis follow-up. 36. A 60-year-old woman presents with stiff, shiny 33. An 8-year-old female presents for initial examina- fingers. The fingers are permanently flexed and tion. She comes in with her parents who say she resemble a claw. Her fingers and toes also expe- has a number of abnormalities. They mention rience blanching when exposed to cold and can she has missing clavicles, is short in stature, and take up to 5 minutes to regain their color after has a large head with prominent frontal bossing. warming. Intraorally, she has multiple petechiae Because of her missing clavicles, she is able to on the lateral borders of her tongue and buc- approximate her shoulders in front of her chest. cal mucosa bilaterally. Further examination re- On skull films, sutures demonstrate delayed clos- veals painless, movable, subcutaneous nodules ing. Intraorally, she has a narrow, high-arched of her skin. These findings are most consistent palate. A panoramic film is taken. What is most with which of the following? likely to be seen on a panoramic radiograph of A. Ehlers–Danlos syndrome this patient? B. Tuberous sclerosis A. Multiple osteomas C. CREST syndrome B. Multiple OKCs D. Osler–Weber–Rendu syndrome Questions: 31-42 199 Pathology

37. A 60-year-old female presents with herpes zoster C. Dentigerous cyst of the face and external auditory canal. The le- D. Ameloblastoma sions on her face are crusted over and follow the course of the V2 branch of the trigeminal nerve 40. A 45-year-old male presents with a red, nodu- stopping at the midline. There is also involve- lar lesion on the anterior hard palate. He states ment of the facial and auditory nerves of the same that it is tender to touch and bleeds easily. He side. She reports facial paralysis, hearing deficits, does not know how long it has been present. and vertigo. These findings are consistent with He was diagnosed 15 years ago with the human which of the following syndromes? immunodeficiency virus. His CD4+ count has subsequently fallen below 200. What is the most A. Exanthema subitum likely diagnosis? B. Chronic fatigue syndrome C. Zoster sine herpete A. Pleomorphic adenoma D. Ramsay Hunt syndrome B. Necrotizing sialometaplasia (NS) C. Kaposi’s sarcoma (KS) 38. A 60-year-old female presents with bilateral D. Mucoepidermoid carcinoma white lesions on her buccal mucosa. She says she has not noticed them before and they are asymp- 41. A 15-year-old boy presents with symptomatic tomatic. She states that she has never smoked submandibular lymphadenopathy for the past and drinks a glass of wine once a week. Clini- 2 weeks. Intraoral and extraoral examinations cally, the lesions appear striated and extend into were unremarkable except for the previously the mandibular vestibule bilaterally. After rul- mentioned lymphadenopathy. A fine needle as- ing out any other etiology, an incisional biopsy piration was performed and Bartonella henselae is performed. The biopsy shows keratinized mu- organisms were isolated. These findings are con- cosal epithelium overlying fibrous connective sistent with which of the following? tissue. At the epithelial-connective tissue inter- A. Cat-scratch disease face, a dense band of lymphocytes is noted. De- B. Mumps generation of the basal cell layer and saw-tooth C. Rubeola rete ridge formation is also seen. What is the D. Rubella diagnosis? A. Leukoplakia 42. A 50-year-old male presents with recurring oral B. Lichen planus (LP) ulcers for the past 3 months. He currently has C. Squamous cell carcinoma a 2-cm ulcer on his left buccal mucosa and a D. Leukoedema similar sized one on his right lateral tongue. His anterior maxillary gingiva is eroded from canine 39. A 40-year-old male presents with an asymp- to canine. Skin lesions are evident and appear as tomatic, unilocular, mixed radiolucent–radio- tiny vesicles that rupture. An incisional biopsy paque lesion in the right posterior mandible that is performed and the histology picture consists extends from the distal of the first molar into the of detached epithelium and mildly inflamed ramus. Inferiorly, it extends to just above the in- connective tissue. The epithelial detachment is ferior border of the mandible. There is no pares- suprabasilar and the basal cells remain attached thesia reported. All three molars are erupted to the underlying connective tissue. What is the and have become more and more mobile over diagnosis? the past year. Slight buccal expansion is noted. A. Benigh mucous membrane pembhigoid Which of the following is the most consistent (BMMP) with this radiographic presentation? B. Pemphigus vulgaris A. Calcifying epithelial odontogenic tumor C. Erythema multiforme (EM) (CEOT) D. Erosive LP B. OKC 200 Chapter 9: Questions: 31-42

43. A 25-year-old male presents with a 0.3-cm nodu- and your wife have children, they will have a lar mass on his right maxillary attached gin- % chance of inheriting hemophilia.” What giva in the area of the cuspid and first premo- percent chance do the Smiths have of having a

Pathology lar. It was submitted for biopsy and the report child with hemophilia? came back as following: Multiple sections show A. 0% stratified squamous epithelium overlying a cel- B. 25% lular lesion composed of haphazardly arranged, C. 50% plump, uniform, fibroblasts set in an immature D. 100% collagenous stroma. Metaplastic calcifications are noted throughout the lesion. What is the di- 46. A 15-year-old female presents with a 1.5-cm neck agnosis? swelling. The lesion is in the midline and at A. Mucocele approximately the level of the hyoid bone. It B. Peripheral ossifying fibroma is asymptomatic, fluctuant, and movable. Upon C. Peripheral giant cell granuloma (PGCG) swallowing, the swelling moves superiorly. This D. Pyogenic granuloma (PG) superior movement of the swelling is also noted when the patient protrudes her tongue. What is 44. A 20-year-old male presents with a multilocu- the most appropriate clinical diagnosis? lar radiolucency in his right posterior mandible. A. Cat-scratch disease The lesion extends from the distal of the ip- B. Metastatic disease silateral canine into the ramus. Teeth are dis- C. Branchial cleft cyst placed, as is the inferior border of the mandible. D. Thyroglossal duct cyst There is considerable expansion of the buccal cortex. No paresthesia is reported. The lesion is 47. A 40-year-old male presents with a diffuse, boggy aspirated and a straw-colored fluid is obtained. swelling of his gingiva. His entire gingiva is Tissue is removed and submitted for histopatho- bluish-green and is hyperplastic. The facial and logical examination. The report is as follows: lingual surfaces of most of his teeth are cov- Multiple sections show fragments of fibrous con- ered by this hyperplastic gingiva. The patient nective tissue containing a tumor composed of reports bruising easily, constant urinary tract in- islands and cords of odontogenic epithelium. fections, and states that overall he “just doesn’t The epithelium is composed of basal cells with feel healthy.” A biopsy of an affected area of hyperchromatic nuclei that exhibit reverse po- the gingiva shows diffuse infiltration of normal larity and subnuclear vacuolization. Within the tissue by sheets of poorly differentiated cells islands, a central edematous area resembling with myelomonocytic features. It is diagnosed stellate reticulum is seen. In areas, prominent as a granulocytic sarcoma. Which malignancy is juxtaepithelial hyalinization is present adjacent granulocytic sarcoma associated with? to the tumor cells. What is the diagnosis? A. Hodgkin’s lymphoma A. Ameloblastoma B. Multiple myeloma B. OKC C. Leukemia C. Odontogenic myxoma D. Ewing’s sarcoma D. Central giant cell granuloma 48. A 35-year-old male presents for routine dental 45. Mr. Smith has hemophilia A and is planning on care. During a head and neck examination, it having children. He along with his wife presents is noted that he has a redundant fold of tissue to a genetic counselor and blood tests are per- on the mucosal side of his upper lip. When formed on the female to determine if she has questioned about it, he says that he has had it hemophilia or is a carrier. It was determined that for approximately 2 years. During the examina- she neither has hemophilia nor carries the trait tion, you also notice that he has drooping upper for it. He inquires about future children. The ge- eyelids and a larger-than-normal thyroid gland. netic counselor says to him, “Mr. Smith, if you He states that his upper eyelid problems started Questions: 43-49 201 Pathology

around the same time as his lip problem began. is along the maxillary branch of the trigeminal No other significant findings are noted. On the nerve. There is involvement of the tip of her basis of this information, which of the following nose. Her medical history is noncontributory. syndromes does this man have? What should be the next step in the manage- ment of this patient? A. Gardner syndrome B. Papillon–Lef`evre syndrome A. Administration of systemic steroids. C. Gorlin syndrome B. Administration of systemic antiviral medica- D. Apert syndrome tion and application of topical steroids. E. Ascher syndrome C. Administration of systemic antiviral medica- tion. 49. A 70-year-old female presents with herpes zoster D. Administration of systemic antiviral medica- on the left side of her face. The distribution tion and referral to ophthalmologist. 202 Chapter 9: Pathology

ANSWERS

1. The correct answer is A. Pemphigus vulgaris atitis may have large, painful lesions that take a Pathology is a rare autoimmune disease in which the while to heal (weeks to months), but skin in- body produces antibodies against desmoglein volvement is not seen. Acyclovir may play a role 3 and desmoglein 1. These proteins are com- in treating stubborn lesions that result from prior ponents of desmosomes (intercellular adhesion herpetic outbreaks (as is also seen in EM). structures between epithelial cells). When these Primary and recurrent herpes simplex virus desmosomes are disrupted, epithelial cells are infections are self-limiting in immunocompe- no longer able to bond to each other and a split tent patients and usually resolve in 10 to 14 days. develops within the epithelium (causing a blis- Also, recurrent intraoral herpes is seen only on ter). The average age is 50 (this patient was a attached mucosa (gingiva and hard palate). This bit younger) and there is no sex predilection. patient had lesions on her buccal mucosa and Oral lesions are distributed haphazardly and ap- ventral tongue. Skin lesions outside of the peri- pear as superficial, ragged erosions and ulcer- oral region are not seen in herpes simplex virus ations. Oral lesions usually precede cutaneous (HSV) infections. Acyclovir, while not indicated lesions, sometimes by a year or more. Patients for treatment of established herpetic lesions, did with pemphigus demonstrate a positive Nikolsky not prevent this patient from getting new lesions. sign (induction of bulla by firm, lateral pressure). Diagnosis can be made histopathologically by 2. The correct answer is D. All leukoplakias re- seeing suprabasilar cleavage of epithelial cells, quire a diagnosis. If trauma is suspected, the eti- but is confirmed by direct immunofluorescence. ology should be removed. In this case, the frac- Treatment consists mainly of immunosuppres- tured cusp should be remedied and the patient sive agents (corticosteroids, azathioprine, etc.). should be re-evaluated in 2 weeks. Nightguards Left untreated, pemphigus vulgaris is fatal. can also be fabricated in suspected cases of Benign mucous membrane pemphigoid trauma. Any white lesion present for more than (BMMP), while in the same differential for the 2 weeks without improvement needs a scalpel intraoral lesions, typically does not present with biopsy to rule out epithelial dysplasia, neoplasm, skin lesions. The ulcers are, instead, limited to etc. It is important to note that obvious traumatic the mucous membranes (oral mucous, eyes). lesions (linea alba, morsicatio) can usually be di- Desquamative gingivitis is a common finding in agnosed clinically. In cases of atypical presenta- patients with BMMP. This patient had multi- tion or change in appearance over time, further ple skin lesions, which are not seen in classic investigation (scalpel biopsy) is required. BMMP. Immediate biopsy is rarely indicated in cases Erythema multiforme (EM) tends to have of leukoplakias. Removing suspected etiology an explosive onset and is self-limiting. Char- and waiting 2 weeks to re-evaluate is perfectly ac- acteristic appearances of EM include crusting ceptable treatment. There will be a significant of the lips and ulceration of the periphery of amount of lesions that will resolve after incit- the tongue. Targetoid lesions, though pathog- ing factors are removed. This will result in nomonic for EM, are rarely seen. This patient patients not having to undergo unnecessary pro- had longstanding lesions and no crusting of her cedures. Any lesion present after 2 weeks with no lips. EM minor is often caused by a prior recur- signs of improvement requires a scalpel biopsy. rent herpetic episode and oftentimes responds Noting a suspicious lesion and waiting 3 to antiviral treatment (as is also seen in major months to evaluate it is below the standard of recurrent aphthous stomatitis). care and not acceptable treatment. Although all Recurrent aphthous stomatitis fits with this lesions should be measured and described (a patient except for the duration of the disease photograph of the lesion is an excellent idea), (most aphthous heal in 10–14 days) and the skin 2 weeks is the recommended re-evaluation time lesions. Patients who have major aphthous stom- for suspicious lesions, not 3 months. Answers: 1-4 203 Pathology

Although this lesion is adjacent to a fractured On the basis of the significant intraoral swelling tooth, there is no way to definitively know that (due to expansile nature of this tumor) and the this lesion is due to trauma. Because of this, it is multilocularity of the lesion, ameloblastoma is recommended to remove the suspected etiology the most likely answer. Ameloblastomas are tu- (address the fractured tooth) and re-evaluate in 2 mors of odontogenic epithelium and are the weeks. Biopsy is indicated if lesion is still present most common clinically significant odontogenic and not improving. tumor. They occur over a wide age range and can develop de novo or arise from the lining 3. The correct answer is C. Ameloblastoma is by of an odontogenic cyst. Radiographically, they far the most common odontogenic tumor. Its fre- can appear as uni- or multilocular radiolucen- quency equals the combined total of all the other cies. Ameloblastomas can be locally destructive odontogenic tumors (excluding odontomas that and treatment and prognosis are determined by some classify as hamartomas). The preferred site the extent of the lesion. for ameloblastoma is the posterior mandible (ap- Dentigerous cysts, by definition, are asso- proximately 66% of total). Overall, mandibular ciated with the crown of an impacted tooth, lesions are more frequent than maxillary lesions usually mandibular third molars. Although any in their respective bony locations (posterior, mid- tooth can be affected, frequent sites (other than dle, anterior). mandibular third molars) include maxillary ca- Adenomatoid odontogenic tumors (AOTs) nines, maxillary third molars, and mandibular account for 3% to 7% of all odontogenic tumors. second premolars. Dentigerous cysts are rarely They have a striking predilection for the anterior seen involving unerupted deciduous teeth. jaws and maxillary lesions are two-thirds as com- Odontogenic tumors and other types of cysts can mon as mandibular lesions. Two-thirds of AOTs arise from the epithelium lining a dentigerous occur in teenage females and are oftentimes cyst. associated with an impacted tooth (usually a Osteosarcoma can present as a radiolucent, canine). radiopaque, or mixed lesion. When opacities Cementoblastomas account for less than 1% are present, they tend to appear as “sunburst” of odontogenic tumors. When they do occur, appearance. Swelling and pain are common 75% occur in the mandible with a striking symptoms. This patient had swelling, but was predilection for the molar and premolar region. asymptomatic. In addition, this lesion is well de- Fifty percent involve the first permanent mo- fined, and has smooth borders. Although there lar. Patients are typically in their 2nd and 3rd is still a possibility for this radiograph to repre- decades. sent an osteosarcoma, a benign lesion would be Calcifying epithelial odontogenic tumor much more likely. (CEOT) (also known as a Pindborg Tumor) is Periapical cysts, by definition, are associated a rare odontogenic tumor accounting for less with the apical area of nonvital teeth. Although than 1% of odontogenic tumors. Two-thirds oc- the apices of the teeth in this radiograph are in- cur in the posterior mandible. Patients are typi- volved, it was noted in the history that all teeth cally middle-aged. were vital. On the basis of this information, peri- Hemangiomas, while seen quite commonly apical cyst can be excluded. It is also important in soft tissue (and less frequently centrally), are to note that it is impossible to distinguish a pe- not odontogenic in origin. riapical cyst from a periapical granuloma or a Odontogenic myxomas are rare tumors of fibrous scar radiographically. odontogenic ectomesenchyme. The average age Buccal bifurcation cysts typically develop on for their occurrence is 25 and they may occur the buccal aspect of a permanent mandibular anywhere in the jaws. first molar. It is that cervical enamel projection in the furcation is responsible for the development 4. The correct answer is E. This lesion is radiolu- of this cyst. Occasionally, these lesions can be cent and has smooth borders with some scal- seen bilaterally. Radiographically, they are some- loping of the inferior border of the mandible. times hard to visualize on a periapical film due 204 Chapter 9: Pathology

to the tendency to expand perpendicularly to the infection of epithelial cells. It is seen almost buccal cortex. A mandibular occlusal film is best exclusively in HIV+ patients, but can be seen in used to diagnose these. With this patient, there other immunocompromised states. Very rarely,

Pathology is no involvement of the furcation of any teeth. it can be seen in immunocompetent patients. This diagnosis does not fit this case because of 5. The correct answer is B. Oral submucous the widespread nature of the disease process in fibrosis is a chronic, progressive disease that in- question. evitably results in oral cancer. It is seen in higher Leukoedema is an extremely common find- frequency in populations of Indian and South- ing in darker-skinned patients. It is considered a east Asian descent. A mixture of nuts, tobacco, variant of normal. Normally, it is seen bilaterally and sweeteners are wrapped in a betel quid leaf on the buccal mucosa with occasional extension and sucked on. Chemicals released from this onto the labial mucosa. On rare occasion, it can mixture are responsible for the clinical find- be seen on the floor of the mouth. These lesions ings. Dense collagenous bands (evident in the characteristically disappear when the mucosa is picture) form and limit the opening of mouth stretched. In this case, the lateral tongue was in these patients. Teeth almost always exhibit affected and the buccal mucosa tissue did not some form of staining, and the leukoplakic areas change in appearance upon stretching. are almost universally dysplastic (premalignant) due to the tobacco component. Despite cessa- 6. The correct answer is A. This is a classic pre- tion of this habit, the changes that have taken sentation of recurrent herpes labialis. A tingling place do not revert. These patients become a prodrome, vesicle formation and coalescence, challenge to treat both dentally (because of de- and eventual crusting over and healing are typ- creased opening) and surgically (to release fi- ical with these lesions. Although he mentions brous bands and excise dysplastic lesions). that he has never had intraoral lesions in the past Although this patient does have a signifi- (a question asked to determine if he had had pri- cant smoking history, smoker’s melanosis typi- mary herpetic gingivostomatitis), only approxi- cally affects the anterior facial gingiva. Smoker’s mately 10% of patients experience any clinical melanosis is also darker in color due to an in- symptoms during primary infection. All of this crease in melanin production by melanocytes information coupled with the patient stating that in the basement membrane of the oral mucosa. he has had these before leads to the diagnosis of The lesions in this patient’s mouth were white recurrent herpes. Treatment for herpes during in nature. the prodrome is antivirals (topical or systemic), Morsicatio buccarum, or chronic cheek but antiviral therapy is not indicated once le- chewing, results from excessive nibbling of the sions are well established and crusted over. Left buccal mucosa, usually in the anterior portions. on their own, herpetic ulcers resolve in 10 to It is quite common and most people are aware 14 days without scarring in immunocompetent that they are doing it. Lesions occur along the patients. Herpetic ulcers that last longer are sus- occlusal plane (just like linea alba) and may ex- picious of immunosuppression and should be tend slightly above or below it. In the case of this investigated as needed. patient, lesions are present high in the vestibules Topicalantivirals are only effective during the (an impossible area to bite) and a definitive prodrome and are not indicated once a lesion is fibrous band can be seen on the buccal mucosa. established. A prescription may be written for The lesions in this patient also appear smooth application during prodrome of next outbreak. and not ragged, as one would expect in a lesion Systemic antivirals are only effective during secondary to chewing. The excessive staining of the prodrome and are not indicated once a le- this patient’s teeth may also hint that something sion is established. The exception to this rule else may be going on. is in an immunocompromised patient, where Oral hairy leukoplakia (OHL) is an entity new lesions have the potential to develop and that occurs exclusively on the lateral borders of the host’s immune system cannot effectively deal the tongue. It is caused by Epstein–Barr virus with them. Answers: 5-8 205 Pathology

Once a strong suspicion of herpes is evident, Gonorrhea is the most common reportable this lesion can be left alone. If, upon follow-up in bacterial infection in the United States. It is a sex- 2 weeks, the lesion is still present, and incisional ually transmitted disease and is usually limited biopsy is indicated. to genital involvement, though oral involvement Recurrent herpes labialis has no ocular man- has been seen. When a patient has oral manifes- ifestations and thus an ophthalmologic referral tations, the most commonly affected site is the is not warranted. oropharynx. Although usually asymptomatic, erythema and sore throat can be present. Tonsils 7. The correct answer is E. Treponema pallidum is may be erythematous and edematous. Left un- a gram-negative spirochete that causes syphilis, a treated, most cases spontaneously resolve. Treat- sexually transmitted disease. The patient in this ment is appropriate antibiotic therapy and is question is exhibiting a classical chancre of pri- indicated to reduce the potential spread of the mary syphilis. These chancres occur at the site disease. of inoculation 3 to 90 days after contact. They are characteristically painless and take 3 to 8 8. The correct answer is C. A Stafne bone cyst weeks to heal. Although the majority of chan- (Stafne defect, static bone defect) is not a true cres occur in the genital region, the most com- cyst. It represents a developmental defect in mon oral site for these lesions is the upper lip. which the submandibular gland causes a de- Treatment for primary syphilis is penicillin G. pression on the lingual aspect of the mandible. Left untreated this lesion could progress to the This depression either contains a portion of sub- disseminated secondary syphilis and eventually mandibular salivary gland tissue or fibrous tis- to the life-threatening tertiary syphilis and neu- sue and muscle. These lesions classically present rosyphilis. as an asymptomatic radiolucency near the an- Histoplasmosis is the most common systemic gle of the mandible and below the level of the fungal infection in the United States. It is en- mandibular canal. Literature states that 80% to demic to the Ohio and Mississippi River Val- 90% of Stafne lesions occur in male patients and, leys. The vast majority of cases produce little or once recognized, require no treatment. no clinical symptoms. When patients are symp- Dentigerous cysts, by definition, are associ- tomatic, the lungs are the most commonly af- ated with the crown of an impacted tooth. There fected sites. Immunocompromised patients can is no tooth associated with this radiolucency. experience disseminated histoplasmosis, which Although certain malignant lesions do metas- is a serious complication. Treatment for histo- tasize to the jawbones, their radiographic ap- plasmosis consists of systemic antifungals. This pearance tends to be ill-defined, indicating a patient had an isolated ulcer, was immunocom- destructive process. This lesion is well defined petent, and had no pulmonary symptoms. with a sclerotic border around it. In this particu- Recurrent herpes labialis (caused by herpes lar case, a metastatic lesion should be considered simplex virus type 1 or type 2) is painful and low on the differential. heals in 10 to 14 days in immunocompetent pa- Calcifying odontogenic cyst is an uncommon tients. This patient was otherwise healthy, had lesion. They can occur in both the maxilla and an ulcer present for 3 weeks, and reported no mandible and approximately 65% of them are pain associated with it. found in the incisor and canine region. Lesions Cryptococcosis is a fungal infection that usu- can be unilocular and well defined, and often ex- ally causes no symptoms in immunocompetent hibit calcifications within the lesion. Although patient. HIV+ patients are very susceptible to this lesion could, theoretically, represent a cal- cryptococcal infection. As with histoplasmosis, cifying odontogenic cyst (COC), there are other cryptococcosis is seen primarily in the lungs lesions that this is more likely to be. and can experience dissemination. This patient Periapical or radicular cysts are associated was immunocompetent and had no pulmonary with the apex of a nonvital tooth. Besides the symptoms. Treatment for cryptococcosis is sys- fact that these teeth were stated to be vital, this temic antifungals therapy. 206 Chapter 9: Pathology

lesion does not appear to be associated with the North American-type Burkitt’s lymphoma is apex of a tooth in any way. similar to the African type but usually initially presents as an abdominal mass.

Pathology 9. The correct answer is B. OHL is seen almost Multiple myeloma is a malignancy of plasma exclusively in HIV+ patients. Its prevalence in- cells that typically affects older patients (median creases once a patient’s CD4 count drop below age between 60 and 70). It represents the most 200. This is a benign lesion in and of itself, common hematologic malignancy among black but it serves as an indicator of a patient’s im- persons in the United States. Bone pain is the mune status. This patient stated that she was most common symptom. Punched-out radiolu- healthy (unaware of her immune status) but cencies can be seen in the skull, jawbones, ver- given her history of IV drug use and the presence tebrae, and other bones. Treatment consists of of this lesion, her HIV status needs to be assessed. chemotherapy and overall prognosis is poor. Ad- Once diagnosed, this lesion requires no further ditionally, with regards to dental care of these treatment. OHL is caused by the inclusion of patients, it should be stated that these patients the Epstein–Barr virus (HHV-4) in the nuclei are typically placed on IV bisphosphonate therapy. of superficial epithelial cells and causes a phe- Ewing’s sarcoma is a malignancy of bone nomenon known as chromatin beading. Any pa- composed of small, undifferentiated round cells tient who has a diagnosis of OHL needs to have of uncertain lineage. Peak prevalence is in the an HIV test. Routine dental care can proceed second decade and the vast majority of those as indicated. Although some patients choose to affected are white. Long bones, pelvis, and rib have this lesion excised for cosmetic reasons, it are most affected. Pain is common and soft- tends to recur. The best treatment for this con- tissue invasion adjacent to affected bone can be dition is to do nothing. Topical steroids are indi- seen. Treatment consists of surgery, radiation, cated only if the patient becomes asymptomatic. and chemotherapy. Frequently, these lesions become suprainfected with candida and require treatment with an an- 11. The correct answer is A. Leukemic cells can tifungal. infiltrate soft tissues and cause a diffuse, non- tender swelling. This is most frequent with the 10. The correct answer is E. Hodgkin’s lymphoma myelomonocytic types of leukemia acute myel- is a malignant lymphoproliferative disorder com- ogenous leukemia (AML), chronic myeloge- posed of Reed–Sternberg cells (bi- and multin- nous leukemia (CML). Tissues affected are most ucleated) in enlarged lymph nodes. The most commonly gingiva, with diffuse gingival enlarge- common sites of initial presentation are the cer- ment that can mimic drug-related gingival hy- vical and supraclavicular lymph nodes (70–75% perplasia. Occasionally, the gingiva can take on of cases). A bimodal pattern with respect to pa- a dark green to black appearance. This is known tient’s age is also noted: between 15 and 35 years, as a granulocytic sarcoma or extramedullary and another peak after age 50. Overall, a male myeloid tumor. predilection is observed. Treatment depends on Thrombocytopenia is a term used to refer to a stage but usually consists of radiation therapy, decrease in the number of circulating platelets. chemotherapy, or a combination of the two. It is often a sign of an underlying disease. Throm- African-type Burkitt’s lymphoma is a type of bocytopenia can be caused by either an in- B-cell lymphoma. It was originally reported in creased destruction of thrombocytes (platelets) African children and has a predilection for the or a decreased production. Clinically, patients jawbones. More than 90% of tumor cells show exhibit signs of inability to achieve hemostasis expression of Epstein–Barr virus nuclear anti- (spontaneous gingival hemorrhage, petechiae, gen. Peak prevalence is 7 years of age. These and ecchymoses, and inability to stop bleeding lesions are extremely aggressive and can grow in after dental treatment, etc.). size in short period of time. Treatment generally Neutropenia is a term that refers to a decrease consists of aggressive chemotherapy. in the number of circulating neutrophils. This can be caused by either an increased destruction Answers: 9-13 207 Pathology

of neutrophils or a decreased production. Clin- developing this tumor. Newer studies still show ically, patients present with bacterial infections. a male predilection, but not as lopsided as the Orally, ulcerative lesions of the gingival mucosa previous numbers (M:F 26:1 vs. M:F 1.6:1). This are common. Treatment consists of treating the may be due to an increased number of women underlying condition and antibiotics for infec- smokers. Because this patient is a nonsmoking, tions. black female, it is unlikely that this represents a Multiple myeloma is a malignancy of plasma Warthin’s tumor. cells that typically affects older patients (median Hodgkin’s lymphoma is a malignant lym- age between 60 and 70). It represents the most phoproliferative disorder composed of Reed– common hematologic malignancy among black Sternberg cells in enlarged lymph nodes. The persons in the United States. Bone pain is the most common sites of initial presentation are the most common symptom. Punched-out radiolu- cervical and supraclavicular lymph nodes (70– cencies can be seen in the skull, jawbones, ver- 75% of cases). A bimodal pattern with respect to tebrae, and other bones. Treatment consists of patient’s age is also noted: between 15 and 35 chemotherapy and overall prognosis is poor. Ad- years, and another peak after age 50. Overall, a ditionally, with regards to dental care of these pa- male predilection is observed. Treatment de- tients, it should be stated that these patients are pends on stage but usually consists of radiation typically placed on IV bisphosphonate therapy. therapy, chemotherapy, or a combination of the Anemia is a generic term that refers to either two. a decrease in volume of red blood cells (hema- Although adenoid cystic carcinomas do oc- tocrit) or in the concentration of hemoglobin. cur in the parotid gland, they are vastly outnum- It can be caused by an increased destruction of bered by pleomorphic adenomas. Adenoid cystic erythrocytes (red blood cells), a decreased pro- carcinomas can also present as painless, discrete duction of erythrocytes, or sequestration of ery- masses, but often patients complain of tender- throcytes in the spleen. Anemia is often a sign of ness, pain, and facial nerve paralysis due to the an underlying disease. Symptoms include tired- propensity of adenoid cystic carcinomas to in- ness, headache, and lightheadedness. Treatment vade nerve. It is also worthy to note that larger le- consists of treating the underlying disease. sions usually become fixed to the skin or deeper surrounding tissues. Although certainly possible, 12. The correct answer is D. Pleomorphic adenoma it is much more likely that this patient has a pleo- (benign mixed tumor) is the most common neo- morphic adenoma. plasm of salivary gland origin. They are most fre- quently encountered in the parotid but can also 13. The correct answer is B. Patients who are suf- occur intraorally (the most common site being fering from hyperparathyroidism have elevated the palate). There is a female predilection and levels of serum calcium and present with lesions the average age of diagnosis is 43 years. Clinically known as brown tumors. These lesions are his- it presents as a slow-growing, asymptomatic, dis- tologically identical to central giant cell granu- crete mass in the superficial (lateral) lobe of the lomas. If this patient has normal serum calcium parotid, although it can also occur in the deep levels, this lesion represents a central giant cell lobe. Treatment for these lesions usually consists granuloma and should be treated appropriately. of superficial parotidectomy with preservation of If he has elevated levels of serum calcium, his the facial nerve. underlying hyperparathyroidism needs to be ad- Warthin’s tumor presents as a painless, some- dressed. Resolution of brown tumor occurs after times fluctuant mass in the parotid gland. It is the balance is restored. second most common benign parotid salivary Circulating platelet levels have no relevance gland tumor. Warthin’s tumor has an extremely in the differentiation of a central giant cell gran- low frequency among black patients (approxi- uloma from a brown tumor of hyperparathy- mately 2.5%). In addition, older studies have roidism. demonstrated a male predilection for Warthin’s Definitive treatment of any pathologic find- tumor. Smoking also seems to increase the risk of ing should not be initiated until a diagnosis has 208 Chapter 9: Pathology

been made. This histology of this lesion is consis- pacted teeth are also common findings. Of par- tent with both a central giant cell granuloma and ticular importance is the presentation of colonic a brown tumor of hyperparathyroidism. Treat- polyps that ultimately transform into adenocar-

Pathology ment should be deferred until it is determined cinoma (usually before the age of 30). Females whether or not the patient has hyperparathy- also demonstrate a 100-fold increase of thyroid roidism. Treatment for brown tumor consists of carcinoma. It is because of the malignant mani- treatment of the underlying condition. Resolu- festations of this syndrome that patients need to tion of the lesion occurs after the metabolic im- be referred to their primary care physician for balance is corrected. evaluation of colon and thyroid gland. Gorlin syndrome (also known as nevoid basal 14. The correct answer is A. Rubeola (measles) is cell carcinoma syndrome) is an autosomal dom- produced by a paramyxovirus. Because of the inant syndrome caused by a mutation in the MMR vaccine, measles incidence is low, but the PTCH tumor suppressor gene on chromosome clinical signs and symptoms still need to be rec- 9. It is characterized by multiple basal cell car- ognized. The incubation period is 10 to 12 days. cinomas and multiple odontogenic keratocysts. Significant lymphoid hyperplasia is noted. In Ascher syndrome is characterized by the fol- this case, the patient is in Stage 1. Stage 1 is char- lowing three features: double lip, blepharocha- acterized by the 3Cs: (coryza, cough, and con- lasis (edema of the upper eyelids), and nontoxic junctivitis). Intraorally, lesions known as Koplik’s thyroid enlargement. spots are noted bilaterally on the buccal mucosa. Klinefelter’s syndrome is the most common Stage 2 shows fading of Koplik’s spots and devel- sex chromosome disorder that affects males. opment of a maculopapular rash. Stage 3 shows These individuals are 47, XXY. Although these the end of the fever and resolution of skin rash. patients are affected with disorders related to Each stage is approximately 3 days long. abnormal hormone production (hypogonadism, Mumps (epidemic parotitis) is also caused by gynecomastia, etc.), the dental abnormalities in a paramyxovirus. Clinical features include sig- the patient described earlier do not fit with Kline- nificant salivary gland changes. These changes felter’s. Also, the fact that this patient is a female include discomfort and swelling in the preauric- eliminates Klinefelter’s as a reasonable choice. ular region extending down the posterior, infe- rior border of the mandible. Another common 16. The correct answer is D. Cryptococcus neo- finding in males is epididymoorchitis, which is formans is uncommon and usually causes no rapid testicular swelling with pain and tender- problems in immunocompetent people. It is as- ness. This patient had none of these findings. sociated with pigeons (living in their excrement) Recurrent intraoral herpes is seen only on at- and can grow in soil. It produces a mucopolysac- tached mucosa (gingiva and hard palate) in im- charide capsule that protects it from host im- munocompetent patients. This patient’s lesions mune defenses. This capsule is visualized using were on his buccal mucosa and they were asymp- a mucicarmine stain. It is acquired by inhala- tomatic. Herpetic lesions are exquisitely painful. tion of spores. Most infections are limited to the Hand-foot-and-mouth disease is caused by an lungs, but dissemination of disease is common enterovirus. Because this patient had no lesions in immunocompromised patients. outside of his mouth, this diagnosis should be Blastomyces dermatitidis grows in rich, moist excluded. soil. Geographically, it is seen in the eastern half of the United States. It is also found north into 15. The correct answer is C. Gardner syndrome is the provinces of Canada surrounding the Great an autosomal dominant syndrome that is part Lakes. Blastomycosis is a rare fungal infection of the spectrum of diseases including familial that is acquired by inhalation of spores and is nor- colorectal polyposis. Patients present with mul- mally no problem in immunocompetent people. tiple osteomas of the jaws 90% of the time and In certain cases, it becomes hematogenously dis- can demonstrate benign cysts and tumors of the seminated. The Blastomyces organisms do not skin. Supernumerary teeth, odontomas, and im- have a capsule. Answers: 14-19 209 Pathology

Paracoccidioides brasiliensis is the causative of uncertain lineage. Peak prevalence is in the agent of Paracoccidioidomycosis (South Amer- second decade and the vast majority of those ican Blastomycosis). It is seen in people who affected are white. Long bones, pelvis, and rib live in South America and has features similar are most affected. Pain is common and soft- to Blastomycosis. The organism is unencapsu- tissue invasion adjacent to affected bone can be lated. seen. Treatment consists of surgery, radiation, Histoplasmosis is the most common systemic and chemotherapy. fungal infection in the United States. It grows as both yeast and a mold. It is endemic to the Ohio 18. The correct answer is A. Plumbism, or lead poi- and Mississippi River Valleys. It is acquired by soning, has widespread systemic effects, includ- inhalation and usually produces no symptoms ing anemia, fatigue, renal dysfunction, and mus- in immunocompetent patients. Disseminated culoskeletal pain. Oral manifestations include disease can be found in immunocompromised ulcerative stomatitis, gingival lead line (Burton’s patients. These Histoplasma organisms are un- line), and grayish discoloration of buccal mu- encapsulated. cosa and tongue. Additional manifestations of lead poisoning include tremor of tongue upon 17. The correct answer is B. Multiple myeloma is thrusting, excessive salivation, and advanced pe- a malignancy of plasma cells that typically af- riodontal disease. fects older patients (median age between 60 and Acrodynia is the result of chronic exposure 70). It represents the most common hematologic to mercury. Oral changes noted include metal- malignancy among black persons in the United lic taste, ulcerative stomatitis, inflammation and States. Bone pain is the most common symptom. enlargement of the salivary lands, gingiva, and Punched-out radiolucencies can be seen in the tongue. skull, jawbones, vertebrae, and other bones. Pa- Argyria is the result of systemic silver intoxica- tients with multiple myeloma can also show focal tion, which can result in coma, pleural edema, deposits of acellular protein known as amyloid. hemolysis, and bone marrow failure. Diffuse These deposits can be found in internal organs, gray discoloration is noted in sun-exposed areas as well as skin and mucosal surfaces. Histolog- of skin. A blue-silver line may also appear along ically, it appears as an amorphous, glassy pink the gingival margins. material. Arsenic poisoning has widespread effects on Hodgkin’s lymphoma is a malignant lym- numerous organ systems. Diffuse macular hy- phoproliferative disorder composed of Reed– perpigmentation and palmar and plantar hy- Sternberg cells (bi- and multinucleated) in en- perkeratosis are noted. Long-term exposure can larged lymph nodes. The most common sites result in basal cell carcinoma and cutaneous of initial presentation are the cervical and supr- squamous cell carcinoma. Oral manifestations aclavicular lymph nodes (70–75% of cases). A are rare. bimodal pattern with respect to patient’s age is also noted: between 15 and 35 years, and another 19. The correct answer is A. Sodium valproate (val- peak after age 50. proic acid) is an anticonvulsant used to treat neu- African-type Burkitt’s lymphoma is a type of rological disorders such as epilepsy. Its use has B-cell lymphoma. It was originally reported in been associated with gingival hyperplasia. African children and has a predilection for the Cyclosporine, while one of the major drugs jawbones. More than 90% of tumor cells show implicated in drug-induced gingival hyperpla- expression of Epstein–Barr virus nuclear anti- sia, is an immunosuppressant agent used in gen. Peak prevalence is 7 years of age. These the treatment of transplant patients, rheumatoid lesions are extremely aggressive and can grow in arthritis, and psoriasis. This patient has no his- size in short period of time. Treatment generally tory of anything other than epilepsy and thus consists of aggressive chemotherapy. would not be taking cyclosporine. Ewing’s sarcoma is a malignancy of bone Verapamil is a calcium-channel blocker composed of small, undifferentiated round cells and is also implicated in the occurrence of 210 Chapter 9: Pathology

drug-related gingival hyperplasia. This patient linear deposition of antibodies subjacent to the has no history of anything other than epilepsy basement membrane. BMMP is normally not and thus would not be taking verapamil. fatal, but blindness may result if left untreated.

Pathology Nifedipine is a calcium-channel blocker and BMMP can sometimes be managed by topi- is also implicated in the occurrence of drug- cal steroids, but should only be done by some- related gingival hyperplasia. This patient has no one with experience with the disease. As with the history of anything other than epilepsy and thus previous explanation, this patient does not have would not be taking nifedipine. BMMP. Pemphigus vulgaris is the correct diagnosis, 20. The correct answer is D. Papillon–Lef`evre syn- but the treatment here is wrong. These patients drome is an autosomal recessive syndrome that need to be seen by someone who is well versed is characterized by palmar–plantar keratosis and in treating patients with this specific malady. dramatically advanced periodontitis in both de- ciduous and permanent dentitions. 22. The correct answer is A. Monomorphic ade- Gorlin syndrome (also known as nevoid nomas are benign salivary gland tumors. They basal cell carcinoma syndrome) is an autoso- are sometimes classified into one of two cate- mal dominant syndrome caused by a mutation gories: canalicular adenoma or basal cell ade- in the PTCH tumor suppressor gene on chromo- noma. They can be found in minor glands and some 9. It is characterized by multiple basal cell major glands. The canalicular type is found al- carcinomas, multiple odontogenic keratocysts, most exclusively in the upper lip. It presents as a palmar/plantar pitting, calcification of the falx slow-growing, painless swelling in older adults. cerebri, and rib abnormalities. Literature states that there is approximately a 2:1 Apert syndrome is a rare condition similar female predominance lesion. This is the most to Crouzon syndrome and is characterized by appropriate clinical diagnosis of the choices pre- craniosynostosis (premature closing of the cra- sented here. nial sutures). Patients typically have cranial mal- Mucoceles are the result of extravasation of formations that result in abnormal-shaped heads mucin into surrounding tissues secondary to (tower skull, cloverleaf skull). Affected individ- a disruption in the continuity of a duct (usu- uals also have a hypoplastic midface and syn- ally caused by trauma). Although mucoceles dactyly of the second, third, and fourth digits of are quite common in the lower lip, they are the hand. Mental retardation is common. exquisitely rare in the upper lip and should not Ascher syndrome is characterized by the fol- be near the top of a differential. lowing three features: double lip, blepharocha- Ameloblastomas are the most common odon- lasis (edema of the upper eyelids), and nontoxic togenic tumor. Although they do occur in thyroid enlargement. extraosseous locations (peripherally), they are confined to the gingiva and alveolar mucosa. 21. The correct answer is C. Pemphigus vulgaris Ameloblastoma would not be an appropriate le- is an autoimmune disorder that is diagnosed by sion for a swelling of the upper lip. direct immunofluorescence that shows a char- A ranula is a mucocele of the floor of the acteristic “chicken wire” deposition of antibod- mouth. By definition, this lesion cannot repre- ies between epithelial cells. The induction of a sent a ranula. bulla secondary to firm, lateral pressure on mu- A Stafne bone cyst (Stafne defect, static bone cosa is known as a positive Nikolsky sign. These defect) is not a true cyst. It represents a devel- patients need to be treated by someone with ex- opmental defect in which the submandibular perience. Left untreated, pemphigus vulgaris is gland causes a depression on the lingual aspect fatal. of the mandible. This depression either contains Although BMMP has a similar appearance a portion of submandibular salivary gland tissue clinically and referral to an ophthalmologist or fibrous tissue and muscle. These lesions clas- is appropriate treatment, the result of the im- sically present as an asymptomatic radiolucency munofluorescence rules it out. BMMP shows a near the angle of the mandible and below the Answers: 20-25 211 Pathology

level of the mandibular canal. Literature states Referral to ophthalmology is required in patients that 80% to 90% of Stafne lesions occur in male diagnosed with BMMP. patients and, once recognized, require no treat- ment. 25. The correct answer is D. OKCs are develop- mental cysts that have a high rate of recurrence. 23. The correct answer is D. Ameloblastic fibro- They can appear uni- or multilocular and have odontomas are odontogenic tumors usually en- a predilection for the posterior mandible. They countered in children. They are most com- can be found as one of the oral manifestations of mon in the posterior jaws with mandible favored nevoid basal cell carcinoma syndrome. Lesion over maxilla. They are frequently encountered size ranges from quite small to large enough to when radiographs are taken to determine why fill entire ramus. Upon aspiration, they typically a tooth has not erupted. The radiographic fea- produce a cheese-like substance (keratin). The tures described earlier are classic for ameloblas- reason the second aspiration was slightly reddish tic fibro-odontomas. The degree of calcification was because there was hemorrhage in the area can range from very little to a solid mass of due to prior aspiration. Treatment of these le- calcified material. Treatment is generally con- sions is controversial and ranges from marsupi- servative, as these do not behave as classic alization to resection. ameloblastomas do. Dentigerous cysts, by definition, are asso- Odontogenic keratocysts (OKCs) are devel- ciated with the crown of an impacted tooth, opmental cysts that have a high rate of recur- usually mandibular third molars. Although any rence. They can appear uni- or multilocular and tooth can be affected, frequent sites (other than have a predilection for the posterior mandible. mandibular third molars) include maxillary ca- They can be found as one of the oral mani- nines, maxillary third molars, and mandibular festations of nevoid basal cell carcinoma syn- second premolars. The lesion described in this drome. Lesion size ranges from quite small to case was not associated with any teeth. large enough to fill entire ramus. Calcifications A Stafne bone cyst (Stafne defect, static bone are rarely (if ever) seen radiographically. defect) is not a true cyst. It represents a devel- Cementoblastomas are rare neoplasms that opmental defect in which the submandibular have a mandibular predilection and occur in the gland causes a depression on the lingual aspect molar/premolar region. Radiographically, this of the mandible. This depression either contains lesion appears as a radiopaque mass fused to one a portion of submandibular salivary gland tissue or more tooth roots and surrounded by a radiolu- or fibrous tissue and muscle. These lesions clas- cent rim. The patient in this question had a mass sically present as an asymptomatic radiolucency coronal to the impacted tooth, not attached to near the angle of the mandible and below the the root(s). level of the mandibular canal. Literature states Radicular (periapical) cysts are found at the that 80% to 90% of Stafne lesions occur in male apex of nonvital teeth. patients and, once recognized, require no treat- ment. 24. The correct answer is C. This histological Traumatic bone cysts (simple bone cysts) are appearance is classic of necrotizing sialometa- not true cysts. They are not lined by conventional plasia. Necrotizing sialometaplasia (NS) is a re- epithelium, but rather by a compressed fibrous active phenomenon thought to be caused by tissue. Upon surgical exploration, these lesions ischemic injury to a portion of the palate. It is are found to be empty. Aspiration may return more common in males and once a diagnosis is a slightly serosanguinous fluid. Treatment is to rendered (via incisional biopsy), no further treat- induce bleeding during biopsy and allow the le- ment is required. Lesions spontaneously heal in sion to fill up with blood. Cause of these lesions 5 to 6 weeks. If lesion does not seem to be heal- was once thought to be due to trauma (hence, ing, another biopsy is warranted. the name), but some seem to be idiopathic. NS spontaneously resolves and requires no further intervention once a diagnosis is made. 212 Chapter 9: Pathology

26. The correct answer is B. The liver is responsi- ture and airway obstruction. This sequence is ble for producing the vitamin K-dependent clot- seen in Stickler and velocardiofacial syndromes ting factors (II, VII, IX, X). Liver disease can most commonly, but has been reported in

Pathology result in the decreased production of these fac- others. tors and place the patient in a hypocoagulated Klinefelter’s syndrome is the most common state. In addition to the other answers not com- sex chromosome disorder that affects males. patible with this question, this patient is a heavy These individuals are 47, XXY. Although these drinker (alcohol is a known risk factor for liver patients are affected with disorders related to disease). If that was not enough, this patient re- abnormal hormone production (hypogonadism, ported frequent acetaminophen use. One of the gynecomastia, etc.), the dental abnormalities in metabolites of acetaminophen is hepatotoxic in the patient described earlier do not fit with Kline- high doses (and even more so in a liver suffering felter’s. The fact that this patient is a female also the effects of alcohol). eliminates Klinefelter’s as a reasonable choice. Patients with acute or chronic renal fail- Apert syndrome is a rare condition similar ure can rarely demonstrate oral manifestations to Crouzon syndrome and is characterized by known as uremic stomatitis. Although rare, it is craniosynostosis (premature closing of the cra- mostly seen in acute rather than chronic disease. nial sutures). Patients typically have cranial mal- Clinically, white plaques appear on buccal mu- formations that result in abnormal-shaped heads cosa, tongue, and floor of mouth. The onset of (tower skull, cloverleaf skull). Affected individ- these plaques is often abrupt. There may also be uals also have a hypoplastic midface and syn- an odor of ammonia or urine on the patient’s dactyly of the second, third, and fourth digits of breath. Lesions resolve once the underlying the hand. Mental retardation is common. kidney disease is treated. Gorlin syndrome (also known as nevoid basal The oral manifestations of Crohn’s disease cell carcinoma syndrome) is an autosomal dom- (an inflammatory disease affecting the distal inant syndrome caused by a mutation in the small bowel and proximal colon) include nodu- PTCH tumor suppressor gene on chromosome lar swelling, cobblestone appearance of oral 9. It is characterized by multiple basal cell carci- mucosa, and deep, granulomatous-appearing ul- nomas, multiple OKCs, palmar/plantar pitting, cers. These findings were not seen in this patient. calcification of the falx cerebri, and rib abnor- Abdominal pain, nausea, diarrhea, and weight malities. loss may also be seen. Gardner syndrome is an autosomal dominant Hodgkin’s lymphoma is a malignant lym- syndrome that is part of the spectrum of diseases phoproliferative disorder composed of Reed– including familial colorectal polyposis. Patients Sternberg cells in enlarged lymph nodes. The present with multiple osteomas of the jaws 90% most common sites of initial presentation are the of the time and can demonstrate benign cysts cervical and supraclavicular lymph nodes (70– and tumors of the skin. Supernumerary teeth, 75% of cases). A bimodal pattern with respect odontomas, and impacted teeth are also com- to patient’s age is also noted: between 15 and mon findings. Of particular importance is the 35 years, and another peak after age 50. Overall, presentation of colonic polyps that ultimately a male predilection is observed. Treatment de- transform into adenocarcinoma (usually before pends on stage but usually consists of radiation the age of 30). Females also demonstrate a 100- therapy, chemotherapy, or a combination of the fold increase of thyroid carcinoma. It is because two. of the malignant manifestations of this syndrome that patients need to be referred to their primary 27. The correct answer is C. The Pierre Robin se- care physician for evaluation of colon and thy- quence is characterized by posterior displace- roid gland. ment of his tongue, cleft palate, and mandibular micrognathia. The retruded mandible results in 28. The correct answer is A. This is a suspected the posterior displacement of the tongue, and malignancy. Given the patient’s age, sex, and also in lack of support of the tongue muscula- social history, he is a prime candidate for oral Answers: 26-30 213 Pathology

cancer. Compounding this is the fact that this le- any pigmented lesion on the palate (a high-risk sion is large, persistent, painless, and indurated. site), they generally take time to evolve (i.e., not Suspected malignancies should have an inci- appear in 1 month’s time). If the patient was sional biopsy performed. This biopsy should in- symptomatic (or becomes symptomatic), or ar- clude both the ulcerated portion and adjacent eas appeared raised, ulcerated, asymmetrical, or normal mucosa. Sutures may be used to orient changing in color, an incisional biopsy would be the pathologist. indicated. Suspected malignancies should have an inci- sional biopsy performed. If the clinical diagnosis 30. The correct answer is B. Although most lesions is appropriate (i.e., malignant), further surgical are given 2 weeks to resolve on their own be- intervention will be warranted. Removing the fore biopsy, this “waiting period” can often times clinical lesion may hinder efforts to evaluate it be skipped based on clinical information and the in the future. biopsy performed immediately (assuming the pa- Performing a biopsy of an ulcer without ad- tient is medically cleared to undergo a surgical jacent tissue leads to a diagnostic dilemma. In procedure). In this particular case, there are a some cases, a diagnosis may still be made. In few hints that allow this to be biopsied imme- other cases, a diagnosis of “nonspecific ulcer” diately. First is the location; anterior maxillary will be rendered and another biopsy will be gingiva and hard palate are high-risk sites for needed. When biopsying ulcers, always take ad- melanoma. Second is that the lesion has in- jacent normal mucosa (this is especially impor- creased in size over time. The last hint is that tant for vesiculobullous diseases such as pem- it has gotten darker. All of these things are point- phigus and BMMP). ing toward a premalignant or malignant pro- Normal adjacent mucosa, while it may con- cess. The ABCDs of melanoma are: Asymmetry, tain some diagnostic information, is far inferior Border irregularities, Color variegation, and Di- to tissue from the lesion itself. ameter greater than 6 mm. All of these have a negative impact on the prognosis of melanoma. 29. The correct answer is D. Malaria has a high Oral mucosal melanoma has a poor overall prog- prevalence in African nations. Subsequently, nosis. people traveling there for any length of time are Although performing a periodontal proce- usually administered chloroquine (or another dure after removing the lesion may be done, quinine derivative). Quinine is known to cause the impetus to remove the lesion should not dark brown-black pigmentation in patients, most be cosmesis. Pigmented lesions on the ante- often in the palate. A good correlation between rior maxillary gingiva and hard palate should beginning of medication and onset of appear- be viewed with skepticism, as these are high-risk ance can be helpful diagnostically. sites for melanoma. What good does nice looking Amalgam tattoos are pigmented and result gingiva do when the patient may require resec- from the traumatic implantation of amalgam tion for a malignant process? into the submucosal. They usually present in iso- Waiting 3 months to re-evaluate a pigmented lated foci (i.e., not the entire palate) and are not lesion in a high-risk site that has undergone symmetrical. This patient also reports no dental changes in size and color is below the standard work prior to the onset. Radiographs can some- of care. This lesion should be biopsied as soon times be handy to see if any radiopaque material as possible. is present within the lesion. Decisions of this nature are not up to the pa- Racial pigmentation is generally seen on the tient. As a licensed medical professional, it is facial aspect of the gingiva. This patient is a Cau- the responsibility of the health care provider to casian and this lesion was not present 2 months make sure this lesion is managed appropriately. prior. All of these reasons rule out racial pigmen- The patient should also not be told he has noth- tation. ing to worry about, especially when all clinical Although melanoma is certainly a concern signs (location, change in size and color) are and should be included in the differential for 214 Chapter 9: Pathology

pointing toward a potentially premalignant or isms to flourish. It is also important for patients malignant process. to finish a course of antibiotics once they have been started on them.

Pathology 31. The correct answer is D. Patients who have This lesion should be easily recognized and a history of breast cancer (and prostate cancer identified based on the patient’s description and in males) are often placed on IV bisphospho- timeline (i.e., onset soon after beginning antibi- nate therapy to help prevent metastatic disease otic therapy). A cytologic smear may be per- to bone. It is known that IV bisphosphonates can formed to confirm the diagnosis. Biopsy is not cause delayed healing and osteonecrosis, espe- indicated unless the lesion does not resolve fol- cially in the mandible. This should be a ma- lowing administration of antifungal agents. jor consideration in how this patient should be Sudden onset of symptoms and clinical signs treated. should never be dismissed. At most, 2 weeks is Although osteoradionecrosis does occur in an acceptable time frame to follow-up on a pa- the jawbones, it is unlikely that this patient will tient. In this case, the patient was symptomatic be affected by it. First off, the scenario did not and should not be dismissed without appropriate even mention that she had radiation therapy. If therapy or a plan. she did, her mandible would most likely not be in the field of radiation. Add this to the fact that 33. The correct answer is D. This is a classic pre- her tooth problem was on the contralateral side sentation of cleidocranial dysplasia. It is caused makes this a relatively low possibility. by a defect in the CBFA1 gene on chromosome Although drug-related xerostomia is a prob- 6p21. These patients often have a large number lem on many different levels (patient comfort, of supernumerary teeth. caries risk, candidiasis, etc.), it falls low on the Multiple osteomas are seen in Gardner syn- list of concerns regarding the actual procedure drome. It is an autosomal dominant syndrome of the extraction of the tooth. that is part of the spectrum of diseases includ- This is most likely unrelated to breast cancer ing familial colorectal polyposis. Patients present and should have little to no impact on the ac- with multiple osteomas of the jaws 90% of the tual extraction of the affected tooth. The cause time and can demonstrate benign cysts and tu- should be determined (is she a diabetic who took mors of the skin. Supernumerary teeth, odon- her insulin but did not eat?) and remedied. Once tomas, and impacted teeth are also common the reason is determined, she can have the tooth findings. Of particular importance is the pre- extracted. In an emergency situation, a patient in sentation of colonic polyps that ultimately trans- a hypoglycemic state can be given oral glycemic form into adenocarcinoma (usually before the agents to raise blood-glucose levels. age of 30). Females also demonstrate a 100-fold increase of thyroid carcinoma. It is because of 32. The correct answer is B. Young, healthy pa- the malignant manifestations of this syndrome tients usually do not get candida infections. They that patients need to be referred to their primary are, however, susceptible to such infections after care physician for evaluation of colon and thy- being on a broad-spectrum antibiotic. The an- roid gland. tibiotic wipes out normal oral bacterial flora and Gorlin syndrome (also known as nevoid allows the native candida organisms (present in basal cell carcinoma syndrome) is an autoso- approximately 70% of the population) to pro- mal dominant syndrome caused by a mutation liferate. This patient should be given a topical in the PTCH tumor suppressor gene on chromo- antifungal to use until antibiotic therapy is com- some 9. It is characterized by multiple basal cell pleted. If the infection returns with no apparent carcinomas, multiple OKCs, palmar/plantar pit- inciting factors, immunosuppression should be ting, calcification of the falx cerebri, and rib ab- ruled out. normalities. Steroids should not be prescribed to patients Multiple giant cell lesions can be seen in both who have infections. Steroids suppress the host cherubism and in hyperparathyroidism. Neither response to the infection and allow the organ- Answers: 31-37 215 Pathology

of these entities have clinical manifestations as and angiofibromas of the skin. These symptoms mentioned earlier. do not match those of the patient mentioned earlier. 34. The correct answer is A. Attrition is loss of Sturge–Weber angiomatosis (encephalotri- tooth structure due to tooth-to-tooth contact dur- geminal angiomatosis) is a nonhereditary devel- ing contact and mastication. Bruxing, premature opmental condition characterized by vascular contacts, and poor-quality enamel can acceler- proliferations involving the brain and face. Pa- ate the damage. tients are born with a capillary vascular malfor- Abrasion is the pathologic wearing away of mation of the face known as port wine stain. tooth structure secondary to the mechanical action of an external agent. Vigorous tooth 36. The correct answer is C. CREST syndrome brushing with abrasive toothpastes is the most is rare and may represent a variant of systemic common cause. Patterns can vary depending on sclerosis. Patients are affected with the follow- cause. ing: Calcinosis cutis, Raynaud’s phenomenon, Abfraction is loss of tooth structure secondary Esophageal dysfunction, Sclerodactyly, and to occlusal stress that creates repeated tooth flex- Telangiectasia. This patient exhibited four of ure and results in failure of enamel and dentin these symptoms (esophageal dysfunction is of- at a point away from the point of loading. This is tentimes only detected by barium radiological most frequently seen in the cervical area of teeth studies). and appears as a wedge-shaped defect. Ehlers–Danlos syndromes (there are at least Erosion is loss of tooth structure caused by a 10) are inherited connective tissue disorders. nonbacterial chemical process. This can include The symptoms are related to production of acidic drinks, certain medications, and reflux of abnormal collagen, the main structural com- gastric secretions both involuntarily (hiatal her- ponent of connective tissue. The symptoms in- nia) and voluntarily (bulimia). Clinically, tooth clude hyperelasticity of skin, hypermobility of loss does not correlate with functional wear pat- joints, papyraceous scarring, and easy bruising. terns. Posterior teeth tend to lose occlusal tooth The hypermobility of the skin seen in Ehlers– structure and they can appear cupped out, with Danlos syndrome is opposite to the tightness of the center being dentin and the edges enamel. the skin in the patient mentioned earlier. The palatal and lingual surfaces of anterior teeth Tuberous sclerosis is a syndrome character- may also be affected. ized by mental retardation, seizure disorders, and angiofibromas of the skin. These symptoms 35. The correct answer is C. Ehlers–Danlos do not match those of the patient mentioned syndromes (there are at least 10) are inherited earlier. connective tissue disorders. The symptoms are Osler–Weber–Rendu syndrome [hereditary related to production of abnormal collagen, the hemorrhagic telangiectasia (HHT)] is an au- main structural component of connective tissue. tosomal dominant syndrome with clinical fea- The symptoms this patient exhibits are consis- tures including frequent episodes of epistaxis tent with the classical type, which is inherited in and multiple telangiectasias in the nasal, oral, an autosomal dominant fashion. and oropharyngeal mucosa. Patients also have Osler–Weber–Rendu syndrome (hereditary telangiectasias on their hands and feet, gastroin- hemorrhagic telangiectasia) is an autosomal testinal, genitourinary, and conjunctival mu- dominant syndrome with clinical features inclu- cosa. Although the patient mentioned earlier ding frequent episodes of epistaxis and multiple does, indeed, have telangiectasias, none of her telangiectasias in the nasal, oral, and oropharyn- other symptoms fit with HHT. geal mucosa. Patients also have telangiectasias on their hands and feet, gastrointestinal, geni- 37. The correct answer is D. Ramsay Hunt syn- tourinary, and conjunctival mucosa. drome classically presents with the symptoms Tuberous sclerosis is a syndrome character- mentioned earlier. Herpes zoster is caused by ized by mental retardation, seizure disorders, 216 Chapter 9: Pathology

the human herpesvirus 3 (HHV-3, or varicella- carcinoma consists of islands or cords of neo- zoster virus). plastic epithelial cells invading the connective Exanthema subitum (roseola) is caused by tissue. Transition from dysplastic epithelium is

Pathology human herpesvirus 6 (HHV-6). It is usually con- often seen. Invasive lesions can be seen infil- tracted at a young age and is asymptomatic. Oc- trating muscle, gland, and vasculature. The his- casionally, an erythematous macular eruption tology mentioned earlier does not work with a with slightly elevated papules may appear. diagnosis of squamous cell carcinoma. Chronic fatigue syndrome is a controversial Although leukoedema may, in some cases, symptom complex thought to be caused by hu- clinically resemble LP, the histology given is that man herpesvirus 4 (HHV-4, or Epstein–Barr of LP. virus). Patients report nonspecific symptoms of chronic fatigue, fever, pharyngitis, myalgia, and 39. The correct answer is A. CEOT (Pindborg tu- headaches. These symptoms are inconsistent mor) is a rare odontogenic neoplasm. It presents with the findings in the patient mentioned ear- as mentioned earlier and may also involve an lier. impacted tooth. Although the lesion may be to- Zoster sine herpete (zoster without rash) is tally radiolucent, scattered calcifications are fre- when a patient experiences a recurrence of a quently seen. This is the only diagnostic choice zoster outbreak but shows no vesiculation of the that may contain calcifications. skin or mucosa. Affected patients experience se- OKCs are developmental odontogenic cysts vere pain and an abrupt onset. The patient men- that exhibit a specific histology. They are impor- tioned earlier has visible healing vesicles and tant because they can be aggressive in nature and thus would not be diagnosed as zoster sine her- have a high rate of recurrence. They can occur pete. anywhere but are most common in the posterior jaws, with the mandible favored over the maxilla. 38. The correct answer is B. This histological de- They can be associated with impacted teeth (but scription is classic for lichen planus (LP). LP do not have to be) and do not have a radiopaque is an autoimmune, dermatologic entity that can component to them. Multiple OKCs are seen in present in the mouth. The two most common nevoid basal cell carcinoma syndrome (Gorlin forms in the mouth are the reticular form (which syndrome). the patient mentioned earlier has) and the ero- Dentigerous cysts, by definition, are asso- sive form. The erosive form is commonly found ciated with the crown of an impacted tooth, on the anterior gingiva, but may be seen in other usually mandibular third molars. Although any patients. These lesions do not need to be treated tooth can be affected, frequent sites (other than if asymptomatic, but should be followed up on mandibular third molars) include maxillary ca- every 6 months. Symptomatic lesions can be nines, maxillary third molars, and mandibular treated with corticosteroids. second premolars. Dentigerous cysts are rarely Leukoplakia is a clinical term only. It is a term seen involving unerupted deciduous teeth. used for a white lesion in the mouth that can- Odontogenic tumors and other types of cysts can not be rubbed off and has no diagnosis. Once a arise from the epithelium lining a dentigerous diagnosis is made (i.e., via histology), the term cyst. All of this patient’s molars were erupted (i.e., leukoplakia no longer applies. Note: The clin- he had no impacted teeth) and thus dentigerous ical term leukoplakia should not be confused cyst should be excluded as a potential diagnosis. with the entity of OHL, which is caused by hu- Also, calcifications are not seen in dentigerous man herpesvirus-4 and is most frequently seen cysts. in HIV+ patients. Ameloblastoma is by far the most common Squamous cell carcinoma of the buccal mu- odontogenic tumor. Its frequency equals the cosa, while it does occur, is quite rare. The fact combined total of all the other odontogenic tu- that this patient also has symmetrical, bilateral mors (excluding odontomas that some classify as lesions also puts it extremely low on the differ- hamartomas). The preferred site for ameloblas- ential diagnosis. The histology of squamous cell toma is the posterior mandible (approximately Answers: 38-42 217 Pathology

66% of total). Overall, mandibular lesions are Mumps (epidemic parotitis) is also caused by more frequent than maxillary lesions in their a paramyxovirus. Clinical features include sig- respective bony locations (posterior, middle, nificant salivary gland changes. These changes anterior). Ameloblastomas, however, do not include discomfort and swelling in the preauric- present as mixed radiolucent–radiopaque le- ular region extending down the posterior, infe- sions. It is for this reason that ameloblastoma rior border of the mandible. Another common should be excluded from the differential diag- finding in males is epididymoorchitis, which is nosis. rapid testicular swelling with pain and tender- ness. 40. The correct answer is C. Kaposi’s sarcoma (KS) Rubeola (measles) is caused by a paramyx- is a malignant vascular neoplasm that is divided ovirus. Because of the MMR vaccine, measles into four types. One of the types is the AIDS- incidence is low, but the clinical signs and symp- related type. This patient is HIV+ and has a toms still need to be recognized. The incubation CD4+ count below 200; this places him into the period is 10 to 12 days. Significant lymphoid hy- AIDS category. KS is associated with human her- perplasia is noted in addition to an erythematous, pesvirus 8 (HHV-8) and although any site may maculopapular rash. be affected, the palate and gingiva are the most Rubella (German measles) is an infection commonly involved. Clinically, it can appear as caused by a togavirus. Because of the MMR vac- a raised, dark-red enlargement of tissue. cine, rubella incidence is low, but the clinical Pleomorphic adenomas are the most com- signs and symptoms still need to be recognized. mon benign salivary gland neoplasm and they Lymphadenopathy is the most common clinical most commonly occur in the parotid. When they feature. An exanthematous rash of the face and do occur intraorally, they are most common on neck is also seen. the posterior hard palate and soft palate. The an- terior one-third of the hard palate does not con- 42. The correct answer is B. Pemphigus vulgaris tain salivary glands; therefore, this would be an is a rare autoimmune disease in which the inappropriate choice to include in a differential body produces antibodies against desmoglein diagnosis. 3 and desmoglein 1. These proteins are com- NS is thought to result from ischemic necro- ponents of desmosomes (intercellular adhesion sis of salivary glands. The anterior one-third of structures between epithelial cells). When these the hard palate does not contain salivary glands; desmosomes are disrupted, epithelial cells are therefore this would be an inappropriate choice no longer able to bond to each other and a split to include in a differential diagnosis. develops within the epithelium (causing a blis- Mucoepidermoid carcinomas are the most ter). The average age is 50 (this patient was a common malignant salivary gland tumor. The bit younger) and there is no sex predilection. anterior one-third of the hard palate does not Oral lesions are distributed haphazardly and ap- contain any salivary glands; therefore, this would pear as superficial, ragged erosions and ulcer- be an inappropriate choice to include in a dif- ations. Oral lesions usually precede cutaneous ferential diagnosis. lesions, sometimes by a year or more. Patients with pemphigus demonstrate a positive Nikolsky 41. The correct answer is A. The isolation of the sign (induction of bulla by firm, lateral pressure). organism Bartonella henselae gives this diagno- Diagnosis can be made histopathologically by sis away. Cat-scratch disease is a bacterial infec- seeing suprabasilar cleavage of epithelial cells, tion that begins in the skin but spreads to the but is confirmed by direct immunofluorescence. lymph nodes. It is the most common cause of Treatment consists mainly of immunosuppres- chronic regional lymphadenopathy in children. sive agents (corticosteroids, azathioprine, etc.). The lymphadenopathy can occur after the cu- Left untreated, pemphigus vulgaris is fatal. taneous lesion has healed. A careful history can BMMP, while in the same differential for the often aid in the diagnosis. intraoral lesions, typically does not present with skin lesions. The ulcers are, instead, limited to 218 Chapter 9: Pathology

the mucous membranes (oral mucous, eyes). a vascular proliferation resembling granulation Desquamative gingivitis is a common finding in tissue. Endothelium-lined vessels are also seen patients with BMMP. This patient had multi- and are arranged in a lobular pattern. There is

Pathology ple skin lesions, which are not seen in classic a mixed inflammatory infiltrate of neutrophils, BMMP. plasma cells, and lymphocytes. Erythema multiforme (EM) tends to have an explosive onset and is self-limiting. Charac- 44. The correct answer is A. This is a classic pre- teristic appearances of EM include crusting of sentation both clinically and histologically of the lips and ulceration of the periphery of the ameloblastoma. Ameloblastoma is by far the tongue. Targetoid skin lesions, though pathog- most common odontogenic tumor. Its frequency nomonic for EM, are rarely seen. This patient equals the combined total of all the other odon- had longstanding lesions, no crusting of his lips, togenic tumors (excluding odontomas that some and skin lesions that were not targetoid in ap- classify as hamartomas). The preferred site for pearance. ameloblastoma is the posterior mandible (ap- Although erosive LP can present in a similar proximately 66% of total). Overall, mandibular fashion to the patient above, the histopathologic lesions are more frequent than maxillary lesions picture described earlier is not that of LP. LP in their respective bony locations (posterior, mid- is a dermatologic disease and can thus have cu- dle, anterior). taneous involvement. The cutaneous manifesta- OKCs are developmental cysts that have a tions of LP are not vesicular in nature, but rather high rate of recurrence. They can appear uni- a flat, itchy rash. or multilocular and have a predilection for the posterior mandible. They can be found as one of 43. The correct answer is B. The histology de- the oral manifestations of nevoid basal cell carci- scribed earlier is that of a peripheral ossifying fi- noma syndrome. Lesion size ranges from quite broma. It occurs exclusively on the gingiva and small to large enough to fill entire ramus. OKCs presents as a nodular mass usually emanating tend to grow in an anterior–posterior direction from the interdental papilla. They can be mis- rather than expand in a buccal–lingual direc- taken for other lesions clinically and thus should tion. The histology presented in this case does be included in the differential with pyogenic not fit and thus OKC is an incorrect diagnosis. granuloma (PG), peripheral giant cell granu- Odontogenic myxomas are rare tumors of loma (PGCG), and irritation fibroma. Excision odontogenic ectomesenchyme. The average age is the treatment of choice. A small recurrence for their occurrence is 25 and they may oc- rate has been reported (8–16%). cur anywhere in the jaws. Again, the histology Mucoceles do not occur on the gingiva, as the present is classic for ameloblastoma and thus gingiva contains no salivary glands. In addition, odontogenic myxoma can be ruled out. the histology given is not that of a mucocele. Central giant cell granulomas can occur any- Clinically, PGCG is an appropriate choice where in the jaws and be uni- or multilocular. to include in the differential. The reason that it Special care should be taken in evaluating the is not is based on the histology given. The his- patient for hyperparathyroidism if they have a tology of a PGCG consists of a proliferation of central giant cell lesion. The histology men- multinucleated giant cells set in a background of tioned earlier is not that of a central giant cell plump ovoid and spindle-shaped mesenchymal granuloma. cells. Hemorrhage is often seen within these le- sions. PGCGs occur exclusively on the gingiva. 45. The correct answer is A. Hemophilia is an Treatment is excision and up to 10% of lesions x-linked recessive disorder. The only scenarios have been reported to recur. in which a male hemophiliac may have a child PG is also an appropriate choice to include with hemophilia are if the mother is a carrier in a clinical differential. PGs can occur any- (Xx) or a hemophiliac (xx) herself. If the mother where on the body. Intraorally, the gingiva is the is a carrier and the father is affected, 50% of the most frequent site. Histologically, PGs consist of male offspring will have hemophilia (XY, xY). Answers: 43-48 219 Pathology

Fifty percent of the female offspring in this sit- 47. The correct answer is C. Granulocytic sarcoma uation will either have hemophilia (xx) or be a (chloroma, extramedullary myeloid tumor) is carrier of the trait (Xx). The only way a male off- the result of diffuse infiltration of the gingiva spring can have hemophilia is if his mother is af- by leukemic cells. It is more common with fected (xx; in which case he has a 100% chance the myelomonocytic type of leukemia (AML, of having hemophilia) or carries the trait (Xx; CML). Patients who present with this should in which case he has a 50% chance of having be referred back to their primary physician for hemophilia). Male hemophiliacs do not pass an further management of a known condition or affected allele to any male offspring. In the case for treatment of a previously unknown disease presented earlier, the mother is neither affected process. nor a carrier and hence cannot produce a male Hodgkin’s lymphoma is a malignant lym- offspring with hemophilia. By the same token, phoproliferative disorder composed of Reed– since all female offspring will have a dominant Sternberg cells (bi- and multinucleated) in en- X allele from their mother and a recessive x al- larged lymph nodes. The most common sites lele from their affected father, 100% of female of initial presentation are the cervical and supr- offspring will be carriers. aclavicular lymph nodes (70–75% of cases). A bimodal pattern with respect to patient’s age is 46. The correct answer is D. Thyroglossal duct cyst also noted: between 15 and 35 years, and another is a developmental cyst that results from a persis- peak after age 50. tence of epithelium of the thyroglossal duct, an Multiple myeloma is a malignancy of plasma embryonic structure that connects the foramen cells that typically affects older patients (median cecum to the final position of the thyroid in the age between 60 and 70). It represents the most anterior neck. The thyroid gland begins devel- common hematologic malignancy among black opment in the ventral floor of the pharynx and persons in the United States. Bone pain is the descends to the anterior neck via the thyroglos- most common symptom. Punched-out radiolu- sal duct. This duct usually undergoes atrophy, cencies can be seen in the skull, jawbones, ver- but can persist and give rise to a cyst. This cyst tebrae, and other bones. Patients with multiple may occur anywhere along the tract. It is inti- myeloma can also show focal deposits of acel- mately associated with the hyoid bone and thus lular protein known as amyloid. These deposits the swelling moves with movement of the hyoid can be found in internal organs, as well as skin (i.e., upon swallowing or tongue protrusion). and mucosal surfaces. Histologically, it appears Cat-scratch disease is an infection caused by as an amorphous, glassy pink material. the organism Bartonella Henselae that begins in Ewing’s sarcoma is a malignancy of bone the skin but spreads to the lymph nodes. It is the composed of small, undifferentiated round cells most common cause of chronic regional lym- of uncertain lineage. Peak prevalence is in the phadenopathy in children. The lymphadenopa- second decade and the vast majority of those thy can occur after the cutaneous lesion has affected are white. Long bones, pelvis, and rib healed. A careful history can often aid in the are most affected. Pain is common and soft- diagnosis. It would be unusual for cat-scratch tissue invasion adjacent to affected bone can be disease present as a fluctuant swelling in the an- seen. Treatment consists of surgery, radiation, terior midline of the neck. and chemotherapy. Although this patient is only 15, it is not un- heard of for young patients to have metastatic 48. The correct answer is E. Ascher syndrome is disease. That being said, metastatic disease to a characterized by the following three features: neck node would be in the lateral neck and firm, double lip, blepharochalasis (edema of the up- not fluctuant. per eyelids), and nontoxic thyroid enlargement. Branchial cleft cysts are developmental cysts In general, no treatment is necessary. Occasion- of the lateral neck that develop from remnants ally, the drooping eyelids impair vision and re- of the branchial clefts. quire surgical correction. The double lip may be excised for esthetics or function. 220 Chapter 9: Pathology

Gardner syndrome is an autosomal dominant craniosynostosis (premature closing of the cra- syndrome that is part of the spectrum of diseases nial sutures). Patients typically have cranial mal- including familial colorectal polyposis. Patients formations that result in abnormal-shaped heads

Pathology present with multiple osteomas of the jaws 90% (tower skull, cloverleaf skull). Affected individ- of the time and can demonstrate benign cysts uals also have a hypoplastic midface and syn- and tumors of the skin. Supernumerary teeth, dactyly of the second, third, and fourth digits of odontomas, and impacted teeth are also com- the hand. Mental retardation is common. mon findings. Of particular importance is the presentation of colonic polyps that ultimately 49. The correct answer is D. Systemic antivirals are transform into adenocarcinoma (usually before indicated for patients with herpes zoster (HHV- the age of 30). Females also demonstrate a 100- 3), especially in elderly patients who have age- fold increase of thyroid carcinoma. It is because related immunosuppression. The treatment reg- of the malignant manifestations of this syndrome imen should also include palliative measures that patients need to be referred to their primary (analgesics) and antipyretics if the patient if care physician for evaluation of colon and thy- febrile. Zoster involving the tip of the nose war- roid gland. rants a referral to an ophthalmologist, since this Papillon–Lef`evre syndrome is an autosomal is most often a sign of involvement of the nasocil- recessive syndrome that is characterized by iary branch of the trigeminal nerve. This is turn, palmar–plantar keratosis and dramatically ad- can lead to ocular infection and potential blind- vanced periodontitis in both deciduous and ness. permanent dentitions. Steroids, be they systemic or topical, are con- Gorlin syndrome (also known as nevoid traindicated in viral infections. If administered, basal cell carcinoma syndrome) is an autoso- they will delay the host response and prolong the mal dominant syndrome caused by a mutation duration of disease and discomfort. in the PTCH tumor suppressor gene on chromo- Although administration of systemic antiviral some 9. It is characterized by multiple basal cell medication is indicated in herpes zoster infec- carcinomas, multiple OKCs, palmar/plantar pit- tions, involvement of the tip of the nose requires ting, calcification of the falx cerebri, and rib ab- mandatory referral to ophthalmology to make normalities. sure there is no ocular infection. Apert syndrome is a rare condition similar to Crouzon syndrome and is characterized by CHAPTER 10 Patient Management, Public Health, Ethics, and Biostatistics

221 222 Chapter 10: Patient Management, Public Health, Ethics, and Biostatistics

QUESTIONS

1. Which of the following behaviors would be con- extraction socket. The patient is eventually sidered unethical with regards to a physician– discharged from the hospital in stable condi- patient relationship? tion. D. Gross negligence or severe overtreatment A. Dating or engaging in a sexual relationship and deviation from standard of care, result- with one of your patients. ing to an injury or death of a patient B. Refusing to date or engage in any sexual re- lationship with your patients. 4. Which of the following situations illustrate un- C. Returning a call from one of your patients ethical behavior of a dental student? who called you to complain of pain from the tooth you worked on yesterday. A. Explaining all the risks and benefits of a pro- D. Requesting to drop off or transfer a pa- cedure to patients before obtaining their sig- tient from your treatment list to a different nature in an informed consent form for ex- provider due to his/her persistent multiple traction of teeth sexual advances directed toward you. B. Creating a root canal during an operative procedure to meet a clinical requirement 2. What is the golden rule that could be used to and graduate on time broadly define all ethical behaviors with regards C. Accepting a Christmas present from a patient to patient treatment? D. Refusing to offer free treatment to a low- income, cash-paying patient A. Be nice only to the patients who are nice to Patient Management, Public Health, Ethics, and Biostatistics you. 5. All of the following statements as regards to eth- B. Administer palatal injections to any patient ical framing are true except one: who annoys you, even though you are treat- ing mandibular teeth. A. Ethical framing can lead to conflicts, be- C. Physicians are expected to respect and treat cause different people have different frames patients as they would like to be treated, us- of reference based on their environments, ing the best of their knowledge and ability, religious or professional backgrounds, and excluding any modifiers or outside bias. moral values. D. Treat patients the way you would like to be B. Ethical framing means one’s perspective of treated except if they are noncompliant with ethics may be different from others. your instructions. C. Personal ego and financial interest are some of the many factors that influence ethical 3. Which of the following scenarios would be con- framing, which often leads to rationalization sidered a type of unethical behavior that might of an ethical decision with attempts to find justify an ethical board to recommend immedi- support for a predetermined conclusion. ate licensure suspension? D. There is usually one black-and-white answer to every ethical situation, which meets com- A. Mistakenly extracting the wrong tooth on a monly held ethical standards, and does not patient, but informing the patient immedi- lead to conflicts or different ethical interpre- ately afterward and offering an apology tations. B. Refusing to extract a wisdom tooth that is near the inferior alveolar nerve, but referring 6. If a patient of record has been out of town for the patient to an oral surgeon for evaluation 10 months and returns to your office to have a and further treatment temporary crown made by another dentist rece- C. Extracting a tooth on a patient taking mented, you may: Coumadin with an INR of 1.7, resulting in the patient’s admission to the hospital for A. Remove and recement the crown if there is 4 days to control excessive bleeding from the compensation prior to treatment. Questions: 1-15 223

B. Remove the crown and recement it with per- 11. Technical assault or battery and maligning a pa- manent cement. tient are forms of: C. Remove the crown and recement it with tem- A. Civil law porary cement. B. Contract law D. Remove the crown, recement it with tempo- C. Tort law rary cement, and refer the patient back to the D. Negligence dentist who made the crown. 12. As dental professionals, we are ethically bound 7. When laboratory procedures are required, the to guarantee patients:

dentist should prescribe treatment for HMO and Biostatistics and Ethics, Health,

private pay patients equally; all patients deserve A. Nothing Public Management, Patient the same quality of treatment. B. The appropriate standard of care C. Successful results A. First statement is true and second is false. D. Professionalism B. Both statements are true. C. First statement is false and second is true. 13. All of the following are correct in regard to the D. Both statements are false. patient records except which one? 8. If Mr. Smith calls the office saying he just moved A. Records are kept for 1 year. to town and needs a cleaning and dental pro- B. Records must include updated medical his- phylaxis, with whom should the appointment be tories. scheduled? C. Records must be recorded in ink or electron- ically. A. The hygienist D. Records include radiographs of the patient. B. The assistant so that the necessary radio- graphs can be taken 14. When treating a patient, whose value system is C. The dentist of the most importance? D. It does not matter with whom the patient is scheduled. A. The dentist B. The patient 9. The PREAMBLE in the ADA’s document on C. Both are of equal importance ETHICS states that our primary goal when treat- D. Value systems should not affect dental treat- ing patients should be to benefit: ment

A. The dental profession 15. If Mrs. Jones shares with you that she has been B. Mankind diagnosed with breast cancer what ethical prin- C. Ourselves ciple must you uphold? D. The patient E. All of the above A. Justice B. Confidentiality 10. If a dentist does not keep up with CE courses and C. Beneficence current trends or advances in dentistry, which D. Veracity of the following principles of ethics is being broken? A. Veracity B. Justice C. Beneficence D. Nonmaleficence E. Autonomy 224 Chapter 10: Patient Management, Public Health, Ethics, and Biostatistics

16. Your brother is in the office to have a deep B. Health Insurance Portability and Account- restoration placed on tooth #19. While you are ability Act of 1996 (HIPAA) waiting for the effect of the anesthetic, he states C. State Law that his back is still bothering him greatly and D. Joint Commission on Accreditation of that the only effective medication is Vicodin. He Healthcare Organizations (JCAHO) asks you to write this prescription for him. What do you do? 19. There are improvements in the following trends in oral health except for: A. Write the prescription for just 10 tablets. B. Give him a prescription for a month of Vi- A. Edentulism and periodontitis among seniors codin, to be taken as needed for pain. B. Prevalence of dental caries, tooth retention, C. Give him some samples of Vicodin that you and periodontitis among adults happen to have in the office. C. Prevalence of dental caries and dental D. Refer him back to his physician or orthope- sealants among youths and adolescents dist to obtain the prescription. D. Dental caries in primary teeth among chil- dren aged 2 to 5 years 17. You have taught your staff members how to mon- E. None of the above itor nitrous oxide analgesia; you are confident that they are all comfortable performing this, but 20. Which of the following statement best described your assistant (although registered) still has not the caries burden among children? taken the nitrous oxide certification course. A A. Approximately 80% of tooth decay is found patient presents with a problem for which ni- in approximately 25% of children. trous oxide analgesia is indicated. You get called

Patient Management, Public B. Tooth decay is evenly distributed among dif-

Health, Ethics, and Biostatistics from the room but not wanting to turn off the ferent racial and socioeconomic groups. nitrous and have to turn it back on again, you C. Approximately 90% of tooth decay is found tell the assistant to sit with the patient. She is in approximately 10% of children. instructed that if the patient has any deleterious D. None of the above effects, “lower the nitrous just a little.” When re- lating this to a fellow colleague, you are informed 21. What percentage of adults have tooth decay that this was not the correct thing to do; you dis- (treated or untreated)? agree since you were “only in the next room.” The two of you get into a discussion about this. A. Approximately 35% of adults aged 20 to 39 Is this an ethical issue or an ethical dilemma? years have coronal decay. B. Approximately 52% of adults aged 20 to 39 A. This is an ethical dilemma. years have coronal decay. B. This is an ethical issue. C. Approximately 87% of adults aged 20 to 39 C. This is not an ethical problem at all. Your years have coronal decay. actions were justified. D. All adults aged 20 to 39 years have coronal decay. 18. In your private office, you keep MSDS (medi- cal safety data sheets); you have one of your staff 22. What percentage of the U.S. population is members in charge of monitoring and updating plagued by moderate to severe periodontitis? them. At monthly meetings, you also have this person inform the staff of any recent OSHA or A. Approximately 1% to 20%, depending on the HIPAA directives. In this situation, MSDS pa- age groups perwork falls under the guidance of which of B. Approximately 20% to 40%, depending on the following agencies? the age groups C. Approximately 40% to 60%, depending on A. Occupational Safety and Health Administra- the age groups tion (OSHA) D. None of the above Questions: 16-31 225

23. What is the incidence of oral and pharyngeal iii. Average rate of hepatitis B virus (HBV) trans- cancer (i.e., new oral and pharyngeal cancer mission to susceptible healthcare workers af- cases) each year? ter percutaneous exposure to HBV-infected blood is 6% to 30% A. Approximately 10,000 new cases of oral and Which statements are correct? pharyngeal cancer each year B. Approximately 35,000 new cases of oral and A. Only i and iii are correct. pharyngeal cancer each year B. Only ii is correct. C. Approximately 50,000 new cases of oral and C. All are correct. pharyngeal cancer each year D. None is correct. elh tis n Biostatistics and Ethics, Health,

D. None of the above Public Management, Patient 28. The portion or percentage of individuals having 24. What is/are the measure(s) of central tendency a disease at a given time is: in statistical methods? A. Frequency A. Mean B. Incidence B. Median C. Prevalence C. Mode D. Mortality D. All of the above 29. Specificity is the proportion of truly diseased per- 25. Currently, nine states mandate the installation of sons who are identified as being diseased. False- amalgam separators in the dental office. Which positive result identifies individuals as having a of the following is/are among the nine states? disease, but in reality they do not. A. California A. Both statements are true. B. Massachusetts B. Both statements are false. C. Alaska C. The first statement is true, the second state- D. South Carolina ment is false. E. A and B D. The first statement is false, the second state- ment is true. 26. What is the most effective community-based in- tervention to control dental decay? 30. After a needlestick or cut exposure, the pathogen with the highest risk of infection is: A. School-based dental sealant programs B. Community health centers with dental A. HBV clinics B. HIV C. Fluoridation of drinking water C. HCV D. Introduction of sugarless chewing gums D. Mycobacterium tuberculosis

27. Up to 800,000 percutaneous injuries occur 31. Latex allergies can result in which of the follow- annually among all U.S. healthcare workers. ing reactions: After percutaneous injury with a contaminated i. Type I sharp instrument, the risks of infection are as ii. Type II follows: iii. Type III i. Average rate of anti–hepatitis C virus (HCV) iv. Type IV seroconversion after percutaneous exposure A. ii and iii to HCV-infected blood is 1.8% (range, B. ii and iv 0–7%) C. i and ii ii. Average transmission rate is 0.3% after per- D. i and iv cutaneous exposure to human immunode- ficiency virus (HIV)-infected blood. 226 Chapter 10: Patient Management, Public Health, Ethics, and Biostatistics

32. The mechanism of action of povidone iodine is: 35. You performed a crown prep on tooth #28 on this gentleman with moderate to poor oral hy- A. Irreversible inactivation of deoxyribonucleic giene. Now, he is back for cementation of his acid and proteins via alkylation PFM crown. You decided to use Glass Ionomer B. Heat inactivation of critical enzymes Cement. Please choose the INCORRECT state- C. Pore formation within the bacterial cell wall, ment concerning Glass Ionomer Cement: resulting in cellular leakage D. Oxidization of free sulfhydryl groups A. Glass ionomer has a high modulus of elas- ticity. 33. Which of the following statements is correct con- B. Fluoride is released from the glass powder at cerning sterilization of dental instruments? the time of mixing and lies free within the A. Gentle washing with bleaching and soaking matrix. for 10 minutes in the bleach is required be- C. Fluoride released has no effect on cariogenic fore putting instruments in the autoclave. bacteria but will harden only the tooth’s B. ADA recommends biological monitors being enamel. used once a month D. Glass ionomer is known for little shrinkage C. Biological indicators contain spores of a heat- and good marginal seal. resistant bacterium Eikenella corrodens. 36. A 6-year-old healthy boy presented to your of- D. Chemical indicators are sterilizer specific. fice for the first time. His mother said he has 34. You just bought a sterilization machine from a never been to a dentist and has never had a ra- retiring dentist for your new office, and then your diograph taken. Intraoral examination revealed that his permanent molars are erupted. The pa-

Patient Management, Public staff tells you that the machine just “failed” the

Health, Ethics, and Biostatistics test. What measures should you take right after tient has no clinically evident dental problems. taking sterilization out of service? What kind of screening radiographic examina- tion will you perform? i. Review the sterilization process being fol- lowed in the office to rule out operator error. i. Occlusal radiographs ii. Check electrical outputs from the wall, since ii. Periapical radiographs this may alter the performance of the steril- iii. Panorex radiographs ization machine. iv. Posterior bitewing radiographs iii. Biological indicator tests should be con- A. i and ii ducted during three consecutive empty- B. i and iii chamber sterilization cycles. C. ii and iii iv. After three consecutive empty-chamber test- D. iii and iv ing, another three consecutive tests should E. iii only be run using only chemical indicators before resterilizing instruments. A. i and ii B. i and iii C. i, ii, and iii D. ii, iii, and iv E. ii and iv Answers: 1-8 227

ANSWERS

1. The correct answer is A. It is unprofessional, 4. The correct answer is B. Despite the pressure unethical, and morally inappropriate for a physi- to fulfill all requirements prior to graduation, cian to abuse his/her position of public trust and dental students are held to the highest moral engage in any form of sexual relationship with and professional standard. So intentionally con- one of his/her patients. A physician’s involve- verting a class I amalgam restoration into a root ment with a patient sexually will compromise canal therapy on a patient to meet the gradua- his/her ability to offer good services to the pa- tion requirement is unethical. It is appropriate elh tis n Biostatistics and Ethics, Health,

tient, thereby creating an ineffective and abnor- for a dental student to accept a gift from a pa- Public Management, Patient mal physician–patient relationship. Physicians tient, as long as it is not in exchange to providing should carefully and tactfully transfer a patient a free treatment to the patient, and the practice in their care to a different provider, if multiple is allowed by the dental school. Dental students attempts to redirect their sexual gestures away are expected to explain all the risks, benefits, al- from the physician failed. Choices B and C are ternatives, and complications to all patients and completely ethical and appropriate. obtain a signed informed consent form prior to any dental procedure, including dental extrac- 2. The correct answer is C. Physicians are ex- tions. pected to treat every patient with courtesy and respect, using the best of their knowledge with- 5. The correct answer is D. There is usually NO out any prejudice or bias. Partial or total devia- black-and-white answer to an ethical situation tion from this golden rule would be considered that meets commonly held ethical standards unethical. All patients should be treated with re- without conflicts or different ethical interpreta- spect irrespective of whether they are nice or not, tions. Ethical framing means one’s perspective or noncompliant with their medications. Inflict- of ethics based on the frames of reference, envi- ing unnecessary pain to patient by administering ronment, religious or professional background, palatal injections while extracting mandibular moral values, personal ego, or financial interest, teeth is unethical and unprofessional. which often leads to rationalization of an ethi- cal decision with attempts to find support for a 3. The correct answer is D. Gross negligence re- predetermined conclusion. sulting in severe injury or patient’s death is a gross deviation for standard of care and might 6. The correct answer is D. It would be unethical lead to licensure suspension or withdrawal. The to not treat the patient but the sole responsibility appropriate thing to do if the wrong tooth is ex- should lie with the dentist who initially made tracted on a patient is to inform your supervisor the crown. or attending, and the patient should be informed immediately and an apology offered. Options A 7. The correct answer is B. Both HMO patients and C are not the best expected outcomes, but and private paying patients should receive the they would not be considered a type of unethi- same quality of care and treatment. The dental cal behavior that might justify an ethical board profession holds a special position of trust with to recommend immediate licensure suspension. society. The ADA Code of Ethical Principles It is appropriate to refer a patient to a different is a written expression of this “contract.” Note: provider with better experience or training to Dentist’s ethical considerations often exceed le- manage a patient with a problem that is beyond gal duties and business obligations, factors that your scope of practice. Treating a patient with a do not excuse them from putting the patient’s problem beyond your level of training, leading to welfare first. an unexpected outcome or harm to the patient, is considered malpractice. 8. The correct answer is C. The dentist should see the patient first to “diagnose” the condition 228 Chapter 10: Patient Management, Public Health, Ethics, and Biostatistics

and dental needs of the patient. The hygienist 12. The correct answer is B. We must meet the min- is not allowed to diagnose. Therefore, if the pa- imum standards of care when treating patients. tient requires further treatment following a “pro- Guaranteeing nothing implies a lack of interest phy,” the dentist must be consulted. The assistant in patient welfare. We are obligated to meet the cannot perform a “prophy” on a patient in most minimum standards of care. No one can pre- states, and the radiographs would need to be pre- dict, promise, or guarantee successful results, scribed by the dentist. It does make a difference but one can state risks and benefits for a vari- who sees the patient first, and that must be the ety of treatments. We are not obligated to guar- physician to diagnose and prescribe treatment. antee professionalism, but we should show the patient respect and should act in a professional 9. The correct answer is D. The patient’s needs manner. come first, and we must offer the patient a choice of the appropriate treatments available. All risks, 13. The correct answer is A. Records must be kept benefits, alternatives, and complications of the for as long as the individual State Dental Practice possible treatment options must be explained to Act dictates or the length of the statute of limita- the patient as part of the informed consent pro- tions within that state. Treatment records must cess. The primary goal in treating patients should include updated medical histories and radio- have nothing to do with the dental profession or graphs of patients. They must also be recorded mankind but the patients themselves. We should in ink (not pencil) and electronically. not treat patients to benefit ourselves since the patient’s needs come first. 14. The correct answer is B. Patients’ value system is what is of the most importance. We should not

Patient Management, Public 10. The correct answer is D. Nonmaleficence judge patients by their values. The only value sys- Health, Ethics, and Biostatistics means not causing harm to be inflicted on any- tem that matters is that of patients’. The practi- one. By taking CE courses, you are providing tioners’ values might be very different from those the patient with the choices of the most effective of patients but must be set aside in favor of pa- and current available treatment. By keeping up tients. with current advances in dentistry, one will not cause undue harm to the patient. This principle 15. The correct answer is B. Regardless of how you is part of the Hippocratic Oath (“Primum non feel, any information shared with you profession- nocere”—“First, do no harm”). Veracity is truth- ally by patients must be kept confidential. Justice fulness. Justice is treating patients fairly. Benef- involves the tenet of treating individuals fairly; icence has to do with the benefit to the patient giving patients their due or what is owed them. by meeting their particular needs. Autonomy has The principle of beneficence can be illustrated to do with the final decision being made by the with the statement “the action is moral if it is patient. good and helps to enhance patient welfare.” Ve- racity involves truthfulness or speaking the truth. 11. The correct answer is C. Tort law is a form of civil law that involves a civil wrong or injury 16. The correct answer is D. A dentist may prescribe to another person, whereas civil law involves a a medication only for a dentally related problem. crime against a person with actions that cause harm to that individual. The civil law suit is filed 17. The correct answer is B. An auxiliary must have with a private attorney. Contract law is a form a nitrous oxide certification course to monitor of civil law that involves a breach of contract. or discontinue nitrous oxide. If there is a state Negligence is synonymous with malpractice; it or federal statute mandating something, it then is carelessness without the intent to harm a pa- becomes an ethical issue. If there is no state or tient. Negligence occurs when the appropriate federal statute mandating something, then it is standard of care is NOT met and some damage just an ethical dilemma with no right or wrong results. answer. Answers: 9-29 229

18. The correct answer is A. Occupational Safety estimated that there will be 35,720 (25,240 men and Health Administration (OSHA) has to do and 10,480 women) newly diagnosed cases of with infection control and the Health Insur- oral and pharyngeal cancer and 7,600 deaths ance Portability and Accountability Act of 1996 from it. (HIPAA) has to do with privacy issues. The OSHA is an agency of the U.S. Department 24. The correct answer is D. In biostatistics, mean is of Labor. State Law is the Dental Practice Act the average, median is the statistic that separates governing the practice of licensed dental pro- the upper half of the population from the lower fessionals in each state. The Joint Commission half, and mode is the statistic that is of the highest

(JCAHO) is a private, not-for-profit organization frequency. Biostatistics and Ethics, Health, that operates accreditation programs for hospi- Public Management, Patient tals and healthcare organizations. It is not a re- 25. The correct answer is B. The nine U.S. states quirement that all private dental offices undergo that have mercury safety laws are the six New evaluation by the JCAHO; however, it is a re- England states (Connecticut, Massachusetts, quirement that all dental offices be OSHA com- Maine, New Hampshire, Rhode Island, and Ver- pliant. mont) and New York, New Jersey, and Oregon. Some other states have local mandates or are in 19. The correct answer is D. These results for trends the process of phasing in statewide mandates. in oral health in the United States between 1988 to 1994 and 1999 to 2004 are from the Na- 26. The correct answer is C. Community water flu- tional Health and Nutrition Examination Sur- oridation is cited as one of the top 10 public vey (NHANES), which showed an increase in health achievements of the 20th century. dental caries in primary teeth among 2- to 5- year-olds. 27. The correct answer is C. This information on infection control/percutaneous injuries was ob- 20. The correct answer is A. The differential caries tained from the National Institute for Occupa- burden among children is such that 80% of den- tional Safety and Health. tal caries are borne by 25% of persons younger than 19 years. 28. The correct answer is C. Prevalence is the por- tion or percentage of individuals having a dis- 21. The correct answer is C. This information about ease at a given time. Frequency is the number of the prevalence of decay among young adults individuals having a disease at a given time. Inci- (aged 20–39 years) was obtained from NHANES dence is the rate of new cases of disease per time. 1999 to 2002. Mortality is the rate of deaths resulting from a specific disease. 22. The correct answer is A. This information re- garding the prevalence of moderate to severe pe- 29. The correct answer is D. The first statement is riodontitis was obtained from NHANES 1999 to false, the second statement is true. Specificity is 2004. Moderate periodontal disease is defined the proportion of truly nondiseased persons who as two or more interproximal sites with 4 mm are identified as such. In a false-positive result, or more clinical attachment loss (CAL), or two the individual does not have the disease but is or more interproximal sites with 5 mm or more identified as having the disease. A true-positive probing pocket depth (PPD). Severe disease is result describes a situation where the individ- defined as two or more interproximal sites with ual has the disease and is correctly identified as 6 mm or more CAL and one or more interprox- such. A false-negative result is when the individ- imal sites with 5 mm or more PPD. ual has the disease and is incorrectly identified as not having the disease. In a true-negative result, 23. The correct answer is B. According to the the individual does not have the disease and is National Cancer Institute’s Surveillance Epi- correctly identified as such. demiology and End Results, in 2009, it was 230 Chapter 10: Patient Management, Public Health, Ethics, and Biostatistics

30. The correct answer is A. The average risk for If the chemical indicator does not change color infection after needle stick or cut exposure is or the spore test result is positive, the following 6% to 30% for hepatitis B virus, 1.8% for hep- steps are recommended: atitis C virus, and 0.3% for human immun- odeficiency virus. Mycobacterium tuberculosis 1. Take the sterilizer out of service. airborne droplet nuclei are generated when in- 2. Review the sterilization process being fol- fected individuals sneeze, cough, and speak. lowed in the office to rule out operator error as the cause of failure. 31. The correct answer is D. Natural rubber latex 3. Correct any identified procedural problems, proteins may result in a type I (immediate) hy- and retest the sterilizer using biological, me- persensitivity reaction. Common presentations chanical, and chemical indicators. include urticaria, rhinorrhea, itchy eyes, and a 4. If the biological indicator test is positive, or the burning sensation at the point of contact. Severe mechanical or chemical test results indicate reactions may result in anaphylactic shock and failure, the sterilizer should not be used until death. Latex allergies can also result in a type IV the reason for failure has been identified and (delayed) hypersensitivity reaction as in allergic corrected. contact dermatitis. These reactions are usually If no procedural errors are identified or fail- localized to the contact area and occur over a ures persist after procedural errors are corrected, 12- to 48-hour period. the sterilizer should not be used until the rea- son for failure has been identified and cor- 32. The correct answer is C. All answers describe rected. a legitimate antimicrobial mechanism of ac- Before the sterilizer can be returned to Patient Management, Public tion. “Irreversible inactivation of deoxyribonu-

Health, Ethics, and Biostatistics service, negative results should be returned cleic acid and proteins via alkylation” describes for biological indicator tests conducted during the mechanism of action of ethylene oxide gas. three consecutive empty-chamber sterilization “Heat inactivation of critical enzymes” describes cycles to ensure that the problem has been the mechanism of action of steam autoclave, dry corrected. heat, rapid heat transfer, and unsaturated chemi- To the extent possible, reprocess all instru- cal vapor. “Oxidization of free sulfhydryl groups” ments that were sterilized since the last negative describes chlorine compounds (hypochlorite). spore test. Povidone iodine causes pore formation within Record the positive test results and all actions the bacterial cell wall, resulting in cellular taken to ensure proper functioning of the steril- leakage. izer in the monitoring log.”

33. The correct answer is D. According to ADA 35. The correct answer is C. According to Qin guideline on sterilization, indicator tapes are et al.’s, “Fluoride Inhibition of Enolase: Crystal sterilizer specific (i.e., tapes for steam steriliz- Structure and Thermodynamics,” and van Lov- ers cannot be used to test chemical vapor ster- eren’s, “The Antimicrobial Action of Fluoride ilizers). The ADA also recommends the weekly and Its Role in Caries Inhibition”, fluoride does use of biological indicators. Eikenella corrodens have bacteriocidal/bacteriostatic effects on oral is a slow-growing, facultative, anaerobic, gram- bacteria. negative bacillus that is part of the normal oral Glass ionomer is described as having the fol- flora and is found in dental plaque. Biological lowing properties: indicators contain spores of a heat-resistant bac- terium Geobacillus stearothermophilus. Advantages Inherent adhesion to tooth structure 34. The correct answer is B. According to the ADA High retention rate guidelines, “What to do when results indicate Little shrinkage and good marginal seal sterilization failure: Fluoride release and hence caries inhibition Biocompatible Answers: 30-36 231

Minimal cavity preparation required, hence Not inherently radiopaque although addition easy to use on children and suitable for of radiodense additives such barium can use even in absence of skilled dental man- alter radiodensity power and facilities Less aesthetic than composite Disadvantages Brittle 36. The correct answer is D. In a child with a transi- Soluble tional dentition, after eruption of the first perma- Abrasive nent tooth, the radiographic examination should Water sensitive during setting phase be individualized and should consist of poste-

Some products release less fluoride than con- rior bitewings and panoramic examination, or Biostatistics and Ethics, Health, ventional GIC posterior bitewings and selected periapical ra- Public Management, Patient diographs. This page intentionally left blank INDEX

Note: Page locators followed by f indicates figure.

A ANB, 109, 126 Autologous bone, 66, 90 Anemia, 207 Automated external defibrillator (AED), 11 Abfraction, 215 Angioedema, 23 Avulsed tooth Abrasion, 43, 215 Angiotensin-converting enzyme (ACE), storage medium for, 138–139, 143, Abscesses, 54, 71–72 20 151, 160 ACE inhibitor, 8, 9, 10, 23, 24 Angiotensin II receptor antagonists, 20 treatment for, 56, 74, 136, 138, 140, 142 Acetaldehyde, 16 Anode, 186 Axonotmesis, 80 Acetaminophen (APAP), 4–5, 8, 18, 23, ANS, 106, 124 68, 91 Antacids, 11 B Acrodynia, 209 Anterior crossbite, 101, 120 Acute apical periodontitis, 146, 153 Band and loop, 130f, 165, 169 Acute necrotizing ulcerative gingivitis, 136, Anterior posterior palatal bar connector, use of, 105, 123 140–141 35, 49 Bands, use of, 104, 109, 112, 122, ADA Code of Ethical Principles, 223, 227, Antibiotic prophylaxis, cardiac conditions 126, 129 228 and, 66, 87–88 Basion, 106, 124 Addition silicone, 29, 43 Antimicrobial therapies, for Beckwith–Wiedemann syndrome, 77 Adenoid cystic carcinomas, 194, 207 high-caries-exposed patients, Bell’s palsy, 60, 79 Adenomatoid odontogenic tumors (AOTs), 137, 142 Benign mucous membrane pemphigoid 203 Antimuscarinic drugs, 11 (BMMP), 202 Aerobic bacteria, 71 Antipseudomonal penicillins, 24 Benzodiazepines, 21, 22, 57, 75 Airway, establishment of, 57, 76 Apert syndrome, 58, 77, 210, 212, 220 Benzonatate, 21 Albuterol, 10, 21 Apexogenesis, 137, 142 Beta-2 adrenergic agonist, 10 Alginate, methods for decreasing setting time Apical constriction, 147, 155 Beta adrenergic receptor blocker, of, 30, 43 Apical force, usage in extraction process, 10–12 All-ceramic crowns, preparation of, 28, 42 58, 77 Bevels, 42 Allergic reactions, to local anesthetics, Applegate’s rules, 33, 46 Bilateral class I malocclusion, correction of, 6, 22 Appositional bone growth, 109, 126, 132 100, 119 Allogeneic graft, 60, 79 Arch, adequate space in, 111, 128 Bilateral cleft lip classification, 138, 143 Allografts, 72 Archwire Bimaxillary protrusion, 109, 126 characteristics of, 165, 170 alloys not used in, 107, 124 Bioavailability of drug, 9, 24 Alloplastic grafts, 51–52 alloys used for, 124 Bisecting technique, 180, 187 Alloys for fixed prosthodontics, classification properties of, 107, 124 Bisphosphonate-related osteonecrosis of jaws system for, 42 ARCON design, advantage over (BRONJ), 65, 86 Alpha-1 receptor antagonists, 23 non-ARCON design, 30, 44 Bisphosphonates, 12 Alprazolam, 22 Argyria, 209 available for use in United States, 86 Aluminum chloride, 43 Arnold’s nerve, 82 Bitewing x-rays, 177, 184 Aluminum disks, 177, 184 Arsenic poisoning, 209 Blastomyces dermatitidis, 208 Alveolar osteitis, 56, 59, 74, 78 Arthrocentesis, 69, 93 Bluegrass appliance, 129f Alveolar ridge, properties of, 107, 125 Articulare, 124 Bolton discrepancy, 110, 127 Ameloblastic fibroodontomas, 196, 211 Articulator, 30 Bonded lingual wire, 131f Ameloblastoma, 192, 200, 203–204, 210, type of, 30, 44 Bone augmentation surgery, 55, 72 216–218 Ascher syndrome, 200–201, 208, 210, 219 Bone grafting, gold standard in, 66, 90 Amelogenesis imperfecta, 26, 40 Aspirin, 15, 18 Bone, growth of, 132 American Academy of Pediatric Dentistry, Asthma, treatment of, 20–21 Border molding, 32 137, 142 Asymptomatic maxillary central incisor, Bradykinin, 10 American Heart Association (AHA), 87 treatment for, 151, 161 Branchial cleft cysts, 219 antibiotic prophylaxis regimens for dental Atenolol, 10 Branemark’s original two-stage technique, procedures, 88 Atropine, 11 66, 88 guidelines for antibiotic prophylaxis, 14 Attrition, 43, 198, 215 Brown tumor, 207–208 Amitriptyline, 3, 15 Auricular nerve, 82 Bruxism, 43 Amoxicillin, 3, 14, 21, 24 Auriculotemporal nerve, 62–63, 82 Buccal bifurcation cysts, 203–204 Amphotericin B, 22 Autogenous bone, 52 Buccal space, 63, 84 Analgesic agents, 4, 16 Autogenous graft, 38, 51 Buccinator, 63, 83 Anatomical posterior tooth setup, 32, 45 Autograft, 72 Burkitt’s lymphoma, 206, 209

233 234 Index

C Class III malocclusion, 96, 117 Cyclosporine, 209 Class III occlusion, 105, 123 CYP3A4 enzyme system, 10 Calcifying epithelial odontogenic tumor Class IV furcation, 166, 171 CYP3A4 substrates, 19 (CEOT), 167, 171, 199, 203, 216 Class V composite preparation, 35, 48 CYP2D6 inhibitors, 20 Calcifying odontogenic cyst, 205 Clearance rate of drug, 9, 24 CYP450 system, 19 Calcium channel blocker (CCB). See also Cleft palate repair, timing of, 58, 76 Nifedipine Cleidocranial dysplasia, 198, 214 Calcium chloride, 36, 49 D Clicking, factors for, 30–31, 44 Calcium hydroxide, 150–151, 160 Clindamycin, 3, 6, 14, 16, 21, 89 Delayed tooth eruption, causes for, 108, 115, Candida infections, management of, 198, 214 Clonidine, 21 125, 134 Capsaicin, 17 Closed fracture, 79 Dementia, 68, 92 Carbamazepine, 71 Codeine, 5, 18, 20 Dens evaginatus, 147, 155 Cardiopulmonary resuscitation (CPR), 57, 75 Cold test, 147, 154 Dental display, 86 Caries, 38, 51 irreversible pulpitis and, 150, 159 Dental elevators, use of, 62, 80 burden among children, 224, 229 Collimation, 179, 186–187 Dental implant replacement, considerations in dentin, 40 Color vision, defects in, 39, 52 for, 68, 90–91 high-risk factors for, 38, 51 Comminuted fracture, 79 Dental instruments, sterilization of, 226, 230 Caries-risk assessment tool, 139, 143 Community-based intervention, to control Dental plaque, 166, 171 Cariogenic bacteria, 51 dental decay, 225, 229 Dental pulp, age-related changes to, 146, 153 Carious enamel, zones of, 38, 51 Complementary metal oxide Dental restorations, dimensions of color for, Carpules of 2% Lidocaine with 1:100,000 semiconductor/active pixel sensor, 185 29, 43 epinephrine, administration of, 70, 94 Complete cast crown, dimensions for, 37, 50 Dental x-rays, retention of, 181, 189 Cartilage, growth of, 132 Complicated crown fracture, 150, 160 Dental x-ray tube, 174, 182 Cast gold restoration, 38, 51 Composite restoration, 48 Dentigerous cysts, 203, 205, 211, 216 marginal designs for, 35, 48 tooth preparation for, 37, 50 Dentin, 36, 49 Cat-scratch disease, 199, 217, 219 Composite, structural components of, 37, Dentinoenamel junction (DEJ), 51 Cavernous sinus thrombosis, 57, 75–76 50–51 Dentistry, definition of, 4, 16 Cavity Class II, 36, 50 Conical pontics, 52 Denture Celecoxib, 4, 17–18 Connective tissue graft, 171 fitting of, 30, 44 Cell death, 104, 122 Conservative tooth preparation, 37, 50 ill-fitting denture, 32–33, 44–45 Cementoblastomas, 203, 211 Consolidation period in alveolar cleft site, Denture delivery/insertion appointment, Central giant cell granulomas, 218 64, 85 32, 45 Centric occlusion, 86 Contrast on radiograph, 167, 171 Developing solution, components of, Cephalometric radiographs Copper, 179, 186 177, 184 application of, 111, 128 Coronal caries, cariogenic bacteria for, 38, 51 Diabetes, 21 study of, 110, 126 Coronal fracture of root, treatment for, 70, 94 Diastema Cephalosporins, 12 Coronoid process of mandible, image of, 180, causes of, 114, 131 Ceramic shade matching, 29, 43 187 closure, treatment for, 104, 122 Cervical burnout, 177, 184 Corticosteroid, 5 methods to close, 114, 131 Chamfer finish line, 42 COX-1 isoenzyme, 17 treatment of, 114, 131 Chamfer margin, 48 COX-2 isoenzyme, 17 Diazepam, 22 Charged-couple device (CCD), 174, 182, 185 CREST syndrome, 198, 215 Digital radiography, advantages of, 174, 178, Charge injection device, 185 Crohn’s disease, 2, 11 182, 185 Cheek biting, by posterior teeth, 36, 49 oral manifestation of, 165, 169 Digital receptors, 174, 182 Chrome metal cast clasps, undercut for, 47 Crossbite, 97, 112, 118, 129 Digital x-rays, 178, 185 Chronic fatigue syndrome, 216 Crossbite of maxillary teeth in children, cause Digoxin, 12 Cimetidine, 2, 10, 22 of, 99, 119 Diltiazem, 22 Circumferential supracrestal fibrotomy, 99, 119 Crowded dentition, method for aligning of, Direct pulp cap, 150, 159 postorthodontic, 114, 131 104, 122 procedure, 48 Clarithromycin, 16 Crown fabrication, considerations for, 27, 41 Disc displacement, 115, 133 Clark’s rule, 55, 72 Crown lengthening Discoloration of tooth, 149, 158 Clasp design, requirements for, 34, 47 candidate for, 166, 171 Distal canal, orientation of files in, 154 Class I fracture, treatment of, 136, 140 goal of functional crown lengthening, 166, Distal shoe, 130f Class I occlusion, 101, 120 171 use of, 105, 123 percent of population with, 105, 123 indications for, 166, 170 Distal step, 114, 131 Class II amalgam preparations, 35, 47 Distal step relationship in primary molars, Crown preparation of all ceramic crown, Class II canine relationship, preservation of, 108, 125 37, 50 64–65, 85–86 Distoangular impactions in mandible, 59, 78 Cryptococcosis, 205 Class II division 2 malocclusion, maxillary Disulfiram, 16 Cryptococcus neoformans, 195, 208–209 centrals and laterals position in, 96, 117 Double lingual bar (Kennedy bar), 33, 46 CT Dentascan, 65, 87 Class II division 1 subdivision right, 110, 127 Doxazosin, 8, 23 Curve of Spee, 86 Class II malocclusion, 165, 170 Doxycycline, 16 Cuspid, 62, 81 population with, 110, 127 Drug holiday, 86–87 Cyclic adenosine monophosphate (cAMP), Class II occlusion, 105, 123 Drug-induced hemolytic anemia, 12 103, 121 Class III furcation, 171 Drug-related pigmentation, 197, 213 Index 235

E Florid cemento-osseous dysplasia, 61, 80 Helicobacter pylori, 10 Flumazenil, 7, 21, 22, 60, 79 Hemangiomas, 203 Eagle syndrome, 77 Fluoride supplementation, recommended Hemoglobin A c (HbA c) value, 164, Echinacea, 17 1 1 amount of, 136, 137, 138, 140, 141, 143 169 Ectodermal dysplasia, 99, 119 Fluoride varnish, 137, 142 Hemophilia, 200, 218–219 Ectopic eruption, 116, 134 Fluoxetine, 5 Heparin, 7, 23 Edema, 12 Flush terminal plane, 101, 107, 120, 125 Hereditary gingival fibromatosis, 115, 134 Edentulous space, restoration of, 40 Focal spot, 178–179, 186, 187 Herpes simplex, 202 Edgewise appliance, 109, 126 Fordyce granules, 136, 141 Herringbone effect on processed film, Edgewise brackets, 102, 120 Foreshortening, 178, 185 179, 187 Ehlers–Danlos syndrome, 198, 215 Foreshortening and elongation, production Histoplasmosis, 205, 209 Eikenella corrodens, 230 of, 180, 188 HMG-CoA reductase inhibitors (statins), 10 Ekta-speed film, 179, 186 Forward flexion of neck, 81 Hodgkin’s lymphoma, 194, 206, 207, Electric pulp tester, contraindication to use Frankel, 97, 112, 118, 128 209, 219 of, 137, 141 Frankfort plane, 105, 124 Hydrochlorothiazide, 7, 22 Electric pulp testing, 150, 159 Free gingival graft, source of final tissue in, Hydrocortisone, 11 Elongation of radiographic image, 178, 186 164, 168 Hydrodynamic theory, 36, 49 Enalapril, 22 Freeway space, 41 Hydroquinone, 176–177, 183–184 Endochondral ossification, 114 Frey syndrome, 82 Hyperthyroidism, 108, 125 Endodontic access form, triangular shape of, Full crown coverage, 26, 40 Hypertrophic cardiomyopathy, 12 147, 155 Functional appliances, 112, 128 Hypoesthesia, 67, 90 Endodontically treated tooth, restoration of, Functional crossbite, 97, 107, 118, 124 Hypoglossal (XII) nerve, 81 38, 51 Hypoglycemia, 11 Endodontic anesthesia, factors affecting, G 147, 154 I Endo–perio lesion, treatment of, 146, 154 Gagging, 49 Endosseous implant placement, 64, 84–85 Gardner syndrome, 116, 134, 195, 208, Ibuprofen, 18, 74 End-stage renal disease patients, 90–91 212, 220 Image magnification, minimization of, Epinephrine, 11 Gemination, 138, 142–143 179, 187 Epstein–Barr virus, 14 Generalized chronic severe periodontitis, Immediate denture (I/D), 30, 43–44 Erosion, 29, 43, 215 166, 171 Impacted maxillary canine, sequence of Erupted teeth, by 19 months of age, 136, 140 Genioglossus muscle, contraction of, 27, 41 treatment for, 105, 124 Erythema multiforme (EM), 202 Geobacillus stearothermophilus, 230 Implant failure, signs and symptoms, 64, 85 Esomeprazole, 8, 23 Gingival hyperplasia, 10 Implant insertion torque, 69, 93 Essix, 107, 124, 130f, 131 Ginkgo balboa, 4, 17 Implant placement Ester local anesthetics, 55, 72 Ginseng, 17 candidates for, 37, 50 Esthetic line of the dentition, 86 Glass ionomer cement, 226, 230–231 from mental foramen, distance for, 59, 78 Ethical framing, 222, 227 advantages of, 35, 47–48 in region of active infection, 70, 94 Ethical principles for dentist, 223, 224, 228 Glickman classification, 166, 171 Incipient caries and enamel Ewing’s sarcoma, 206, 209, 219 Glipizide, 20 hypocalcification, differential diagnosis, Exanthema subitum (roseola), 216 Glucocorticoids, 9, 21, 24 26, 40 Excisional biopsy, 60, 79 Glutathione, 18 Incision for drainage, 149, 158 of mucocele, 61, 79–80 Gnathion, 124 Incomplete canal debridement, 148, 156 Extraction, 151, 160 Golf cast clasps, undercut for, 47 Indirect digital imaging, 178, 185 Extraction wounds, healing process for, 61, 79 Gonorrhea, 205 Indirect pulp cap, 48 Gorlin syndrome, 208, 210, 212, 214, Indirect retainers, 34, 47 Inferior alveolar nerve, and deep needle F 220 Granulocytic sarcoma, 200, 206, 219 placement, 58, 77 Facial nerve function, examination of, 58, Greater palatine and incisive nerve blocks, Inferior border of maxillary sinus, 175f, 183 76–77 59, 78 Infraorbital margin, 176f, 183 Facial prominences, 81 Guedel’s stages of anesthesia, 55, 72 Instrument aspiration, prevention of, 148, Facial (VII) nerve, 81 Guided tissue regeneration, 165, 170 156–157 False-negative result, 229 Gypsum, ways to decrease setting time of, Instrument separating inside root canal, cause False-positive result, 229 35, 48 of, 137, 141 Famotidine, 22 Insulin, 5, 21, 22 Fascial space infection, treatment for, 60, 78 Interarch elastics, 100, 119 H Feather edge finish line, 42 Internal bleaching procedure, material for, Fentanyl, 6, 38, 52, 60, 74–75, 79 Hairy leukoplakia, 14–15 149, 158 Ferric sulfate, 43 Hand-foot-and-mouth disease, 208 Interproximal caries, 26, 40 Fiber-reinforced polymers, 169 Hand–wrist radiographs, 101, 120 detection of, 177, 184–185 Filtration, 178, 179, 185, 187 H1 antihistamines, 8, 24 Intralesional steroid injections, 194–195, Fistula, method for diagnosing origin of, 151, Hard palate, 176f, 183 207–208 160–161 Hawley retainer, 107, 124, 130f Intramembranous ossification, 114 Fixing time, 180, 188 Headgear, 104, 111, 123, 128 Intrapulpal hemorrhage, 158 Floor of nasal fossa, 175f, 183 mechanism of action of, 109, 126 Intrusion, 108, 125 236 Index

Invisalign treatment, 102, 120, 121 Local anesthetics Mesiodistal distance, measurement of, Ipratropium, 21 Clark’s rule of pediatric dosing of, 55, 72 68–69, 92 Irreversible pulpitis, 146, 153 effect on heart, 61, 79 Metformin, 23 IV bisphosphonate therapy, 198, 214 infection and, 56, 73 Methemoglobinemia, 56, 73–74 measurement of percentage of, 56, 73 Methyldopa, 12, 22 J mechanism of action for, 55, 73 Metoprolol, 4, 11–12 onset time, 58, 77 Metronidazole, 4, 16, 20 Jaw thrust, 57, 75 overdose, signs of, 58, 77 Midazolam, 7, 22 Joint function, indicator of, 115, 132 Local anesthetic toxicity, 55, 72–73 Midline deviation, 30, 44 Long cone, 180, 187 Mid-root fracture, 56, 74 K Lorazepam, 22 Milk, to store avulsed tooth, 151, 160 Lost-wax casting technique, 29, 43 Miller class I defects, 164, 168 Kaposi’s sarcoma (KS), 199, 217 Lower lingual holding arch, 113, 129 Miller class III defects, 164, 168 Kennedy classification for partial denture, Lower lip sucking, 98, 118 Miller’s index, class II mobility, 165, 169 33, 46 Luting ceramic restorations, 43 Mineral trioxide aggregate, 165, 170 Kennedy classification system, 165, 170 Minor connectors, 46 Kennedy Class IV classification, 34, 47 M Misoprostol, 9, 24 Ketoconazole, 22 Missing or delayed tooth eruption, 99, 119 k-file Macrophage, 164, 169 Mixed dentition analysis, purpose of, measure of diameter of tip of, 147, 154 Malignant hyperthermia, 57, 75 109, 126 measure of length of, 147, 154 Mandible fractures, sequence for, 56, 74 MO amalgam, 27, 40 Klinefelter’s syndrome, 208, 212 Mandible, growth of, 115, 133 Mode, 225, 229 Koplik’s spots, 208 Mandible landmarks, on final impression, 31, Modified ridge lap, 29, 42 44–45 Modified ridge lap pontics, 52 Mandibular growth process, 99, 119 L Molar uprighting Mandibular leeway space, 116, 134 complication in, 101, 120 Labetalol, 12 Mandibular lingual bony exostosis removal, contraindication to, 111, 127–128 Labial bar, 46 complications after, 54, 71 goal of, 107, 124 Lacrimal gland, 82 Mandibular molars, 59, 78 Monomorphic adenomas, 196, 210 Lactic acidosis, 23 Mandibular plane angle, 96, 111, 117, 128 Montelukast, 5, 20–21 Lamina dura, 175f, 183 Mandibular primate space, 114, 131 Morsicatio buccarum, 204 Lamina propria of connective tissue, Mandibular third molar impaction position, Motor fibers, 60, 79 164, 168 59, 78 Mouth preparation appointments, sequence Larger diameter implants, 92 Mandibular vertical growth, site for, 99, 119 of, 34, 47 Latent period, 177, 185 Masseter muscle, 76, 83 MPD (maximum permissible dose), yearly Lateral pterygoid muscle, 57, 76, 83 Mastication muscle, 63, 83 for nonoccupationally exposed person, Latex allergies, 225, 230 Mature bone cells, 178, 186 179, 186 Ledging, 148, 156 Maxilla and mandible, primary denture for radiation workers, 179, 186 Leeway space, 108, 116, 125, 134 support areas on, 31, 44 MSDS (medical safety data sheets), 224, 229 LeFort I maxillary osteotomy, 62, 81 Maxillary and mandibular bone Mucoceles, 210 Lesion biopsy, 150, 158–159 formation of, 108, 126 Mucoepidermoid carcinomas, 217 Leukemia, 194, 200, 206–207, 219 healing time for, 66, 88 Mucogingival junction (MGJ), 168 Leukoedema, 204 reaction to edentulism, 31, 44 Mucosal cells, 177, 184 Leukoplakia, 216 Maxillary and mandibular permanent first Multiple giant cell lesions, 214–215 Leukoplakias, 182, 202–203 molars, calcification of, 136–137, 141 Multiple myeloma, 195, 206, 207, 209, 219 Levator veli palatini muscle, 62, 80–81 Maxillary first premolar, 63, 83 Multiple supernumerary teeth, 198, 214 Levodopa, 12 Maxillary leeway space, 116, 134 Mycobacterium tuberculosis, 14 Licensure suspension, recommendation of, Maxillary major connectors, 33, 46 Mylohyoid muscle, 60, 79 222, 227 Maxillary permanent canine, eruption of, Lichen planus (LP), 15, 199, 216 138, 143 N Lidocaine, 3, 13 Maxillary primate space, 114, 131 Linea alba, 14 Maxillary prognathism, 109, 126 Nabumetone, 15 Lingually positioned file in mesial, 147, Maxillary protrusion, 97, 117–118 N-acetyl-benzoquinoneimine (NAPQI), 18 154 Maxillary removable partial denture, 35, 49 N-acetylcysteine, 8, 18, 23–24 Lingual nerve, 63, 84 Maxillary sinus, 62, 80 Nadolol, 12 Lingual plate, 33, 46 Maxillary zygomatic arch, 175f, 183 Naloxone, 6, 21, 22, 24 Lip biting, after denture fabrication, 49 Mean, 225, 229 Naltrexone, 22 Lip depressor muscle, 81 Mechanical consideration, in crown Nance appliance, 105, 113, 123, 130 5-Lipooxygenase, 21 preparation design, 28, 41 Narcotics, 18 Lisinopril, 10, 20 Medial pterygoid muscle, 76, 83 Nasion, 106, 124 Lithium, 20 Median, 225, 229 Nasopalatine neurovascular bundle, 176f, 183 Liver disease, 197, 212 Meibomian gland, 82 Necrotizing fasciitis, 54–55, 72 Loading dose (LD), 7, 22 Melanoma, management of, 197, 213–214 Necrotizing sialometaplasia (NS), 196, 211 Local anesthesia and pediatric dentistry, 138, Mental nerve, 62, 81 Needlestick/cut exposure, infection risk in, 142 Meperidine, 57, 75 225, 230 Index 237

Neurapraxia, 67, 80, 90 Osteoradionecrosis, 177, 180, 184, 187 Phalanges of fingers, 104, 123 Neuroleptic malignant syndrome (NMS), 75 Osteosarcoma, 203 Physiologic age of patient, determination of, Neurotmesis, 62, 80 Ovate pontic, 39, 52 101, 120 Neutropenia, 206–207 Overjet in class I occlusion, 109, 126 Physostigmine, 15 Nifedipine, 2, 10, 210 Oxazepam, 6, 22 Pierre Robin sequence, 197, 212 Nitroglycerine, 11 Oxygen E-cylinders, estimates of time Pilocarpine, 15 Nitroglycerin dose in anginal discomfort remaining in, 69, 93 Pindborg tumor. See Calcifying epithelial patient, 164, 169 Oxygen supplementation, 69, 93 odontogenic tumor (CEOT) Nonhealing ulcer, 3, 14 Piperacillin, 7, 23 Nonmaleficence, 223, 228 P Platysma muscle, 81 Nonnutritive sucking habit, 114 Pleomorphic adenomas, 194, 207, 217 Nonresorbable suture materials, 59, 78 Paget disease, 167, 171 Plumbism, 195, 209 Nonrigid archwire, placement of, 102, 120 Palatal bonded retainer, drawback of, 107, Pocket reduction, 166, 171 Nonrigid splints, 56, 74 125 Pogonion, 106, 124 Nonsteroidal anti-inflammatory drug Palatal connector, 35, 49 Pontic design selection, 29, 42–43 (NSAID), 8, 15, 17–18, 24 Palatal embrasures, 164, 169 Porion, 111, 128 Nonworking condyle, movement of, Palatal expander, 98, 107, 119, 124 and orbitale, 97, 117 27, 41 Palatal palate, 49 Posterior bite plate, 105, 123 Nylon, 59, 78 Panoramic radiograph, advantages of, 177, Posterior bitewing radiographs, 226, 231 Nystatin, 6, 21 184 Posterior denture teeth, and cheek biting, 49 Panorex radiographs, 226, 231 Posterior teeth, selection of, 45 Panorex, reverse occlusal plane curve on, O Post placement, after endodontic treatment, 177, 184 27, 41 Occlusal rim, 45 Papillon–Lef`evre syndrome, 196, 210, Post, use of, 41 Occupational Safety and Health 220 Post width, during placement of post, Administration (OSHA), 224, 229 Para-aminobenzoic acid (PABA), 22 165, 169 Odontogenic infections, 71 Paracoccidioides brasiliensis, 209 Povidone iodine, mechanism of action of, Odontogenic keratocysts (OKCs), 60, 78–79, Parafunctional movement, 26, 40 226, 230 196, 211, 216, 218 Paralleling technique, 175, 183 Prazosin, 8, 23 Odontogenic myxomas, 203, 218 disadvantage of, 178, 186 Prefabricated posts, materials for, 165, 169 Odontomas, 61, 80 Parel’s classification system, 72 Pregnancy-induced hypertension (HTN), 12 Oil of cloves, 49 Parotid glands, 62, 82 Prevalence, 225, 229 Omeprazole, 6, 11, 22 Paroxetine, 7, 22 Prilocaine, 58, 77 Open bite, 112, 129 Pathologic fracture, 60, 79 Primary canines, premature loss of, 110, 127 Open-coil springs, usage of, 103, 121 Patient education for denture care and use, Primary intention, 79 Open reduction of subcondylar fracture, 80 31, 44 Primary radiation, 179, 186 Opioids, 9, 24 Patient records, 223, 228 Primary resistance form feature, in amalgam Oral and pharyngeal cancer, incidence of, Patients’ visits, steps in, 148, 155–156 preparation, 165, 170 225, 229 Patient treatment, rule for ethical behaviors Primary second molar, early loss of, 113, 130 Oral candidiasis, 6 in, 222, 227 Primary teeth, 136, 141 Oral hairy leukoplakia (OHL), 194, 204, Patient value system, 223, 228 Primate space, 106, 124, 137, 141 206 Pediatric behavior management techniques, in primary dentition, 104, 122 Oral infections, bacteria in, 54, 71 139, 143 Primers, 36, 49 Oral submucous fibrosis, 192–193, 204 Pemphigus vulgaris, 182, 199, 202, 210, 217 Proliferation stage, 138, 142 Oral tissue, benefits of resting of, 32, 45 Penicillin, 6, 9, 12, 21, 24 Propoxycaine, 6, 22 Orbicularis oris muscle, 82 Percussion test, 150, 159 Propranolol, 7, 22 Orbitale, 106, 124 Percutaneous injury, information on, 225, 229 Prostaglandin analogs, 15 Oroantral communication, 56, 74 Periapical cysts, 203, 205–206 Prosthetic instability, 89 Orofacial implant position, 69, 93 Periapical x-rays of the maxilla, 180, 187–188 Prosthetic obturation of postablative Orthodontic diagnostic records, routine set of, Periodontal abscess, 164, 168 palatomaxillary defects, 66, 89 108, 126 Periodontal ligament, 175f, 183 Protamine, 22 Orthodontic therapy, adult patients in, 99, Periodontal surgery, 3, 14 Proton pump inhibitors (PPIs), 11 119 objective of, 166, 171 Provisional restoration, requirements of, 35, Orthodontic treatment, negative sequelae, Periodontitis, 166, 170, 171 49 107, 112, 124, 128 in U.S. population, 224, 229 Proxabrush, 171 Orthostatic hypotension, 12 Perioral vesicles, 14 Pseudo–class III malocclusion, 96, 110, 117, Osler–Weber–Rendu syndrome, 215 Peripheral giant cell granuloma (PGCG), 218 127 Osseointegrated endosseous implants, Peripheral nerves, blockade of, 79 Pterygomaxillary fissure, 176f, 183 placement of, 68, 92 Peripheral ossifying fibroma, 200, 218 Pulpal depth of amalgam preparation, 35, 48 Osseous periodontal surgery, palatal approach Permanent first molars, 36, 50 Pulp damage during cavity preparation, 36, 50 for, 164, 169 Permanent incisors, crowding of, 110, 127 Pyogenic granuloma (PG), 218 Osteoclasts and osteoblasts, 108, 125 Permanent mandibular canines, 115, 134 Osteoclasts in dark zones, 103, 122 Permanent second premolars, 36, 49 Q Osteogenesis, 66, 89 PFM (Porcelain Fused to Metal) crown Osteoprogenitor cells, 114 restoration, 37, 50 Quinidine, 12 238 Index

R Sanitary (hygienic) pontics, 52 Supragingival margin, advantages of, 28, 42 Scar tissue, 152, 161 Supramentale, 96, 117 Radicular (periapical) cysts, 211 Sealants, retaining of, 49, 50 Surgery, 96–97, 117 Radiograph Secondary intention, 61, 79 Suspected malignancies, 197, 212–213 brown, 180, 188 Second stage implant surgical procedure, Swallowing, 26, 40 dark, 181 188 69, 93 Syphilis, 205 light, 181, 188 Sensors, 185 Systemic antiviral medication, in herpes zigzagged pattern, 174, 182 Separated instrument, strategy for, 148, 156 zoster infections, 201, 220 Radioresistant cells, 177, 178, 184, 186 Serial extraction, 108, 116, 125–126, 134 Radiosensitive cells, 177, 184 Setting time, 48 RAD (radiation absorbed dose), 181, 189 T Short cone, 180, 187 Rampant caries, 38, 51 Shoulder finish line, 28, 42 “T” and “D” sounds, formation of, 36, 49 Ramsay Hunt syndrome, 199, 215–216 Sialolith, 60, 78 Teeth removal in patient with atrial Ranitidine, 22 Silver, 165, 169 fibrillation, 59, 78 Ranula, 210 Simvastatin, 10 Temazepam, 22 Rapid palatal expander (RPE), 110, 127 Single-tooth implants, 38–39, 52 Temporalis muscle, 76, 83 Rebasing, denture, 45 Single-unit implant, 40 Temporary anchorage device (TAD), Recurrent aphthous stomatitis, 202 Skeletal class II malocclusions, 110, 127 110, 127 Recurrent herpes labialis, 193, SLOB rule, 154, 175, 183 Temporomandibular disease (TMD), 115, 204–205 SLUDGE, 16 132, 133 Recurrent intraoral herpes, 208 Smear layer, 36, 49 classifications, 115, 133 Refractory periodontitis, 167, 171 Smile arc, 65, 86 Temporomandibular joint (TMJ), 54, 71 Regeneration, 170 Smoker’s melanosis, 204 Terazosin, 8, 20, 22, 23 Relining, denture, 45 Sodium perporate, 149, 158 Tetracaine, 6, 22 Removable appliances, indications for, Sodium valproate, 195, 209 Therapeutic index (TI), 12–13, 19 111, 128 Solid-state detectors, to capture direct digital Three-unit fixed partial denture, 26, 40 Removable retainers, 107, 113, 124, 130 x-rays, 178, 185 Thrombocytopenia, 206 REM (Roentgen equivalent man), Space maintainers, 113, 130 Thyroglossal duct cyst, 200, 219 181, 189 of prematurely missing primary molar, Time to refer patient to orthodontist, 103, 121 Renal toxicity, 20 105, 123 Timolol, 12 Repair, 170 Specificity, 229 Tipping, 103, 108, 121, 125 Resin-modified glass ionomer-luting agent, Speech sounds, and related malocclusion, Tissue-borne functional appliance, 112, 128 29, 43 110–111, 127 Tongue biting, after denture fabrication, 49 Resinous balsam, 49 Sphenooccipital synchondrosis, 124 Tongue carcinoma, 63, 83 Resistance form of preparation, 47 Squamous cell carcinoma, of oral cavity, 63, Tongue crib, 129f Retention, 113, 131 83, 216 Tongue thrust swallow, 114 Retention and resistance form, 28, 41–42 Stafne bone cyst, 193, 205–206, 210–211, 211 Tooth decay, in adults, 224, 229 Retention form of preparation, 47 Stafne defect, 61, 80 Toothpastes, fluoride in, 136, 141 Retentive clasp, 34, 46 Staphylococcus aureus infections, 21 Tooth preparation, strategies for, 37, 50 Retraction cord, 29, 43 Stensen’s duct, 58, 63, 77, 84 Tooth structure reduction, 38, 51 Retrograde treatment or orthograde Sterilization failure, measures to be taken in, Tort law, 223, 228 treatment, 149, 157 226, 230 Townes projection Reverse-pull headgear, 97, 118, 165, 170 St. John’s Wort, 17, 19 for condyles evaluation, 180, 188 Reye syndrome, 18 Straight-pull headgear, 112, 129 Tramadol, 57, 75 Rheumatoid arthritis, 9, 24 Straight-wire appliance, 112, 128 Transection of cervical branch of facial nerve, Richards analysis, 110, 126 Stress–strain curve, 102–103, 121 62, 81–82 Rickets, 99, 119 Sturge–Weber angiomatosis, 215 Translation movement, 101, 104, 120, 122 Rifampin, 14, 16 Styloid process, 176f, 183 Transpalatal bar, 129f Right unilateral cleft lip, 62, 81 Subcondylar fracture and parasymphyseal Traumatic bone cysts, 211 Rituximab, 12 fracture, management of, 61, 80 Treacher–Collins syndrome, 61, 77, 79 Rockets power dancers (RPD), 27, 41 Subdivision to classification, 105, 123–124 Treponema pallidum infection, 193, 205 Root canal failure, 148, 157 Subject contrast, 182 Triamcinolone, 21 Root canal therapy, 146, 147, 148, 149, 153, Sublingual gland, 82 Triazolam, 22 155, 156, 157 Submandibular gland, 82 Tricyclic antidepressant (TCA), 8, 15, 23 Rosin, 49 Submentovertex projection, for zygomatic Trigeminal nerve, 82 Rotational movement of forceps, use arch fractures, 177, 184 Trigeminal neuralgia, 54, 69, 71, 94 of, 83 Sugar, for management of hypoglycemia, 11 Trigeminal (V) nerve, 80–81 Rubber base (polysulfide), 32, 42 Sulfonylureas, 8, 20, 23 True-negative result, 229 Rubeola (measles), 195, 208 Supernumerary teeth, 99, 108, 119, 125 True-positive result, 229 gender predilection for, 116, 134 Tuberculosis (TB), 14 S multiple, 116, 134 Tuberous sclerosis, 215 Saddle-ridge lap, 42 site for, 105, 123 Turbid dentin, 26, 40 Safety index, 12 Supracrestal circumferential fibrotomy, 107, Type III alloys, for fabrication of Crowns and Saline rinses, 177, 184 125 FPDs, 28, 42 Index 239

U W X-ray developer, 180, 188 X-ray fixer, 178, 185 Unethical behavior, 227 Wall convergence angles in crown X-ray imaging, 174, 182 of dental student, 222, 227 preparation, 28, 42 film contrast, 182 in physician-patient relationship, 222, 227 Warfarin, 18–19 higher kilovoltage images, 189 Unilateral crossbite, 100, 119 accidental ingestion of, 6, 22 lower kilovoltage images, 189 Unmyelinated C fibers, 146, 153–154 teratogenic effect, 7, 23 rules for, 174, 179, 182, 187 Upright teeth, 103, 121 Warthin’s tumor, 207 subject contrast, 182 U.S. states with mercury safety laws, 225, 229 Waters projection zigzagged pattern on, 182 for maxillary sinus evaluation, 180, X-ray operator and patient, distance between, 187–188 V 178, 186 for midface fractures evaluation, 180, 188 X-rays, 179, 186 Valproic acid, 8, 23 Wharton’s duct, 77 Verapamil, 209–210 Wires, load–deflection rates of, 103, 121 Vertical dimension of occlusion (VDO), 27, Wisdom teeth, removal of, 74 Z 41 Wrap-around retainer, 130f guidelines for evaluation, 32, 45 Wrist–hand radiograph, 104, 123 Zileuton, 21 Vertical fracture of mesial root, 148, 157 Wrought alloy clasps, undercut for, 34–35, 47 Zinc oxide eugenol impression paste, Vertical mandibular opening, normal range 36, 49 of, 63, 83 Zoledronic acid, 2, 12 X Vital pulp therapy/apexogenesis, 149, 158 Zoster sine herpete, 216 Vitamin K, 6 Xenografts, 51, 72, 165, 169 Zygomaticus implants, 63, 84 Volume of distribution (Vd), 9, 24 Xerostomia, 3, 12, 107, 124 ABOUT THE AUTHORS

Jason E. Portnof, DMD, MD Dr. Portnof is currently full-time faculty of the Oral and Maxillofacial Surgery residency program at Beth Israel Medical Center/Jacobi Medical Center/Albert Einstein College of Medicine. He is Assis- tant Professor in the Departments of Dentistry and Otorhinolaryn- gology, Albert Einstein College of Medicine. He completed his undergraduate education at Washington University in St. Louis, Missouri, and received his doctorate in dental medicine (DMD) from Nova Southeastern University in Ft. Lauderdale, Florida. His medical school training (MD) was completed at Weill Cornell Medical College in New York City. He completed a residency in oral and maxillofacial surgery and an internship in general surgery at New York Presbyterian Hospital/Weill Cornell Medical Center, where he also served as Chief Resident in oral and maxillofacial surgery. Dr. Portnof completed a fellowship in pediatric maxillofa- cial surgery/craniomaxillofacial surgery in the Department of Plastic and Maxillofacial Surgery, Royal Children’s Hospital, Melbourne, Australia.

Tim Leung, DMD, MHSc, MD Dr. Leung is presently in private practice oral and maxillofacial surgery in Toronto, Ontario, Canada and New York, NY. He is also a faculty member of the oral and maxillofacial surgery residency pro- gram at Jacobi Medical Center Department of Dentistry, Oral and Maxillofacial Surgery, Bronx, NY. He completed a residency in oral and maxillofacial surgery and an internship in general surgery at New York Presbyterian Hospital—Weill Cornell Medical Center, where he also served as Chief Resident in oral and maxillofacial surgery. He received an MD degree from Weill Cornell Medical College, and a DMD from University of Pennsylvania School of Dental Medicine. He earned an MHSc from the University of Toronto. His undergrad- uate education (BA) was completed at Cornell University.

Melvyn S. Yeoh, DMD, MD Dr. Yeoh is currently the Chief Resident in oral and maxillofacial surgery at New York Presbyterian Hospital, Cornell Medical Center. He completed an internship in general surgery at New York Pres- byterian Hospital—Weill Cornell Medical Center. He received his dental degree from the University of Pennsylvania and his medical degree from Cornell University. His undergraduate education was at Claremont McKenna College in Claremont, California.